OTR Practice Questions

अब Quizwiz के साथ अपने होमवर्क और परीक्षाओं को एस करें!

What is the leading cause of death in individuals over 65 years of age? a. Cerebrovascular disease b. Coronary heart disease c. Chronic obstructive pulmonary disease d. Cancer

b. Coronary heart disease

Which model of practice views a disability as being caused by an injury or event? a. Education model b. Medical model c. Telehealth model d. Community model

b. Medical model

an ot provides consultation services to members of a town chamber of commerce who have expressed interest in improving their business accessibility. The minimum door width that the ot recommends as being acceptable and not requiring modification is: 32 inches 28 inches 30 inches 34 inches

32 inches

According to Occupational Safety and Health (NIOSH) guidelines, what is the maximum weight an OT practitioner should handle before using a lift to assist with patient handling becomes necessary? 15 lbs 25 lbs 35 lbs 40 lbs

35 lbs

An occupational therapist provides home-based early intervention evaluations. A referral for an 18-month-old child notes that the child is able to finger feed effectively but is not able to use a spoon or suck from a straw. The occupational therapist puts together supplies to bring to the child's home and plans activities to use during the developmental evaluation. Which developmental age is most important for the therapist to consider when selecting objects and activities to bring to the evaluation session? A. 6-9 months .B. 9-12 months. C. 12-18 months. D. 18-20 months.

.B. 9-12 months. -straw and utensil at 10mo

Upon evaluation a client for a wheelchair, the OT determines that a standard narrow adult chair would be suitable for the individual. The dimensions of the chair will be: 18 wide x 18 deep x 20 high 16 wide x 16 deep x 18.5 high 14 wide x 16 deep x 18.5 high. 16 wide x 16 deep x 20 high

16 wide x 16 deep x 20 high

An OTA is screening a patient with cognitive decline using the Allen Cognitive Level screening tool. When the OTA asks the patient to imitate the running stitch, the patient appears to be confused and disoriented. What should the OTA do next? Ask the patient to imitate the single cordovan Ask the patient to imitate the whipstitch Administer the higher cognitive test Administer the lower cognitive test

Administer the lower cognitive test

Which type of wheelchair cushion is the MOST appropriate for a patient who has a history of developing pressure ulcers? Honeycomb Foam Contoured Air

Air

Child with mild spastic diplegia wants to participate in neighborhood activities with peers. The family's main goal is for the child to ride a bike. Which bike is the best for the OT to recommend? A. A hand-propelled bike with hand brakes B. A foot-propelled bike with hand brakes C. A foot-propelled bike with foot brakes

B. A foot-propelled bike with hand brakes -mild spastic diplegia is mild LE and min-no UE, foot peddles will be good for strength development and hands are best for safety

An inpatient who has Guillain-Barré syndrome is participating in a rehabilitation program. The patient requires 25-50% assistance to complete self-feeding tasks. What terminology BEST describes the level of assistance the client requires for feeding? A.Maximal assist B.Moderate assist C.Minimal assist

B.Moderate assist

A patient who has hemiplegia currently transfers to a variety of surfaces using a stand-pivot technique with moderate assistance from a caregiver. The main bathroom in the patient's home has a standard tub/shower combination. Which type of durable medical equipment would be MOST BENEFICIAL for supporting the patient's participation in bathing at home? Hydraulic bath lift Shower chair with armrests Three-in-one shower commode Tub transfer bench

Tub transfer bench

A 20-year-old male patient had recently been involved in a car crash and suffered a TBI. After reviewing his chart, you read that he has cortical blindness and cannot see. What part of the brain is this most likely an injury from?Select one: a. Frontal lobe b. Occipital lobe c. Temporal lobe d. Parietal lobe

b. Occipital lobe

Which precaution is established for a patient who is coughing and sneezing from influenza? a. contact b. standard c. droplet d. airborne

c. droplet

OT receives a referral to eval individual's EF following mild CVA. Which are most relevant foci for this eval? A. Person's attention and memory B. Person's job interests and efficacy C. Person's spatial relations and praxis D. Person's initiation and planning

D. -mild CVA

An OT is completing a wheelchair assessment for an adult client who has progressive cerebellar degeneration and requires the use of a power wheelchair for mobility. What type of control switch would be MOST BENEFICIAL for enabling this client to independently operate the wheelchair? A. Sip and puff B. Joystick C. Chin-activated toggle D. Proximity-sensing microswitch

A. Sip and puff -no coordination involved

A client's family purchased an ultra lightweight wheelchair (K0005) for the client. It has the correct seat height, seat width, and seat depth, but the client has difficulty propelling it. The OTR® asks the client to demonstrate moving the chair forward. What two potential problems would the OTR® FIRST focus on? A. Location of the axle in relation to the client's center of gravity, seat-back height B. Height of seat back, position of leg rests C. Position of leg rests, location of axle in relation to the client's center of gravity D. Distance of axle from the floor; location of the axle in relation to the client's center of gravity

A. Location of the axle in relation to the client's center of gravity, seat-back height -too high seat will prevent elbow extension

What MUST be addressed as part of the intervention plan for children who meet eligibility for services under the mandates of the Individualized Family Service Plan (IFSP) component of the Individuals With Disabilities Education Act (IDEA, Part C)? A. Needs of both the child and the family B. Rationale for providing one-on-one sessions C. Activities for achieving curriculum-based goals D. Reasonable accommodations beneficial to the child

A. Needs of both the child and the family

An OT evaluates a child with mild spastic hemiplegia. The child uses palmar grasp to pick up small objects and is unable to oppose the pads of the thumb and index finger. Which grasp should the therapist establish for the child to achieve as a short-term goal? A. Radial palmar grasp B. Inferior pincer grasp C. Ulnar palmar grasp D. Scissors grasp

A. Radial palmar grasp

A client is being discharged from a rehab facility. During the discharge interview, the OT asks questions such as, "Who is available to assist the client in each physical context the client needs to be in?" "When are they available?" "Who will check the Roho cushion and reinflate it as needed?" "Who will clean the cushion cover?" Under which assessment category do these questions fall? A. Physical context B. Social context C. Physical skills context D. Equipment context

B. Social context

An OT practitioner working in a long-term care facility needs to evaluate the long-term memory of a resident. Which of the following methods is BEST for evaluating memory of personally experienced events (episodic memory)? A. Show the person a series of objects and ask him to recall the objects within 60 seconds. B. Ask the individual how he spent New Year's. C. Have the individual state the place, date, and time. D. Ask the client to remember to bring a specific item to the next therapy session.

B. Ask the individual how he spent New Year's.

The MOST helpful information gained after using Allen's Cognitive Disability theory to determine the level of cognitive function in a developmentally disabled adult is: _______" Identifying the client's difficult behaviors Improving prevocational skills Estimating a patient's cognitive level Improving social skills

Estimating a patient's cognitive level

Which communication device is used to turn pages and point on a keyboard when a patients hand and arm function are inadequate? Head control Prone stander Head pointer/wand Sip and puff

Head pointer/wand

Which device is typically recommended to turn pages and point on a keyboard when a patient's hand function is limited? AAC device Prone stander Head pointer/wand Sip and puff

Head pointer/wand

The OT provides home-based services to a child with developmental delays. The parents have identified a goal of including the child in the completion of household chores. The child can put toys away with reminders and wipe up spills. Which activity should the therapist focus on during intervention? A. Coping parents doing chores B. Helping to sort laundry C. Making a bed D. taking out the trash

B. Helping to sort laundry

An occupational therapist observes that a child can open a combination lock, open a lock with a key and turn a pencil over to erase. In documenting the child's in-hand manipulation, which is most accurate for the occupational therapist to report the child can effectively perform? A. Finger-to-palm translation. B. Rotation. C. Palm-to-finger translation. D. Shift.

B. Rotation.

An OT treats a child with severe spastic CP and cleft palate to improve feeding and swallowing skills. The child's positioning includes hips flexed at 90 degrees and knees at 90 degrees. To facilitate normal swallowing and prevent aspiration, how should the therapist set up the tray table and position the child's head? A. Table at elbow height and head slightly tipped back B. Table at elbow height and chin tucked downwards C. Table at chest height and chin tucked downwards. D. Table at chest height and head slightly tipped back

B. Table at elbow height and chin tucked downwards

An OTR® is working with a 6-year-old child with attention deficit hyperactivity disorder on self-monitoring energy and arousal levels inside the classroom. Which intervention is BEST to use for this goal? A. Train the parent to use a sensory diet to help the child remain calm throughout the day. B. Use the ALERT Program to train the child to self-regulate. C. Train the child to use breathing and relaxation techniques when feeling hyperactive. D. Provide a therapy ball for seating in the classroom.

B. Use the ALERT Program to train the child to self-regulate.

When positioned in supine, at which age does an infant FIRSTdisplay maintaining the head in a midline position and holding the legs together? A 7-8 months B 5-6 months C 3-4 months D 0-2 months

C 3-4 months

Toddler with spastic quadriplegia CP demonstrates consistent tonic bite reflex. Which technique should the OT use to help inhibit this reflex? A. Move spoon from side to side on tongue B. Walk a spoon down the tongue, going proximal to distal C. Press spoon down firmly on center of tongue D. Stroke the tongue in circular motion w/ a firm object

C. -B. should be distal to proximal -A. and D. should be avoided bc circles and side to side cause gag reflex

An OT consults with the parents of a child with anxiety disorder and challenging behavior that interfere with homework completion. They consider modifications to the child's bed room and homework routine. What recommen dation would BEST help the child with self organization skills? A. Providing open storage space for clothing and school supplies for easy access. B. Painting walls and furniture in bright colors to create a cheerful environment and hanging-a picture schedule. C. Adding cabinets with labeled compartments to store items out of sight and creating a consist ent nightly routine. D. Giving the child a choice about when he wants to complete homework.

C. Adding cabinets with labeled compartments to store items out of sight and creating a consist ent nightly routine.

To ensure provision of best practice, a new entry-level OT will be provided with supervision of their caseloads. At what level should this sup be provided? A. Routine B. General C. Close D. Minimal

C. Close -direct, daily contact -intermediate is general (every 2 wks/monthly) -advanced is min

An occupational therapist works in a school system with a child with developmental delays. One of the goals of treatment is to develop prewriting skills. The child exhibits the ability to grasp a pencil proximally with crude approximation of the thumb, index, and middle fingers and the ring and little fingers slightly flexed. The therapist develops an intervention plan. Which grasp should be the focus for the implementation of intervention? A. Digital pronate grasp. B. Static tripod posture grasp. C. Dynamic tripod grasp. D. Palmar supinate grasp.

C. Dynamic tripod grasp. -progress to dynamic

Following a left CVA, an individual receives OT services in subacute rehab. The pt's goal is to be indep. w/ dressing. The pt demonstrates decreased memory, poor sequencing, and ideational apraxia. Which is the most effective for the therapist to provide when teaching one-handed dressing techniques to this pt? A. Step-by-step verbal instructions B. Sequenced photos of the steps in dressing C. Physical prompts to initiate steps in dressing D. A full-length mirror for the client to observe self-dressing performance

C. Physical prompts to initiate steps in dressing -kinesthetic to enhance learning -B. and A. may be confusing with apraxia

House painter is referred to OT after reoccurrence of rotator cuff tendinitis. The physician prescribes a conservative intervention approach. Which rec is best for the OT to take? A. Continue performing above shoulder activities to build rotator cuff strength B. Use an extension handle in the paint roller when painting ceilings C. Sleep with shoulder extended and adducted D. Sleep with shoulder fully flexed and adducted

C. Sleep with shoulder extended and adducted -rest shoulder -B. would still need overhead since he is painting ceilings

OT reevaluates client prior to discharge from rehab. Client exhibits residual cog deficits in problem-solving. To develop requisite problem-solving skills for indep. functioning at home, which ability should the OT work on with the client? A. Performing routine morning self-care B. Making a shopping list of grocery staples C. Washing the family's laundry D. Reading a bedtime story to the client's children

C. Washing the family's laundry -stain removal, sorting fabrics, etc. -B. is structured bc its "staple"

While playing cards with a patient, the patient blurts out "The mail just came." What Rancho level is this patient exhibiting? Generalized Response Localized Response Confused and Inappropriate Confused and Appropriate

Confused and Inappropriate

An occupational therapist is creating a resting hand splint. What is the appropriate position for the wrist? A 30-45 degrees flexion B 30-45 degrees extension C 20-30 degrees flexion D 20-30 degrees extension

D 20-30 degrees extension

What is the MOST appropriate first intervention after a fall? A Call 911 if appropriate B Report the fall to a supervisor C Ask others what happened D Assess for injury

D Assess for injury

An OTR® is working in an inpatient psychiatric setting where the average length of stay is 5 days. Which group activity would be MOST appropriate for patients with depression? A. A parallel group in which participants work on a free-form clay project of their choice B. A cooperative group in which participants plant a garden C. A project group in which participants paint a watercolor on a blank canvas D. A parallel group in which each participant completes his or her own solid-color ceramic tile trivet

D. A parallel group in which each participant completes his or her own solid-color ceramic tile trivet - lots of structure and allows for them to be around each other but not have to interact much

An individual is about to be discharged to home following a brief hospitalization for sub stance abuse. The family asks the OT what they should do the first weekend at home. Which of the following suggestions is MOST appropriate? A. Throw a welcome home party for some close friends. B. Go out to hear a favorite band. C. Go on a minivacation. D. Attend an AA meeting.

D. Attend an AA meeting. -build a support system -A. and B. are parties and are bad -C. is stressful

An OTR® is interviewing a new client who physically presents with a "barrel chest." This appearance is most often associated with what condition? A. Cystic fibrosis B. Asthma C. Collapsed lung D. Emphysema

D. Emphysema

During which home maintenance tasks would symptoms related to constructional apraxia be MOST EVIDENT? A. Washing the interior sides of single pane windows B. Using a roller brush to paint an interior wall of the home C. Vacuuming floors in the main living area of the home D. Following written instructions for installing curtain rods

D. Following written instructions for installing curtain rods

Over whom does the AOTA Ethics Commission have jurisdiction? A. All AOTA members B. All graduates of accredited OT and OTA programs C. Licensed OTs and OTAs but not occupational therapy educators D. OTAs and OTs with AOTA membership at the time of an alleged incident

D. OTAs and OTs with AOTA membership at the time of an alleged incident -must be a member

An individual who sustained partial thickness burns is ready for the rehabilitative phase of treatment after 4 weeks in intensive care. The wounds have all closed and there are no open areas remaining. Prior to performing ADL training, what is the MOST important action for the OT to take? A. Debride the wounds. B. Remove compression garments. C. Complete UE strengthening activities. D. Perform skin conditioning techniques.

D. Perform skin conditioning techniques. -A. may occur in acute phase and isnt related to ADLs

An OTR® at an acute care hospital is evaluating a client in the onset stage of Guillain-Barré syndrome (GBS) who requires total assistance for most tasks. At this stage, what goals should the OTR® focus on? A. Patient and caregiver education on the progressive nature of this disease B. ADL performance, especially adaptive equipment training and other compensatory strategies C. ROM and strengthening to prevent muscles from further weakening D. Positioning to prevent skin breakdown or contractures and to allow access to needed items

D. Positioning to prevent skin breakdown or contractures and to allow access to needed items -This client is likely not able to reposition him- or herself or independently clean up incontinence issues. -ADLs are too high level

A client is being evaluated for a new wheelchair cushion because a Stage I pressure ulcer developed on the skin over the right ischial tuberosity. The OT is using pressure mapping to determine which cushion best redistributes pressure for the client and determines that one type of cushion best redistributes the pressure. However, the client refuses it because it feels unstable. Which cushion type would MOST likely elicit this response? A. Foam B. Foam molded C. Hybrid foam and gel D. Air filled

D. air-filled air-filled responds to clients movements

An OT is working with a patient in the in-patient rehab department. The patient recently sustained a TBI during a MVA and has become very agitated. When the OT presents the patient with a simple grooming activity of brushing his teeth, he does not follow the OT's instructions and instead throws the toothbrush on the floor. He then states that he wants to be left alone. How should the OT best document the session in the "A" part of the SOAP note? Patient became agitated and continues to demonstrate difficulty following instructions Patient appears to be agitated and threw the toothbrush on the floor Patient stated "he wants to be left alone" OT will continue to work on grooming activities with the patient

Patient became agitated and continues to demonstrate difficulty following instructions

An OTR Face fabricating a split for a client with median nerve injury. What position should be avoided to protect the clients wrist in hand? a. thumb palmar abduction b. metacarpophlangeal hyperextension c. thumb opposition d. wrist flexion

b. metacarpophlangeal hyperextension

In regard to object use, at what age is a child's participation with a toy characterized by the movement he performs with the toy? a. 3-6 months b. 9-12 months c. 6-9 months d. 12-15 months

a. 3-6 months

An OT conducts a home evaluation for an individual with a complete T10 level spinal cord injury. The only entrance to the home has five steps, a total of 35 inches in height. The therapist recommends that the family have a ramp constructed that is: a. 35 feet long b. 17.5 feet long c. 48 feet long d. 70.5 feet long

a. 35 feet long -FEET long 1:12 you are dumb

An occupational therapist conducts an on-site accessibility assessment with a building contractor who is remodeling an apartment building. The therapist recommends modifications for doorways which do not meet minimum accessibility width standards. The contractor states the building's owner wants to exceed minimum standards without incurring unreasonable costs. Which is the most preferred doorway measurement for the occupational therapist to recommend the contractor use for the remodeling? a. 36 inches. b. 32 inches. c. 34 inches. d. 38 inches.

a. 36 inches.

Following nerve injury repair surgery, an individual is evaluated for sensory return. Which measurement tool is best for the occupational therapist to use to assess for the return of vibration? a. A tuning fork. b. Nylon filament. c. A disk-criminator.

a. A tuning fork. -b is semmes and c is two-point disc.

An OT is evaluating a 10-month-old child who was referred to OT for developmental delays. The infant exhibits extension of the extremities on the face side and flexion of extremities on the skull side. Which reflex is not integrated and will likely affect the child's ability to creep and crawl?Select one: a. ATNR b. STNR c. TLR d. BOB

a. ATNR

Which classification of amputation is described when a patient has an amputation through the mid-shaft of the humerus? a. Above-elbow amputation b. Elbow disarticulation c. Shoulder disarticulation d. Forequarter amputation

a. Above-elbow amputation

Teaching methods vary depending on patient situation and abilities. Which teaching method is used when a therapist shows a patient how to dress the upper extremity using only one hand, followed by the patient attempting the task? a. Demonstration and performance b. Audiovisual aids c. Simulation d. Explanation and discussion

a. Demonstration and performance

After a patient undergoes posterolateral hip replacement surgery, it is important that the occupational therapist instruct the patient in hip precautions. Which of the following is NOT a hip precaution for a posterolateral total hip replacement? a. no hip flexion b. no external rotation c. no hip abduction d. no internal rotation

b. no external rotation

An individual attends a community day treatment program to assist in recovery from a major depressive disorder. The client has fair eye contact and responds verbally to interactions initiated by others. The person's cognition is intact. Which group level is best for the occupational therapist to recommend this client attend? a. Project. b. Parallel. c. Cooperative. d. Mature.

a. Project.

An occupational therapist is providing intervention to allow for improved leisure activity for an adolescent with a C3 level SCI. What is an appropriate intervention? a. Educate in the use of a head pointer to turn the pages of a book b. Educate in the use of a typing splint to use email and the internet c. Educate in the use of a tenodesis grasp to hold a phone d. Educate in the use of a sports wheelchair to play wheelchair basketball

a. Educate in the use of a head pointer to turn the pages of a book

An occupational therapist is training a patient with a level C4 SCI in self-care tasks. What is an appropriate intervention? a. Educating the patient about using a long straw to drink from a glass b. Educating the patient about using a button hook for dressing c. Educating the patient about using a rocker knife for feeding d. Educating the patient about using a cup with large handles for drinking

a. Educating the patient about using a long straw to drink from a glass

What is the FIRST action the OTR should take before completing an evaluation of a patient in a forensic psychiatric hospital? A.Interview the guards to determine the reason for incarceration. B.Contact the patient's family to determine prior level of function. C.Review the patient's medical record for safety precautions.

C.Review the patient's medical record for safety precautions.

An occupational therapist provides an accessibility consultation to a business that has hired a new employee who uses a wheelchair for mobility. The only entrance to the business has four steps, each seven inches high. Which ramp length is best for the therapist to recommend the business have constructed? a. 14 feet. b. 28 feet. c. 35 feet. d. 46 feet.

b. 28 feet.

At what age would one expect an infant to display an fine pincer grasp when grasping a pellet, according to Erhardt Prehension Developmental Levels? a. 9 months b. 6 months c. 8 months d. 12 months

d. 12 months

What is the PRIMARY reason for recommending an air-filled wheelchair cushion to a client with a C5 (ASIA A) spinal cord injury? A.To provide even relief from pressure B.To offer low daily maintenance needs C.To provide a stable base of support

A.To provide even relief from pressure

A 5-year-old child with Down syndrome and delayed motor skill development is learning to self-toilet. Over the past several months, the child has made significant progress and is now able to complete age-appropriate toileting tasks, with the exception of independently pulling up pants. Based on the child's current abilities, what is the MOST EFFECTIVE technique to include in the intervention plan to support this child's success with toileting? A.Use preferred games to increase upper extremity strength and coordination. B.Read a social story to reinforce successful daily bowel and bladder routines. C.Develop a pictorial reward chart using a system of positive reinforcement.

A.Use preferred games to increase upper extremity strength and coordination.

An OTR is evaluating an inpatient who has chronic obstructive pulmonary disease. The client reports feeling comfortable when lying in bed but becomes short of breath during ADL. What is the BEST option to describe this occurrence? A.Dysmetria B.Dyspnea C.Dysphonia

B.Dyspnea

When a patient has limited range of motion in their upper limbs, what bathing adaptations would help create more independence? Long-handled sponge, soap on a rope, hooded towel Handheld shower, shower shoes, loofah Gel soap instead of bar soap, shower bench, bath mitt Bath mitt, soap container with a pump, magnifier to read bottles

Long-handled sponge, soap on a rope, hooded towel

A _____ assists movement of the arms when the shoulder and elbow muscles are weak and can be mounted to a wheelchair, table, or belt. It also uses gravity in an inclined plane. Mobile arm support (MAS) Overhead suspension sling Spasticity reduction splint Rood cone

Mobile arm support (MAS)

An individual who uses a wheelchair and has limited financial support is moving into a new apartment and wants to select a floor sur face for easy maneuverability. Which of the fol lowing surfaces is the MOST appropriate for this situation? a .Vinyl floor. b. Short-pile carpeting. c. Deep-pile carpeting. d. Several area rugs.

a .Vinyl floor.

An occupational therapist reviews the departments to determine potential physical and financial liabilities. What type of review does this describe? a. risk management b. peer review c. prospective review d. utilization review

a. risk management

When working with a patient with motor control dysfunction an occupational therapist can observe the patient during an assessment of basic activities of daily living and instrumental activities of daily living. The therapist must observe how problems in motor control affect motor performance. What other area(s) of patient's performance should the therapist be concerned with? a. cognition b. sensation c. medical status d. perception e. all of the above

e. all of the above

What type of sensory system is TYPICALLY associated with a child constantly touching the face of teachers and peers? A.Tactile B.Proprioceptive C. Vestibular

A.Tactile

An inpatient in the intensive care unit was medically unstable for 3 weeks secondary to multi-trauma and respiratory failure. Currently, the patient is medically stable and the OTR is completing a chart review to prepare for the initial evaluation. What is the MOST IMPORTANT action for the OTR to take as part of the chart review process? A.Understand the patient's medical information in relation to the assessment process. B.Identify key interprofessional team members who typically provide care to the patient. C.Analyze the medical documentation recorded throughout the patient's length of stay.

A.Understand the patient's medical information in relation to the assessment process.

What type of spoon has a mechanism that keeps the spoon level to prevent spills and can be used to compensate for lack of movement in the forearm? Lightweight Angled Weighted Swivel

Swivel

A therapist is observing an individual who is unable to transfer attention from one task to another. For example, when he is playing a card game, answering the phone, then returning to the card game. Which skill is deficient? a. Attentional switching b. Divided attention c. Selective attention d. Sustained attention

a. Attentional switching

At what age is a child FIRST able to stack blocks horizontally and vertically? a. 24-30 months b. 12-15 months c. 21-24 months d. 36-48 months

a. 24-30 months

According to Erhardt Prehension Developmental Levels for pellet grasp, at what age would one expect an infant to only visually attend to an object? a. 3 months b. natal c. 6 months d. 7 months

a. 3 months

In the acute stage of cardiopulmonary rehabilitation, which of the following is permissible? a. breathing exercises b. performing triceps extension exercises overhead c. isometric exercises d. holding the breath while performing light exercise reps

a. breathing exercises

An occupational therapist is marketing his services to the elderly at a community retirement complex. Which type of marketing is being used? a. concentrated marketing b. undifferentiated marketing c. unethical marketing d. differentiated marketing

a. concentrated marketing

A patient presents with an impaired ability to supinate her forearm due to peripheral nerve injury. Which other motion would likely also be impaired? a. elbow extension b. elbow flexion c. forearm pronation d. wrist flexion

a. elbow extension -radial nerve damage

In regard to physical agent modalities, which form uses convection for heat transfer? a. fluidotherapy b. ultrasound c. laser d. paraffin

a. fluidotherapy

A patient presents with weakness on one side of his body due to a CVA, impacting both upper and lower extremities. As part of interdisciplinary team intervention, the occupational therapist discusses functional ambulation with the physical therapist. Which of the following assistive devices is the MOST appropriate to provide stability for ambulation in this patient? a. hemi walker b. crutches c. standard walker d. rolling walker

a. hemi walker

Which of the following is FALSE regarding general principles of motor development? a. progresses from controlled mobility to stability b. progresses from proximal to distal direction c. displays stages of "equilibrium" and "disequilibrium" d. gross motor develops prior to fine motor

a. progresses from controlled mobility to stability

What form of grasp is displayed when a six-month-old infant holds a small toy block and uses the thumb? a. radial-palmar grasp b. radial-distal grasp c. palmar grasp d. primitive squeeze grasp

a. radial-palmar grasp

Client has been receiving OT in an outpatient center for six weeks and it's at the end of the plan of care. The client is preparing to return to work in a busy legal firm with multiple demands. Which goal area would be most appropriate for the OTR to implement in order to prepare the client for return to work? a. work hardening b. improving cognition c. work conditioning d. improving strength

a. work hardening

What major construct of Jean Piaget's process of cognitive development is described as the ability to process experiences in order to provide a response? a. Assimilation b .Operations c. Mental schemes d. Adaptation

b .Operations

During an intervention session focused on developing home management skills, a client with a diagnosis of schizophrenia made a grocery list. She grouped needed items together to make shopping easier and listed eggs separately from all of the other items. When explaining how she composed her list, the client stated, "Eggs break, they should be on top". The OT documents in the progress report that the client's approach to this task is indication of: a. diminished insight b. concreteness c. anosognosia d. poor sequencing

b. concreteness

Validity is an important element of standardized assessments to ensure you are obtaining an accurate measurement of whichever skill or ability you wish to measure. Which type of validity uses a similar previously standardized assessment to compare and determine potential validity? a. face validity b. criterion validity c. content validity d. predictive validity

b. criterion validity

Which type of qualitative research is performed when the researcher conducts observations and interviews in a skilled nursing facility to determine residents' desires for therapeutic intervention? a. heuristic research b. ethnographic research c. phenomenological research d. true experimental research

b. ethnographic research

A hospital volunteer requests transfer training from the overseeing occupational therapist, so that he may assist in performing safe transfers for patients in the hospital. What is the MOST appropriate response? a. set up a transfer training with all interested volunteers b. explain to the volunteer that this is not appropriate or allowed c. delegate this task to an OTA to maintain productivity d. teach the volunteer on the next available patient

b. explain to the volunteer that this is not appropriate or allowed - volunteers are not trained staff and about not perform hands-on transfers with clients

An occupational therapist is designing an anti-deformity splint. In which position should the IPs be placed? a. 70-90 degrees flexion b. in extension c. 30-45 degrees flexion d. 0-20 degrees flexion

b. in extension - intrinsic plus splint

When evaluating a child who is at risk for shunt malfunction, it is MOST important for the therapist to observe for: a. back pain b. irritability and nausea c. decreased tone d. increased tone

b. irritability and nausea

A client with a lower brain stem injury is referred to occupational therapy. During the evaluation, which reflex would the person most likely demonstrate? a. Tonic lumbar. b. Body-on-body righting. c. Tonic labyrinthine. d. Labyrinthine righting.

c. Tonic labyrinthine.

What form of grasp is displayed when a eight-month-old infant holds a small toy block effectively between the thumb and fingertips? a. radial-palmar grasp b. radial-distal grasp c. palmar grasp d. primitive squeeze grasp

b. radial-distal grasp

Regarding important reflexes, which early reflex is essential for searching for and locating the mother's nipple or a bottle? a. traction b. rooting c. suck-swallow d. moro

b. rooting

What would NOT be an appropriate intervention for modification of an environment to reduce fall risk? a. use bright tape on the edge of the stairs b. store frequently used kitchen items in a cabinet above the sink c. install a gab bar in the shower d. use a night-light in the hallway

b. store frequently used kitchen items in a cabinet above the sink

A 38 year old women was referred to therapy for lateral epicondylitis of her right dominant UE with a positive Cozen's test. During the acute phase of treatment, sessions should focus on all but: a. splinting b. strengthening c. AROM d. Reducing pain

b. strengthening

A patient is unable to navigate a familiar or common environment (a home) due to decreased memory and cognition. This is an example of: a. astereognsis b. topographical disorientation c. ideational apraxia d. asomatognosia

b. topographical disorientation

An occupational therapist works with adolescents with intellectual disabilities who are participating in a community integration program in an urban area. When implementing a community mobility group focused on the use of public transportation, which should the occupational therapist have the group do first? a. Read a subway or bus map. b. Take a subway or bus as a group. c. Determine a destination. d. Purchase a subway or bus fare card.

c. Determine a destination.

An occupational therapist working in a school system must incorporate the Individuals with Disabilities Education Act (IDEA) in the program. In which location should the therapist provide intervention? a. Regular classroom while general education classes are not in session. b. Special education classroom while other children with disabilities are present c. .Regular classroom while general education classes are in session. d. Private occupational therapy room designed for children with disabilities.

c. .Regular classroom while general education classes are in session.

At what age is an infant first able to turn his head and attempt to pick his head up when positioned in prone? A. 7-8 months b. 4-5 months c. 0-2 months d. 5-6 months

c. 0-2 months

Regarding the typical developmental sequencing for toileting, at what age does the child begin to express displeasure with a dirty diaper? a. 3 years b. 4 years c. 1 year d. 2 years

c. 1 year

At what age does a child FIRST begin to try multiple actions to achieve his goals, continuing until one is successful? a. 21-24 months b. 9-12 months c. 12-15 months d. 18-21 months

c. 12-15 months

A client with a diagnosis of trigger finger is seen in an outpatient hand clinic for an initial occupational therapy evaluation. The client reports having a trigger finger release surgery done 1 month prior and complains of pain and limited ROM. Which of these treatment interventions would NOT be appropriate?Select one: a. Edema control b. Tendon gliding exercises c. Strengthening exercises with repetitive gripping d. Scar massage

c. Strengthening exercises with repetitive gripping

Validity is an important element of standardized assessments to ensure you are obtaining an accurate measurement of whichever skill or ability you wish to measure. Which type of validity refers to how well the assessment encompasses and accounts for all possible responses? a. face validity b. criterion validity c. content validity d. predictive validity

c. content validity

Volition is comprised of motivating thoughts and emotions that encourage an individual to participate in occupations. What is NOT a factor of human volition? a. interests b. personal causation c. habits d. values

c. habits

In regard to physical agent modalities, which form uses radiation sourced heat transfer? a. fluidotherapy b. ultrasound c. laser d. paraffin

c. laser

A hospital setting pays for a newly hired occupational therapist's registration for an advanced continuing education course on hand-splint use and adaptation. What is the MOST important outcome for the occupational therapist? a. tax write-off for professional development b. maintaining licensure requirements for continuing education c. improved professional skills and professional competency d. networking with potential referral sources

c. improved professional skills and professional competency

An occupational therapist is working with a patient with a known seizure disorder. The patient is experiencing a tonic-clonic type of seizure. What is an appropriate intervention? a. place a towel roll in the mouth to prevent biting the tongue b. place the patient in a side-lying position immediately after the seizure begins c. protect the patient's head as much as possible d. keep the bed rails down in the hospital

c. protect the patient's head as much as possible

An occupational therapist assesses planned or prospective treatment to determine necessity and potential benefit. What type of review does this describe? a. risk management b. peer review c. prospective review d. utilization review

c. prospective review

A patient presents with wrist drop. What is the likely injury or disorder? a. proximal median nerve injury b. ulnar nerve lesion c. radial nerve lesion d. distal median nerve injury

c. radial nerve lesion

Which of the following rotator cuff muscles is innervated by the subscapular nerve? a. teres minor b. infraspinatus c. subscapularis d. supraspinatus

c. subscapularis

Which muscle of the rotator cuff functions for external rotation and is innervated by the suprascapular nerve? a. Teres minor b. Supraspinatus c. Subscapularis d. Infraspinatus

d. Infraspinatus

An adolescent referred for OT has oppositional defiant disorder. During the initial interview, the parent reports that the youth never listens to requests, become angry several times a day, blames mistakes on an older sibling. Which area is most related? a. ADLs b. Rest and Sleep c. Leisure d. Social participation

d. Social participation

Which writing posture/grasp is demonstrated by a four-year-old child who grasps the pencil between her fingers but uses whole hand movement to write letters? a. Palmar-supinate grasp b. Dynamic tripod posture c. Digital-pronate grasp d. Static tripod posture

d. Static tripod posture

While evaluating an individual with arthritis, the OT observes PIP joint hyperextension and DIP joint flexion in the digit. The OTR will MOST likely document this as a: a. Boutonniere deformity. b. Mallet finger deformity. c. Congenital deformity. d. Swan neck deformity.

d. Swan neck deformity.

Your client at a SNF is recovering from a right CVA infarct. She presents with severe left hemiplegia, resulting in poor trunk control and flaccidity of her left UE. After assessing scapular movement during PROM, you find that she has no upward rotation of her scapula or external rotation of the humerus. Based on proper handling techniques of a hemiparetic upper extremity, what should be AVOIDED when handling your client's affected UE? a. Placing tape on the shoulder and scapula for support b. having the client self-manage ROM and positing of her affected arm c. Using the gait belt, not her affected arm, to assist her in moving her body during movement d. Utilizing PROM and AAROM of the affected arm in full shoulder flexion and abduction

d. Utilizing PROM and AAROM of the affected arm in full shoulder flexion and abduction

A patient s/p CVA does not recognize her left hand as being her own and often displays neglect of the left side of her body. This is an example of: a. astereognsis b. topographical disorientation c. ideational apraxia d. asomatognosia

d. asomatognosia

An occupational therapist must provide CPR (cardiopulmonary resuscitation) to an unresponsive adult. What is the recommended compression rate? a. at least 20/min b. at least 60/min c. at least 30/min d. at least 100/min

d. at least 100/min

An individual with a head injury is frequently observed watching the person next to her instead of performing her assigned task. The OT practitioner should document this as a problem with: a. memory. b. spatial operations. c. generalization of learning. d. attention.

d. attention.

When assessing an individual who is suspected of having carpal tunnel syndrome, the OT tests for Tinel's sign by gently tapping the median nerve at the level of the a. elbow b. mid-forearm c. palmar crease d. carpal tunnel

d. carpal tunnel

Which is NOT a symptom of a lesion of the lower motor neuron system? a. lack of deep tendon reflexes b. muscle atrophy c. flaccidity d. clonus

d. clonus

In regard to physical agent modalities, which form uses conduction for heat transfer? a. fluiotherapy b. ultrasound c. laser d. paraffin

d. paraffin

An occupational therapist designs a community class that educates new mothers about infant care to promote newborn health. Which type of intervention does this describe? a. tertiary prevention b. management intervention c. secondary prevention d. primary prevention

d. primary prevention -addressing before it happens

An occupational therapist reviews the departments to determine what resources are most required for skilled intervention and how cost-effective the current supplies and tools are for patient care. What type of review does this describe? a. risk management b. peer review c. prospective review d. utilization review

d. utilization review

A patient presents with an impaired ability to pronate her forearm due to peripheral nerve injury. Which other motion would likely also be impaired? a. elbow extension b. elbow flexion c. forearm supination d. wrist flexion

d. wrist flexion -median nerve damage

What is the likely time of hypertrophic scar formation for a patient whose third-degree burn is no longer open? a. 10-12 weeks b. 2-4 weeks c. 6-8 weeks d. 1-2 weeks

c. 6-8 weeks

OT interviews a COTA for a position in a HS for gay and lesbian youth. The position involves provision of transitional vocational programming and life skills training. The COTA uses a w/c for mobility and has dysarthric speech. Which of the following is most relevant for the OT to ask the COTA about during the interview? A. Verbal group leadership skills B. Sexual orientation C. Personal beliefs about homosexuality D. Accommodations needed due to the evident disability

A. Verbal group leadership skills -it is up to COTA to request accommodations

A private practice OTR is providing an intervention, in the therapy clinic, to address the handwriting of a pediatric client with ADHD. At first, the client actively participates in the session, but begins to perseverate on the same stroke while forming letters. The client's level of arousal declines, and the client starts to drool and then vomit. What action should the OTR take FIRST in this situation? A.Activate emergency medical services by calling 911. B.Contact the parents to receive consent for medical intervention. C.Determine if the child has a history of adverse reactions to medication.

A.Activate emergency medical services by calling 911.

What is the PRIMARY reason for selecting a gel wheelchair cushion to relieve pressure on the sacrum of a client with a C7 spinal cord injury? A.To provide a stable base of support B.To permit airflow for ventilation C.To offer low daily maintenance needs

A.To provide a stable base of support -gel is stable

After screening a patient with dementia using the Allen Cognitive Level Screening tool, the OT determines that the patient is functioning at level 5. What would be the best treatment intervention? Baking a cake Flipping through a magazine Riding a bicycle Performing overall body exercises

Baking a cake

A client has lost considerable weight since his admission to a long-term care facility, and his family is concerned because he constantly complains about how sore he gets from sitting in his wheelchair now that he is "nothing but bones." Why would the OT recommend a wheelchair cushion for this client? A. It will provide a cushioned surface on which to sit. B. It will raise the seat height for better mobility. C. It will redistribute pressure on his sitting surface. D. It will prop up the client so he is sitting upright.

C. It will redistribute pressure on his sitting surface.

Which condition related to vestibular impairment is connected to short periods of vertigo due to positional changes? A Meniere's disease B Cerebrovascular disease C Migraine D Benign paroxysmal positional vertigo (BPPV)

D Benign paroxysmal positional vertigo (BPPV)

Upon completion of an early intervention screening, the OT recommends a more intensive evaluation for an 8-month old. Which reflex would the infant demonstrate to need for further evaluation? A. Neck righting on body reflex B. Body righting on body reflex C. STNR D. ATNR

D. ATNR

A child is referred to OT. The teacher reports that the child is able to assemble jigsaw puzzles, color within the lines, and play dodge ball, but demonstrates difficulty forming letters and performing simple connect the dot activities. Based upon this referral information, which deficits should the therapist screen the child? A. Form and space perception B. Laterality C. Figure-ground perception D. Visual closure

D. Visual closure

Which type of common UE fracture in the radius is characterized by the fractured segment being pushed toward the dorsal side of the wrist? a. Smith's fracture b. Scaphoid fracture c. Boxer's fracture d. Colles' fracture

d. Colles' fracture

Which ADL would a client who has a complete C6 spinal cord injury be expected to complete independently? A.Transfer from a wheelchair to the driver seat of a car. B.Use a tenodesis grasp to hold a hair brush by its handle. C.Lean forward to wash legs while seated on a bath chair.

B.Use a tenodesis grasp to hold a hair brush by its handle

An inpatient in a rehabilitation facility has a C6 tetraplegia with a rating of "A" on the ASIA impairment scale. The patient has achieved BADL goals and now wants to be as independent as possible with homemaking tasks. Which intervention approach would be effective to use as the PRIMARY strategy for promoting progress toward the patient's goal? A.Biomechanical B.Behavioral C.Compensatory D.Remedial

C.Compensatory

A patient who has hemiplegia and cognitive-perceptual deficits has been transferred from an acute care facility to a skilled nursing rehabilitation unit. When should discharge planning for this patient take place? A.When the majority of short-term goals have been met B.After determining if the patient has potential to return home C.Throughout the rehabilitation phase of treatment

C.Throughout the rehabilitation phase of treatment

Which of the following would be the MOST DIFFICULT activity for a client who has stage IV Alzheimer's disease to perform? A.Reaching to pick up a jacket B.Writing a letter to a friend C.Knowing how to drink from a cup

B.Writing a letter to a friend - stage IV is moderate cog decline

An OT practitioner is evaluating an individual who sustained a complete T8 spinal cord injury. Which level of independence in bathing, dressing, and transfers would the practitioner MOST likely expect this individual to achieve? a. Complete independence with self-care and transfers. b. Independence with self-care and minimal assistance with transfers. c. Minimal assistance with self-care and moderate assistance with transfers. d. Dependence with both self-care and transfers.

a. Complete independence with self-care and transfers.

Which type of assessment involves evaluation of a specific performance area using a standardized test such as the AMPS? a. Performance tests b. Observation c. Interview d. Self-report

a. Performance tests

A patient demonstrates weakness in elbow flexion when the forearm is positioned in pronation. Which is the primary muscle responsible for this motion? a. Brachioradialis b. Brachialis c. Biceps d. Anconeus

b. Brachialis

Which of the following reflexes does NOT persist throughout the lifetime? a. Landau b. Prone tilting c. Downward parachute d. Forward parachute

a. Landau

At what age does a child FIRST begin to develop simple plans before acting on them to gain what he wants? a. 21-24 months b. 9-12 months c. 12-15 months d. 18-21 months

d. 18-21 months

A client with schizophrenia is being discharged from a long term care facility to a group home. The client is independent with self-care and enjoys cooking, but requires assistance during meal preparation tasks due to a disregard for safety. The group home staff is willing to assist the client during meal preparation activities. What recommendations should the OTR provide the staff to maximize the client's participation in meal preparation? A.Provide step-by-step verbal cues throughout a cooking task. B.Limit the client's meal preparation activities to pre-made microwaveable meals. C.Allow the client to prepare meals using recipes familiar to the client.

A.Provide step-by-step verbal cues throughout a cooking task.

An OTR is providing consultative services at an assisted living facility. Facility administrators are concerned that some residents may be at risk for falls. Using a primary prevention model, what INITIAL recommendation should the OTR make to the management of the facility? A.Screening of all residents prior to determining needs of the facility B.Analysis of resident injuries that have occurred at the facility over the past 5 years C.Ongoing monitoring and observation of the residents to decrease liability

A.Screening of all residents prior to determining needs of the facility

An OTR is preparing to complete an initial evaluation of a new client. What ethical obligation does the OTR have for upholding professional responsibilities during the evaluation process? A.Select and administer assessment tools to obtain accurate, client-centered information. B.Develop and adhere to a consistent assessment protocol to be used with all new clients. C.Interpret results and draw conclusions based on one evidence-based assessment method.

A.Select and administer assessment tools to obtain accurate, client-centered information.

An inpatient is recovering from partial and full thickness burns on the dominant upper extremity and has recently developed heterotopic ossification (HO) at the elbow. Prior to the onset of HO symptoms, the patient was independent with self-feeding. Now the patient uses only the non-dominant hand for holding utensils. Which assistive device would be MOST BENEFICIAL for improving the patient's functional abilities when eating? A.Swivel spoon and elongated utensils B.Universal cuff with elongated utensil C.Mechanical feeder with supinator attachment D.Rocker knife with a built-up handle

A.Swivel spoon and elongated utensils -HO effects flexion, extension, and supination of AROM in elbow

An OTR is evaluating an adult client with autism spectrum disorder who wants to work full-time in a competitive work environment. What is the PRIMARY reason for administering a combination of non-standardized and standardized assessments as part of the initial evaluation? A.To provide a dynamic picture of the client's performance level in clinical and natural settings B.To gather holistic information about the client's vocational interests and career aspirations C.To compare the client's work behaviors and personal attributes to a normative sample

A.To provide a dynamic picture of the client's performance level in clinical and natural settings

Which dressing task requires the MOST challenging integration of performance skills and patterns for a typically developing 3-year-old child? A.Taking off a pair of ankle-high socks B.Unfastening the zipper of a front-opening jacket C.Finding armholes in a pull-over shirt D.Pulling down a pair of elastic waist pants

B.Unfastening the zipper of a front-opening jacket

An OTR is providing instruction in lower extremity dressing techniques to a patient who has had a total hip arthroplasty and is following posterior lateral hip precautions. What should the OTR include in the instructions? A.Sit on a low stool to put on shoes and socks. B.Use a reacher to put on a pair of pants. C.Position the shoulders over the knees when standing up.

B.Use a reacher to put on a pair of pants. -dont bend past 90

The OT is thinking ahead to a child's reha bilitation following a traumatic brain injury. His current program, while he is in acute care, is focused on positioning and movement to maintain joint movement, sensory stimulation, and family education. Which is the OT MOST likely to consider to help identify when the child's awareness increases to a point at which his occupational therapy program can shift its focus? A. The child's muscle tone increases. B. Child's visual disturbances have diminished. C. Parents have prioritized ADL independence as a goal. D. Scores on the Ranchos Los Amigos Cognitive Recovery Scale.

D. Scores on the Ranchos Los Amigos Cognitive Recovery Scale.

A woman recovering from a traumatic brain injury is functioning at Rancho Level 7. The OT plans an activity with the woman to help her plan for her future after discharge from the rehabilitation unit. What would the OT expect to hear from the woman during the activity? Responses that show a lack of orientation to person and place Responses that show total lack of awareness of safety Responses that show an overestimation of abilities and unrealistic expectations Responses that show a realistic awareness of abilities but difficulty adjusting prior activities to new abilities

Responses that show an overestimation of abilities and unrealistic expectations -7 may display superficial awareness of their conditions/overestimate abilities

A patient presents with an incomplete SCI with motor function preserved below neurological level and the majority of key muscle groups below the neurological level have a muscle grade of less than 3/5. What type/grade of spinal cord injury is described, based on the ASIA Impairment Scale? a. C b. B c. D d. A

a. C

During assessment, an OT practitioner observes an individual who is able to place his dentures in his mouth, but has difficulty applying denture cream to the appropriate place on the dentures and attempts to place the cap on the tube backwards or on the wrong end of the tube. The OT is MOST LIKELY to interpret this as: a. Constructional apraxia b. Ideomotor apraxia c. Dressing apraxia d. Unilateral neglect

a. Constructional apraxia -related to 3D spatial relationship - ideomotor would be more like the entire task

An OT practitioner is observing an individual who has had a stroke and notes that the person seems unable to correctly position slices of cheese and ham onto bread or to then place another piece of bread on top to assemble a sandwich. This behavior would MOST likely be reported as: a. Constructional apraxia. b. Ideomotor apraxia. c. Visual agnosia. d. Unilateral neglect.

a. Constructional apraxia.

When evaluating an individual with dysphagia for MOTOR problems associated with swallowing, the OT practitioner should look for: a. Coughing or choking. b. Disorientation or confusion. c. Pain while swallowing. d. Decreased smell and taste.

a. Coughing or choking.

Proprioceptive neuromuscular facilitation (PNF) is used therapeutically for many individuals with neurological conditions; it focuses on using movement patterns to improve motor skills for functional tasks. Which upper extremity pattern is used when the therapist is trying to promote improved function of wiping during toileting? a. D2 extension b. D2 flexion c. D1 extension d. D1 flexion

a. D2 extension

An individual is recovering from deep partial-thickness burns on the upper extremities, chest, and lower neck. The occupational therapist provides equipment to prevent positions that can result in contractures. Which are the most important positions for the therapist to prevent? a. Positions of comfort. b. Antideformity positions. c. Positions resulting in edema. d. Positions of pain.

a. Positions of comfort.

A patient s/p CVA presents with impaired sensation in the effective limb as well as homonymous hemianopsia and Alexia? a. Posterior cerebral artery b. Anterior cerebral artery c. Right middle cerebral artery d. Left middle cerebral artery

a. Posterior cerebral artery

Which type of intervention seeks to prevent negative health occurrences and promote overall well-being? a. Prevention b. Management c. Meeting health needs d. The change process

a. Prevention

An adult is referred to occupational therapy for ADL training. The patient incurred a fracture of the right proximal humerus and is using a shoulder immobilizer for the first two weeks to aid healing and help control pain. The patient is right-hand dominant. Which activity will be the most difficult for the patient? a. Putting on a pullover top. b .Taking off a heavy coat. c. Brushing their teeth.

a. Putting on a pullover top.

An OT practitioner is consulting in a long term care setting to recommend programs for residents who seem isolated and disengaged from the other residents. Which type of group would be BEST for enhancing self-esteem, pro viding opportunities for socialization, and as sisting residents in integrating past experiences with present life? a. Reminiscence. b. Meditation. c. Grooming. d. Movement.

a. Reminiscence.

Which hand and wrist disorder is characterized by an injury to the ulnar collateral ligament in the thumb? a. Skier's Thumb/Gamekeeper's Thumb b. Dupuytren's disease c. Colles' fracture d. De Quervain's tenosynovitis

a. Skier's Thumb/Gamekeeper's Thumb

Which tool would be the LEAST helpful in promoting increased sleep for an individual who consistently wakes up during the night? a. Television in the bedroom b. Soft music playing c. Sound machine at night d. Blackout curtains

a. Television in the bedroom

Which of the following hemoglobin levels indicate a safe reading to get a patient up out of bed, in an acute setting? a. 8.0 L b. 12.0 L c. 3.0 L d. 5.0 L

b. 12.0 L

An OT working at an outpatient clinic is in the process of evaluating a new client. This client presents with restricted passive range of motion in their shoulder. Their greatest limitation is in external rotation, then abduction, internal rotation, and flexion. Which condition is this client likely to have?Select one: a. Rotator Cuff Tendonitis b. Adhesive Capsulitis (frozen shoulder) c. Peripheral Nerve Injury d. Shoulder Dislocation

b. Adhesive Capsulitis (frozen shoulder)

A patient has a full-thickness ulcer that is not improving. Which pressure reducing device should the occupational therapist recommend? a. Gel cushion b. Air-fluidized bed c. Foam mattress d. Low-air-loss mattress

b. Air-fluidized bed -electric device that provides the highest level of pressure management

Which of the following muscles does NOT contribute to elbow flexion motion? a. Brachioradialis b. Anconeus c. Brachialis d. Biceps

b. Anconeus

A patient s/p CVA presents with hemiplegia, and potentially incontinence confusion and inability to speak. What is the likely site of infarct? a. Posterior cerebral artery b. Anterior cerebral artery c. Right middle cerebral artery d. Left middle cerebral artery

b. Anterior cerebral artery

When evaluating an individual with coronary artery disease for controllable risk factors, the OT practitioner should: a. Determine the individual's age and gender. b. Assess the individual's lifestyle and dietary habits. c. Observe the individual for obesity and cholesterol levels. d. Determine if the individual has a family history of heart disease.

b. Assess the individual's lifestyle and dietary habits.

A patient presents with an incomplete SCI with no motor function but sensory function is preserved below the neurological level and extends through the sacral segments. What type/grade of spinal cord injury is described, based on the ASIA Impairment Scale? a. C b. B c. D d. A

b. B

When working with a patient 1 day post a total shoulder arthroplasty it is appropriate to do: a. resistive strengthening exercises b. Codman's pendulum exercises c. AROM exercises hitting full ROM in all planes

b. Codman's pendulum exercises

A patient presents with weakness of shoulder internal rotation, and the evaluating therapist suspects weakness in the rotator cuff. Which of the following rotator cuff muscles is responsible for internal rotation of the shoulder? a. Supraspinatus b. Subscapularis c. Teres minor d. Infraspinatus

b. Subscapularis

In regard to key terms for payment of occupational therapy services, which describes insurance companies? a. Providers b. Third-party payers c. Vendors d. Prospective payment systems

b. Third-party payers

In regard to physical agent modalities, which form uses deep level heat transfer through conversion? a. fluidotherapy b. ultrasound c. laser d. paraffin

b. ultrasound

At what age is a child FIRST able to correctly match shapes and drop objects into a slot? a. 24-30 months b. 12-15 months c. 21-24 months d. 36-48 months

c. 21-24 months

A patient is unable to understand what a toothbrush is used for s/p CVA. This is an example of: a. astereognsis b. topographical disorientation c. ideational apraxia d. asomatognosia

c. ideational apraxia

When working with a child in an early childhood setting, the therapist must consider the role of family education. Which is an INCORRECT statement in regard to family education? a. identify elements in the home environment that may encourage development b. train families to advocate for themselves in schools c. teach the same strategies regardless of a family's cultural background d. educate on methods to discourage behavior that is a deterrent to participating effectively

c. teach the same strategies regardless of a family's cultural background

What is an appropriate intervention for an immobilized distal radius due to fracture? a. Strengthening of forearm musculature b. TENS use c. Contrast baths d. AROM of the fingers and elbow

d. AROM of the fingers and elbow

In regard to early reflexes, which of the following is a reflex that does not integrate but persists throughout life? a. Asymmetric tonic neck b. Tonic labyrinthine-supine c. Landau d. Labyrinthine/optical righting

d. Labyrinthine/optical righting

Which of the following toys is NOT appropriate to have in a pediatric occupational therapy setting? a. Rubber bouncing ball b. Plastic interlocking rings c. Climbing play equipment d. Stuffed teddy bear

d. Stuffed teddy bear - can't clean it man

Regarding qualitative research, which criterion is defined as the study's ability to correlate with related studies? a. Dependability b. Confirmability c. Credibility d. Transferability

d. Transferability

What is NOT a sign/symptom of schizophrenia? a. delusions b. hallucinations c. significant weight loss d. disorganized behavior

c. significant weight loss

In regard to fiscal management of an occupational therapy practice or department, which term refers to income still awaiting payment? A Accounts receivable B Accounts payable C Operating expense budget D Break-even analysis

A Accounts receivable

An OTR has completed an initial evaluation of an 18-month-old child who has hypotonicity. The child's mother reports the child does not crawl and resists being placed in a hands and knees crawling position. During the evaluation the child is able to maintain head control, roll, and sit unsupported for several minutes. After the OTR places the child in a crawling position, the child is able to maintain the position for about 30 seconds. The child does not spontaneously initiate locomotion. What statement accurately reflects the child's current motor skills? A. The child exhibits tactile hypersensitivity that impacts the child's ability to maintain postural stability .B. The child has decreased postural stability for completing developmentally appropriate gross motor tasks. C. The child has the ability to initiate transitional movement patterns and achieve developmentally-appropriate locomotion. D. The child has the potential to develop skills for engagement in age-appropriate childhood occupations.

.B. The child has decreased postural stability for completing developmentally appropriate gross motor tasks.

During evaluation, an individual who had a severe myocardial infarction 2 weeks ago displays good memory of information processed before the MI, poor recall of the first week after the MI, but good recall of information from the past week. The OT practitioner would MOST likely document this as: A. Disorientation. B. Long-term memory deficits .C. Anterograde amnesia. D. Retrograde amnesia.

.C. Anterograde amnesia. antero: memory loss after episode retro: memory loss before episode

Put in order the play development progression 1. symbolic 2. exploratory 3. creative 4. game-based

1. exploratory (0-2) 2. symbolic (2-4) 3. Creative (4-7) 4. game-based (7-11)

An occupational therapist is assisting a community in designing a ramp that includes a sharp turn of 180 degrees. What is the MINIMUM required landing size at the turn in order to allow for proper maneuverability in a wheelchair? A 4' x 8' landing B 2' x 2' landing C 4' x 4' landing D 4' x 6' landing

A 4' x 8' landing

The Glasgow Coma Scale assesses consciousness following a traumatic brain injury. Which of the following is NOT a component of assessment? A Auditory response B Motor response C Verbal response D Eye response

A Auditory response

A patient has weakness with elbow flexion when the forearm is supinated. Which is the primary muscle used to perform this motion? A Biceps B Brachialis C Anconeus D Brachioradialis

A Biceps

A patient who has recently been diagnosed with Parkinson's disease is experiencing difficulty using standard cutlery. Her muscle weakness, fatigue and bradykinesia are impacting on her ability to eat in an efficient manner. What type of adaptation is the BEST to introduce to this patient during this stage of her disease? A swivel spoon A built-up handle spoon A weighted spoon A coated spoon

A built-up handle spoon -will help with grip and lightweight may facilitate faster movements

For a 13-year-old patient who has a progressive condition which is causing him to lose his vision, what is the BEST adaptation that can be recommended to help him continue to use his computer despite his poor vision? A recommendation for a modified, smaller keyboard A recommendation for a voice recognition input program A recommendation for a screen reading program A recommendation for single-switch scanning with a keyboard on the screen

A recommendation for a screen reading program

Carol, a 58-year-old housewife who recently sustained a mild TBI, has been admitted to a rehabilitation facility. She is currently working with a OTR® and the focus of the session is on identifying the components of ADL tasks using card sorting exercises. Once Carol is able to achieve this, the OTR® asks her to separate clothing into 2 piles of light colors and dark colors, before washing the clothing. What approach is the OTR® using in this scenario? Using a remedial approach to improve color differentiation An adaptive approach to compensate for Broca's aphasia A compensatory approach to minimize inattention A transfer-of-training approach working on the skill of categorization

A transfer-of-training approach working on the skill of categorization

An older adult client has lost significant vision and reports difficulty with reading, sewing, writing, recognizing faces, and responding to social gestures. Which visual deficit is the client exhibiting? A. Age-related macular degeneration B. Glaucoma C. Myopia D. Presbyopia

A. Age-related macular degeneration

A client has been referred to occupational therapy for a wheeled mobility assessment. The client has a standard wheelchair (K0001), which he can propel, but wants the OT to recommend a power wheelchair so that he can visit around the neighborhood with greater ease. His wife states that she is able to push him in the neighborhood if he needs help, but he tells the OT that he needs to get out of the house ALONE. Why is the client ineligible for a Certificate of Medical Necessary for a power wheelchair? A. He is able to propel a standard wheelchair. B. He prefers, but does not need, a power wheelchair. C. He does not want caregiver assistance, even though it is available. D. He wants a power wheelchair to visit in the neighborhood

A. He is able to propel a standard wheelchair.

An OT is performing a physical examination to determine the client's seating and positioning needs before ordering a new wheelchair. What rule of thumb would the OT use for the client's ability to sit upright? A. Minimal hip flexion of 90° B. Minimal hip flexion of 80° C. Minimal hip flexion of 70° D. Minimal hip flexion of 60°

A. Minimal hip flexion of 90°

A client is being discharged home after spending 2 weeks in rehab after a stroke. She has arrived for transfer safety training. For a wheelchair-to-bed standing pivot transfer, which of the following steps would be FIRST in the sequence of the transfer? A. Positioning the chair at a 45° angle to the bed B. Telling the client to scoot forward in the wheelchair. C. Swinging the footrests out of the way D. Setting the brakes on the wheelchair

A. Positioning the chair at a 45° angle to the bed

When measuring a client for a wheelchair, what are the four basic measurements the OT MUST take? A. Seat width, seat depth, seat height, seat-back height B. Seat width, seat depth, seat height, footrest length C. Seat width, seat depth, seat-back height, footrest length D. Seat width, seat depth, footrest length, seat-to-back angle

A. Seat width, seat depth, seat height, seat-back height -seat height is how high off ground -seat back hight is back of chair -footrests can be adjusted later don't worry about them

A young adult with schizophrenia is scheduled for discharge from an inpatient setting to a halfway house and psychosocial clubhouse. The OT is assisting the team w/ a discharge plan. What is the most important info for the OT to provide the team w/ about this person? A. The person's IADL skills B. The person's vocational potential C. the person's social interaction skills

A. determines how much support will be needed at house and during transition to clubhouse

An OT who is employed by a senior center has been asked to develop groups to address the motor needs of people with Parkinson's disease. Which of the following activities would be MOST appropriate to include? A. A game of rhythmic exercises performed to music. B. Creating time capsules for their grandchildren. C. Wheelchair races performed in pairs. D. Taking turns reading out loud from the newspaper

A. A game of rhythmic exercises performed to music.

An occupational therapy student is working in a rehabilitation setting that treats many clients with osteopenia. The fieldwork educator requests that the student determine whether a walking program can prevent osteoporosis. What kind of research article would provide the MOST trustworthy evidence to support a walking intervention? A. A systematic review B. A randomized clinical trial C. A cohort study D. A case-control study

A. A systematic review

The parents of an 18-month-old bring their child to a free community developmental screening. The child can attend to shapes and use them appropriately. However, the parents are worried because the child cannot match shapes or manipulate different shaped objects into a shape sorter. Which is the best response for the occupational therapist to make in response to the parents' expressed concern? A. Advise the parents that the child is showing a typical, age-appropriate skill. B. Complete an occupational therapy evaluation of the child's cognitive skills. C. Refer the child to the early intervention program for developmental delay. D. Provide the parents with activity recommendations to develop shape recognition.

A. Advise the parents that the child is showing a typical, age-appropriate skill.

An 18-year-old client is admitted to an inpatient psychiatric hospital. The psychiatrist describes an uninterrupted 18-month period of illness that includes visual and auditory hallucinations. What type of issues would the OTR® expect to see when the client attempts to do a task? A. Aggressive outbursts of behaviors when frustrated with a project B. Overassertive, controlling behaviors during group activities C. Difficulty engaging in problem solving D. Panic over task choices

A. Aggressive outbursts of behaviors when frustrated with a project -others are more related to bipolar disorder (depressive/manic)

A carpenter recovering from injuries after a fall from ladder has decreased strength in triceps, bilaterally. Most recent MMT indicated triceps muscle strength was 3/5. The OT provides the individual with tabletop wood project to complete. to develop triceps muscle strength, how should the OT position the tabletop when they sand the project? A. At a 45 degree incline angled so individual's hands are above elbows when elbows are flexed. B. At individual's waist height so their hands and elbows are on the same plane when elbows are flexed C. At individual's chest height so their hands and elbows are on the same plane when elbows are flexed D. At a 45 degree incline angled so individual's hands are below elbows when elbows are flexed.

A. At a 45 degree incline angled so individual's hands are above elbows when elbows are flexed. -forces movement against gravity -D. is gravity-assisted

An OTR® is observing a client's driving performance in a contained environment; the client is performing tasks and responding to demands involving car controls and car handling or maneuvers. Which testing approach is the OTR® using? A. Closed route B. Simulator testing C. Instrumented vehicle D. Naturalistic driving

A. Closed route -A closed route is a contained environment that allows a client to perform tasks and respond to car handling or maneuvers. -instrumented vehicle has cameras and sensors to objectively measure the person's vehicle input and output responses. -simulator dosnt involve actual car

An OTR is facilitating a group for clients diagnosed with a polysubstance use disorder. During the first group session, the OTR describes how trigger events and irrational thinking can lead to substance misuse. Which theoretical model does this approach exemplify? A. Cognitive-behavioral B. Psychodynamic C. Human Occupation

A. Cognitive-behavioral

An OT conducts an initial home visit to a family with a premature infant who, at 4 mos. and 5 lbs., has just been discharged from the hospital. The child has multiple DDs. Which is the most important for the OT to do during the first session w/ the family? A. Communicate effectively to develop a therapeutic relationship w/ the family. B. Teach the family proper body mechanics for lifting the child C. Teach the family assertiveness training to develop advocacy skills. D. Determine whether adaptive aids or positioning equipment is needed.

A. Communicate effectively to develop a therapeutic relationship w/ the family.

An occupational therapist completes an early intervention screening of an 8-month-old child. The results indicate that the child is able to sit independently by propping forward on both arms. Which is the best action for the occupational therapist to complete next? A. Evaluate the child's sensorimotor skills using a standardized evaluation. B. Inform the parents that the child exhibits typical behavior. C. Develop goals to improve sitting balance. D. Provide play activities to develop sitting balance.

A. Evaluate the child's sensorimotor skills using a standardized evaluation. -after screening is eval

A 4-year-old child with spina bifida has a lesion at the lumbar level resulting in a flaccid bladder. The parents are requesting a bladder training program. What is the OT's MOST ap propriate response? A. Explain to the parents that toilet training is not a feasible option. B. Recommend waiting until the child is 5 years old. C. Develop a toilet-training program together with the parents. D. Assess the child's ability to remove lower extremity garments.

A. Explain to the parents that toilet training is not a feasible option.

Elementary school aged child referred to OT by pediatrician. They have ruled out ADHD and note the child demos poor school performance and unexplained clumsiness. The OT screens the child and determines the child is a candidate for a SP eval. Which skills are most likely observed to be intact during the screening and need no further eval? A. FM coordination B. Sequencing motor tasks C. Initiation of activities D. Ocular pursuits

A. FM coordination -usually fine with sensory processing disorder -all others are related to SP

A child with hemophilia attends a middle school on a large campus. The child develops hemarthroses in both wrists. Two weeks after draining of the hemarthroses, the child returns to school. To safely manage the books they need for each class, which is the best for the therapist to recommend? A. Have subject books in each classroom an extra set of all class texts at home B. Use a light-weight wheeled suitcase to transport books between classes C. Use a light-weight backpack with adjustable straps to carry books D. Frequently shift books from one arm to the other arm when carrying them

A. Have subject books in each classroom an extra set of all class texts at home

An OTR is developing a community-based program for clients who live independently in the community and have diabetes. One of the goals of the program is for the clients to identify lifestyle changes to improve self-management of the disease. Which type of assessment tool represents a wellness model that can be used as part of the program evaluation to determine clients' progress toward this goal? A. Health risk appraisal B. Quality of life measure C. Contextual checklist

A. Health risk appraisal

An individual with a history of anger self control issues has met the goals of indepen dently identifying anger-provoking stressors in his home and work life and identifying his typ ical behaviors in anger-provoking situations. What action should the OT take NEXT? A. Initiate training in conflict resolution strategies. B. Utilize creative media to explore situations that trigger anger. C. Implement use of journaling to identify stres sors and responses. D. Recommend discontinuation of occupational therapy services.

A. Initiate training in conflict resolution strategies.

A patient with a fractured radial head has the arm immobilized from above the elbow to below the wrist. The patient has requested ex ercises that will help maintain strength in the elbow and forearm muscles while awaiting permission to move the arm. In response, what types of exercises should the OT provide in structions to perform? A. Isometric. B. Isotonic. C. Progressive resistive. D. Passive.

A. Isometric.

A client with Down syndrome is preparing for transition from high school to work and is seeking a work placement. The client needs to identify vocational interests and abilities for general job tasks. To which service should the OTR® refer the client? A. Job coaching B. Functional capacity evaluation C. Job site evaluation D. Prework screening

A. Job coaching

While developing an occupational therapy program for an adolescent with significant learning disabilities, the OT MOST likely includes which strategies to promote the developmental changes associated with adolescent development? A. Opportunities to explore new areas of inter est, helping to identify strengths and needs. B. Encouragement to join clubs in high school to meet new friends. C. Recommendation that the family pursue OT outside of school to prevent negative peer pressure. D. Intervention in private area of the school to protect privacy.

A. Opportunities to explore new areas of inter est, helping to identify strengths and needs.

An individual has sustained a large, full thickness burn to both upper extremities and is in the acute phase of treatment. Which of the following BEST represents a short-term rehabilitation goal in the acute stage of recovery? A. Maintain and/or prevent loss of joint and skin mobility. B. Issue adaptive equipment. C. Provide compression and vascular support garments. D. Prevent scar hypertrophy through scar man agement techniques.

A. Maintain and/or prevent loss of joint and skin mobility.

An OTR® is planning an occupational therapy intervention focused on leisure exploration for a client with oppositional defiant disorder who likes graffiti, body piercings, and tattoos. Which activity would be MOST appropriate? A. Making posters with colored markers on poster paper B. Leather tooling a wallet with premade designs C. Knitting a blanket following an assigned pattern D. Freestyle painting a design on the wall in a public area

A. Making posters with colored markers on poster paper

After several months of working with an OT, a client with severe and persistent mental illness has obtained a job cleaning empty offices for 4 hours a day. While taking public transpor tation to the job site, the client experiences rid icule from school-age children taking the same bus to school in the morning. Using an environ mental adaptation approach, what is the BEST action for the OT to take so that this client can keep this job? A. Negotiate work hours that would allow the client to travel at times that children are less likely to be on the same bus. B. Have the client take a taxi rather than a city bus. C. Work with the client on coping skills so that the schoolchildren's taunting is more manageable. D. Volunteer to offer a mental health awareness day for the local school district.

A. Negotiate work hours that would allow the client to travel at times that children are less likely to be on the same bus. -B. works but it is more expensive

A child who has had a traumatic brain in jury (TBI) demonstrates tongue thrust. What should the OT do FIRST, prior to feeding? A. Position the child's trunk, head, neck, and shoulders in proper alignment. B. Recline the child's seat backward and position neck in slight extension. C. Place the OT's digits directly under the child's chin, facilitating tongue retraction. D. Provide upward pressure under the child's lower jaw prior to chewing.

A. Position the child's trunk, head, neck, and shoulders in proper alignment.

What clinical symptom is typically present in the affected hand of a client who has severe carpal tunnel syndrome? A. Positive Tinel's sign at the wrist B. Decreased light touch of all digits C. Numbness of the small finger D. Atrophy of the extrinsic muscles

A. Positive Tinel's sign at the wrist

The Occupational Therapy Code of Ethics (2015) states, "Occupational therapy personnel shall treat clients, colleagues, and other professionals with respect, fairness, discretion, and integrity." What principle does this statement directly relate to? A. Principle 6, Fidelity B. Principle 1, Beneficence C. Principle 2, Nonmaleficence D. Principle 4, Justice

A. Principle 6, Fidelity

For a client with C5 spinal cord injury (SCI) in the acute phase of rehabilitation, what is the BEST position to place the client's forearms in? A. Pronation B. Supination C. Flexion D. Extension

A. Pronation -The forearm should be pronated because clients with C5 SCI are prone to contractures in their supinators because of their inability to pronate actively. -D. is correct but doesn't prevent contractures

An individual with borderline personality disorder is admitted to the hospital following a suicide attempt. After attending OT orientation group, the pt tells the OT, "You are the only therapist who has ever been really helpful." The patient asks to meet with the therapist privately on a regular basis instead of the assigned primary individual therapist. Which action is best for the OT to take in response to the patient's request? A. Refer the pt to the assigned individual therapist B. Agree to meet with the patient since a positive therapeutic connection has been expressed C. Tell the patient that an OT provides only occupation-based group treatment D. Explain that this type of manipulative behavior is not acceptable

A. Refer the pt to the assigned individual therapist

An adolescent client who has muscular dystrophy uses a manual wheelchair that has a backrest cushion and a gel seat insert. The client reports a recent onset of shoulder pain while propelling the wheelchair over a carpeted surface. The OTR® observes that the wheelchair has a backrest cushion that positions the client's pelvis beyond the rear wheel axle. Which wheelchair modification should the OTR® recommend to reduce the client's shoulder pain? A. Remove the backrest cushion. B. Change to an air-filled seat cushion. C. Raise the seat height. D. Adjust the axle plate.

A. Remove the backrest cushion.

During an intervention session in a school, the occupational therapist observes a young child turn the pages of a book. The therapist identifies this behavior as an example of an in-hand manipulation task. Which task should the occupational therapist document the child is capable of performing? A. Shift. B. Simple rotation. C. Translation. D. Translation without stabilization.

A. Shift. -simple rotation is moving objects between thumb and fingertips -shifting is moving fingers linearly (i.e. fanning cards during poker)

During a kitchen activity, an individual with decreased shoulder ROM demonstrates difficulty retrieving items from the higher shelves. Which of the following recommenda tions will BEST facilitate home management for this individual? A. Store the most frequently used items on shelves just above or below the counter. B. Use the largest joint available to move or lift items. C. Perform shoulder range-of-motion exercises 10 times each, twice a day. D. Continue reaching for items on high shelves because it will help improve range of motion

A. Store the most frequently used items on shelves just above or below the counter.

When planning services for a preschool child using the Model of Human Occupation to guide the intervention, what is the OT is MOST likely interested in learning about the child? A. Teacher's report of how the child spends classroom free play time and choices made during outdoor recess. B. How the child's posture and seating positions impact visual-motor control and use of hands during learning activities. C. Instructions, assistance, and cues child is given in preparation for transition between activi ties, and response following attempts to change activities. D. Child's preferences among activities that re quire movement, such as playground activity, "move and sing" activities, and sand and wa ter play.

A. Teacher's report of how the child spends classroom free play time and choices made during outdoor recess.

An OTR® is working on the playground with a child who has sensory integration dysfunction. The OTR® is trying to foster an adaptive response, which is best described by which statement? A. The behavioral manifestation of optimal sensory organization that results in an efficient goal-directed action B. Engagement in rough-and-tumble play and other activities that provide the child with muscle resistance C. A reflection on primitive neural functions in children with sensory processing problems D. An individualized plan that provides a specific child with optimal sensory experiences

A. The behavioral manifestation of optimal sensory organization that results in an efficient goal-directed action

An OTR® receives evaluation orders for a client who has recently experienced traumatic brain injury. The client's chart lists a Glasgow Coma Scale (GCS) score of 7. What does this score imply? A. The client has experienced a severe head injury and will provide minimal feedback. B. The client has experienced a moderate head injury and may not be able to provide discernible feedback. C. The client has experienced a mild head injury and may be confused but able to follow simple commands. D. The client has experienced an extreme head injury and will provide minimal feedback.

A. The client has experienced a severe head injury and will provide minimal feedback. - severe is 8 or less (no such thing as extreme)

Supervising OTs meet ot plan the workload of the recently hired COTAs. Which task is best for them to assign to the COTAs? A. The in-home evaluation of the adult clients' IADLs B. The determination of LTGs to include in the adult clients' OT home care intervention plan C. The design of home-based sensory integration protocols for infants/toddlers w/ SPDs D. The administration and interp. of the Hawaii Early Learning Profile (HELP) to infants/toddlers

A. The in-home evaluation of the adult clients' IADLs -they can complete IADL evals with supervision

An occupational therapist observes that an 18-month-old child is not able to creep more than a few steps. When the child looks up, both hips and knees flex and the child ends up W sitting with both arms extended and propped forward. When documenting this observation, which is most accurate for the occupational therapist to report the child is demonstrating? A. The influence of the symmetrical tonic neck reflex (STNR) resulting in delayed gross motor skills. B. An obligatory asymmetrical tonic neck reflex (ATNR) resulting in delayed gross motor skills. C. An intact tonic labyrinthine reflex which facilitates balance responses.

A. The influence of the symmetrical tonic neck reflex (STNR) resulting in delayed gross motor skills.

An individual with bilateral knee replacements and cataracts is newly admitted to a skilled nursing facility. The individual retains some residual vision. During the initial evaluation, which is the most effective placement for the occupational therapist to use when presenting materials to the person? A. To the side of the person, with no direct lighting. B. Directly in front of the person, at eye level. C. Directly in front of the person, at tabletop level. D. To the side of the person, with a strong light shining.

A. To the side of the person, with no direct lighting. -peripheral with no glare

The spouse of a client with Alzheimer's dementia tells the OTR® that the client has become more agitated, wanders, resists care, and displays inappropriate and sometimes destructive behavior. The spouse confides that feelings of stress have begun to feel overwhelming. Which occupational therapy intervention is most appropriate? A. Training the spouse to provide simple, one-step directions and avoid abstractions B. Role playing with the spouse and the client ways of providing explanations that will clarify the client's confusion. C. Assertiveness training with the spouse D. Conflict resolution sessions between the spouse and client

A. Training the spouse to provide simple, one-step directions and avoid abstractions

An OTR®; is treating a client who is in a vegetative state after a traumatic brain injury (TBI). The OTR has identified the need to use restorative strategies with the client. Which restorative strategy is MOST appropriate for this client? A. Upright positioning strategies in a wheelchair to normalize muscle tone and facilitate arousal B. A sensory stimulation program to facilitate return to consciousness C. A self-feeding program that simplifies the task and provides success for the client D. A behavioral reinforcement program to reinforce the client's on-task performance

A. Upright positioning strategies in a wheelchair to normalize muscle tone and facilitate arousal -minimize abnormal tone, visual and vestibular input -B. is good but not the MOST important

An occupational therapist reviews the use of the OT department's resources to determine medical necessity and cost efficiency. Which service management task is the therapist performing? A. Utilization review B. Retrospective peer review C. Total quality management D. Risk Management

A. Utilization review

An OT is working in an outpatient pediatric clinic with a child who has juvenile rheumatoid arthritis. The parents state that the child experiences pain in the wrists during BADL. Which is the pest splint to recommend for this child? A. Volar wrist cock-up splints B. Dynamic wrist extension splints C. Dorsal wrist cock-up splints D. Resting hand splints

A. Volar wrist cock-up splints -support wrists -D. is overall inflammation

OT provides services to homeless shelter including residents who are HIV+. OT conducts several groups. What should the OT do while working with this population? A. Wash hands before and after each group session B. Always wear latex gloves during groups C. Wear latex gloves when handling foods D. Implement transmission-based precautions

A. Wash hands before and after each group session -trick question its a standard precaution for everyone

An OTR® completes a work tolerance screening with a worker at a manufacturing plant. What should be included in the documentation? A. Weight limits for various job tasks B. Worker's reported desire to remain in the job C. Worker's ability to perform ADLs D. Financial issues the worker experiences

A. Weight limits for various job tasks

Parents of a 2 y/o with unilateral congenital UE ampuations express concern to the home-care OT about their child's complete disinterest in toilet training. At which point should the OT advise the parents to begin toilet training? A. When the child indicates discomfort w/ being wet/soiled B. Immediately, b/c toileting is a developmentally appropriate task C. When the child is 3 y/o, as this is the typical developmental age for toilet training

A. When the child indicates discomfort w/ being wet/soiled

The parents of a two-year-old child with unilateral congenital upper extremity amputations express concern to the home-care occupational therapist about their child's complete disinterest in toilet training. At which point should the occupational therapist advise the parents to begin toilet training? A. When the child indicates discomfort with being wet or soiled. B. Immediately, because toileting is a developmentally appropriate task. C. When the child is 3 years old, as this is the typical developmental age for toilet training.

A. When the child indicates discomfort with being wet or soiled. -initiate it when child is ready

According to the AOTA Ethics Commission's advisory opinion on balancing patient rights and practitioner values, when is it ethically acceptable for an occupational therapy practitioner to refrain from providing services? A. When the environment poses a clear threat to the practitioner's safety B. When a client uses repugnant, harsh, or inappropriate language C. When the moral values of the client and practitioner are clearly in conflict D. When the practitioner feels an overwhelming aversion to the client

A. When the environment poses a clear threat to the practitioner's safety

According the ethical principle of Justice, an OTR® is obliged to be aware of certain laws such as the Americans With Disabilities Act (ADA). An OTR who is also a rehab manager in a long-term care facility has an incoming fieldwork student who is requesting an accommodation for a disability. How should the OTR respond to the request? A. Work with administration to determine whether the site can reasonably accommodate the request. B. Deny the request, because the health care environment already accommodates people with disabilities. C. Work with the student and the student's occupational therapy program to determine specific needs. D. Refer the question to the facility's human resources department.

A. Work with administration to determine whether the site can reasonably accommodate the request. -collaborate with management (not D.)

An OT provided home-based services to a person recovering from a recent CVA. The individual lives alone and receives home care Medicare Part A benefits. The OT arrives at the clients house at the scheduled session time, but there is no response. A neighbors reports seeing the client leave w/ a friend. Which is the best action for the OT to take? A. document that no on answered the door and the appointment will be rescheduled. B. call the nurse case manager to report the missed appointment and need to reschedule C. document that no on was home and the appointment will be rescheduled D. document that the client is engaged in community mobility activities and should be evaluated for d/c

A. document that no on answered the door and the appointment will be rescheduled. -keep it objective (don't rely on neighbor)

An individual with a history of chronic ob structive pulmonary disease (COPD) has lim ited endurance. The long-term goal for this in dividual is to prepare three meals a week. The MOST relevant short-term goal for the OT prac titioner to focus on is independent use of: A. energy conservation techniques. B. work-hardening activities. C. graded activities to increase strength. D. safety skills in the kitchen

A. energy conservation techniques.

A client sustained a C4 spinal cord injury in a car accident, and the OT is recommending a power wheelchair with a sip-and-puff controller. Which of the following options would be best for providing pressure relief to the buttocks by changing orientation in space but not body position? A. Tilt in space B. Recline C. Elevating leg rests D. Pelvic bar

A. tilt in space

Which of the following symptoms are associated with a client who is experiencing an absence (petit mal) seizure? A.Abruptly stopping an activity with a blank stare and fluttering eyelids B.Sudden movement of a single body part with no change in vital signs C.Stiffening of extremities with rhythmic, bilateral jerking of limbs

A.Abruptly stopping an activity with a blank stare and fluttering eyelids

A client sustained a TBI 3 months ago and is functioning at Level VIII (Purposeful, Appropriate, Stand-by Assist) on the Rancho Los Amigos scale. The client is participating in a meal preparation task. As part of the task, the client is asked to prepare vegetable soup using a five-step printed recipe. The client is able to read the recipe steps aloud but does not act on any of them. When the OTR covers over all but the first step of the recipe, the client follows through with the step. What conclusion can the OTR make about the client based on this observation? A.Adaptive strategies compensate for attention deficits. B.Ideational apraxia interferes with task initiation. C.Anchoring techniques improve visual perception.

A.Adaptive strategies compensate for attention deficits.

To influence modifiable risk factors for readmission, which action should be a priority for an OTR working in an acute care hospital to complete prior to a patient's discharge home? A.Address unmet IADL and ADL needs. B.Identify an adequate support network. C.Discuss chronic disease and co-morbidities.

A.Address unmet IADL and ADL needs. -MODIFIABLE risk factors

An OTR is selecting a seat cushion for a client who has tetraplegia. The primary objective is to select a cushion that provides even distribution of pressure relief to the buttocks when the client is seated in a wheelchair. Which type of cushion would be MOST EFFECTIVE for this purpose? A.Air-filled B.Soft foam C.Honeycomb gel

A.Air-filled -even distribution

A client with primary progressive multiple sclerosis has an atonic bladder that causes urinary incontinence. The OTR and the nurse are collaborating to teach the client self-catheterization strategies. The client has poor perineal sensation and wants to be independent in self-catheterization. Which of the following is the MOST BENEFICIAL assistive device to increase self-catherization? A.An adjustable handheld mirror B.A built-up handle attached to tubing C.A no spill urinal

A.An adjustable handheld mirror

An OTR who has 6 years of clinical experience is starting a private practice outpatient clinic. Medicare and workers' compensation will be the primary source of reimbursement. In addition to obtaining state licensure, which action MUST the OTR take prior to billing for services provided at this clinic? A.Apply for a National Provider Identifier number through the Centers for Medicare & Medicaid Services. B.Ensure active member registration with the American Occupational Therapy Association. C.Maintain current professional certification with the National Board for Certification in Occupational Therapy

A.Apply for a National Provider Identifier number through the Centers for Medicare & Medicaid Services.

An OTR is teaching compensatory strategies to an older adult client who lives alone and receives weekly visits from a family member. One of the client's priorities is to be independent with IADLs, including medication management. However, when filling the pill box, the client made 8 medication errors in the past month. To respect the client's stated priorities and to maintain client safety, which statement represents the BESTethical action for the OTR to take? A.Ask for medications to be pre-packaged at the pharmacy according to prescription instructions. B.Encourage the client's family member to organize the medication in the pill box once per week. C.Have the client fill the pill box with the family member present to monitor for possible mistakes.

A.Ask for medications to be pre-packaged at the pharmacy according to prescription instructions. -Safety first -It is the ethical responsibility of the OTR to select an intervention strategy that will ensure the client's safety (principle of beneficence) and respect the client's wishes (principle of autonomy).

A client who has moderate cognitive decline lives in a skilled nursing facility. The client's spouse visits every morning after breakfast. The spouse informs the OTR that the client's frequency of interacting during the visits has decreased over the past few weeks, and the client now has intermittent verbal outbursts that disrupt the social interaction. In addition to informing the care team, what action should the OTR take based on this observation? A.Ask the spouse open-ended questions about the situation, and identify possible triggers for the change in level of interaction. B.Sit with client and the spouse during a visit, and ask the client direct questions to facilitate appropriate communication. C.Provide the spouse with information about providing immediate feedback to the client when there are signs of agitation.

A.Ask the spouse open-ended questions about the situation, and identify possible triggers for the change in level of interaction.

An OTR, who works in an inpatient rehabilitation setting, completes a critical appraisal of outcome measures for patients with TBI. The OTR wants to propose including these measures in the department's clinical protocols but finds that the fast pace of the setting is a barrier to communicating with colleagues. What is the MOST BENEFICIAL method for the OTR to present the benefits of modifying the department's clinical protocols for the TBI population? A.Ask the supervisor to include program planning on the agenda during a staff meeting. B.Encourage each therapist to read scholarly articles that support a change in protocol. C.Discuss the evidence with colleagues intermittently throughout the workday.

A.Ask the supervisor to include program planning on the agenda during a staff meeting.

A client who has substance use disorder is referred to OT for assistance with leisure skill acquisition to help maintain sobriety. Which intervention priority is MOST IMPORTANT for the OTR to include in the intervention plan? A.Assist the client to identify activities that are of interest B.Schedule the client to work as a volunteer at the clinic C.Provide the client with a list of nearby recreational facilities

A.Assist the client to identify activities that are of interest

What is the FIRST action an OTR should take if a client who has paranoid personality disorder becomes suspicious and reports feeling exploited during an intervention session? A.Avoid confrontation and remind the client of the intentions of the therapeutic relationship. B.Inform the client that therapy will be most effective through maintaining a positive attitude. C.Report the behavior immediately to the referring physician and interprofessional team.

A.Avoid confrontation and remind the client of the intentions of the therapeutic relationship.

A 6-year-old child who has developmental delay is learning to self-dress. Over the past several months, the child has made significant progress and is now able to complete age-appropriate dressing tasks with the exception of independently tying shoe laces. The parent reports the child becomes extremely frustrated with this task and has behavioral outbursts when an adult provides assistance. Which technique should the OTR incorporate into the intervention to MOST EFFECTIVELY support the child's success with shoe-tying? A.Backward chaining B.Gesture prompting C.Forward chaining

A.Backward chaining

A high-school student with autism spectrum disorder excels academically but has difficulty organizing assignments and homework. The OTR plans to evaluate the student's relative strengths and weaknesses to guide intervention planning. One of the student's goals is to learn an organization system. Which of the following is the BEST assessment for this purpose? A.Behavior Rating Inventory of Executive Function (BRIEF) B.Executive Function Performance Test (EFPT) C.Canadian Occupational Performance Measure (COPM)

A.Behavior Rating Inventory of Executive Function (BRIEF) -The objective of the Behavior Rating Inventory of Executive Function (BRIEF) is to evaluate self-regulation and executive function in adolescents who have neurological or developmental conditions.

An inpatient who has COPD is participating in a dressing session while seated at bedside. While putting on a pair of pants, the patient begins to have dyspnea. Pulse oximetry indicates the patient's oxygen saturation level is 93%. After stopping the activity, what should the OTR have the patient do NEXT? A.Breathe in deeply through the nose and slowly exhale through pursed lips. B.Inhale through pursed lips and quickly exhale through the nose. C.Breathe through a nasal cannula using supplemental oxygen. D.Take several short shallow breaths through the mouth.

A.Breathe in deeply through the nose and slowly exhale through pursed lips. -Pursed lip breathing is an intervention technique that assists in mitigating the effects of dyspnea.

An OTR who works in an elementary school is preparing to evaluate a student in fifth grade who has a learning disability and ADHD. Which standardized assessments would be MOST EFFECTIVE for the OTR to include as part of the evaluation process? A.Bruininks-Oseretsky Test of Motor Proficiency (BOT) and Evaluation Tool of Children's Handwriting (ETCH) B.Hawaii Early Learning Profile (HELP) and Barthel Index of ADL C.Wee-FIM™ and Ranchos Los Amigos Levels of Cognitive Function-Revised™

A.Bruininks-Oseretsky Test of Motor Proficiency (BOT) and Evaluation Tool of Children's Handwriting (ETCH)

An OTR is developing seating recommendations for a client with cerebral palsy who uses a wheelchair for functional mobility. The client has a fixed hip contracture causing pelvic obliquity with the iliac crest positioned higher on the right. What type of custom seating system should the OTR recommend the client sit on when positioned in the wheelchair? A.Build up the right lateral side of the seat cushion and lower the left side to even out contact area throughout the cushion. B.Drop the front part of the seat base downward and build up the posterior part of the cushion to prevent a posterior pelvic tilt. C.Provide a pre-ischial shelf on the left, anterior part of the cushion to stop the client from sliding forward.

A.Build up the right lateral side of the seat cushion and lower the left side to even out contact area throughout the cushion. - you want to accommodate for hip positioning; not fix it

A young adult client sustained transfemoral amputations to both legs after a recent accident. The OTR is gathering information to identify the client's priorities and personal goals regarding engagement in daily activities. Which standardized assessment tool would be MOST BENEFICIAL to use for obtaining this information? A.Canadian Occupational Performance Measure (COPM) B.Kohlman Evaluation of Living Skills (KELS) C.Functional Independence Measure (FIM™)

A.Canadian Occupational Performance Measure (COPM)

An OTR is completing a functional visual screening of a client who has macular degeneration. The OTR asks the client to read a passage from a magazine. The client misses several letters and words, and has difficulty finding their place in the text when scanning. What aspect of visual function should the OTR investigate further based on the client's performance? A.Central visual field B.Visual acuity C.Contrast sensitivity

A.Central visual field -reading is in center and macular degeneration is central field issues

A student in the second grade has spastic quadriplegia secondary to cerebral palsy and uses a power wheelchair for mobility. The OTR observes the student throwing and catching a ball with peers during gym class. As the student attempts to throw the ball, the trunk shifts laterally and the student has difficulty moving back to midline to continue the game. What modification to the wheelchair would be MOST EFFECTIVEfor promoting postural stability to maintain a proper base of support while this student is throwing and catching the ball? A.Chest harness B.Thigh and pelvic strap C.Lap tray attachment

A.Chest harness

A student in the second grade has autism spectrum disorder and is scheduled to begin school-based OT. The teacher reports the student has difficulty attending to academic tasks and typically has outbursts when in close proximity to other people. In which environment should the majority of the student's intervention sessions take place? A.Classroom during routine curriculum-based activities B.Playground in an area apart from other students C.Self-contained occupational therapy treatment room

A.Classroom during routine curriculum-based activities -Providing services in the natural classroom environment during typical routines would increase the likelihood for carry-over and consistency.

An inpatient in a rehabilitation facility recently had transtibial amputations of both legs. The patient is currently scheduled for discharge in 2 weeks. During the weekly interprofessional meeting the OTR reports the patient has met the OT intervention goals. The physical therapist reports the patient will continue to require physical therapy until discharge from the facility. The prosthetist will also be working with the patient during this time, and nursing will continue to perform wound care. What action should the OTR take based on the meeting reports? A.Complete a discharge summary and discontinue the patient from OT services. B.Identify any other activities in which the patient would like to participate. C.Schedule the patient for at least one hour of OT services per day.

A.Complete a discharge summary and discontinue the patient from OT services.

An OTR, who works in the home health setting, is evaluating a client with pneumonia who was recently discharged home from the hospital. The OTR plans to interview the client and administer a standardized assessment to gather information about the client and the home environment. Which additional evaluation method is MOST BENEFICIAL for the OTR to include in the information-gathering process to understand the client's priorities? A.Conduct skilled observations while in the client's home. B.Measure spaces and features in the natural environment. C.Complete a comprehensive social participation inventory.

A.Conduct skilled observations while in the client's home.

An OTR is developing an intervention plan using a bottom-up approach for clients who have hemiplegia and hemi-neglect secondary to having a CVA more than one year ago. Which of the following intervention techniques has evidence supporting its efficacy for reducing the effects of "learned non-use" through cortical reorganization? A.Constraint-induced movement therapy (CIMT) B.Occupational adaptation (OA) C.Proprioceptive neuromuscular facilitation (PNF)

A.Constraint-induced movement therapy (CIMT)

An OTR is a contract employee at a preschool and works 2-3 shifts each month. During a scheduled shift, the OTR receives a call from a student's physician asking for a progress update for a student. The therapist locates the student's file but is unsure of the school's confidentiality policy. What action should the OTR take NEXT? A.Consult with the therapy supervisor to understand the privacy restrictions. B.Request that the family provide written consent for consultation with team members. C.Provide the physician with a summary of functional gains made by the student.

A.Consult with the therapy supervisor to understand the privacy restrictions.

An OTR has completed an initial neuromotor assessment of a 4-year-old child who has moderate athetoid cerebral palsy. Results indicate persistent primitive reflexes and decreased oral motor control interfere with feeding and functional communication. The parents want the child to be able to self-feed, eat family meals and communicate with others. What additional information is MOST IMPORTANT for the OTR to collect prior to developing the intervention plan? A.Contextual features that support the child's typical participation in occupation B.Early intervention programs available for supporting the child's academic readiness C.Medical reports that include the child's past medical history and developmental prognosis

A.Contextual features that support the child's typical participation in occupation

Which prewriting skill would a TYPICALLY developing 3-year-old child be expected to perform? A.Copying a horizontal line B.Writing uppercase letters C.Imitating intersecting lines

A.Copying a horizontal line

An inpatient had a CVA with flaccid hemiplegia 8 days ago. When the OTR arrives for the scheduled session, the patient reports pain in the axillary region of the affected upper extremity. The OTR notes significant edema in the arm and that the arm is warm to the touch. Which of the following is the BEST option for why the OTR should alert the physician about these observations? A.Deep vein thrombosis B.Subluxation of the shoulder C.Late effect of cerebrovascular disease

A.Deep vein thrombosis

An OTR is using a top-down approach to select interventions for a client who has unilateral neglect secondary to a CVA. Which intervention would be MOST BENEFICIAL to include as part of the client's intervention when using this approach? A.Determine compensatory options the client can use in the home environment. B.Use tactile-kinesthetic guiding to the client's involved extremity during a dressing task. C.Place commonly used toiletry items to the client's affected side during self-care tasks. D.Teach drills for practicing head turning to find an object placed near the affected side.

A.Determine compensatory options the client can use in the home environment. -This represents a top-down approach because it focuses on a strategy the client can use in everyday life. -top down is whole person

An inpatient is recovering from a second major depressive episode. The patient plans to resume family and homemaking responsibilities after discharge from the hospital. Which INITIAL action should the OTR take as part of the intervention for promoting progress toward this goal? A.Determine the patient and family priorities for role resumption. B.Educate the patient and family about pacing typical daily tasks. C.Develop a program that promotes gradual integration into family routines. D.Recommend family share responsibilities based on typical home routines.

A.Determine the patient and family priorities for role resumption.

A young adult client with high functioning autism attends a community-based day program. The client goes to a restaurant for lunch twice per week with staff and other clients from the program. The client enjoys this activity but becomes extremely anxious and engages in negative self-stimulation behaviors when it is time to transition from center activities to go to the restaurant. One of the intervention goals is to help the client learn to manage these behaviors during this transition time. Which strategy would be MOST EFFECTIVE to include as part of the intervention for supporting this goal? A.Develop a social story. B.Teach visualization techniques. C.Create a color-coded behavior chart.

A.Develop a social story. -good for transitions between environments

An OTR completed a screening with an older adult inpatient who had surgery for an abdominal hernia repair 2 days ago. When interpreting the results of the screening, the OTR learns the patient has hearing loss that was not noted during the patient's intake procedures. What influence would undetected hearing loss have on the patient's occupational performance when completing a standardized assessment? A.Difficulty following verbal test directions B.Impaired sequencing skills during ADL C.Easily distracted by ambient auditory stimuli

A.Difficulty following verbal test directions

During an OT session a patient stops working on the activity and states, "My grandson would like this activity. What's it costing me to match up these colors and patterns?" What action should the OTR take in this situation? A.Discuss activities that would be more meaningful to the patient. B.Review costs and insurance coverage with the patient. C.Remind the patient about the importance of the activity

A.Discuss activities that would be more meaningful to the patient.

A client, who has depression, has accomplished the stated goal of completing ADL by 9 a.m. daily for one week. After a medication adjustment, the client has difficulty waking in the morning and ADL performance has decreased. What is the FIRSTaction the OTR should take in response to the client's change in function? A.Discuss the effects of the medication change with the interprofessional team. B.Discontinue intervention until the client adjusts to the medication change. C.Advise the client to take the medication earlier in the evening.

A.Discuss the effects of the medication change with the interprofessional team.

An OTR is completing an initial evaluation of an inpatient diagnosed with a lesion in the orbitofrontal cortex. The patient is oriented to person, place, and time but constantly makes sexually inappropriate comments to the OTR. Which option BESTdescribes this behavior? A.Disinhibition B.Disorientation C.Distractibility

A.Disinhibition

An OTR is preparing discharge recommendations for an inpatient who recently has been diagnosed with relapsing-remitting multiple sclerosis. The patient is an executive chef at a restaurant and wants to resume work as soon as possible after discharge. What would be MOST BENEFICIAL for the OTR to include as part of the discharge planning process for supporting this goal? A.Education about wearing a cooling vest to regulate body temperature B.Handouts outlining the general purpose and principles of energy conservation C.Instructions on using pursed lip breathing techniques to regulate dyspnea

A.Education about wearing a cooling vest to regulate body temperature -Heat intolerance is a symptom of multiple sclerosis that may be exacerbated in a client who works in a kitchen. Recommending a cooling vest is a solution to decrease this risk of overheating.

An OTR is completing home set-up for a client who has a C5 spinal cord injury and is transitioning from an inpatient setting to home. Which type of bed would be MOST BENEFICIAL for this client to use at home? A.Electric hospital bed with adjustable side rails B.Standard bed that is level to the wheelchair seat height C.Adjustable frame with overhead trapeze and attached rope ladder

A.Electric hospital bed with adjustable side rails -C5 spinal cord injury has active elbow flexion and is able to use the side rails to assist with rolling and bed mobility. The electric hospital bed will decrease risk of injury to a caregiver providing assistance during ADL.

A client in an outpatient setting sustained a frontal lobe TBI 2 months ago. The client has good motor control but has residual problems with executive functioning. One of the client's goals is to be independent with homemaking tasks. During a meal preparation session, the client cooks a meal, but makes no attempt to clean the cooking utensils and dishes or put the food items away after completing the cooking task. Which area of executive function appears to be MOST affected by the TBI as evidenced by this behavior? A.Emergent awareness B.Environmental gnosia C.Selective attention D.Episodic memory

A.Emergent awareness

An OTR who works in an outpatient setting is providing assistive technology recommendations to a client who has amyotrophic lateral sclerosis. Which is the MOST EFFECTIVE method for the OTR to use during the intervention session to evaluate the client's understanding of the information presented? A.Engage the client in a discussion to allow the opportunity for questions to be answered. B.Explain to the client evidence that supports benefits and outcomes associated with each device. C.Provide the client with the written information from the manufacturers of the devices.

A.Engage the client in a discussion to allow the opportunity for questions to be answered.

Which precaution should be taken during intervention sessions with an inpatient in an acute care setting who is in the initial acute phase of Guillain-Barré syndrome? A.Ensure hemodynamic stability prior to assisting with bed-to-chair transfer. B.Avoid substitution movement patterns during resistive exercise programs. C.Check that the mean arterial pressure is less than 110 mmHg before starting the intervention.

A.Ensure hemodynamic stability prior to assisting with bed-to-chair transfer. -it is essential to measure the patient's blood pressure and heart rate, and determine that the patient has the stability to tolerate the change in body positions and the transfer. -Haemodynamic stability is the medical term used to describe a stable blood flow.

Which precaution should be taken during intervention sessions with an inpatient in an acute care setting who has an indwelling urinary catheter? A.Ensure the collection bag is below the level of the patient's bladder without placing tension on the tubing. B.Hang the urine collection bag on an intravenous pole next to the patient to avoid displacement of the tube. C.Place the urine collection bag on the patient's lap when transporting the patient in a wheelchair.

A.Ensure the collection bag is below the level of the patient's bladder without placing tension on the tubing. -prevent back flow of urine

An OTR is working with a client who has a visual field deficit. Which option represents the BEST type of activity for teaching compensatory scanning techniques? A.Everyday household activities such as paying bills and reading email. B.Pencil and paper tasks, including mazes and crossword puzzles C.Visual tracking of a penlight moved slowly by the OTR.

A.Everyday household activities such as paying bills and reading email.

An OTR, who works in a long-term acute care setting, is following contact precautions when providing intervention to an inpatient who has MRSA. In addition to putting on gloves and gown, what other action MUST the OTR take? A.Follow hand hygiene procedures B.Discard equipment when possible. C.Wear a custom-fit N95 mask.

A.Follow hand hygiene procedures -they MUST do this

For a typically developing child, which postural reflex becomes evident between 6-9 months of age and continues throughout life? A.Forward protective extension B.Asymmetrical tonic neck C.Head righting D.Tonic labyrinthine

A.Forward protective extension

An OTR who works in a hospital setting is preparing to evaluate an inpatient who recently had a C5 spinal cord injury. What action MUST the OTR take prior to commencing the evaluation process? A.Gather all necessary materials and supplies to complete the planned assessments. B.Request a family member be available to provide emotional support and answer questions. C.Ensure members of the interprofessional team understand the unique contribution of OT.

A.Gather all necessary materials and supplies to complete the planned assessments.

An OTR is developing an intervention plan for an inpatient who has major depressive disorder. The patient reports poor concentration, decreased pleasure in activity, inability to plan or prioritize tasks, and feelings of incompetence. What would be the BEST action for the OTR to take during the intervention planning process? A.Have the client identify meaningful short-term goals. B.Have the client participate in social and leisure groups. C.Have the client engage in a multistep project.

A.Have the client identify meaningful short-term goals.

An outpatient client has increased sensitivity of the residual limb following a transradial amputation of the dominant arm. The incision is fully healed. One of the intervention priorities is to reduce hypersensitivity of the residual limb in preparation for a trial with a prosthetic device. Which intervention activity would be MOST BENEFICIAL to support this objective? A.Have the client immerse the limb into containers filled with items of various textures. B.Apply transcutaneous electrical nerve stimulator for 10 minutes before functional activity. C.Provide a strengthening program for the shoulder girdle and the muscles around the elbow.

A.Have the client immerse the limb into containers filled with items of various textures.

A client who has generalized anxiety disorder has difficulty conceptualizing and organizing future events, and is experiencing decreased feelings of competence. What method should the OTR include in the INITIAL intervention session? A.Have the client prioritize short-term goals that will have a high chance of success. B.Assist the client to plan social activities with friends and start new projects at home. C.Encourage the client to freely explore meaningful, multiple-step projects.

A.Have the client prioritize short-term goals that will have a high chance of success.

An OTR is preparing an intervention plan for a student in second grade who has attention deficit hyperactivity disorder. The student has difficulties getting ready for school, often misplaces homework assignments, and forgets to bring assignments from home to school. The OTR plans to help the student learn strategies to use in preparation for going to school each day. Which activity would be MOST EFFECTIVE for supporting initial progress toward this objective? A.Help the student develop an organizational checklist to use as part of a treasure hunt game. B.Ask the student's parents to assist the student in completing a school needs checklist each morning. C.Have the student follow a step-by-step instruction sheet during a class art project.

A.Help the student develop an organizational checklist to use as part of a treasure hunt game.

An inpatient recovering from pneumonia has decreased strength and endurance, and functional active ROM. The patient ambulates independently for short distances with the use of a walker. Medical history indicates the patient has recently been diagnosed with moderate macular degeneration. The patient will be discharged in one day to live alone in a single-story home. Which service should the OTR recommend as part of the patient's discharge plans? A.Home health services to assess ADL needs and teach compensatory strategies B.Extension of current hospital stay for comprehensive rehabilitation services C.Short-term placement in a skilled facility to maximize independence in ADL

A.Home health services to assess ADL needs and teach compensatory strategies

An OTR working in an outpatient setting is planning intervention for a client who has a 12-month history of fibromyalgia. The client reports symptoms associated with this condition are limiting the ability to participate in social activities. This has led to significant frustration and progressive social isolation over the past few months. The client wants to reverse this trend. What should the OTR include as part of the INITIAL intervention planning process with this client? A.Identify options for modifying the client's preferred community activities. B.Recommend the client maintain a journal as a means of venting frustration. C.Explore hobbies the client can try at home that do not exacerbate pain.

A.Identify options for modifying the client's preferred community activities.

An inpatient, who has secondary-progressive multiple sclerosis, had surgery 3 days ago for placement of an intrathecal baclofen pump to influence severe spasticity in both lower extremities. When the OTR arrives for the next scheduled session, the patient is in respiratory distress with decreased mental status. What is the FIRSTaction the OTR should take based on this observation? A.Immediately alert the primary care nurse assigned to the patient. B.Place the patient in a reverse Trendelenburg position and administer oxygen. C.Activate the emergency response system and start chest compressions.

A.Immediately alert the primary care nurse assigned to the patient.

A home health client has good visual acuity but has difficulty seeing cooked chicken when slicing it on a white cutting board. What is the PRIMARY reason for this occurrence? A.Impaired contrast sensitivity B.Decreased peripheral visual field C.Weak oculomotor function

A.Impaired contrast sensitivity

An adolescent client who has a moderate traumatic brain injury is participating in an initial BADL session. During the session, the client frequently seeks physical contact with the OTR. What action should the OTR take in response to the client's behavior? A.Inform the client that the behavior is inappropriate and set reasonable limits. B.Advise the client of generally accepted distances for personal space. C.Provide the client with alternate methods for obtaining proprioceptive input.

A.Inform the client that the behavior is inappropriate and set reasonable limits. -Establishing clear and reasonable limits is an important part of a behavior management plan.

An OTR working in a pediatric outpatient setting approaches the therapy manager and states, "I believe my colleague is billing for interventions that are not being provided during client sessions." After acknowledging the practitioner's concern, what actions should the manager take NEXT? A.Initiate a review of internal clinical service coding and billing practices. B.Develop a remediation program for resolving future billing issues. C.Ask the OTR to provide supporting proof of the colleague's misconduct.

A.Initiate a review of internal clinical service coding and billing practices. -The manager is responsible for the oversight of billing practices utilized by the occupational therapy staff.

What INITIAL actions should the OTR take to establish rapport when meeting a client for the first time? A.Introduce self to the client and explain the purpose of the visit. B.Provide encouragement for behavior change and define intervention outcomes. C.Interact with family members and address the client by first name.

A.Introduce self to the client and explain the purpose of the visit.

Which assessment would be MOST BENEFICIAL to administer to an inpatient who is functioning at a Level III (Localized response) on the Rancho Los Amigos scale? A.JFK Coma Recovery Scale B.Contextual Memory Test C.Katz Index of ADL

A.JFK Coma Recovery Scale

An OTR who works in a home health setting is developing educational handouts for clients who have low vision. Which guidelines are MOST BENEFICIAL for the OTR to use when developing educational handouts for this population? A.Large print with a minimum of 16-point font with high contrast between the text and the paper B.Many different colors to highlight important information printed on high-gloss paper C.Pictorial illustrations and written information placed in columns on the page

A.Large print with a minimum of 16-point font with high contrast between the text and the paper

An OTR, who wants to learn best evidence for occupational therapy interventions that improve ADL performance in clients with low vision, is reading a systematic review that summarizes multiple randomized control trials. Based on the hierarchy of evidence, which level of evidence is a systematic review? A.Level I B.Level V C.Level III

A.Level I -A systematic review involves analysis of several randomized controlled trials and, in the hierarchy of evidence, is considered the highest level of evidence.

Which of the following symptoms TYPICALLY indicates that a client who has been on prolonged bed rest is experiencing orthostatic hypotension? A.Lightheadedness upon moving from a supine to seated position B.Pounding headache upon moving into a semi-reclined position C.Diaphoresis when turning over from supine to side-lying position D.Shortness of breath when sitting up from a supine position

A.Lightheadedness upon moving from a supine to seated position

A patient in a rehabilitation facility has a C6 (ASIA A) spinal cord injury. The patient is learning techniques to maintain forward sitting balance while sitting on the edge of the bed. Which technique is MOST BENEFICIAL for the OTR teach the patient to support progress toward this goal? A.Lock the elbows in full extension; and use shoulder depression, protraction and external rotation, and full wrist extension B.Position the elbows in slight flexion; use available shoulder movement while maintaining the wrist in neutral C.Alternate between full elbow extension and flexion; use shoulder elevation, retraction and internal rotation

A.Lock the elbows in full extension; and use shoulder depression, protraction and external rotation, and full wrist extension -elbows have no strength (0/5) so use shoulders and keep elbows locked

An OTR is using the principles of evidence-based practice to guide clinical decision making and wants to generate a clinical question that is pertinent to clients on the caseload. Which option represents a useful strategy for formulating a clinical question to initiate the evidence-based practice process? A.PICO method B.Meta-analysis C.Normative research

A.PICO method

A client sustained a distal radius fracture of the dominant upper extremity 6 weeks ago. A short arm cast was applied on the day of injury and was removed one day ago. The client holds the affected arm close to the chest in a protected position due to pain, which the client rates as 6 out of 10 using a visual analog scale. The hand and forearm are moderately edematous. The client is able to flex the fingers to within one inch (2.54 cm) from each fingertip to the distal palmar crease. The client refused to move the wrist due to pain. Which intervention should be a priority to include as part of the treatment plan during the INITIAL phase of the client's rehabilitation? A.Manual edema mobilization of the affected extremity B.Fabrication of a dynamic finger flexion orthotic C.Passive ROM exercises for the wrist and fingers D.Gentle stress loading exercises as tolerated

A.Manual edema mobilization of the affected extremity -edema reduction is priority

Using the levels of evidence hierarchy, which type of research methodology is considered the strongest type of evidence to inform practice decisions? A.Meta-analysis B.Randomized control trial C.Single-subject case study

A.Meta-analysis

An inpatient had a myocardial infarction 2 days ago and is beginning phase I cardiac rehabilitation. Which activity is an ESSENTIAL component of the initial assessment with this patient? A.Monitoring the patient's orthostatic tolerance during movement B.Determining the patient's typical daily energy expenditure C.Measuring the patient's upper extremity grip and pinch strength

A.Monitoring the patient's orthostatic tolerance during movement

An OTR who works on a general medical unit in an acute care hospital is evaluating an older adult patient who has pancreatitis, diabetes mellitus, and atrial fibrillation. During the evaluation, the patient appears confused and is disoriented to person, place, and time. What assessment should the OTR include in the initial evaluation of this patient? A.Montreal Cognitive Assessment B.Canadian Occupational Performance Measure C.Draw a Clock Test

A.Montreal Cognitive Assessment

An OTR, who works in an inpatient rehabilitation setting, is working with a patient who had a traumatic T10 (ASIA A) spinal cord injury 3 weeks ago. Prior to the injury, the patient worked as a pilot for a commercial airline. The patient is distressed about the future and says, "I don't think I will ever be able to fly an airplane again." Which of the following provides the BEST description of the patient's response to this unexpected change in functional abilities? A.Mourning B.Shock C.Denial

A.Mourning

An OTR working in an industrial setting is documenting the success of a wellness program. One of the program goals is to reduce the number of job-related injuries among workers in a warehouse shipping and receiving department. The initial assessment cited risks related to poor worker fitness, improper lifting techniques, and inadequate work routines. What information about the program should be documented in the program reports to management to provide the MOST BENEFICIAL information about the effectiveness of the program? A.Number of new or recurring injuries during the reporting period B.Pre- and post-test results from a worker questionnaire about job-related ergonomics C.Evidence of improved body mechanics among workers during lifting tasks

A.Number of new or recurring injuries during the reporting period -goal of program is to reduce injuries

An OTR is administering the Functional Independence Measure (FIM™) as part of the initial evaluation of an inpatient who had a left CVA with aphasia 5 days ago. The patient has right hemiparesis and requires moderate assistance to maintain balance while standing. What action should the OTR take when completing this assessment? A.Observe as the patient performs ADL routines with the typical methods used prior to admission. B.Introduce a reacher and other adaptive equipment to maximize patient's level of performance. C.Provide the patient with visual and tactile cues to sequence each step of the task.

A.Observe as the patient performs ADL routines with the typical

An OTR and a COTA are collaborating to evaluate a student in fifth grade who is legally blind and had a CVA with right-sided hemiplegia 6 years ago. Which task can the OTR delegate to the COTA as part of the evaluation process? A.Observe the student's ability to navigate in the hallway during transition from the classroom to the playground. B.Select specific assessment tools to understand the influence of sensory and process skills on academic performance. C.Develop an intervention plan that includes accessing resources for individuals who are visually impaired.

A.Observe the student's ability to navigate in the hallway during transition from the classroom to the playground.

An OTR working in an outpatient setting refers a client to a driving rehabilitation specialist. The OTR wants to provide the specialist with a summary of the client's progress during OT intervention. To be in compliance with the Health Insurance Portability and Accountability Act (HIPAA), what action must the OTR take? A.Obtain written authorization from the client to release the information to the specialist. B.Give a copy of the therapy documentation to the client to deliver to the specialist. C.Provide the specialist with access to the client's online medical record.

A.Obtain written authorization from the client to release the information to the specialist.

A family member, who has not attended therapy sessions and is unknown to the medical team, attends the discharge planning meeting for an inpatient who will be discharged in 3 days. In order to be in compliance with the Health Insurance Portability and Accountability Act (HIPAA), what action MUST occur before initiating the meeting? A.Obtain written consent from the patient for the family member to attend. B.Determine if the family member will assist the patient after discharge. C.Request that the family member sign a patient privacy confidentiality agreement.

A.Obtain written consent from the patient for the family member to attend.

An OTR is evaluating the visual function of a patient who is emerging from coma. The patient opens eyes spontaneously and in response to auditory stimulation. What visual function is MOST IMPORTANT for the OTR to observe in a patient at this level of function? A.Oculomotor control B.Scanning C.Pattern recognition

A.Oculomotor control

An OTR who works in an intensive care unit is selecting a communication system for an inpatient who recently had a C2 spinal cord injury and uses a ventilator to assist with breathing. The patient has intact cognitive function but is unable to communicate verbally. Which option would be MOST BENEFICIAL to support the patient's functional communication needs? A.Partner-assisted scanning with a picture communication symbol board B.Direct selection interfaced with a dynamic display communication system C.Auditory scanning using a clock face communication device

A.Partner-assisted scanning with a picture communication symbol board -keep it low tech at this stage of recovery

Which interprofessional team member would be MOST BENEFICIAL for an OTR to refer a middle-school student who has expressed suicidal ideation? A.Physician B.Psychologist C.Social worker

A.Physician -immediate contact

Which option BEST represents instructions an OTR should provide an inpatient who had a total hip arthroplasty 2 days ago and post-operative precautions include non-weight-bearing (NWB)? A.Place no body weight through the operative extremity. B.Touch toes on the operative extremity on the floor but do not bear weight. C.Apply up to 50% of body weight through the operative extremity.

A.Place no body weight through the operative extremity.

A kindergarten student has mild ataxic cerebral palsy. Social and cognitive skills are intact. Which task would be BEST to ask the student to complete when screening fine motor skills? A.Place school supplies into a backpack. B.Slide down a slide on the playground. C.Follow verbal directions in the classroom.

A.Place school supplies into a backpack.

Which option represents the MOST EFFECTIVE method to screen an 8-month-old infant for neck-on-body automatic righting reaction? A.Place the child in a supine position and observe body movements while rotating the child's head to one side and then to the other. B.Place the child in a sitting position and observe for arm extension while gently pushing the child off balance to the front, back, and each side. C.Hold the child in vertical suspension and observe head position while gently tilting the child from side to side then from front to back.

A.Place the child in a supine position and observe body movements while rotating the child's head to one side and then to the other. -B is protective -C is equilibrium

An OTR who works in a hospital setting is searching for a standardized assessment tool to include in the initial evaluation of an inpatient who has a progressive neurological condition. However, the OTR is unable to find a tool that matches the needs of the patient and is specific to the hospital setting. What action would be MOST BENEFICIAL for the OTR take next? A.Plan to use available non-standardized assessment measures in combination with observation and interview methods to develop a client-centered intervention plan. B.Develop an assessment tool that meets the needs of the patient and plan to administer the same assessment throughout the patient's entire length of stay. C.Select a standardized assessment tool that is typically used by the rehabilitation department staff and adapt the administration protocol to meet the needs of the patient.

A.Plan to use available non-standardized assessment measures in combination with observation and interview methods to develop a client-centered intervention plan.

A student in second grade has mild diplegic cerebral palsy. Social and cognitive skills are intact. Which task would be BEST to ask the client to complete when screening gross motor skills? A.Play outside on the playground equipment. B.Use scissors to cut paper along a dotted line. C.Follow instructions during a group activity.

A.Play outside on the playground equipment.

A patient has flaccid hemiplegia and dysphagia secondary to a CVA one month ago. The patient is participating in an interprofessional rehabilitation program. One of the intervention goals is for the patient to regain independence with self-feeding and become safe when eating. What information about the patient is MOST IMPORTANTfor the OTR to present to the interprofessional team during each care coordination meeting? A.Positioning, adaptive devices and caregiver assistance needed during mealtimes B.Specific evidence-based techniques that are being used during intervention sessions C.Improvements in upper extremity movement patterns used for self-feeding D.Ability to select nutritious foods from the hospital dining menu that are safe to swallow

A.Positioning, adaptive devices and caregiver assistance needed during mealtimes

An OTR working in an outpatient setting is planning intervention for a client who has a trigger finger of the third digit on the dominant hand and wants to avoid surgical intervention. The client reports symptoms associated with this condition are limiting the ability to participate in everyday activities. What should the OTR include as part of the INITIAL intervention with this client? A.Prescribe a static immobilization orthosis to position the affected MCP joint at 0° and instructions to avoid MCP flexion. B.Place the hand in an intrinsic plus position and provide education on how to perform a PROM and AROM program 3 times per day. C.Support the MCP, DIP, and PIP in maximal extension and provide a list of activities that encourage MCP flexion.

A.Prescribe a static immobilization orthosis to position the affected MCP joint at 0° and instructions to avoid MCP flexion.

In order to promote professional development and maintain certification, an OTR is developing a continuing competency plan. What action should the OTR take FIRST? A.Prioritize professional needs based on a self-assessment. B.Review current professional goals with an OT supervisor. C.Determine specific requirements of the state OT regulatory body. D.Identify professional development activities needed for recertification.

A.Prioritize professional needs based on a self-assessment. -An effective component of maintaining continuing competence is to systematically perform a self-assessment of current skills and abilities followed by developing a learning plan that prioritizes professional need

An OTR administered a standardized tool to screen the gross and fine motor skills of a student in the fourth grade. The student's total motor composite scores are listed in the table below: 99th percentile What conclusion can the OTR make about the student's motor skills based on these results? A.Proficient level of function B.Below average performance C.Commensurate with peers

A.Proficient level of function

An OTR who works in an outpatient hand therapy clinic fabricated a forearm orthosis for a client. The OTR is providing instructions to the client on the wearing schedule and care of the orthosis. Which is the MOST EFFECTIVE method for the OTR to use in the education session to promote the client's understanding of the information presented? A.Provide a detailed written handout with illustrations to reinforce key points. B.Encourage the client to independently place the orthosis on and off during the session. C.Ask the patient an open-ended question that invites a descriptive response.

A.Provide a detailed written handout with illustrations to reinforce key points. -Written documentation allows the patient to review the instructions after the session and refer to the handout when needed.

An inpatient in the cardiac intensive care unit had a myocardial infarction 3 days ago. The patient is overwhelmed and shocked by the unexpected change in functional abilities. To facilitate a positive adaptive response to this medical situation, what action would be MOST BENEFICIAL for the OTR to take? A.Provide support and empathy while the patient expresses feelings. B.Teach the patient that feelings of grief and depression are to be expected. C.Provide the patient with information on current abilities and limitations.

A.Provide support and empathy while the patient expresses feelings.

An inpatient who has pneumonia and mild cognitive decline has been stabilized in the hospital and is medically ready for discharge. Currently, the patient requires constant supervision for ADL and IADL due to safety concerns and impulsive behavior. The patient and family have declined the treatment team recommendation that the patient move into an assistive living facility. Which option represents the MOST IMPORTANTdischarge recommendation the OTR should provide the family? A.Provide the patient with 24-hour supervision. B.Involve the patient in a structured adult day program. C.Modify the home environment prior to discharge.

A.Provide the patient with 24-hour supervision.

An adolescent was recently admitted to an inpatient psychiatric unit due to symptoms associated with a conduct disorder. Evaluation results indicate the adolescent has a poor self-concept, decreased fine and gross motor coordination, and is socially aggressive. What should be the focus of the INITIAL sessions with this adolescent? A.Providing opportunities for success in a consistent structured environment B.Presenting options for pre-vocational exploration and practice C.Enhancing physical abilities for completing responsibilities at home D.Encouraging participation in self-expression group activities

A.Providing opportunities for success in a consistent structured environment

A client who had a CVA one month ago now has moderate-severe flexor spasticity and scapular immobility of the involved upper extremity. Which technique is CONTRAINDICATED to use for minimizing the impact of spasticity on passive mobility for dressing and hygiene? A.Reciprocal overhead pulley exercises using wall mounted pulleys B.Long-arm air splinting prior to completing a self-care task C.Self-ROM exercises in supine several times per day D.Upper extremity weight-bearing during functional tasks

A.Reciprocal overhead pulley exercises using wall mounted pulleys

An OTR is working with a client who has post-traumatic stress disorder and reports persistent symptoms of anxiety and interrupted sleep, which impairs function in ADL. Which intervention method should the OTR focus on FIRST? A.Relaxation techniques B.Cognitive retraining C.Routine physical activity

A.Relaxation techniques

An OTR, who is working in an acute psychiatric setting, observes that an occupational therapy aide, who was recently hired, has violated departmental policy by leaving a transfer belt attached to a walker in the hallway. What is the FIRST action the OTR should take after observing this? A.Remove the walker and transfer belt from the hallway, and remind the aide of the policy. B.Inform the rehabilitation supervisor and document the incident. C.Locate the aide and explain that leaving the walker and transfer belt in the hall is a safety hazard.

A.Remove the walker and transfer belt from the hallway, and remind the aide of the policy.

An OTR, who works in an outpatient setting, receives a referral to evaluate a new client. The client and the OTR have several mutual friends, see each other regularly at social functions, and maintain contact through a social media platform. Which statement represents the BEST ethical action for the OTR to take? A.Report the relationship to the supervisor, and request the referral be passed to another OTR. B.Maintain professional boundaries, and only provide intervention during in-clinic sessions. C.Provide OT services to the client, but ensure that no preferential scheduling occurs.

A.Report the relationship to the supervisor, and request the referral be passed to another OTR. -It is the ethical responsibility of the OTR to maintain professional boundaries and avoid a dual relationship to protect the client, the therapist and the therapeutic process.

Which information MUST be documented in a letter of medical necessity to support reimbursement for a wheelchair prescription? A.Results of a face-to-face mobility evaluation, justification for each recommended feature, and a statement related to the client's enhanced participation in frequently performed activities B.Details of the occupational profile and living situation, an outline of the wheelchair maintenance plan, and a summary of the client's preferred activities and mobility goals C.Summary of a recent annual physical examination by a medical doctor, competitive pricing options for each custom feature, and a completed checklist of activity limitations

A.Results of a face-to-face mobility evaluation, justification for each recommended feature, and a statement related to the client's enhanced participation in frequently performed activities

What is the FIRST action the OTR should take before completing an evaluation of a patient in a forensic psychiatric hospital? A.Review the patient's medical record for safety precautions. B.Contact the patient's family to determine prior level of function. C.Interview the guards to determine the reason for incarceration.

A.Review the patient's medical record for safety precautions.

Which federal law in the United States regulates the provision of reasonable accommodations for a student with a disability to have equal access to academic programs and to fully participate in sports, after-school care, and extracurricular activities? A.Section 504 of the Vocational Rehabilitation Act B.Individuals with Disabilities Education Act (IDEA) Part B C.Individuals with Disabilities Education Act (IDEA) Part C

A.Section 504 of the Vocational Rehabilitation Act -Section 504 of the Vocational Rehabilitation Act is the federal law in the United States that regulates the provision of reasonable accommodations for a student with a disability to have equal access to academic programs and to fully participate in sports, after-school care, and extracurricular activities.

An inpatient had a L5 spinal cord injury one week ago. Results of the American Spinal Injury Association (ASIA) Impairment Scale (AIS) indicate the classification of the patient's injury is ASIA B. What do the results of this scale suggest? A.Sensory incomplete impairment below the level of the lesion B.Complete motor and sensory impairment below the level of the lesion C.Motor incomplete impairment below the level of the lesion

A.Sensory incomplete impairment below the level of the lesion

An OTR working in an outpatient setting is completing an initial interview with an older adult client who has recently been diagnosed with a progressive neurological disease. What method should the OTR use during the course of the interview to communicate effective listening and client-centered understanding? A.Share examples of adaptations that help to overcome adversity. B.Use head gestures to indicate empathy and understanding. C.Provide examples of how other clients have overcome similar adversity. D.Offer suggestions as the client discloses concerns and problems.

A.Share examples of adaptations that help to overcome adversity.

A 12-month-old infant has moderate hypotonia resulting in developmental delay and poor oral motor control. Which position is recommended for this infant for promoting oral motor function during feeding? A.Slightly reclined with trunk fully supported and the neck and head at midline B.Fully upright in sitting with the head and neck resting in slight extension C.Seated upright in a standard high chair with a lap tray positioned close to the chest D.Semi-reclined in a position of comfort on a soft beanbag chair

A.Slightly reclined with trunk fully supported and the neck and head at midline

A client with a non-operable cerebellum tumor is participating in OT to increase independence with self-feeding. Which assistive devices should the client use to promote progress toward this goal? A.Suction plate and cup holder B.Universal cuff with mobile arm support C.Plastic cup and lightweight utensils D.Side-cutting fork and rocker knife

A.Suction plate and cup holder

A school-based OTR is making universal design for learning (UDL) recommendations for a student who has difficulty reading printed text because of low vision. Using the principles of UDL, which recommendation would be MOST BENEFICIAL for this student? A.Suggest the student try a digital version of a novel or a book with enlarged print. B.Provide the student with a classroom aide to read assigned materials. C.Recommend the student purchase a text-to-speech reader from the school's library.

A.Suggest the student try a digital version of a novel or a book with enlarged print.

An OTR is establishing therapeutic rapport with a young adult client who has a personality disorder and struggles with relationships. As a child, the client lived in several foster homes and briefly attended college before dropping out to live with a partner. Currently, the client is not working and the partner has ended the relationship. What action is MOST IMPORTANT for the OTR to take when establishing rapport with this client? A.Support motivation to make changes to personal behaviors. B.Focus on developing an external locus of control. C.Establish flexible rules of conduct during intervention.

A.Support motivation to make changes to personal behaviors.

An OTR who works in a hospital setting has been asked by the Readmission Prevention committee to provide an in-service to the team about OT services in the care of a patient with diabetes. Which intervention is MOST IMPORTANT to include in this presentation to explain the role of OT in maintaining glycemic control? A.Support the development of healthy routines and participation in activities. B.Explain how food composition and portion size affect blood sugar. C.Provide instructions for correctly timing and measuring medication dosages.

A.Support the development of healthy routines and participation in activities.

An OTR working in an inpatient mental health setting is using a cognitive behavioral approach to select an intervention activity for a young adult undergoing treatment for symptoms associated with body dysmorphic disorder. The patient is employed in a successful career and enjoys outdoor activities such as jogging and hiking, but has difficulty forming and maintaining interpersonal relationships. One of the patient's goals is to improve self-image and the ability to express feelings. Which activity would be MOST BENEFICIAL to include as part of the intervention for supporting this goal when using this approach? A.Teaching the patient to use a dysfunctional thought record for describing distressing situations and emotional responses B.Guiding the patient through avocational exploration of activities that provide opportunities for social interaction C.Engaging the patient in an art project using a variety of media during which the patient is encouraged to release emotions

A.Teaching the patient to use a dysfunctional thought record for describing distressing situations and emotional responses -CBT

An inpatient had a right CVA one week ago. Prior to the CVA, the patient was relatively healthy and worked a full-time job. During a dressing assessment, the patient puts on a pull-over sweater and then realizes the need to put on a T-shirt underneath the sweater. To do this, the patient attempts to insert the T-shirt down the neckhole of the sweater, instead of removing the sweater to put on the T-shirt. Which statement MOST ACCURATELY describes the reason for the patient's action? A.The patient is not able to conceptualize steps of a task due to ideational apraxia. B.Spatial integration deficits may be affecting the ability to recognize items. C.The central nervous system is not receiving complete visual information. D.Right hemispheric damage appears to have resulted in perseverative behaviors.

A.The patient is not able to conceptualize steps of a task due to ideational apraxia.

An OTR is teaching medication management strategies to an older adult client who has diabetes and low vision. One of the intervention goals for the client is to increase compliance in adhering to the prescribed medication regimen. In addition to teaching the client to combine the medication schedule with familiar ADL routines, what would be the MOST EFFECTIVE approach in improving medication adherence in this client? A.Trial several types of adaptive aides for medication management. B.Dedicate set periods of time in the day for taking medication. C.Establish a habit of counting the remaining pills in each bottle.

A.Trial several types of adaptive aides for medication management.

An OTR is evaluating a client who has an ulnar nerve injury at the wrist level of the right dominant extremity. During which task would this injury be MOST evident? A.Turning a key in the car ignition B.Holding coins in the palm of the hand C.Carrying a briefcase D.Operating a desktop calculator

A.Turning a key in the car ignition

An OTR who works in an inpatient rehabilitation unit is completing an evaluation of a patient who had bilateral total knee arthroplasties 3 days ago. The patient is in generally good health, plans to live independently at home, and wants to resume volunteer work in the community. The patient currently requires OT services to increase independence in bathing, dressing, and toileting. What MUST the OTR consider when scheduling the patient for OT? A.Understand documentation requirements for reimbursement by the third party payer B.Patient's endurance for completing activities according to a clinical pathway C.Availability of caregivers to participate in daily sessions for education and training

A.Understand documentation requirements for reimbursement by the third party payer -The OTR has the responsibility of understanding policies and procedures related to reimbursement when establishing the intervention plan.

An OTR is leading a community-based class for self-referred clients who have rheumatoid arthritis. Through class discussion and observation, the OTR determines that one of the clients in the class would benefit from bilateral hand orthotics. After discussing this observation with the client and determining the client intends to pay for services using insurance benefits, what action should the OTR take NEXT to address the client's needs? A.Understand the state regulatory requirements for direct access to OT services. B.Arrange a clinic appointment time to fabricate the orthotics for the client. C.Contact the insurance company to obtain reimbursement authorization for the orthotics. D.Complete a comprehensive evaluation to justify the need for the orthotics to the primary care physician.

A.Understand the state regulatory requirements for direct access to OT services.

A client who has low vision is learning methods for handling money when shopping for personal items in a store. The client is able to identify coins using touch, but wants suggestions for handling paper monetary denominations. Which recommendation should be included as part of the INITIAL intervention with this client? A.Use a consistent method of folding individual denominations. B.Pay for purchases using a bank debit card instead of paper money. C.Consistently place money denominations in a specific order in a wallet.

A.Use a consistent method of folding individual denominations. -uses touch for assistance not sight (C. involves sight)

Which option BEST represents instructions an OTR should provide an inpatient who had a total knee replacement 2 days ago and post-operative precautions include weight-bearing as tolerated (WBAT)? A.Use pain as the limiting factor and place up to 100% of body weight through the operative extremity. B.Place the toes on the operative extremity on the floor to maintain balance but do not bear weight. C.Learn to apply up to 50% of body weight through the operative extremity.

A.Use pain as the limiting factor and place up to 100% of body weight through the operative extremity.

What action should an OTR, who works in early intervention, take when challenging the emerging play skills of a young child with a disability? A.Use toys that are available in the child's home to encourage carry-over of play into everyday life. B.Select games from a resource closet in the therapy office that will create a just-right challenge. C.Purchase new and economical play materials that the child selects from an illustrated catalogue.

A.Use toys that are available in the child's home to encourage carry-over of play into everyday life.

A client has a peripheral neuropathy of the dominant hand. A screening indicates thenar muscle atrophy with loss of thumb opposition and palmar abduction, inability to pick up a key or coin from a table top, and decreased grip and pinch strength compared to the non-affected hand. Based on these findings, where on the client's hand would an OTR expect to find sensory disturbances during a Semmes-Weinstein monofilament assessment? A.Volar surface of the thumb, index, long, and radial half of the ring fingers B.Volar and dorsal surfaces of the small finger and radial half of the ring finger C.Entire palm and tips of the index, long, ring, and small fingers

A.Volar surface of the thumb, index, long, and radial half of the ring fingers -Sensory distribution for the median nerve is to the volar surface of the thumb, index, long, and radial half of the ring fingers.

An outpatient had a myocardial infarction two weeks ago. One of the patient's goals is to resume household tasks. The cardiologist is allowing the patient to resume an activity level of no more than 3.0 metabolic equivalent (METs) units. Which activity represents a maximum allowable MET-level activity for this patient? A.Washing dishes standing at a kitchen sink B.Mowing the lawn with a manual push mower C.Paying bills electronically with a computer

A.Washing dishes standing at a kitchen sink

Which of the following joint protection techniques should a client with rheumatoid arthritis use when completing kitchen tasks? A.Stir foods with weighted long-handled utensils. B.Transport items using a wheeled cart. C.Twist a jar lid open with the least affected hand. D.Grasp cookware with the fingertips.

B.Transport items using a wheeled cart.

A patient is seen in his home after having a middle cerebral artery stroke 6 weeks ago. The patient is modified independent in most ADLs and IADLs using assistive devices for grooming and ambulation. In order to resume gardening, which modification would be the most necessary? Adaptive strategies during graded outdoor gardening tasks Routine maintenance care from a professional gardener A raised garden box and planter Mobility skills training to get to the garden level

Adaptive strategies during graded outdoor gardening tasks

Which assistive device is the best to recommend for a patient who presents with significant limited hand function, affecting their ability to write, turn pages, and activate switches and controls? Typing aid Page turner Adjustable Head Pointer Sip and puff

Adjustable Head Pointer

A client is being evaluated for a standard wheelchair (K0001), but after the intake interview, the OT is concerned that Medicare will not reimburse the cost of the wheelchair. Which of the client's statements could make him ineligible? A. "I'm unable to ambulate safely with a cane or walker." B. "We don't have enough room to move the wheelchair between rooms in our home." C. "I'm unable to push myself up a ramp in a wheelchair." D. I'm unable to remove the armrest to transfer."

B. "We don't have enough room to move the wheelchair between rooms in our home." -home must allow access

A client was provided with an ultra lightweight wheelchair. Because the client is at risk for falls, the drop seat was tilted posteriorly to help prevent the client from getting out of the chair; also, because of the client's short stature, the axle was moved forward to make it easier for the client to propel the chair. What two accessories would the OT recommend to improve the client's safety while using the wheelchair? A. Solid rubber casters, antitippers B. Antitippers, brake lever extenders C. Brake lever extenders, flip-up footrests D. Flip-up footrests, antitippers

B. Antitippers, brake lever extenders -brake extenders bc they are short and can't reach brakes

The topic of the February 7, 2013, newsletter MLN Matters, published by the Medicare Learning Network under the Centers for Medicare and Medicaid Services, was related to changes in the criteria for mobility assistive equipment and mobility-related ADLs (MRADLs), which OT's working with positioning, seating, and wheelchairs need to know for reimbursement. Which of the following is NOT an MRADL specifically cited by Medicare? A. Feeding B. Cooking C. Bathing D. Grooming

B. Cooking -"toileting, feeding, dressing, grooming, or bathing."

A client is being considered for a power wheelchair, because her functional status has deteriorated and she is unable to move around her home. She currently has a wheelchair that her family purchased for her at a drug store. According to the Healthcare Common Procedure Coding System (HCPCS) for durable medical equipment, including wheelchairs and their accessories her current standard wheelchair would have which HCPCS code? A. K0005 B. K0001 C. K0002 D. K0007

B. K0001

A frail older adult client who uses an ultra lightweight manual wheelchair with a foam cushion comes to the clinic complaining of low back pain. The client's caregiver indicates that the skin on the client's coccyx is very pink, and there is concern about skin breakdown. The OT observes that the client's seated position is stable without supports, the client's upper extremities rest on the armrests without the shoulders hiking, the feet rest on the foot plates bilaterally, and the hips are at a 45° angle, which keeps the knees higher than waist level. The client indicates that this sitting posture is usual and is maintained most of the day. What intervention would the OT try INITIALLY to relieve the client's pain and protect the skin? A. Add an adjustable-tension back support so that the client can lean back further B. Lower the footrests so that the thighs are level with the seat of the chair and the feet rest flat on the footrests C. Replace the client's foam cushion with a gel cushion D. Analyze how pressure is distributed on the pelvis using pressure mapping

B. Lower the footrests so that the thighs are level with the seat of the chair and the feet rest flat on the footrests

Although fitted with an ultra lightweight wheelchair (K0005), a client with bilateral above-the-knee amputations is prone to losing sitting balance and falling forward when propelling the chair. The position of the axle is in front of the pelvis. The OT needs to find a balance between stability and the effectiveness of arm propulsion. Because the chair is a K0005, which solution will the OT MOST likely suggest? A. Move the axle forward so that the client's center of gravity is behind the axle B. Move the axle back so the client's center of gravity is over the axle and add antitippers C. Exchange the client's pushrims for ones that have knobs for easier pushing D. Make no changes to the axle but add antitippers to prevent falls

B. Move the axle back so the client's center of gravity is over the axle and add antitippers

An OT is developing a retirement planning protocol for a group of older adults at a community senior center, based on the transtheo retical model. Which activity would be MOST appropriate for individuals in the pre-contemplation stage? A. "Identify one life role you wish to continue after retirement." B. "List five potential changes you will face following retirement." C. "List the steps you need to take to accomplish one of your goals." D. "Choose one goal to address for this session."

B. "List five potential changes you will face following retirement." -develop an awareness of change -A. would come next (contemplation) -D. is preparation stage -C. is action phase

Which occupational therapy group would benefit MOST from a highly directive and authoritative leadership style? A. A group of preschool children exploring sensory experiences B. A group of adolescents attending a gang prevention group in an alternative education setting in which they have been placed for behavioral issues stemming from a diagnosis of oppositional defiant disorder C. A group of middle-aged clients in an outpatient day treatment setting who are learning to manage their depression following inpatient hospitalization D. A group of healthy older adults in an apartment complex exploring ways to stay fit and health

B. A group of adolescents attending a gang prevention group in an alternative education setting in which they have been placed for behavioral issues stemming from a diagnosis of oppositional defiant disorder -dont think too much about this one; teens need authority

A 21-month-old child with severe spastic quadriplegia is evaluated by an occupational therapist. The therapist determines that the child is cognitively intact, exhibiting age-appropriate cognitive skills despite major sensorimotor deficits. The therapist recommends a play activity to enhance these cognitive abilities and provide the child with a fun and pleasurable experience. Which is the best object for the therapist to recommend? A. A multi-colored mobile of objects of interest placed over the child's stroller. B. A mechanical toy with a chin controlled on/off switch. C. A shape sorter with foam squares, triangles, and circles. D. A battery controlled hammock swing.

B. A mechanical toy with a chin controlled on/off switch. -probably can't do C. and its a little too early

An OTR® has been working on increasing the concentration and attention span of an 8-year-old child with autism who is distractible during handwriting activities in the classroom. Using a sensory integration approach, which environment is the BEST to address concentration and attention skills? A. An isolated room free from any kind of distraction B. A small classroom with two other children and soothing music C. A small corner of a sensory gym where two other children are swinging D. A quiet corner in the child's classroom with a chalkboard divider

B. A small classroom with two other children and soothing music -in between isolation and actual classroom

A parent of a newborn infant has bilateral shoulder weakness and is referred to OT for training in energy conservation techniques for performance of parenting and home management tasks. Which adaptation(s) is/are most effective for the OT to recommend the parent use? A. A top-loading washer and drying for clothing care B. A steamer, steamer basket, and/or slow cooker for meal prep C. A front-pack carrier for holding infant D. Cloth diapers and use of weekly diaper care service

B. A steamer, steamer basket, and/or slow cooker for meal prep -slowcooker would eliminate lifting pots/pans -C. is wrong bc it is still on shoulders

An OTR® is working with a client with chronic obstructive pulmonary disease on therapeutic exercise and how to measure exercise tolerance. The client is performing seated bilateral shoulder flexion with 3-lb weights and rates this activity as "very, very easy." How would the OTR® appropriately grade this activity? A. Change the muscle groups used by switching to shoulder abduction. B. Advise the client to perform the task in a standing position. C. Switch the client to a resistance band. D. Increase the number of repetitions and have the client perform the exercises while standing.

B. Advise the client to perform the task in a standing position.

An OTR® is establishing reasonable accommodations for a client diagnosed with osteoarthritis in the left shoulder. The client is employed as a plumber. Essential job functions include reaching, bending, crawling, lifting, pushing and pulling, using tools, and lifting materials weighing 25-50 lb. Which accommodation is MOST appropriate for the OTR® to recommend? A. Provide the client with modified-handle tools to reduce torque on the shoulder joint. B. Allow the client a flexible work schedule and use of leave time as needed. C. Ensure materials and equipment are within functional reach. D. Recommend use of wheeled cart to transport tools.

B. Allow the client a flexible work schedule and use of leave time as needed. -reasonable accommodation -D. won't work for plumber is small spaces

An OTR® is working on sitting balance at the edge of the bed with a client who has had a CVA with residual left-sided hemiplegia. The client requires minimal assistance to maintain static sitting because of lateral leaning. How can the OTR® BEST adapt or modify this activity to improve sitting balance? A. Prop a wedge and pillows behind the client. B. Allow the client to bear weight on the left forearm on a small stool. C. Position the client's hips in more of an anterior pelvic tilt. D. Instruct the caregiver on proper hand placement to support the client.

B. Allow the client to bear weight on the left forearm on a small stool. -only answer provide lateral support

An OTR® is ordering a wheelchair for a client with bilateral transfemoral amputations. The client does not have prostheses and has no intention of using prosthetic limbs in the future. Based on this information, which design feature would be MOST BENEFICIAL to include as part of the wheelchair prescription? A. Backrest with vertical zipper B. Anti-tipping attachments C. 5-inch (8-cm) diameter pneumatic front casters D. Adjustable cushioned arm rests

B. Anti-tipping attachments -best for weight shifting since client can't balance feet on floor

A 4-year-old child with attention deficit hyperactivity disorder is using a tire swing for vestibular input to feel more organized. After a few minutes, the child appears to have some autonomic reactions to the fast swinging. Which course of action for the OTR® is BEST? A. Immediately ask the child to stop and sit in a corner to relax B. Ask the child to slow down the swinging by pulling a trapeze bar for proprioceptive input C. Ask the child to slow down while doing slow, deep breathing D. Let the child continue because the vestibular stimulation ceases with autonomic activation

B. Ask the child to slow down the swinging by pulling a trapeze bar for proprioceptive input -proprioception slows down vestibular

An OTR® continues to work with and charge for services with a client after all goals are met and there is no further need for occupational therapy services. This behavior is unethical. Which Occupational Therapy Code of Ethics (2015) principle best describes this situation? A. Nonmaleficence B. Beneficence C. Justice D. Veracity

B. Beneficence - not serving well-being of client -integrity to goals of treatment

An adolescent has a genetic condition that has resulted in significant deterioration in their motor, language and social skills. The adolescent frequently chews on hands and has open woulds that are infected and non-healing. Medication and behavior management techniques to decrease this behavior have been unsuccessful. The physician refers the adolescent to OT to determine a method to keep their hands from their mouth for several hours daily. Which should the therapist recommend? A. Candy on a stick to redirect attention to a different hand-to-mouth activity B. Bialteral soft elbow splints 30 degrees elbow flexion to be worn intermittently during the day. C. Thermoplastic resting splints to be worn at the times when the behaviors tend to increase. D. Soft leather gloves to be worn most of the day to deter chewing on the hands

B. Bialteral soft elbow splints 30 degrees elbow flexion to be worn intermittently during the day. -D. would increase it -A. is not long term -C. would decrease function

A client with advanced amyotrophic lateral sclerosis (ALS) presents with a stiff forward-flexed neck that is at risk for developing a contracture and impairing social participation. Which intervention would the OTR® use to BEST prevent a contracture? A. A physical agent modality such as diathermy or ultrasound B. Caregiver training on gentle neck stretching and how to help don a soft cervical collar C. A home exercise program of isometric neck exercises in all planes D. Education on proper positioning at rest and during functional tasks

B. Caregiver training on gentle neck stretching and how to help don a soft cervical collar -ALS is progressive and caregiver training is BEST -D. is good but neck is probably already too weak to be positioned on its own

An OTR® is evaluating a new client and notes that the client has a bluish tinge to the skin and lips. The client also presents with significant edema. Which medical condition is this client MOST likely to have? A. Emphysema B. Chronic bronchitis C. Cystic fibrosis D. Cirrhosis of the liver

B. Chronic bronchitis -B. Chronic bronchitis - poor oxygenation, resulting in a bluish A. Emphysema - pinkish skin C. Cystic fibrosis - yellowing of skin D. Cirrhosis of liver - orange tint to skin

An OTR® receives a referral to complete a job demand analysis on a client who is a diesel mechanic. At the job site, what should the OTR® do FIRST? A. Assess the design of the workstation. B. Conduct interviews with staff at the work site. C. Assess the client's material handling skills. D. Complete force gauge measurements on all equipment.

B. Conduct interviews with staff at the work site. -determines job demands -A and D are related to ergonomics, not job demands -C is related to FCE

The OT plans an intervention to address goals to engage in meaningful occupations with a pt that has degenerative joint disease and incurred a R hand injury and also has CRPS Type I (presents with has severe pain, stiffness, extreme temp changes in hand, pitting edema, and blotchy, shiny skin). Which PAM should the OT use in preparation for functional activity? A. TENS B. Contrast baths C. Whirlpool D. None; PAMS are contraindicated for this diagnosis

B. Contrast baths -good for blood flow and edema reduction -TENs may be too painful at first -whirlpool is wounds

A school-based occupational therapist receives an evaluation referral for a third-grader. The teacher reports that the student has illegible handwriting, poor attending behaviors, questionable visual skills, and problems with pencil management. After speaking with the teacher, reviewing classroom work samples, and reading the student's history, which action should the therapist take next? A. Provide pencil grips and specialized paper as a trial to determine interventions. B. Directly observe the student during a naturally occurring writing time. C. Administer a standardized visual perceptual and visual motor assessment. D. Administer a standardized handwriting assessment.

B. Directly observe the student during a naturally occurring writing time.

What is the most important safety recommendation for clients with middle-stage Alzheimer's disease (AD)? A. Do not allow this client to complete ADLs independently because the client might not dress appropriately for the weather. B. Do not leave this client alone because the client might get lost, even in a familiar environment. C. Provide this client with activity to engage the client throughout the day to minimize behavioral disturbances. D. Provide this client with redirection to minimize behavioral outbursts and prevent self-injurious behaviors.

B. Do not leave this client alone because the client might get lost, even in a familiar environment.

15 y/o with asymptomatic HIV attends outpatient clinic. OT protocol for persons diagnosed with HIV includes presentation of info on safe sex. The adolescent's parents refuse to allow this info to be presented to their child. Which is the best action for the OT to take in response? A. Ask the adolescent's opinion and act on their refusal or consent B. Document that the parents refuse the intervention for their child C. Refer the family to the social worker for counseling D. Have the parents sign a waiver that they refused the intervention for their child

B. Document that the parents refuse the intervention for their child -right to refuse treatment

An OT is using a visual perceptual frame of reference to plan an intervention for a child with visual perceptual problems. The FIRST activities planned should address which type of visual perceptual skills? A. Visual memory skills. B. Visual attention skills. C. General visual discrimination skills. D. Specific visual discrimination skills.

B. Visual attention skills.

An OTR is completing a self-care assessment with a resident in a long term care facility. Nursing staff referred the resident to OT after observing a decline in the resident's function following an exacerbation of COPD one month ago. Medical history is also significant for residual cerebellar symptoms secondary to an excision of a brain tumor 5 years ago. The client has Fair Plus (3+/5) functional strength of bilateral upper extremities. During the assessment, the OTR asks the resident to "pick up your shirt from the night stand." When reaching for the shirt, the resident has to stabilize the arm to prevent reaching beyond or short of the shirt. When documenting the session what term should the OTR use to BEST describe the resident's action? A. Decreased motor planning B. Dysmetria C. Dysdiadochokinesia

B. Dysmetria

Following evaluation of an elderly individ ual, an OT determines that the individual no longer leaves her bed primarily because of a fear of falling when moving from the bed to the wheelchair. What is the MOST important concept for the OT to remember when working with this individual? A. Teach the client to increase lower extremity strength. B. Educate the client to increase confidence in the area of functional transfers. C. Modify the environment to reduce safety risks. D. Reduce the use of medications that may be contributing to falls.

B. Educate the client to increase confidence in the area of functional transfers. -all the other options are related to increasing confidence -adaptions can be made for each context

An OTR® in an adult day treatment program decides to use a group intervention with clients with schizophrenia who live in a supported living environment with 24-hour supervision. The OTR believes that a sensorimotor approach would help the clients reach their goal to increase independence in overall function and goal-directed activity in the home context. What would be an appropriate group activity that uses the sensorimotor approach? A. Discussing task performance B. Exercising to music C. Using a sensory room D. Baking cookies

B. Exercising to music

An individual covered by Medicare who has been receiving OT and PT in the home is now able to transfer in and out of the car with supervision of a caregiver and visit friends 30 minutes away. OT services are still required to improve mobility, upper extremity function, and home management skills. Which of the fol lowing actions should the OT practitioner take FIRST? A. Provide a home program and discharge the individual. B. Explain to the individual and caregiver that one must be "homebound" in order to be eli gible for home care services. C. Refer the individual for outpatient therapy and provide a comprehensive discharge sum mary to the outpatient setting. D. Inform the PT of the individual's status.

B. Explain to the individual and caregiver that one must be "homebound" in order to be eli gible for home care services. -talk to PT after

When fabricating an orthotic device for an individual with swan-neck deformities, the or thotic should be designed to prevent what positions? A. Hyperextension of the PIP and DIP joints. B. Hyperextension of the PIP joint and flexion of the DIP joint. C. Flexion of the PIP joint and hyperextension of the DIP joint. D. Hyperextension of the MP joint and flexion of the PIP joint.

B. Hyperextension of the PIP joint and flexion of the DIP joint.

The OT observes that an individual's brain injury causes difficulty using a bus schedule to get to work on time. What would be the FIRST step toward developing this skill, using a problem-solving training approach? A. Generate as many solutions as possible for the problem. B. Identify using a bus schedule as a problem area. C. Weigh the pros and cons of various solutions. D. Recognize faulty solutions, self-correct errors, and develop alternate hypotheses.

B. Identify using a bus schedule as a problem area. -FIRST step

The parents of an infant with severe osteogenesis imperfecta ask the OT the best way to put on the child's diaper. Which recommendation should the therapist make for positioning the child during this activity? A. Sitting on the parent's lap, supported the parent's stomach and place the diaper between the child's legs. B. In supine, and move the child gently side to side while supporting the child's hip, and neck to place the diaper under the child's hips. C. In supine, and gently grasp the child by the ankles to lift up the hips and place the diaper under the child. D. Cradle the child in one arm, and place the diaper under the child's hips with the other hand.

B. In supine, and move the child gently side to side while supporting the child's hip, and neck to place the diaper under the child's hips.

An occupational therapist is planning discharge for a toddler with a brachial plexus injury. Which should the therapist teach the parents to do in the home program the therapist develops with them? A. Mirror the intervention provided by the therapist B. Incorporate proper positioning into activities of daily living C. Include new treatments before they are initiated by the home care therapist

B. Incorporate proper positioning into activities of daily living

A client who had a recent CVA would like to return to work on an assembly line at a manufacturing plant. An OTR® preparing to provide job coaching to this client should review which document FIRST? A. Functional capacity evaluation B. Job analysis for essential functions C. Performance reviews of worker D. ADL and IADL checklist results

B. Job analysis for essential functions

An OTR is evaluating the effectiveness of an OT home program for clients who have been diagnosed with an anxiety disorder. Which qualitative measure would be MOST BENEFICIAL to use as part of this evaluation process? A. Retrospective record reviews to measure the percentage of clients who attained occupation-based goals B. Occupational role checklists administered at the start and then several months after completion of the program C. Standardized, norm-referenced or criterion-referenced occupation-based assessments

B. Occupational role checklists administered at the start and then several months after completion of the program -best because it is client specific

A client who has cardiac disease is participating in outpatient occupational therapy and is able to complete activities in the 1-3 metabolic equivalent (MET) range. Dyspnea and angina limit physical activity above 5 METs. Which activity is safe to include as part of the initial intervention for this client? A. Completing self-care tasks including dressing, bathing, grooming, and hygiene for 5-minute intervals each with 1-2 minutes rest between activities B. Participating in very light stationary biking for 5 minutes with a short rest of 1 minute and then standing to pack moderate weight items into boxes for 5 minutes C. Pedaling a stationary bicycle for several 5-minute intervals followed by a 1- to 2-minute rest between interval and continuing with 5-minute intervals until fatigued D. Walking on a treadmill at 10 miles per hour (16 km per hour) for 5 minutes followed by a 1- to 2-minute rest, then walking on treadmill at 5 miles per hour (8 km per hour)

B. Participating in very light stationary biking for 5 minutes with a short rest of 1 minute and then standing to pack moderate weight items into boxes for 5 minutes -A. is for MET 1.0-2.5 - others don't involve functional activity

An OT is using a sensory integration ap proach with a group of regressed residents on an inpatient unit who display a very low en ergy level, hyposensitivity to stimuli, and poor visual and tactile perception. What is the BEST activity for beginning a session focusing on identifying pleasant memories? A. Go around the circle and ask each patient to introduce himself or herself. B. Pass around a scent box and ask each patient to smell the contents. C. Ask each patient to select a favorite poem and read it. D. Discuss the lunch menu and healthy eating habits.

B. Pass around a scent box and ask each patient to smell the contents.

An OTR® is reviewing a chart on an infant who was referred to the developmental follow-up clinic. The OTR® notes that the mother transmitted syphilis to the baby during birth. Which impairments might be seen in the child? A. Poor bladder control B. Poor tolerance of passive range of motion C. Poor balance D. Poor feeding and vomiting

B. Poor tolerance of passive range of motion -Congenital syphilis may result in osteochondritis at the joints, other bone anomalies, dental anomalies, and visual and auditory deficits. PROM may be painful.

An OTR completed a screening with an older adult inpatient who had surgery for an abdominal hernia repair 2 days ago. When interpreting the results of the screening, the OTR learns the patient has hearing loss that was not noted during the patient's intake procedures. What influence would undetected hearing loss have on the patient's occupational performance when completing a standardized assessment? A.Impaired sequencing skills during ADL B.Difficulty following verbal test directions C.Easily distracted by ambient auditory stimuli

B.Difficulty following verbal test directions

An order came in for a hand splint for a new client. A newly graduated OTR® evaluated the client for the splint. The new OTR® had not made a splint on a client before and requested an experienced OTR® hand therapist's assistance. In doing so, the OTR® abided by the ethical principle that involves taking "responsibility for maintaining high standards and continuing competence?" Which principle is that? A. Nonmaleficence B. Procedural Justice C. Prudence D. Veracity

B. Procedural Justice

he OT is working individually with a cli ent who has demonstrated anger and aggres sion on several occasions. When the OT ob serves the client becoming increasingly restless, then yelling, what is the FIRST action the OT should take? A. Remove himself/herself from physical prox imity of the client altogether. B. Provide the client with the choice to talk about it or end the session. C. Call for help to escort the client to the emer gency room. D. Leave the door to the treatment room open

B. Provide the client with the choice to talk about it or end the session. -deesclate at this point -D. should be done prior to session -A. and C. are if situation escalates

A child with a sensory processing disorder and Level 1 autism spectrum disorder without intellectual impairment (Asperger syndrome) is attending a regular classroom. Which reasonable accommodation would be BEST for this child under the Individuals With Disabilities Education Improvement Act? A. Place the student's desk at the back of the classroom B. Reduce the number of math problems to one per page C. Increase the number and variety of spontaneous activities in the school day D. Increase the number of other students at the student's lunch table

B. Reduce the number of math problems to one per page -focusing on sensory issues: reduces clutter

A client sustained a non-displaced distal radius fracture 8 weeks ago. The OTR receives a consult from the client's physician to initiate ultrasound and therapeutic exercise. The state regulatory board allows occupational therapists to use physical agent modalities (PAM), however, the OTR assigned to the client has not had PAM training. How should the OTR respond to this referral? A. Follow the protocol for ultrasound as outlined in the instruction manual. B. Refer the client to an occupational therapist who is experienced in ultrasound. C. Apply moist heat to the affected hand and wrist instead of ultrasound.

B. Refer the client to an occupational therapist who is experienced in ultrasound.

An adolescent with myelomeningocele participates in an OT session in a private practice setting. The client reports experiencing intermittent headaches, decreased UE strength and shortened attention span during the past week. Which is the therapist's best response? A. Discuss typical adolescent behavioral and physical changes with the client B. Report these symptoms of possible shunt malfunction to the physician C. Report these symptoms of possible tethered cord to the physician D. Suggest that the client talk to the school psychologist about these behaviors

B. Report these symptoms of possible shunt malfunction to the physician

A client has constructional disorder secondary to an excision of a brain tumor. One of the client's goals is to resume work in a garden center. During which of the following gardening activities would this deficit be MOST evident? A.Determining how much water to use when watering plants B.Duplicating garden designs based on a magazine picture C.Distinguishing between weeds and flowers in a garden bed D.Placing seeds in small containers filled with potting soil

B.Duplicating garden designs based on a magazine picture

An individual with a below-elbow amputa tion lacks sensation in the residual limb. What is the MOST appropriate intervention for the OT to teach the patient? A. Techniques of tapping, rubbing, and applica tion of textures to the residual limb. B. Routinely inspect the skin closely for signs of skin breakdown. C. Deeply massage the residual limb. D. Perform proper skin hygiene.

B. Routinely inspect the skin closely for signs of skin breakdown. -use vision to compensate for sensation -D. is important but dosnt address lack of sensation

During a meal preparation evaluation, an individual with a brain injury repeatedly cuts up the vegetables before washing them and turns on the stove before filling up the tea ket tle with water. Which of the following activities MOST appropriately addresses the deficit area demonstrated? A. Using rubber stamps in a random design to create notecards. B. Stringing beads for a necklace, following a pattern. C. Putting together a 100-piece puzzle and then framing it. D. Setting a table with dishes and utensils for four people.

B. Stringing beads for a necklace, following a pattern.

During an early intervention planning meeting, an occupational therapist explains the results of a play assessment to the parents of an 18-month-old toddler with multiple developmental disabilities. The child has been assessed as delayed by 6-8 months in all developmental parameters. Which intervention approach is most effective for the therapist to recommend using first to develop the toddler's play skills? A. The use of toys that encourage creative and imaginative play. B. The use of toys that are visually and auditorily stimulating. C. Participation in a small parallel play group with other toddlers. D. Engagement in activities that use sensorimotor skills prerequisite to play.

B. The use of toys that are visually and auditorily stimulating.

A 75-year-old client presents to an OTR® after a distal radius fracture 2 weeks earlier. The surgeon used a primary healing technique. What should the OTR focus on FIRST? A. Therapy must focus on edema reduction, because the client is still in a cast. B. Therapy can begin with passive and active range of motion (PROM and AROM), because the client's bone is fixed with a plate. C. Therapy should not begin for another 2 weeks, because the client is elderly and the client's bones are probably brittle. D. Therapy can be aggressive and include strengthening from the first day of therapy.

B. Therapy can begin with passive and active range of motion (PROM and AROM), because the client's bone is fixed with a plate. -primary healing = ORIF - stable and fixed so can being moving as long as therapy is allowed by doc

An occupational therapy professional education program provides an after-school play program for typically developing children to help students understand typical development. The students observe a child who is beginning to use blunt scissors to snip paper. The child opens and closes the scissors and moves them in a controlled forward motion, but the child cannot cut circles or figure shapes. At which age are these behaviors typical? A. Two years old. B. Three years old. C. Four years old. D. Five years old.

B. Three years old.

A child with ataxic CP attends OT twice weekly and has met all ADL goals. The current goal is for the child to become involved in community activities. Which is the best intervention strategy for goal attainment? A. Treat the child twice weekly while locating community resources B. Treat the child once per week for issues relating to activity selection C. Give the family a home program and ask them to call if they have any difficulties D. Discharge the child from therapy so that the family will have time to pursue community activitiies

B. Treat the child once per week for issues relating to activity selection

6 y/o has thumb weakness that is most evident in the child's poor ability to perform thumb opposition. During eval, which activity will the therapist most likely observe the child having difficulty performing? A. Rolling a piece of clay into a ball B. Turning a pencil over to erase C. Picking up coins D. Separating two pieces of paper

B. Turning a pencil over to erase -A. and D. are shifting -C. is finger to palm

An older adult expresses concerns about the ability to perform daily tasks. The occupational therapist assesses that the individual has somatosensory deficits consistent with the normal aging process. The therapist recommends adaptive equipment to assist with task performance. Which adaptive equipment should the therapist recommend the person use during meal preparation and feeding? A. Utensils with narrow, smooth grips. B. Utensils with wide, textured grips. C. A rocker knife and spork

B. Utensils with wide, textured grips.

An OTR is working with a client who has sustained injuries in a knife attack. The injury to the flexor tendon is in what is known as "no man's land." The stitches in the fingers are between the distal palmar crease and the proximal interphalangeal joints. In what flexor tendon zone are the injuries located? A. Zone I B. Zone II C. Zone III D. Zone IV

B. Zone II

In preparing a patient with a unilateral below-knee amputation for discharge from a re habilitation facility, the MOST important adap tive equipment for the OT practitioner to recommend is: A. lightweight cooking utensils. B. a tub bench and toilet rails. C. long-handled dressing devices. D. a reacher.

B. a tub bench and toilet rails.

A client is a hands-dependent sitter. What goals would the OT have for such clients when providing a positioning and seating system? A. Support the upper extremity and hands for activities B. Support the pelvis and trunk to free the hands for activities C. Provide armrests to support the arms and hands for activities D. Provide a lapboard to support the client and free the hands for activities

B. support the pelvis and trunk to free the hands for activities - Hands-dependent sitters require the use of one or both hands to maintain an upright seated position. Positioning must begin with the pelvis and trunk; if they are supported, the client will not need to use the hands to sit upright

An OTR is using the Modified Ashworth Scale to measure the muscle tone of an inpatient who has hemiplegia. The patient has full passive ROM, no increase in muscle tone, and no active ROM of the affected upper extremity. Which score on the Modified Ashworth Scale is MOST CONSISTENT with this response? A.2 - marked increase in tone through the entire ROM with full passive ROM B.0 - flaccid C.4 - rigid in flexion and extension

B.0 - flaccid

An OTR is providing ramp specifications for a client with a disability who uses a power wheelchair for mobility. What is the minimum gradient for the ramp that the OTR should recommend to this client? A.1:10 B.1:12 C.1:6

B.1:12

At what age does an infant develop a radial digital grasp in which the thumb opposes the index finger and the middle finger? A.19 - 24 months B.6 - 12 months c. 13-18 months

B.6 - 12 months - pick up block

Which of the following options represent a condition that requires close monitoring of a client's wound due to having the GREATEST risk for infection? A.Vascular ulcer left uncovered during healing B.Abdominal surgical wound left open to heal C.Traumatic laceration immediately closed with clips for healing

B.Abdominal surgical wound left open to heal

An OTR who works in a hospital setting is evaluating an inpatient who recently had a severe stroke. The patient currently requires a ventilator for respiratory support and is not able to verbalize goals. To complete the evaluation process, what ethical obligation does the OTR have for upholding professional responsibilities? A.Conclude the patient is not eligible for services until medically stable. B.Acquire information for the occupational profile from other sources. C.Document that the patient has poor potential for rehabilitation.

B.Acquire information for the occupational profile from other sources.

An outpatient client is undergoing medical treatment for breast cancer and has debilitating chemotherapy-induced peripheral neuropathy of the hands and feet. The client wants to attend social events with friends, but feels distressed over a fear of falling and frequently dropping objects. In addition to providing fall prevention strategies and recommending adaptive equipment, which intervention priority would be MOST BENEFICIAL for the OTR to include in sessions to support progress toward this goal? A.Recommend a yoga class at a community center. B.Address the client's psychosocial well-being. C.Teach the client to manage symptoms of neurological pain.

B.Address the client's psychosocial well-being. -fear of falling

An older adult client, who is recovering from pneumonia and sepsis, has been participating in home health OT since being discharged from an acute care hospital 3 weeks ago. During a transfer training session in the client's bedroom, the OTR notes the smell of urine and evidence of incontinence. What is the FIRST action the OTR should take based on this observation? A.Establish a new short-term goal for independence in toileting. B.Ask the client if it is difficult to get to the bathroom in time. C.Plan to discretely discuss the findings with the client's spouse.

B.Ask the client if it is difficult to get to the bathroom in time.

An OTR working in a home health setting is collaborating with a client who has mild dementia to establish client-centered goals and to obtain informed consent for the intervention plan. Which action should the OTR complete NEXT as part of the informed consent process for this client? A.Request that the client's caregiver review the care plan. B.Assess the client's decision-making skills. C.Interview the client to determine cognitive capacity.

B.Assess the client's decision-making skills.

An OTR at a long-term care facility sees that a colleague has left a resident, who has Stage III Alzheimer's disease, unattended during a cooking activity. What is the FIRSTaction the OTR should take? A.Locate the supervisor and report the incident. B.Attend to the resident until the treating OTR returns. C.Assist the resident in returning to the room.

B.Attend to the resident until the treating OTR returns.

An OTR, who works in medical oncology, plans to have an inpatient walk to the bathroom and complete all steps for showering. Prior to the session, the OTR reviewed the medical record and noted that the international normalized ratio (INR) value for the patient is greater than 5.0. What action should the OTR take based on this INR value? A.Have the patient walk to the sink for a sponge bath. B.Cancel the intervention session planned for the day. C.Proceed with the showering activity as planned.

B.Cancel the intervention session planned for the day. - INR>5 is at risk for bleeding and on bedrest

An OTR, who works in an outpatient pediatric clinic, is preparing to fabricate an orthosis for a 4-year-old child who has cerebral palsy and a wrist flexion contracture. The child is crying and is visibly distressed in anticipation of the pending procedure. What action should the OTR take prior to starting the fabrication procedure to provide a positive experience for the child? A.Play with the child in an adjoining therapy space near the hand therapy room. B.Collaborate with the child to fabricate an orthosis for a doll or stuffed animal. C.Teach the child each step associated with fabricating the forearm-based orthosis.

B.Collaborate with the child to fabricate an orthosis for a doll or stuffed animal.

client who has severe depression has been participating in a partial hospitalization program. The client has made some improvements, but continues to have difficulties concentrating on tasks and coping with day-to-day stressors. The interprofessional team agrees that the client has made sufficient progress to transition to the next level of care. Which type of program would BEST assist the client toward the goal of returning to work as a healthcare provider? A.Impaired provider program B.Community mental health program C.Support group for working professionals

B.Community mental health program -good b/c they continue to have issues with process skills and mood

A 2-year-old toddler, who has a developmental delay, recently learned to release a 2-inch (5.08 cm) ball into a large toy box. What fine motor skill should the OTR work on NEXT? A.Color within the lines on a page. B.Complete a 3- to 4-piece puzzle. C.Transfer toys from hand to hand.

B.Complete a 3- to 4-piece puzzle.

A client in an outpatient setting has early relapsing-remitting multiple sclerosis. The client lives at home with a spouse and two adolescent children. The client wants to remain independent with homemaking tasks, but finds these daily routines are physically exhausting. What INITIAL action should the OTR take to acquire more information related to the client's priorities? A.Administer standardized assessments of client factors. B.Complete a client-centered occupational profile. C.Collaborate with the family regarding typical occupational roles.

B.Complete a client-centered occupational profile.

An OTR is evaluating an inpatient diagnosed with a glioblastoma multiform in the right frontal lobe. The patient has an extensive fund of knowledge and intact long-term memory, but does not understand the meaning of a joke when engaged in social conversation with family members. Which option BEST describes this behavior? A.Flight of ideas B.Concrete thinking C.Confabulation

B.Concrete thinking

A client sustained a closed fracture of the humeral shaft 6 weeks ago. The physician refers the client to OT with a consult that reads: "Begin elbow and shoulder ROM". An initial screening of the affected upper extremity indicates the client has elbow and shoulder stiffness and mild swelling of the hand. The client has full active flexion and full passive extension and flexion of the wrist and digits. Active extension of the wrist and digits is absent. What INITIAL action should the OTR take based on these findings? A.Complete a comprehensive manual muscle test of the affected upper extremity. B.Confirm whether the client has a secondary radial nerve injury. C.Fabricate a dynamic orthotic to compensate for loss of finger extension.

B.Confirm whether the client has a secondary radial nerve injury. -The OTR should contact the physician to confirm this diagnosis and to clarify the consult before proceeding with rehabilitation for the hand.

An inpatient had a myocardial infarction 4 days ago. While sitting on a chair at the sink during a bathing session, the patient's systolic blood pressure increases by 10mmHg above the patient's resting blood pressure. What INITIAL action should the OTR take based on this observation? A.Discontinue the activity and help the patient transfer to a supine position in bed. B.Continue to monitor vital signs and proceed with the planned intervention session. C.Document this response and other clinical observations in the client's medical record.

B.Continue to monitor vital signs and proceed with the planned intervention session.

A patient with a borderline personality disorder was admitted to an inpatient facility 4 days ago secondary to an exacerbation of suicidal and self-mutilating behavior. The patient's condition is now stable and the patient is functioning at Allen Cognitive Level 5 (Exploratory Actions). The patient reports being overwhelmed by a new personal relationship, experiencing job dissatisfaction, and feeling a lack of control in most daily situations. Which intervention would be MOST BENEFICIAL for addressing problems in performance skills and patterns secondary to the concurrent symptoms? A.One-on-one sessions to encourage the patient to contract for safety B.Coping skills groups that address a variety of adaptive strategies C.Structured one-step craft activities to promote successful outcomes D.Daily self-care sessions that focus on structured BADL

B.Coping skills groups that address a variety of adaptive strategies -Patients functioning at this cognitive level typically are able to use problem solving and inductive reasoning. This type of group provides the patient with opportunities to learn adaptive coping strategies that can be used in a variety of situations relative to the patient's typical performance skills and patterns.

Which handwriting skill emerges between 4 - 5 years of age in a TYPICALLY developing child? A.Imitating a vertical line on a paper B.Copying some letters and numerals C.Printing the letters in their own name

B.Copying some letters and numerals

An OTR is using the Modified Ashworth Scale (MAS) to monitor a client's degree of spasticity in response to a newly implemented medication regimen. At the start of the regimen, the OTR rated the client's wrist and finger spasticity on the MAS as grade 3. Two weeks after the regimen, the rating changed to grade 1. What does this change represent? A.Positive change in muscle tone resulting in increased functional use of the affected upper extremity B.Decrease in overall muscle tone when the affected wrist and digits are passively moved into flexion and extension C.Slight change in muscle tone through most of the ROM, but negligible change in active use of the wrist or hand

B.Decrease in overall muscle tone when the affected wrist and digits are passively moved into flexion and extension

An OTR is using universal design for learning (UDL) principles to make recommendations to optimize academic performance for students in a middle school. Using the principles of UDL, what action should the OTR take to meet this objective? A.Provide environmental design principles to adapt the playground to include students who have a disability. B.Determine the needs of the students and the influence of the educational environment on learning outcomes. C.Recommend desk heights and ergonomic computer workstations that benefit most students in the classroom.

B.Determine the needs of the students and the influence of the educational environment on learning outcomes.

A young adult client is participating in a community-based OT program after completing inpatient treatment for an acute episode of major depression. Evaluation results indicate the client has difficulty concentrating on simple tasks, has poor personal hygiene, and has limited insight about the impact of the depression on areas of occupation. The client states the primary goal for attending OT is "to get a job". What should be the INITIAL focus of sessions with this client? A.Finding the client a transitional job involving routine and repetitive work tasks B.Determining the client's work habits and current abilities for job readiness C.Assigning the client to a job in a highly supervised sheltered work environment D.Teaching the client how to locate job opportunities and submit job applications

B.Determining the client's work habits and current abilities for job readiness

An OTR is developing an intervention plan for a 4-year-old child with pervasive developmental delay. The child demonstrates impaired ideation and has limited play skills. A primary goal is for the child to successfully interact with age-appropriate toys. Which strategy would be MOST BENEFICIAL to include in the intervention plan to support progress toward this goal? A.Encourage the child to select a toy that facilitates pretend play as a favorite superhero. B.Develop a social story with pictures of a peer using appropriate skills to play with the toy. C.Provide frequent verbal guidance while the child plays with a preferred toy.

B.Develop a social story with pictures of a peer using appropriate skills to play with the toy. -pervasive developmental delay (i.e. autism) does not know how to engage in play and social story will teach

A hospital-wide interprofessional continuous quality improvement (CQI) committee has identified reduction in pressure sores related to hospitalizations as an opportunity for improvement. After discussing the initiative with the rehabilitation manager, what should the OTR do NEXT to contribute to this process? A.Explain to all rehabilitation staff the Department of Health and Human Services (DHHS) initiative to decrease the incidence of pressure sores related to hospitalizations. B.Develop a survey for the rehabilitation staff to determine the frequency of assessment and provision of interventions related to wound management and care. C.Develop an in-service education program for all rehabilitation staff including information about pressure sores being preventable and a major concern of health care.

B.Develop a survey for the rehabilitation staff to determine the frequency of assessment and provision of interventions related to wound management and care.

An OTR is designing a process improvement plan to reduce the use of patient restraints in a skilled nursing facility. What is the INITIAL step toward promoting corrective actions? A.Use incident reports to track staff members who most frequently use restraints. B.Develop procedures for tracking and monitoring restraint use within the facility. C.Survey other skilled nursing facilities to identify typical reasons for use of patient restraints. D.Implement an operating procedure for the correct application of patient restraints.

B.Develop procedures for tracking and monitoring restraint use within the facility.

A client has constructional disorder secondary to an excision of a brain tumor. One of the client's goals is to resume work in a garden center. During which of the following gardening activities would this deficit be MOST evident? A.Distinguishing between weeds and flowers in a garden bed B.Duplicating garden designs based on a magazine picture C.Placing seeds in small containers filled with potting soil D.Determining how much water to use when watering plants

B.Duplicating garden designs based on a magazine picture

A resident, who lives in a nursing home, has schizophrenia and tardive dyskinesia. The OTR is using the Abnormal Involuntary Movement scale (AIMS) to measure uncontrollable movements of the resident's face and upper body. At what frequency should the OTR administer the AIMS for this resident? A.Prior to each of the resident's medical appointments. B.During every intervention session with the resident. C.After a change in the resident's medication regimen.

B.During every intervention session with the resident. - anti-psychotic medication should be measured every session

In which pediatric practice setting does the OTR collaboratively assess the child using a family-centered philosophy that considers the concerns, priorities, and strengths of the family throughout the evaluation process? A.Outpatient clinic B.Early intervention C.School based

B.Early intervention

An OTR fabricated an immobilization orthosis for a 4-year-old child who had a syndactyly release. The purpose of the orthosis is to protect the skin grafts, maintain the interdigital spaces, and keep the fingers properly positioned. What action should the OTR take as part of the fabrication procedure to proactively support a positive orthotic-wearing experience for this child? A.Ensure the orthosis is wrapped snuggly on the hand with a brightly colored elasticized wrap. B.Engage the child in an activity to make a sock puppet to wear over the orthosis. C.Explain the purpose of the orthosis to the child using age-appropriate language.

B.Engage the child in an activity to make a sock puppet to wear over the orthosis. -always pick play with kids: functional

Which intervention activity would be MOST BENEFICIAL to use with an inpatient who has depression and poor initiation? A.Recommend that the patient attend a planned exercise program with peers. B.Engage the patient in a 1:1 intervention activity involving a previously enjoyed leisure activity. C.Have the patient independently navigate the facility to locate spaces used for social activities.

B.Engage the patient in a 1:1 intervention activity involving a previously enjoyed leisure activity.

An OTR has been leading discussions about evidence-based practice during monthly rehabilitation team meetings. The discussion at the next meeting will focus on reviewing the final step associated with this systematic process of searching and appraising literature. What information should the OTR include in the presentation when summarizing the final step in the evidence-based practice process? A.Applying best evidence to guide practice decisions B.Evaluating the effectiveness of each step in the process C.Appraising and synthesizing the evidence

B.Evaluating the effectiveness of each step in the process

An OTR is establishing a pediatric private practice to provide comprehensive in-home services. According to Part C of the Individuals with Disabilities Education Improvement Act of 2004, what MUST the OTR include as part of each child's intervention? A.Child-focused goals emphasizing curricular standards for successful transition to school B.Evidence of family-centered treatment sessions in the child's natural environment C.Individualized family service plan based on the child's special education goals

B.Evidence of family-centered treatment sessions in the child's natural environment -Part C of the Individuals with Disabilities Education Act (IDEA) mandates that early intervention services occur in the child's natural environment and be family-centered.`

A client had an open reduction external fixation of a distal radius fracture several days ago. Evaluation results indicate moderate swelling of the hand, decreased active ROM of the digits, and protective posturing of the involved arm close to the chest at all times. Which intervention would be MOST BENEFICIAL to include in sessions during this initial phase of the client's recovery? A.Use of a dry whirlpool modality to manage edema of the affected hand B.Exercises to promote capsular gliding and ROM of the shoulder of the affected arm C.Education about proper positioning in a standard pouch sling to minimize swelling D.Static splinting to prevent MCP joint collateral ligament tightness

B.Exercises to promote capsular gliding and ROM of the shoulder of the affected arm

An OTR, who works in a home health setting, is using a laptop computer to document assessment results during an initial evaluation at the client's home. The client asks to see the information the OTR is entering into the computer. To be in compliance with the Health Insurance Portability and Accountability Act (HIPAA), how should the OTR respond to this request? A.Provide the client with a printed copy of the electronic medical record, and answer any specific questions. B.Explain to the client their right to read and request a written copy of the occupational therapy evaluation note. C.Give the client a verbal summary of the evaluation results, and collaboratively establish client-centered goals.

B.Explain to the client their right to read and request a written copy of the occupational therapy evaluation note.

An OTR is assessing the reflexes of a 4-month-old infant. The OTR places the infant in a sitting position and encourages the infant to actively flex the neck forward to look at an object held near the infant's chest. Which of the following responses to this movement indicates the presence of the symmetrical tonic neck reflex? A.Flexion of the lower extremities and extension of the upper extremities B.Flexion of the upper extremities and extension of the lower extremities C.Flexion in both the upper and lower extremities

B.Flexion of the upper extremities and extension of the lower extremities

An OTR working in a children's hospital has been providing services on a daily basis for the past several weeks to a school-age child hospitalized for treatment of a systemic congenital condition. When the OTR arrives at the child's room for the next scheduled session, there is a sign posted on the door indicating enteric contact precautions are in place. What action MUST the OTR take based on this observation? A.Rub hands with an alcohol-based hand sanitizer at the end of sessions with the child. B.Follow transmission precaution procedures when interacting with the child. C.Cancel sessions with the child until these precautions are no longer in place.

B.Follow transmission precaution procedures when interacting with the child.

An older adult in an inpatient setting has moderate-severe debilitation from prolonged bed rest secondary to general medical-surgical post-operative complications. The patient's primary goal is to be as independent as possible with BADL prior to discharge home. The patient has full passive ROM and Fair minus (3-/5) functional muscle strength of the upper extremities. The patient can ambulate for several feet using a walker and contact guard assistance, but uses a wheelchair in the hospital room and depends on caregivers for wheelchair transport to various areas of the hospital. Based on the patient's current status, which dressing activity would be MOST difficult for this patient to complete while seated in the wheelchair? A.Crossing one leg over the other and putting on loose-fitting slip-on shoes B.Getting a pair of pants hanging in the closet and putting them on C.Putting on a front-opening shirt after reaching for the shirt off a bedside stand D.Washing hands at a sink and drying the hands using a towel placed next to the sink

B.Getting a pair of pants hanging in the closet and putting them on

An OTR, who works in a hospital setting, is working with a patient who has the herpes zoster virus and is on contact and airborne precautions. Which of the following options represent the MOST EFFECTIVE sequence for putting on personal protective equipment at the start of the session? A.Mask, gloves, gown B.Gown, mask, gloves C.Gloves, mask, gown

B.Gown, mask, gloves

A resident of a skilled nursing facility has Wernicke's aphasia secondary to a left CVA and is frustrated during intervention sessions. Which communication techniques should the OTR use to limit the resident's frustration? A.Sign language and written instructions B.Hand gestures and tactile cues C.Alphabet and low tech communication boards

B.Hand gestures and tactile cues

An inpatient had a TBI one month ago and is functioning at Level VII (Automatic-appropriate) on the Rancho Los Amigos scale. Currently, the patient is able to follow two-step instructions and attends to a familiar task for up to 15 minutes at a time. The patient wants to return home to resume homemaking roles. One of the patient's short-term goals is to independently bake cookies for a family member's upcoming birthday. How should the activity be graded to support the patient's successful participation in this task? A.Have the patient prepare cookies using a boxed cookie mix with pre-measured ingredients. B.Have the patient prepare cookies using slice and bake packaged cookie dough. C.Mix the ingredients together and have the patient drop the cookies onto the cookie sheet. D.Provide the patient with a recipe of ingredients for mixing and baking cookies.

B.Have the patient prepare cookies using slice and bake packaged cookie dough.

A client who has hemiplegia has a subluxation of the affected shoulder. Which intervention is CONTRAINDICATED for this client? A.Teaching the client self range of motion within a limited arc of motion B.Having the client use wall-mounted reciprocal pulleys for maintaining ROM C.Applying electrical stimulation to the posterior deltoid of the affected arm D.Engaging the client in bilateral upper extremity weight-bearing activities

B.Having the client use wall-mounted reciprocal pulleys for maintaining ROM -Use of overhead pulleys and aggressive PROM is contraindicated

Which option represents the MOST EFFECTIVE method to screen a 5-month-old infant for automatic head righting reaction? A.Place the child in a supine position and observe body movements while rotating the child's head to one side and then to the other. B.Hold the child in vertical suspension and observe head position while gently tilting the child from side to side then from front to back. C.Place the child in a sitting position and observe for arm extension while gently pushing the child off balance to the front, back, and each side.

B.Hold the child in vertical suspension and observe head position while gently tilting the child from side to side then from front to back.

A 6-year-old child, who has immature postural balance and gravitational insecurity, wants to be able to climb a cargo net with peers. To support progress toward this goal, the OTR initially provides physical support as the child plays a board game while sitting on a therapy ball. Once this skill is mastered, what action should the OTR take NEXT to progressively grade the intervention plan? A.Ask the child to play with peers who are sitting on the swings during recess. B.Hold the child's hand when the child is walking on an uneven surface. C.Encourage the child to walk upstairs while holding on to a hand rail.

B.Hold the child's hand when the child is walking on an uneven surface. -sitting w/ support to walking w/ support

An OTR working in an acute care setting is contributing to the discharge recommendations for an older adult inpatient. The patient has mild cognitive impairment and was admitted to the hospital 4 days ago due to a syncopal event with a diagnosis of atrial fibrillation and hypertension. The patient lives with family who are available to provide care as needed, but the patient wants to be independent in ADL. Currently, the patient completes BADL with supervision, but occasionally relies on physical assistance from a caregiver to stand up from the toilet. Which discharge setting would be MOST BENEFICIAL for the OTR to recommend for the patient's next level of care? A.Inpatient rehabilitation B.Home with home health services C.Skilled nursing facility

B.Home with home health services

An inpatient is undergoing treatment in an acute rehabilitation facility after sustaining bilateral ankle fractures one week ago. Medical records indicate the patient will be non-weight-bearing for at least 6 weeks. Currently, the patient is independent with upper body dressing; requires moderate assistance with lower body dressing, bathing, and transfers; and independently propels a standard wheelchair. One of the intervention priorities is for the patient to be independent with IADL prior to discharge. Which action is MOST IMPORTANT for the OTR to take in advance of scheduling intervention activities for this patient? A.Ensure the hospital bathroom is equipped with durable medical equipment. B.Identify the patient's expected discharge context and available resources. C.Discuss the projected timeline for recovery with the interprofessional team.

B.Identify the patient's expected discharge context and available resources.

A client is developing pitting edema of the hand secondary to flaccid hemiplegia. What should the OTR teach the client and caregiver as part of the INITIALintervention for managing this client's edema? A.Methods for using elasticized compression wraps for the digits and hand B.Importance of proper upper extremity positioning and elevation C.Procedures for providing retrograde massage of the upper extremity D.Techniques for completing passive ROM exercises of the digits

B.Importance of proper upper extremity positioning and elevation

An inpatient has been participating in ADL, social skills, and prevocational groups as part of a multidisciplinary program for the treatment of anorexia. What information about the patient's participation should be documented in the "A" section of the SOAP note to provide the MOST BENEFICIAL information about the patient's progress? A.Patient reports of compliance with caloric intake requirements B.Impressions about the patient's ability to use appropriate judgment related to own skills and assets C.Evidence of specific decision-making skills the patient uses during assigned group tasks D.Examples of patient's conformity with strict limits set on personal behavior

B.Impressions about the patient's ability to use appropriate judgment related to own skills and assets

An OTR is planning an intervention session for a 6-year-old child who has fetal alcohol syndrome. The child has difficulty coping when frustrated and frequently refuses to speak in social situations and during therapy. The OTR has attempted to engage the child using play therapy and sensory-based approaches, but the child refuses to participate. How should the OTR modify the intervention approach based on the child's responses during previous interventions? A.Engage the child in computer games that promote positive feedback. B.Include expressive media using cartoon character puppetry. C.Use role-play scenarios about interacting with super heroes.

B.Include expressive media using cartoon character puppetry. -increase expression

Which federal law in the United States regulates the provision of early intervention, special education, and related services to children with disabilities from birth to two years of age? A.Section 504 of the Vocational Rehabilitation Act B.Individuals with Disabilities Education Act (IDEA) Part C C.Individuals with Disabilities Education Act (IDEA) Part B

B.Individuals with Disabilities Education Act (IDEA) Part C

Which action is MOST IMPORTANT for the OTR to take when recommending driving cessation for a client who is assessed to be unsafe to drive a motor vehicle? A.Report the safety risk associated with fitness to drive to state licensing authorities and educate the family on strategies to prevent the client from driving. B.Inform the client and the referring physician about safety concerns and know state laws about legal obligation for reporting. C.Collaborate with the client and family members to determine best plan of action and develop goals for community mobility alternatives.

B.Inform the client and the referring physician about safety concerns and know state laws about legal obligation for reporting. -safety and laws first

What is the FIRST action the OTR should take when a patient who has schizophrenia exhibits an increase in lip smacking, tongue protrusion, and facial grimacing? A.Request that the patient stop the behavior. B.Inform the patient's physician. C.Suggest that the patient change medications.

B.Inform the patient's physician. -side effects of anti-psychotic meds

An OTR is evaluating a client who was recently diagnosed with osteoarthritis of the CMC joint of the dominant hand. The client works as a chef and reports thumb pain and progressive difficulty in using cooking utensils to complete food preparation. The client wants to continue working as a chef. Which intervention should the OTR provide to support INITIAL progress towards this goal? A.Fabricate a forearm-based thumb spica orthosis. B.Instruction and practice using joint protection strategies. C.Initiate a resistive pinch and grip strengthening program.

B.Instruction and practice using joint protection strategies.

In addition to achieving client-centered short- and long-term goals, which of the following is the BEST example of an optimal overall outcome for a home health client who has multiple chronic conditions? A.Knowledge of the trajectory of disease processes to identify needed pharmaceuticals and medical interventions B.Integration of self-management strategies to positively influence symptoms associated with the health conditions C.Understanding the role of various payer sources to successfully navigate reimbursement systems

B.Integration of self-management strategies to positively influence symptoms associated with the health conditions

An OTR, who works in an acute care setting, encourages a patient who recently had surgery, to use an incentive spirometer. What is the PRIMARY purpose for post-operative incentive spirometry? A.It prevents thromboembolic conditions such as deep vein thrombosis. B.It expels anesthesia from the lungs and improves lung function. C.It offers an alternative to pharmaceutical pain control methods.

B.It expels anesthesia from the lungs and improves lung function.

Which of the following symptoms TYPICALLY indicates that a client who has been on prolonged bed rest is experiencing orthostatic hypotension? A.Pounding headache upon moving into a semi-reclined position B.Lightheadedness upon moving from a supine to seated position C.Shortness of breath when sitting up from a supine position D.Diaphoresis when turning over from supine to side-lying position

B.Lightheadedness upon moving from a supine to seated position

An OTR completes a home modification evaluation and is writing a report to submit to a potential funding source. What information is MOST IMPORTANT to include in this report? A.Statement outlining the client's financial situation and preferred DME vendors B.List of the required modifications and justification for recommendations C.Results of a standardized assessment and a summary of the client's rehabilitation goals

B.List of the required modifications and justification for recommendations

An OTR working in a pediatric acute care setting is contributing to the discharge recommendations for a patient who was admitted to the hospital due to a pulmonary exacerbation of cystic fibrosis. Current medical interventions include mechanical ventilation support for breathing and multiple intravenous medications. The patient is weaning from the ventilator and long-term therapy goals include optimizing the patient's performance in ADL and IADL. Which discharge setting would be MOST BENEFICIAL for the OTR to recommend for the patient's next level of care? A.Subacute rehabilitation B.Long-term acute care C.Inpatient rehabilitation

B.Long-term acute care A long-term acute care setting is the most beneficial discharge disposition for this patient due to the high level of medical complexity including the need for a ventilator for respiratory support.

An OTR is selecting a wheelchair for a patient in an acute rehabilitation center who has transhumeral amputation of both arms and a distal transtibial amputation of one leg. The patient plans to alternate between using a manual wheelchair and ambulating with a prosthesis. Which wheelchair feature would be MOST BENEFICIAL for meeting the patient's daily needs? A.High back height B.Low seat-to-floor height C.Off-set rear axle position

B.Low seat-to-floor height -A wheelchair that has a low seat-to-floor height will allow this client to use the unaffected leg to propel the wheelchair.

An OTR is fabricating a static orthotic for a client who has a partial thickness burn to the dorsum of the hand. The primary purpose of the orthotic is to maintain the length of the MCP joint collateral ligaments. How should the MCP joints, IP joints and wrist be positioned in the orthotic to achieve this goal? A.MCP joint in 20° - 30° flexion, IP joint in 20° - 25° flexion and wrist in 25° - 30° extension B.MCP joint in 60° - 70° flexion, IP joint in 0° - 5° flexion and wrist in 25° - 30° extension C.MCP joint in 35° - 45° flexion, IP joint in 30° - 40° flexion and wrist in neutral D.MCP joint in 5° extension, IP joint in 45° - 60° flexion and wrist in neutral

B.MCP joint in 60° - 70° flexion, IP joint in 0° - 5° flexion and wrist in 25° - 30° extension

Which option represents the code of professional behavior associated with the ethical principle of beneficence? A.Protecting the privacy of the client's information and being respectful of the client's independence B.Maintaining concern for the safety and well-being of the client throughout the occupational therapy process C.Attaining and maintaining competency in selecting, administering, and interpreting evaluation tools to avoid doing harm to the client

B.Maintaining concern for the safety and well-being of the client throughout the occupational therapy process

A client has spasticity secondary to an upper motor neuron lesion. Which assessment would be MOST RELIABLE for monitoring muscle tone over time? A.Nine Hole Peg test B.Modified Ashworth scale C.Fugl-Meyer Motor assessment

B.Modified Ashworth scale -spasticity assessment

A student who is 8-years-old has severe diplegia and is non-ambulatory. The student is referred to OT after the family relocates to a new school district. Which area of occupation should be a priority for the school-based OTR to assess during the INITIAL evaluation? A.Prehensile skills for manipulating eating utensils during lunchtime B.Motor skills for managing clothing during toilet transfers C.Perceptual abilities for copying from the classroom lesson board

B.Motor skills for managing clothing during toilet transfers

An OTR has completed an evaluation of a client who has amyotrophic lateral sclerosis. When reviewing the results of upper extremity goniometric measurements, the OTR notes that the client's active ROM is significantly less than passive ROM. What should the OTR conclude is the PRIMARY cause for this discrepancy? A.Bony ankylosis B.Muscular weakness C.Soft-tissue shortening

B.Muscular weakness

An OTR is reviewing the lab values of a patient in an acute care hospital. Which medical condition is typically associated with upward trending troponin levels greater than 0.04 ng/mL? A.Hyperglycemia B.Myocardial infarction C.Pulmonary embolism

B.Myocardial infarction -troponin is a heart muscle protein

A resident of a skilled nursing facility, who has moderately severe cognitive decline, becomes increasingly agitated during mealtime. What is the FIRST action the OTR should take based on this observation? A.Arrange for the resident to eat meals in a calm, quiet location. B.Observe the resident at mealtime to identify behavioral triggers. C.Instruct the resident in relaxation exercises to use at mealtime.

B.Observe the resident at mealtime to identify behavioral triggers.

An OTR is evaluating the visual function of a patient who is emerging from coma. The patient opens eyes spontaneously and in response to auditory stimulation. What visual function is MOST IMPORTANT for the OTR to observe in a patient at this level of function? A.Scanning B.Oculomotor control C.Pattern recognition

B.Oculomotor control

Which symptoms of cranial nerve dysfunction are TYPICALLY associated with a diagnosis of multiple sclerosis? A.Tinnitus and hearing loss B.Optic neuritis and diplopia C.Trigeminal neuralgia and facial pain

B.Optic neuritis and diplopia

Which data collection set is part of a comprehensive assessment completed by a home health agency to support payment from Medicare for home health services? A.Minimum Data Set (MDS) B.Outcome and Assessment Information Set (OASIS) C.Functional Independence Measure (FIM®)

B.Outcome and Assessment Information Set (OASIS)

An OTR is working with a client who has type 1 complex regional pain syndrome (CRPS) affecting functional use of the dominant upper extremity. Pain interferes with the ability to complete ADL and tasks that involve reaching, lifting, and carrying. The client reports that pain fluctuates in intensity, but does not decrease below a 6/10 on the analogue pain scale, even after taking analgesic medication. The OTR notes that the client's affected upper extremity is cold to the touch and there are signs of muscle atrophy. Based on these results, what would be the MOST EFFECTIVE intervention for the OTR to include in sessions with this client? A.Apply ice or a cooling agent prior to initiating a therapeutic exercise program. B.Perform active ROM exercises and bilateral functional activities that promote weight bearing. C.Recommend immobilizing the affected limb in a pouch sling for 1-2 weeks.

B.Perform active ROM exercises and bilateral functional activities that promote weight bearing.

An OTR is completing an ADL evaluation for a patient who has an acquired brain injury. The patient can independently comb hair while standing at the sink but requires tactile and verbal cues from the OTR to terminate the activity. Which option BEST describes this behavior? A.Executive dysfunction B.Perseveration C.Ideomotor apraxia

B.Perseveration

An OTR who works in a community mental health program is preparing to evaluate a 15-year-old client who has bulimia nervosa and frequently engages in self-injurious behaviors by superficially cutting the skin. Which standardized assessments would be MOST EFFECTIVE for the OTR to include as part of the evaluation process? A.Numeric Pain Rating Scale and Sensory Integration and Praxis Test B.Piers Harris Children's Self Concept Scale and Adolescent Role Assessment C.Sensory Profile and Ranchos Los Amigos Levels of Cognitive Function-Revised™

B.Piers Harris Children's Self Concept Scale and Adolescent Role Assessment

An OTR is collaborating with a client and a building contractor to have wall-mounted grab bars installed in the client's bathroom. The client lives independently in a mobile home and is at-risk for falls due to cerebellar ataxia. The OTR determines the optimal placement of the grab bars to meet the needs of the client, however the contractor informs the OTR there are no available studs behind the wallboard in the recommended locations. Which alternative option would be MOST BENEFICIAL for the OTR to discuss with the contractor and the client? A.Install 1/2-inch (1.2cm) plywood over existing wallboard. B.Place blocks between the wall studs under the wallboard. C.Apply suction cup bathroom safety bars on the wallboard.

B.Place blocks between the wall studs under the wallboard. -apparently this is right but I didn't get a doctorate in fricken construction

Which option represents the MOST EFFECTIVE method to screen a 14-month-old toddler for protective responses? A.Place the child in a supine position and observe body movements while rotating the child's head to one side and then to the other. B.Place the child in a sitting position and observe for arm extension while gently pushing the child off balance to the front, back, and each side. C.Hold the child in vertical suspension and observe head position while gently tilting the child from side to side then from front to back.

B.Place the child in a sitting position and observe for arm extension while gently pushing the child off balance to the front, back, and each side.

An OTR has completed the initial evaluation of an outpatient who has Medicare as the primary insurance. What MUST be included in the occupational therapy documentation to meet Medicare requirements for reimbursement? A.Medical order written by the physician; client-centered, occupation-based intervention priorities; and recommendations for post-discharge community support services B.Plan of care certified by the physician; justification for the skills of an occupational therapy practitioner; and services specific to the needs of the patient C.Health form completed by the physician; summary of occupational profile data; and description of outcome measurement tools to track the patient's progress

B.Plan of care certified by the physician; justification for the skills of an occupational therapy practitioner; and services specific to the needs of the patient

An OTR is working with a client who has Stage 5 Alzheimer's disease and lives with a full-time caregiver. Based on this stage of the disease process, which option would be the MOST BENEFICIAL recommendation for the OTR to make to the caregiver? A.Post an illustrated daily schedule for the client to follow. B.Provide assistance for dressing and grooming activities. C.Encourage the client to participate in a new hobby.

B.Provide assistance for dressing and grooming activities.

An OTR has been providing home health services for 2 weeks to a patient who has had cardiac bypass surgery. The OTR determines that the patient could benefit from an outpatient cardiac rehabilitation program. The OTR informs the supervisor of these conclusions, but the supervisor recommends that the patient continue with home health services to maintain Medicare reimbursement. What action should the OTR take in this situation? A.Ask the patient and family about their service preference. B.Provide justification for the referral in a discharge summary. C.Continue to provide services based on the initial plan. D.Contact the patient's physician to discuss a referral to the program.

B.Provide justification for the referral in a discharge summary. -medicare = must be medically necessary

A client in the outpatient setting recently underwent plastic surgery to correct facial disfigurement. During a grooming session, the client states, "I will never look in the mirror again." What action should the OTR take in response to the client's concern? A.Validate the client's concerns and transition to the next intervention that does not require a mirror. B.Provide support and ask open-ended questions to gain insight into the client's view on their body image. C.Provide positive affirmations and encourage the client to seek reassurance from close friends.

B.Provide support and ask open-ended questions to gain insight into the client's view on their body image.

An OTR is planning intervention for an outpatient client who had a CMC thumb arthroplasty one week ago. What should the OTR include as part of the INITIALintervention plan at this stage of recovery? A.Educate the client to complete a progressive strengthening program of the affected hand. B.Provide the client with a ROM program for the shoulder, elbow, fingers and thumb IP joints. C.Teach the client one-handed techniques using the non-affected hand for ADL.

B.Provide the client with a ROM program for the shoulder, elbow, fingers and thumb IP joints.

An 8-year-old student who has an autism spectrum disorder shows signs of distress during routine planned fire drills conducted at the school. This interferes with the teacher's ability to evacuate the students from the classroom. Which approach should the OTR recommend to the teacher for reducing the student's stress level during these fire drills? A.Ensure an aide is in the classroom to calm and escort the student during planned fire drills. B.Provide the student with a picture board illustrating the sequence of steps the class will use during the fire drill. C.Provide the student with headphones to put on whenever the fire drill alarm begins to sound.

B.Provide the student with a picture board illustrating the sequence of steps the class will use during the fire drill. -allow for greater preparedness in the event an actual emergency

An OTR, who works in a hospital setting, is investigating the perceptions of patients with neurodegenerative disease who have had several hospitalizations over a 3-month period. What type of research method would be MOST BENEFICIAL for this purpose? A.Meta-analysis B.Qualitative C.Quantitative

B.Qualitative

A patient in an acute care setting is undergoing treatment for an exacerbation of cardiomyopathy. The patient reports avoiding most preferred activities because they typically cause shortness of breath and extreme exhaustion. Which self-monitoring assessment should the OTR teach the patient to use for supporting the patient's engagement in these preferred activities? A.Visual analog fatigue scale B.Rating scale of perceived exertion C.Pulse oximetry measurements

B.Rating scale of perceived exertion -rate how you feel to see if you need to change

An inpatient in a rehabilitation facility has hemiplegia secondary to a CVA. The patient is independent with BADL. The OTR, who uses the ecology of human performance model, is preparing the patient's discharge summary. What information reflects this approach and should be included as part of this report? A.Current functional status and anticipated occupational performance upon return home B.Recommendations for home modifications to maximize accessibility and task performance C.Support groups for promoting the patient's acceptance of the physical impairments D.Exercise protocols for maintaining physical strength and cardiovascular endurance

B.Recommendations for home modifications to maximize -ecology = environment/context and person interaction

An OTR is evaluating an older adult in patient who has had a rapid decline in self-feeding and dressing skills. The patient was confused and unable to respond to verbal instructions. Before the OT intervention plan was implemented, medications were adjusted and the patient's mental status improved. What action should the OTR take FIRST in this situation? A.Write new short-term goals B.Reevaluate the patient C.Follow the established plan

B.Reevaluate the patient

An OTR is providing home health services to a 9-month-old infant who underwent surgery for a Type I Chiari malformation one month ago. The infant has been progressing without complication since the date of surgery. During a follow-up home visit, the infant's parent reports lack of progress with the home program because the infant has been extremely lethargic for the past few days and has had several episodes of vomiting within the past 12 hours. What should the OTR advise the parent to do based on the infant's current condition? A.Reschedule the appointment for the next week when symptoms should be resolved. B.Report the infant's symptoms as soon as possible to the primary care physician. C.Monitor the infant's flu-like symptoms for the next 24 hours and ensure fluid intake.

B.Report the infant's symptoms as soon as possible to the primary care physician.

An OTR, who works in a hospital setting that uses an electronic medical record for clinical documentation, mistakenly entered the wrong information in a patient's medical record. What is the FIRST action the OTR should take in this situation? A.Reflect on personal strategies to prevent and reduce documentation errors. B.Request information from a supervisor on how to correct the documentation error. C.Ask the department manager to schedule a training session on error-free documentation.

B.Request information from a supervisor on how to correct the documentation error.

An OTR is teaching stand-pivot transfers to a client who has Stage 2 Parkinson's disease and uses a wheelchair for mobility. After instructing the client to properly position the chair in relation to the transfer surface, and asking the client to lock the wheelchair brakes, what should the OTR ask the client to do NEXT? A.Rock forward while reaching toward the transfer surface. B.Scoot the hips forward to the edge of the wheelchair. C.Position both feet perpendicular to the transfer surface. D.Come to standing by pushing up on the wheelchair arm rests.

B.Scoot the hips forward to the edge of the wheelchair.

An OTR and a vocational rehabilitation team are using the Individual Placement and Support (IPS) model to support a client, who has schizophrenia and anxiety, in gaining employment. One of the intervention goals is for the client to achieve competitive employment. When using the IPS model, which approach should be included in the vocational intervention to support this goal? A.Provide education in general work behaviors, then conduct an extensive job search. B.Secure a job placement site, then teach the client the specific job skills for that workplace. C.Complete a detailed resume of job skills, then teach the client effective work behaviors.

B.Secure a job placement site, then teach the client the specific job skills for that workplace. -The Individual Placement and Support (IPS) model is a supportive employment approach that involves securing a job placement site, then teaching the client the specific job skills for that workplace.

An OTR is working with an 8-month-old infant who has severe hypotonia. What is the BEST position to place the infant in for feeding? A.Reclined with the neck in slight extension and the shoulders depressed. B.Semi-reclined with the neck in neutral and the head at midline. C.Forward flexion with the head and neck resting in the caregiver's hand.

B.Semi-reclined with the neck in neutral and the head at midline. -best position to reduce aspiration

An inpatient had a TBI 3 weeks ago and is functioning at Level II (Generalized response) on the Rancho Los Amigos scale. Which intervention should be a priority to include in treatment sessions during this phase of the patient's rehabilitation? A.Self-feeding program using assistive devices and hand-over-hand cues B.Sensory stimulation program using graded and consistent stimuli C.Personal hygiene tasks using cueing to minimize outbursts D.Simple life skills tasks using compensatory cognitive strategies

B.Sensory stimulation program using graded and consistent stimuli

Which interprofessional team member would be MOST BENEFICIAL for an OTR to refer an elementary student who may need modification to the academic curriculum to achieve grade level success in reading and writing? A.Speech language pathologist B.Special education teacher C.Pediatric ophthalmologist

B.Special education teacher

For which client condition is the use of a hot, moist pack as a physical agent modality INDICATED? A.Superficial skin graft 3 days ago with acute pain B.Spinal cord injury (ASIA B) with painful muscle spasms C.Breast cancer with upper extremity edema

B.Spinal cord injury (ASIA B) with painful muscle spasms

An OTR is about to conduct an evaluation of a student who has a developmental delay and is transitioning from preschool to kindergarten. Which data gathering method will provide the OTR with the MOST OBJECTIVE information about the student's occupational performance? A.Structured interview B.Standardized assessment C.Review of previous records

B.Standardized assessment

An 8-year-old student with severe achondroplasia has limited hand function with no isolated finger movements. The student is learning to use an adapted computer to optimize performance during classroom assignments. Which modification would BEST support the student's success to access the computer? A.Single-switch control interface B.Trackball mounted on a stationary base C.Continuous joystick input device

B.Trackball mounted on a stationary base

A 6-year-old child has developmental delay associated with multiple hospitalizations for a congenital heart defect. The parents currently provide in-home care to meet the child's ongoing medical needs and daily BADL. One of the intervention goals is to teach the parents methods for including in-home playtime into daily routines. What information would be MOST BENEFICIAL for the OTR to provide the parents for supporting this goal? A.Guidance on directing specific play sequences for age-appropriate activities B.Strategies for developing accessible play spaces the child can access independently C.Explanation about the benefits of playtime for the typically developing child

B.Strategies for developing accessible play spaces the child can access independently

An OTR has been asked by the director of nursing in an acute-care hospital to provide an in-service to the nursing staff to describe the role of OT in preventing deep venous thrombosis and pulmonary embolus. Which intervention is MOST IMPORTANT to include as part of this presentation to explain the role of OT in thromboembolic disease prevention? A.Prescribe anticoagulation therapy to thin the blood and prevent clots from forming. B.Support early mobilization to facilitate patient engagement in self-care activities. C.Recommend mechanical interventions such as compression stockings and pumps.

B.Support early mobilization to facilitate patient engagement in self-care activities.

An OTR is interviewing an adult client who has schizoaffective disorder. The client responds to the questions but jumps from one unrelated topic to another. What is the BEST option to describe this behavior? A.Dysphonia B.Tangential speech C.Stereopsis

B.Tangential speech

An inpatient with relapsing-remitting multiple sclerosis has been making steady progress during morning BADL. For the past 3 mornings, the patient reportedly completed the morning self-care routine independently. During a reevaluation of BADL, the patient becomes physically exhausted while dressing after taking a shower and asks to return to bed. In addition to talking with the patient about energy conservation, what action should the OTR take based on the patient's physical response? A.Talk with the patient about pacing self-care tasks throughout the day. B.Teach the patient to monitor symptoms while incorporating appropriate rest-activity ratios into self-care tasks. C.Ask the patient to identify the BADL tasks that typically take the most time to complete. D.Advise the patient to complete self-care while seated upright in bed in order to reduce energy expenditure.

B.Teach the patient to monitor symptoms while incorporating appropriate rest-activity ratios into self-care tasks.

A client in an outpatient setting sustained an acquired brain injury 2 months ago. Evaluation results indicate the client has functional ROM and strength, but continues to require assistance with ADL due to moderate visual and vestibular processing deficits. Which intervention represents an adaptive approach for improving the client's performance in areas of occupation? A.Incorporating progressively more challenging tasks into a functional activity B.Teaching the client to use proprioceptive cues during functional activities C.Providing the client with an exercise program for improving gaze stabilization D.Engaging the client in valued activities that promote postural stability and balance

B.Teaching the client to use proprioceptive cues during functional activities -adaptive approach

An OTR is evaluating the biceps strength of a client recovering from a musculocutaneous nerve injury. The OTR asks the client to fully flex the elbow while the client is seated upright with the shoulder adducted, the elbow fully extended, and the forearm in supination. The OTR observes that the client's forearm consistently moves into midposition on each attempt to flex the elbow despite prompting the client to maintain the forearm in supination. What conclusion can the OTR make based on this observation? A.The muscle strength of the biceps should be graded as Poor (2/5). B.The brachioradialis muscle is substituting for the weaker prime mover. C.The pronator teres muscle should be blocked on future testing. D.The movement should be retested with the client positioned in prone.

B.The brachioradialis muscle is substituting for the weaker prime mover.

An OTR who works in an inpatient rehabilitation setting is using the FIM™ as a clinical measurement tool to evaluate and monitor the progress of a patient who had a stroke. What additional information can the OTR obtain from interpreting the results of the FIM™ with this patient? A.The patient's potential to return to daily activities at home and in the community B.The future discharge status of the patient related to level of assistance required for the patient to perform self-care tasks C.The health outcomes for the patient related to stroke, including ADL, IADL, and participation levels

B.The future discharge status of the patient related to level of assistance required for the patient to perform self-care tasks -The FIM™ is a measurement tool that has been shown to predict the discharge disposition for patients who have had a stroke.

An OTR is fabricating a hand orthosis for a 10-year-old child who has progressive upper extremity symptoms secondary to Charcot-Marie-Tooth disease. What is the PRIMARY purpose for fabricating this orthosis? A.To prevent ulnar drift and swan neck deformities B.To improve function for active grasp and release C.To rest the hand during periods of acute pain

B.To improve function for active grasp and release -Charcot-Marie-Tooth disease is a neurological disorder that impacts voluntary muscle control and muscle strength

An OTR, who works in the hospital setting, is starting an ADL session at the bedside of an inpatient who has mitochondrial encephalopathy and flaccid hemiplegia. The patient is lying supine in bed and is wearing a prefabricated foot drop orthosis on the affected lower extremity. What is the PRIMARY purpose for having the patient wear a foot drop orthosis? A.To provide stability when sitting on the edge of bed and during transfer training B.To offer pressure relief while supporting the foot in a neutral position C.To increase circulation in the lower extremity and prevent thromboembolism

B.To offer pressure relief while supporting the foot in a neutral position

An OTR working in a skilled nursing facility frequently teaches residents how to use adaptive devices during BADL. The OTR wants to evaluate if the instruction methods and devices are effective in helping to improve the residents' functional performance. Which method would be MOST EFFECTIVE to use for gathering this information? A.Ask the residents how often during the week they use the devices for BADL. B.Track how many times the residents spontaneously use the devices during BADL. C.Note the number of times the residents bring the devices to BADL sessions. D.Have the residents demonstrate the use of the devices during a session.

B.Track how many times the residents spontaneously use the devices during BADL. -Tracking spontaneous use of the devices during BADL provides the best indication of transfer of learning and the impact of the devices on residents' functional performance.

An OTR, who works in an acute care hospital, is planning an intervention for an inpatient who recently had a total shoulder arthroplasty. One of the intervention priorities is to teach the patient strategies to use during ADL while maintaining ROM precautions. What INITIAL action should the OTR take to address this priority? A.Use a numeric pain scale to measure the patient's level of pain. B.Understand the precautions outlined by the orthopedic surgeon. C.Search a scholarly database for evidence-based intervention methods.

B.Understand the precautions outlined by the orthopedic surgeon.

An OTR who works in a school is planning a session for a kindergarten student who has Down syndrome. The student can push down their pants during toileting, but is unable to pull them up. Which technique would be MOST BENEFICAL to include as part of the initial intervention for this student? A.Train the staff on methods to support the student. B.Use games and activities that support pinch strength. C.Assist the student with pulling up their pants.

B.Use games and activities that support pinch strength.

An OTR is providing recommendations to the caregiver of a client who has moderate severe cognitive decline secondary to Alzheimer's disease. The caregiver reports the client wanders throughout the home at night trying to find the bathroom. On several occasions the client has walked out the front door of the home thinking it was the door leading into the bathroom. Which environmental adaptation should the OTR recommend to the caregiver based on this report? A.Install a video monitor in several locations in the house. B.Use movement sensitive audio-visual assistive technology. C.Keep hallway and bedroom lights on at night. D.Place a commode chair in the client's bedroom.

B.Use movement sensitive audio-visual assistive technology. -safest option for promoting a safe home environment that contains the least restrictions. Caregivers will be alerted to the client's movements if the client wanders at night.

An OTR who works in a school-based setting is planning a session for a fifth-grade student who has autism spectrum disorder. The student has average cognitive ability but poor organization skills. Which support would be most beneficial to include in the INITIAL intervention for improving the student's organization skills? A.Provide extra supplies for use in all educational environments. B.Use visual supports to identify items needed for each class. C.Develop a behavior management plan to reward the student for planning ahead.

B.Use visual supports to identify items needed for each class.

A client with fibromyalgia reports hand pain and stiffness make it difficult to grasp a standard knife and fork during meals. Which assistive device would be MOST BENEFICIAL for improving the client's functional abilities when eating? A.Universal cuff with wrist support B.Utensils with built-up handles C.Lightweight utensils with non-slip grips D.Rocker knife and plate with raised sides

B.Utensils with built-up handles

In preparation for a facility-organized dance class, an OTR has a resident with Parkinson's disease rock in a rocking chair. What is the PRIMARY body system being challenged during this preparatory activity? A.Pulmonary system B.Vestibular system C.Neuromuscular system

B.Vestibular system - Primary!!!

An OTR is working with a 5-year-old child who has severe spastic diplegic cerebral palsy and will soon be attending kindergarten. The child has been approved for a power wheelchair. Before beginning training in the use of the chair in a school setting, which client factor is MOST IMPORTANT for the OTR to re-evaluate? A.Upper extremity range of motion B.Visual perceptual skills C.Object permanence

B.Visual perceptual skills -Cerebral palsy is associated with visual impairments that need to be screened prior to initiating power mobility training.

A home health client with amyotrophic lateral sclerosis has normal lower extremity strength but has difficulty lifting each leg up to dress lower extremity when supine in bed. What is the PRIMARY reason for this occurrence? A.Decreased vital capacity B.Weak abdominal muscles C.Impaired vestibular function

B.Weak abdominal muscles

A client has a one-week history of symptoms consistent with a work-related carpal tunnel syndrome. The client works on an assembly line and is required to stand at a workstation and tighten bolts into sheet metal at waist level using a pneumatic pistol-grip drill. The client uses the drill approximately 6 hours each 8-hour shift. Which ergonomic modification would be MOST BENEFICIAL for reducing symptoms associated with this condition? A.Lowering the level of the workstation B.Wearing anti-vibration gloves C.Adapting the handle of the drill

B.Wearing anti-vibration gloves

An OTR who works in an inpatient rehabilitation setting is preparing to evaluate a 6-year-old patient who recently had a traumatic brain injury. The patient is alert and oriented to person, place, and situation. Which standardized assessments would be MOST EFFECTIVE for the OTR to include as part of the evaluation process? A.Sensory Integration and Praxis Test and School Function Assessment B.Wee-FIM™ and Ranchos Los Amigos Levels of Cognitive Function-Revised™ C.Sensory Profile and Canadian Occupational Performance Measure (COPM)

B.Wee-FIM™ and Ranchos Los Amigos Levels of Cognitive Function-Revised™

A client has an open wound on the dorsum of the dominant hand. The medical record indicates mechanical debridement to promote wound healing. Which of the following wound management options represents mechanical debridement? A.Wound dressings that keep the wound moist to trap the body's natural enzymes. B.Whirlpool as a preparatory activity to remove dead tissue from the wound. C.Topical enzymes on the wound to selectively remove necrotic tissue and debris.

B.Whirlpool as a preparatory activity to remove dead tissue from the wound.

A home health client had surgery for a total hip arthroplasty 2 weeks ago. During an ADL session, the OTR notes that the area near to the surgical site is warm to the touch and has new erythema. The wound has purulent drainage and a foul odor. Which of the following conditions is TYPICALLY associated with these reported symptoms? A.Tissue necrosis B.Wound infection C.Venous stasis

B.Wound infection

Sustaining a closed head injury may result in a patient demonstrating aphasia. What are the key differences between Broca's aphasia and Wernicke's aphasia that would allow you to distinguish between these 2 types of aphasia? Wernicke's is a type of non-fluent aphasia and Broca's is a type of fluent aphasia Broca's aphasia is characterized by slow and incorrectly articulated speech and with Wernicke's aphasia the patient exhibits well-articulated speech, which lacks meaning Broca's aphasia is characterized by the patient exhibiting well-articulated speech, which lacks meaning and Wernicke's aphasia is characterized by slow and incorrectly articulated speech Broca's area is located in the temporal lobe and Wernicke's area is located in the occipital lobe

Broca's aphasia is characterized by slow and incorrectly articulated speech and with Wernicke's aphasia the patient exhibits well-articulated speech, which lacks meaning

According to Exner's Classification System, what manipulating skill is present at 12 to 15 months and involves a movement such as picking up coins? A Shift B Simple rotation C Finger-to-palm translation D Palm-to-finger translation

C Finger-to-palm translation

A client has difficulty swallowing secondary to multiple sclerosis. The OTR manually assists the client in performing a chin tuck prior to the client swallowing a bite of food. What is the PRIMARY benefit of facilitating this position? A. To hasten overal oral-motor responses for chewing and forming a bolus of food B. To maximize tongue elevation when pushing the food bolus to the back of the mouth C. To prevent food and secretions from entering the larynx below the level of the vocal cords

C. To prevent food and secretions from entering the larynx below the level of the vocal cords

What is an effective strategy to facilitate maximal independence in self-feeding with a client who has middle- to late-stage dementia? A. Change the client's diet to foods that are palatable and easy to chew. B. Change the consistency of foods to allow for easier chewing and swallowing. C. Provide hand-over-hand guidance for food retrieval and utensil-to-mouth motions. D. Provide constant verbal cues throughout feeding to improve attention to the self-feeding task.

C. Provide hand-over-hand guidance for food retrieval and utensil-to-mouth motions. -A. and B. don't encourage independence - D. may be overwhelming

A client has been referred to the occupational therapy clinic to be assessed for a new wheelchair and positioning support system. Currently, the client has a standard wheelchair with a custom-contoured cushion shaped to support a spinal deformity associated with scoliosis. The OT considers several options with the client but again recommends a custom-contoured cushion? What is the most important reason for making this recommendation? A. The cushion requires little maintenance. B. The cushion does not change shape. C. The cushion can be molded to fit the client and support him in a stable position. D. The cushion is made of lightweight materials

C. The cushion can be molded to fit the client and support him in a stable position.

An OT and a Level II Fieldwork student at a long-term care facility observe a new resident moving his wheelchair forward by gripping the front of the pushrim and giving it multiple tiny pushes. The OT asks the student, "Why do you think he is pushing his wheelchair that way?" The student observes a bit longer and responds: A. The seat back is too low. B. The wheelchair is too wide. C. The seat back is too high. D. The armrests are too low.

C. The seat back is too high.

An OT is measuring a client for a new wheelchair. The client's hip width is 18 inches. The client does not use any lateral trunk supports but lives in a cold climate and frequently wears bulky clothing. What seat width should the OT recommend? A. 16 inches B. 18 inches C. 20 inches D. 22 inches.

C. 20in

A child with congenital anomalies has severe developmental delay. The child demonstrates motor and cognitive skills at the nine-month level. Which is the best adaptation for the occupational therapist to use during intervention to develop the child's visual and auditory awareness? A. A hand-held rattle of the child's favorite cartoon character. B. A wrist bracelet with blinking lights that makes noise when moved. C. A button switch that activates a CD player when the switch is pressed

C. A button switch that activates a CD player when the switch is pressed

An OTR® is working with a client with Down syndrome to develop cooking skills. The client successfully makes a peanut butter and jelly sandwich. Which therapeutic activity would be BEST for the next treatment session? A. A turkey-and-cheese sandwich B. A microwavable dinner C. A grilled cheese sandwich D. A stir-fry vegetable dish over brown rice

C. A grilled cheese sandwich

An OT is working as part of a multidiscipli nary team that contributes to an Individualized Education Program (IEP). How is an IEP MOST ACCURATELY described? A. A contract that lists the student's goals and services intended to help achieve those goals. B. A document that outlines the district's educa tional curriculum together with the state's learning standards. C. A process and legal document that outlines strengths, needs, and educational services for a student with disability. D. A plan of services and supports for families and their infants and toddlers with disabilities

C. A process and legal document that outlines strengths, needs, and educational services for a student with disability.

A young adult on an inpatient surgical unit 4 days after the excision of a brain tumor is medically stable and participates in therapy 3 hours per day. Currently, the patient requires moderate assistance with BADL due to perceptual deficits and decreased balance. The patient will be discharged from this facility in one day. Which transitional setting would BEST support the patient's current level of function? A. Assisted living center B. Residential skilled nursing facility C. Acute inpatient rehabilitation unit D. Patient's home with outpatient therapy

C. Acute inpatient rehabilitation unit

An OTR® is working on the intensive care unit of a hospital. The client has hypertension and has an arterial line catheter inserted in the radial artery. What change in the client is the OTR MOST likely to record during treatment?A. A decrease in blood pressure when the head of the bed is elevated B. A decrease in heart rate when client is positioned from supine to short sitting at the edge of the bed C. An inaccurate blood pressure reading when the wrist is moved D. An increase in respiration rate when the client is positioned side lying

C. An inaccurate blood pressure reading when the wrist is moved

Individual with incomplete C6 SCI has secondary diagnosis of thromboangiitis obliterans. The OT conducts a predischarge home eval of client's rented apartment. Which is the most important area for the OT to assess? A. Apartment's electrical capacity for an environmental control unit B. Apartment's electrical capacity for an emergency call system C. Apartment's water temp D. The landlord's willingness to modify the bathroom

C. Apartment's water temp -Thromboangiitis obliterans or 'Buerger's disease' results in diminished temperature sense, paresthsias, pain, and cold extreme -B. you don't need apartment for -D. and A. arnt decided till after home eval

When implementing a school-based lunch-time feeding program, the therapist must consider the consistency and control of the food. Which of the following is most likely to present difficulties in sequential swallowing and control during feeding interventions? A. Dry cereal B. Peanut Butter C. Apple Juice D. Pudding

C. Apple Juice -thinnest

An OTR® is working with a child who has scoliosis with a curve of 70°. The child would like to engage in sports activities. On the basis of this medical condition, which factor would most likely limit the child's ability to participate in sports? A. The need to wear a therapeutic brace B. Weak abdominal muscles C. Cardiopulmonary function D. Ability to manage pain

C. Cardiopulmonary function -Children with a scoliosis curve between 65° and 80° may have reduced cardiopulmonary function. -B. is true but but breathing is more important

An OT is seeing a home health-care client who is in the moderate stages of Alzheimer's disease and whose memory loss is now interfer ing with the performance of daily self-care activities. What is the MOST relevant OT inter vention for this client? A. Memory-retraining activities for the client. B. ADL-retraining program for the client. C. Caregiver instructions in problem-solving. D. Leisure activity planning.

C. Caregiver instructions in problem-solving.

An OTR® is performing a home evaluation for a client with Stage IV amyotrophic lateral sclerosis (ALS). On what will the OTR®'s recommendations MOST LIKELY focus? A. Modifications to keep the client's lifestyle as close as possible to occupations preferred before diagnosis B. Environmental adaptations such as moving frequently used items to easy-to-reach and nearby areas C. Creation of a first-floor setup and increased accessibility to a wheelchair or durable medical equipment D. Technology changes such as a motorized stair lift

C. Creation of a first-floor setup and increased accessibility to a wheelchair or durable medical equipment -no longer able to ambulate -B. is for earlier stages, A. won't happen bc ALS is progressive, and D. requires caregivers to carry wheelchair upstairs

A 5-year-old is referred to occupational therapy. Upon the completion of a standardized test evaluation, the occupational therapist determines that the child demonstrates age-appropriate cognitive and fine motor skills. Which activity would the child be able to complete at this developmental level? A. Cutting long thin strips with scissors. B. Holding and snipping with scissors. C. Cutting simple figure shapes with scissors. D. Opening and closing scissors in a controlled fashion

C. Cutting simple figure shapes with scissors.

An OT provides services to child with severe CP who has decreased oral-motor control. During a feeding session, the child has a strong gag reflex and coughs in response to a small spoonful of pudding placed on the tongue. Which is the therapist's best action? A. Decrease the amount of pudding and place the spoon in the same place on the tongue. B. End the session and resume intervention at a later time. C. Decrease the amount of pudding and place the spoon close to the tip of the tongue D. Place the spoon in the same place on the tongue and increase the amount of pressure used

C. Decrease the amount of pudding and place the spoon close to the tip of the tongue

Individual with post-polio syndrome receives OT reevaluation of functional status. How should the therapist initiate sensory testing with this client? A. Demo the test with individual's vision occluded B. Proceed proximal to distal C. Demo the test with individual's vision not occluded D. Proceed distal to proximal

C. Demo the test with individual's vision not occluded -demonstrate first -SCI is proximal to distal -nerve is distal to proximal

An OTR®; wants to find an evidence-based rationale for using a sensory integration intervention with at-risk youths. What must the OTR do FIRST to construct an evidence base for practice? A. Access available databases to search the literature for evidence regarding sensory integration and at-risk youths. B. Evaluate whether peer-reviewed articles from five journals support using a sensory integration intervention with at-risk youths .C. Develop a question to guide a literature search for information regarding a sensory integration intervention with at-risk youths. D. Apply existing evidence about sensory integration to practice with at-risk youths.

C. Develop a question to guide a literature search for information regarding a sensory integration intervention with at-risk youths.

Individual has Parkinson's disease with poor trunk rotation during ambulation and while performing ADL. According to neurophysiologic approaches, which is the most effective therapeutic intervention for the OT to use with this person? A. Facilitation of trunk rotation using neurodevelopmental handling techniques. B. Slow rolling with the person in supine with knees and hips flexed. C. Engagement in ADL using diagonal patterns. D. Provision of rolling walker to compensate for limited rotation and enhance mobility.

C. Engagement in ADL using diagonal patterns. -PNF with functional tasks -A. and B. are neuro but not functional

A child increased tone of the back extensors. The therapist provides intervention to decrease tone and increase co-contraction. Which actions should the therapist take when workin with the child on a therapy ball? A. Stand behind the child who is prone on the ball and roll the ball away from the therapist. B. Stand behind the child who is supine on the ball and roll the ball away from the therapist. C. Face the child who is sitting on the ball and roll the ball away from the therapist D. Face the child who is sitting on the ball and roll the ball towards the therapist

C. Face the child who is sitting on the ball and roll the ball away from the therapist

A 2-year-old toddler, who has a developmental delay, recently learned to release a 2-inch (5.08 cm) ball into a large toy box. What fine motor skill should the OTR work on NEXT? A.Transfer toys from hand to hand. B.Color within the lines on a page. C.Complete a 3- to 4-piece puzzle.

C.Complete a 3- to 4-piece puzzle. -memorize: release then puzzle

An individual who recently experienced a myocardial infarction has been evaluated by the OT and is currently receiving intervention as part of the OT. A primary concern of the car diac team is to prevent "overexertion" in therapy. What is the BEST way to address this con cern? A. Teach the client independent medication management strategies. B. Retrain the client in ADLs to prevent exces sive strain. C. Monitor the client's heart rate and blood pres sure regularly throughout the intervention. D. Teach energy conservation techniques to de crease stress.

C. Monitor the client's heart rate and blood pres sure regularly throughout the intervention.

The OT is evaluating a child with DD characterized by hypotonicity. According to the Rood approach, which is the first stability pattern the OT should facilitate during intervention? A. Roll over B. Quadruped C. Neck co-contraction D. Prone on elbows

C. Neck co-contraction -first in development -Rood means normal muscle tone priority and developmental progression

An OTR is preparing to administer a standardized developmental screening to a premature infant who is at-risk for developmental delay. In addition to noting the assessment date and the infant's birth date, what information MUST be gathered in order to calculate the infant's corrected age? A. Original projected due date B. Typical gestational age C. Number of weeks premature

C. Number of weeks premature

A client was referred to occupational therapy because of persistent pain in the neck and shoulder and the recent onset of paresthesia in the right index and middle fingers. The OTR® decides that the symptoms may be work related and proceeds with an onsite ergonomic assessment. During the ergonomic assessment, the OTR® observes that the client uses the mouse 80% of the time. What will the OTR® MOST likely recommend? A. Using a vertical mouse and placing it at desktop level B. Changing to a laptop so that the client can use the touchpad instead of a mouse C. Performing full upper body stretches after every 20 minutes of mouse use D. Assigning the client other tasks that require less frequent use of the mouse

C. Performing full upper body stretches after every 20 minutes of mouse use -A. desktop is too high for mouse use (shoulder pain)

A 15-month-old child is demonstrating mo tor performance similar to that of an 8-month old child. In which activities would the OT MOST likely engage the child to elicit develop ing protective reactions? A. Play while prone on mat, with child reaching to grab toys suspended forward and out to child's sides. B. Play while floor sitting with toys placed on floor in front of child, or suspended for child's reach to front. C. Play while floor sitting with toys placed on floor to child's sides, or suspended for child's reach, out to child's sides. D. Play while in supported quadruped position, with child reaching for toys placed on floor more than arm's reach from child's body.

C. Play while floor sitting with toys placed on floor to child's sides, or suspended for child's reach, out to child's sides.

OT is providing home-based OT services on a bed positioning plan for a person recovering from a CVA. The person is receiving care from family members and personal care assistants employed by a home care agency. Which action should the OT take to ensure accurate implementation of this plan by the client's caregivers? A. Provide verbal step-by-step directions of desired positions to client's caregivers B. Post written step-by-step directions of desired positions on the wall by the client's bed C. Post pictures of the desired positions next to the headboard of the client's bed D. Require each caregiver to demonstrate the replication of the desired positions

C. Post pictures of the desired positions next to the headboard of the client's bed -pictures are better than words so it won't get misinterpreted -D. is true but there are too many caregivers in this scenario to get them all in one session

In a cooking group activity, each child makes their own pizza from a set of ingredients that have been set out from them with little interaction among the children. When the level of interaction increases, which group will the therapist most likely implement? A. Mature B. Cooperative C. Project D. Parallel

C. Project -short term group interaction around a task

A toddler with developmental delays attends an early intervention program. Over the past two weeks, the toddler has successfully completed the activities the occupational therapist has provided in order to develop a palmar grasp. Which action should the therapist take next in response to the child's progress? A. Continue providing the child with the activities to refine palmar grasp. B. Review the initial evaluation to determine new goals. C. Provide activities to develop a radial palmar grasp. D. Provide activities to develop an ulnar palmar grasp

C. Provide activities to develop a radial palmar grasp.

An OT in a state psychiatric hospital, using the recovery model, is working with a group of older adults with severe and persistent mental illness. They have been hospitalized for decades and will be moved into community living when the hospital closes in several months. Which of the following discharge planning activities is most appropriate for these individuals? A. Using a map, show them where the new house will be in comparison to where the state hospital is. B. Allow them to select their roommates. C. Provide catalogs from which they can select bedspreads and curtains. D. Provide collage activity to explore/express feelings about moving.

C. Provide catalogs from which they can select bedspreads and curtains. -focus on choice and self-goals -B. isnt safe option for wellness

What is the PRIMARY purpose for completing a dysphagia screening for an inpatient who had a right CVA several days ago? A. To detect silent aspiration and identify foods that are safe for the patient to eat B. To identify causes for aspiration and the need for referral to speech therapy C. To observe the patient's oral motor control and determine a need for further evaluation D. To determine the patient's tolerance to various food textures prior to requesting a modified diet

C. To observe the patient's oral motor control and determine a need for further evaluation

A client has visual acuity of 20/200 in the right eye and 20/400 in the left eye. With what technology will the client MOST likely be able to operate a microwave with a flat panel? A. A prescribed magnifier B. Color-coded buttons C. Raised dots on the panel D. Task light over the microwave

C. Raised dots on the panel - all the others rely on vision

An inpatient, who has a C8 spinal cord injury, is brushing teeth while sitting upright in a wheelchair at the sink. The patient reports a pounding headache and nasal congestion, and the vital signs are as follows: heart rate = 50 bpm blood pressure = 150 / 110 mmHg. After calling for immediate medical assistance, what action should the OTR take NEXT in this situation? A.Place a cool cloth on the patient's neck, and offer the patient small sips of cold water or ice chips. B.Transfer the patient to a supine position in bed and elevate the head off the bed up to 10°. C. Remove any restrictive clothing, including thromboembolic stockings, and check catheter tubing.

C. Remove any restrictive clothing, including thromboembolic stockings, and check catheter tubing. -based on Pt. condition

In a social skills group, members have learned about social skills and their components, and the OT practitioner has spent time in modeling and demonstrating ways to perform social skills. Based on a Social Skills Training approach, which of the following would be the NEXT step in the process of developing social skills? A. Self-evaluating one's social behavior. B. Independently practicing in real-life situations. C. Role-playing of social situations. D. Providing feedback on the client's hygiene.

C. Role-playing of social situations.

A child with autism and self-abuse behavior is a student in a classroom for children with behavioral problems. Which should the school-based therapist include in the intervention program for this student. A. ADL training B. Cooperative games with peers C. Self-regulation activities D. Development of pre-writing skills

C. Self-regulation activities

The accountant asks the private practitioners to present their budget for anticipated direct expenses of their growing practice. Which is the most appropriate item to include in this budget request? A. The rent/utilities of the practice's primary office B. An integrated computer system for paperless documentation by all staff C. Staff vacation and sick time D. Supplies of items used in in-home therapy sessions

C. Staff vacation and sick time -direct expense are costs with OT services (salaries/sick time); day-to-day expenses -computers are capital expenses; long-term benefit ti business -supplies are variable expense -rent are fixed expense -utilities are indirect expense

A client is recovering from an exacerbation of rheumatoid arthritis and is participating in outpatient occupational therapy. The initial evaluation indicates decreased active ROM of bilateral wrists and hands, bilateral ulnar drift, moderate edema, and a pain rating of 7 out of 10 on the visual analog scale. The client currently depends on the spouse to assist with all ADL, but has a goal to increase independence with self-care tasks. Which of the following statements would be BEST to include as a short-term goal in the client's intervention plan? A. Control MCP joint ulnar deviation by the end of the second session using bilateral hand-based splints during self-feeding. B. Increase active ROM of the digits to within functional limits for independence with grooming by the sixth occupational therapy session. C. Subjective report of pain during dressing tasks will decrease by 2 levels on a visual analog scale within 2 weeks. D. Verbalize daily use of joint protection techniques during oral hygiene tasks within 2 weeks.

C. Subjective report of pain during dressing tasks will decrease by 2 levels on a visual analog scale within 2 weeks.

An OTR® has been treating a client who sustained a chemical burn to both hands 6 months ago while cleaning up a spill at the adhesive manufacturing plant where the client is employed as a janitor. The client has been participating in a work conditioning program for the past 6 weeks and has made gains in all areas. The client plans to transition back to full-time, full-duty employment within the next 2 weeks. The OTR® included instruction in proper body mechanics as part of the client's treatment program to reduce the client's risk for reinjury. What type of intervention would this treatment program be considered? A. Primary intervention B. Secondary intervention C. Tertiary intervention D. Wellness intervention

C. Tertiary intervention -the whole treatment program, not just body mechanics

The client lives in a long-term care facility and uses a wheelchair to get to the dining room. The OTR® has removed the client's wheelchair footrests. Which reason BEST explains why the OTR® removed the footrests from the wheelchair? A. Staff push the client, and the footrests get in the way. B. The client's legs are short, so the client does not need the footrests. C. The client propels the chair using only the feet. D. The client propels the chair using only the hands.

C. The client propels the chair using only the feet.

An OTR® is reviewing the medical chart of a client who has Parkinson's disease. The neurologist has indicated that the client has a festinating gait. What does the client's gait look like? A. The client's steps are stiff and slow. B. The client's steps are halting and unsteady. C. The client's steps are small and rapid. D. The client's steps are marked by pauses.

C. The client's steps are small and rapid

A toddler with arthrogryposis has poor extremity strength, minimal active ROM and bilateral elbow extension contractures. The child sits independently when placed and rolls, but does not crawl. To increase functional ability, which should the therapist provide? A. An UE strengthening program B. Bilateral volar cock-up splints C. Trunk rotation and control activities D. Mobile arm support

C. Trunk rotation and control activities

A Level II fieldwork OT student and a supervising OTR are meeting a client for the first time. The OT student greets the client by stating, "I am the occupational therapist who will be working with you." The supervising OTR immediately informs the client that the student is completing a fieldwork rotation and has not yet graduated from an OT program. After the session, the OTR meets with the student to review the professional code of ethics. Which ethical principle directly relates to this situation? A. Beneficence B. Fidelity C. Veracity

C. Veracity

An infant with bilateral UE transhumeral amputations is referred by the pediatrician for an OT evaluation. In the initial interview the parent states, " I don't want my child using the feet for playing or anything like that." Which is the therapist's best response? A. Tell the parent that the child will use the feet naturally for ADL and that it would best to get use to this B. Discharge the child from OT since the parent will not comply with the program. C. Work with the parent on what action to take when the child uses feet during activities in OT D. Refer the parent to counseling and ask the parent to return to therapy after counseling has been initiated.

C. Work with the parent on what action to take when the child uses feet during activities in OT

A clubhouse program hires an OT as a consultant. The clubhouse board of directors requests that the OT consultant focus on the development of evening and weekend leisure activities. Which recommendations should the OT make? A. activities to be selected and completed by clubhouse members according to guidelines provided by OT B. activities to be selected by OT with written instructions provided to the group attendees for activity completion C. activities to be selected by clubhouse members with activity completion led by clubhouse members

C. activities to be selected by clubhouse members with activity completion led by clubhouse members clubhouse promotes active involvement and decision making between all members

When performing a preprosthetic evalua tion of a patient who had a long above elbow amputation, the OT practitioner should mea sure the circumference of the residual limb at the: A. distal radio-ulnar joint. B. proximal radioulnar joint. C. distal humerus. D. proximal humerus.

C. distal humerus.

One of the key positioning strategies an OT practitioner plans to provide for a patient with a hip replacement while the patient is sitting or supine is to: A. place pillows on the lateral side of the hips. B. elevate the foot on the side of the hip surgery. C. use a foam wedge between the legs of the patient. D. insert a roll under the patient's knees.

C. use a foam wedge between the legs of the patient.

A patient who has global aphasia and flaccid hemiplegia secondary to a CVA has been participating in OT. During an employee safety in-service, a customer service employee, who is a distant relative of this patient, asks the OTR about the patient's progress in OT. Which statement represents an appropriate response for the OTR to provide to this inquiry? A."I think the patient will have a difficult time returning home." B."The patient seems to be satisfied with the overall progress." C."I can't talk about the patient without the patient's approval." D."Based on the recent evaluation, the patient has a long way to go."

C."I can't talk about the patient without the patient's approval."

An OTR is teaching a client, who has COPD, a breathing technique to use when experiencing shortness of breath during an activity. The OTR instructs the client to place one hand on the stomach, inhale deeply, and allow the stomach to rise. Which breathing technique do these instructions represent? A.Pursed lip breathing B.Relaxation breathing C.Abdominal breathing

C.Abdominal breathing

What critical functional advantage is TYPICALLY observed in a client who has a complete C6 spinal cord injury compared to a client who has a complete C5 spinal cord injury? A.Improved gross grasp from innervation of the extrinsic flexors B.Ability to use triceps strength during transfers C.Ability to use the radial wrist extensors to supplement grasp D.Improved trunk control to bend side to side without falling

C.Ability to use the radial wrist extensors to supplement grasp -tenodesis grasp

An OTR who works in a home health setting is providing an intervention to address BADL of a client who had a stroke. At first, the client actively participates in the intervention activity, but begins to perseverate on the same action while brushing teeth. The client's level of arousal declines, and the client starts to drool and then vomit. What action should the OTR take FIRST in this situation? A.Determine if the client has a history of adverse reactions to medication. B.Contact a family member to receive consent for medical intervention. C.Activate emergency medical services by calling 911.

C.Activate emergency medical services by calling 911.

An OTR is completing a feeding evaluation of a 4-year-old child who has mild hypotonia, immature oral motor control, and oral hypersensitivity. The child sits in a standard dining chair during meals and requires moderate to maximum assistance from a caregiver for feeding. When attempting to swallow food the child hyperextends the neck, elevates both shoulders, and has poor lip closure. What information should the OTR include in the INITIAL caregiver instructions based on this observation? A.Methods for using cryotherapy to stimulate facial muscles prior to feeding the child B.Handling techniques for facilitating full forward neck flexion during feeding C.Adaptive positioning techniques for promoting trunk alignment D.Neuromuscular facilitation techniques for promoting head and trunk stability

C.Adaptive positioning techniques for promoting trunk alignment

An OTR is employed at a skilled nursing facility on an hourly basis and is typically scheduled to work 2 shifts each month. The facility-specific clinical protocol directs the OTR to use a standardized assessment tool that is unfamiliar to the OTR. What is the FIRST action the OTR should take in this situation? A.Follow the step-by-step procedural instructions on the test form. B.Request a colleague administer the assessment tool when needed. C.Alert the therapy supervisor that additional training is needed.

C.Alert the therapy supervisor that additional training is needed.

Which situation listed below warrants a referral to a gastroenterologist? A.A resident who recently started only eating the food placed on the right side of the plate. B.A client who has diabetes and is experiencing more frequent episodes of hyperglycemia. C.An adolescent who has cerebral palsy who refuses to eat and reports frequent episodes of reflux.

C.An adolescent who has cerebral palsy who refuses to eat and reports frequent episodes of reflux.

An OTR is working with a 4-year-old child, who has autism spectrum disorder and oral hypersensitivity. The child's parent is concerned about an upcoming dental appointment, and asks the OTR for strategies that would enable the child to have a positive experience at the dentist. Which of the following is the MOST BENEFICIALintervention activity to prepare the child for the dental appointment? A.Teach the child to gesture with both arms when experiencing overstimulation. B.Identify a meaningful reward to give to the child after the appointment. C.Apply vibration to the child's gums and cheeks using an electric toothbrush.

C.Apply vibration to the child's gums and cheeks using an electric toothbrush.

A patient has moderate flexor spasticity in the dominant upper extremity secondary to a TBI. During a self-care session, the OTR plans to inhibit the patient's muscle tone prior to encouraging active movement. Which method would be MOST EFFECTIVEto include as part of the intervention session? A.Administering a light, quick stretch to the patient's triceps when reaching forward B.Applying resistance to the patient's maximally contracted biceps at the start of the session C.Applying firm, prolonged manual pressure across the patient's biceps tendon prior to a BADL task D.Positioning the patient's shoulder in abduction and external rotation during the task

C.Applying firm, prolonged manual pressure across the patient's biceps tendon prior to a BADL task -Applying firm, prolonged manual pressure across the patient's biceps tendon is an inhibition technique.

An OTR is developing an intervention plan for a child who has multiple physical disabilities. Which contextual factors are MOST IMPORTANT for the OTR to consider as part of this decision making process? A.Number of intervention sessions approved by the child's health care plan B.Techniques and equipment needed to implement the intervention plan C.Parent goals and research studies supporting the proposed intervention

C.Parent goals and research studies supporting the proposed intervention -using the principles of evidence-based and client-centered practice are most important.

An OTR is using the multicontext approach to guide intervention with an older adult client who has executive dysfunction secondary to a traumatic brain injury. One of the client's goals is to be able to construct a birdhouse. What is the FIRST action the OTR should take to incorporate this approach in the intervention session? A.Teach the client methods to prevent risk of personal injury during the activity. B.Provide the client with step-by-step instructions needed to build the birdhouse. C.Ask the client to predict the steps of the task that will be most challenging.

C.Ask the client to predict the steps of the task that will be most challenging. - multi context emphasizes self-awareness and prediction of performance abilities

Which action should an OTR, who works in a hospital setting, take at the start of an intervention session with an inpatient who has frequent seizures? A.Dim the lights and create a calm therapeutic environment for the session. B.Determine if the patient takes anti-seizure medication as prescribed. C.Ask the patient if an aura is typically associated with seizure activity.

C.Ask the patient if an aura is typically associated with seizure activity. -Knowing this information allows the OTR to be prepared to take necessary action and maintain the patient's safety in the event of seizure activity. -aura is a feeling that seizure will happen

A client who works as a hair stylist has cubital tunnel syndrome. The OTR is providing the client with instructions about work modification techniques the client should use to minimize exacerbation of symptoms. What should the OTR include as part of these instructions? A.Use a small handheld vacuum instead of a push broom to remove hair from around the styling chair. B.Wear a neoprene elbow sleeve on the affected arm as much as possible when cutting and styling hair. C.Avoid holding the affected elbow in 90°of flexion or greater, and actions that require repetitive elbow flexion.

C.Avoid holding the affected elbow in 90°of flexion or greater, and actions that require repetitive elbow flexion.

An OTR working in an inpatient rehabilitation facility is scheduled to complete an initial grooming and hygiene assessment with a patient. The patient has mild hemiplegia and neurobehavioral deficits secondary to a CVA one week ago. Which area of the facility should the assessment take place in order to obtain the MOST BENEFICIAL information about the impact of these symptoms on the patient's occupational performance? A.Simulated environment in the OT clinic B.Bedside in the patient's hospital room C.Bathroom in the patient's hospital room

C.Bathroom in the patient's hospital room

An 8-year-old child sustained second-degree burns to the first web space of both hands one month ago. Results of a reevaluation indicate the child's web space is contracting despite wearing pressure garments, using night orthotics and completing home program activities. What additional action should the OTR take based on these findings? A.Use a paraffin modality during OT sessions to soften the scar prior to functional activity. B.Advise the caregiver to increase the intensity and frequency of passive ROM exercises. C.Begin serial splinting that incorporates a polymer gel sheet over the affected areas. D.Provide the caregiver with a list of age-appropriate games that will promote hand use.

C.Begin serial splinting that incorporates a polymer gel sheet over the affected areas.

What information MUST be documented in the Individualized Education Program plan for students who meet eligibility criteria for occupational therapy services? A.Statement of medical necessity for skilled school-based OT services B.Number of therapy minutes required for each OT session C.Beginning and ending dates including location of OT services

C.Beginning and ending dates including location of OT services

A high-school student with autism spectrum disorder excels academically but has difficulty organizing assignments and homework. The OTR plans to evaluate the student's relative strengths and weaknesses to guide intervention planning. One of the student's goals is to learn an organization system. Which of the following is the BEST assessment for this purpose? A.Canadian Occupational Performance Measure (COPM) B.Executive Function Performance Test (EFPT) C.Behavior Rating Inventory of Executive Function (BRIEF)

C.Behavior Rating Inventory of Executive Function (BRIEF)

An OTR places a 6-month infant in a supine position and observe body movements while rotating the child's hips to one side and then to the other. Which automatic reaction is the OTR screening when following this procedure? A.Equilibrium reaction B.Protective response C.Body on body righting

C.Body on body righting

An OTR is providing information about assistive devices to a client who has osteoarthritis of the first CMC joints of both hands. Which devices would be MOST BENEFICIAL for protecting these joints during meal preparation? A.Mixing bowls with non-slip bottom, scoop dish, and cutting board with slicing guide B.Right-angled knife with ergonomic handle, over-stove mirror, and swivel serving utensils C.Built-up utensils, adjustable bowl tipper, and mountable electric can opener D.Washable universal cuff, rocker knife with contoured handle, and wall-mounted jar opener

C.Built-up utensils, adjustable bowl tipper, and mountable electric can opener

A student in the second grade has autism spectrum disorder and is scheduled to begin school-based OT. The teacher reports the student has difficulty attending to academic tasks and typically has outbursts when in close proximity to other people. In which environment should the majority of the student's intervention sessions take place? A.Self-contained occupational therapy treatment room B.Playground in an area apart from other students C.Classroom during routine curriculum-based activities

C.Classroom during routine curriculum-based activities

An OTR is using the principles of evidence-based practice (EBP) to guide professional decision making. The OTR knows the first step of EBP involves generating a pertinent question and identifying a need. Which option represents the MOST BENEFICIAL source for identifying a relevant topic for investigation? A.Quantitative results of a randomized control trial B.Analysis of practice trends in a scholarly database C.Clinical issues encountered during everyday practice

C.Clinical issues encountered during everyday practice

An OTR working in a pediatric hospital-based clinic wants to begin using a newly developed listening device as part of the OT intervention for children diagnosed with autism. What information is MOST IMPORTANT for the OTR to determine prior to using this device with these children? A.Third-party payers reimburse the device as a therapeutic modality. B.Standard protocol to use with each child who will use the device. C.Clinical practice guidelines and evidence related to using the device. D.Plan to monitor the effectiveness of the new intervention.

C.Clinical practice guidelines and evidence related to using the device.

An inpatient is preparing for discharge to home after completing 3 months of inpatient rehabilitation. The OTR is reviewing documentation in the patient's contact notes and determines the patient is still working to achieve several short-term goals related to the current treatment plan. What INITIAL action should the OTR take based on this finding? A.Ensure durable medical equipment delivery and home health visits are scheduled in preparation for the patient's discharge. B.Prepare a discharge summary providing a rationale for the goal shortcomings noted in the contact notes. C.Complete a comprehensive re-evaluation to identify current function in relation to the discharge plan. D.Discuss options with the interprofessional team for extending inpatient rehabilitation until goals are achieved.

C.Complete a comprehensive re-evaluation to identify current function in relation to the discharge plan.

A family practice physician referred a client to outpatient OT for conservative treatment of carpal tunnel syndrome of the dominant hand. The client reports a 3 month history of numbness and difficulty manipulating objects. Symptoms interfere with work as a jack-hammer operator for a road works department. Evaluation results indicate loss of protective sensation, 11 mm two-point discrimination, and thenar muscle atrophy. Active ROM of the affected hand is within normal limits. What INITIAL actions should the OTR take in addition to fabricating a volar wrist orthosis? A.Advise the client to wear the orthosis when sleeping and a padded glove when at work. B.Arrange an appointment with an orthopedic surgeon and begin a sensory reeducation program. C.Conduct Phalen's test and report findings of the overall evaluation process to the physician.

C.Conduct Phalen's test and report findings of the overall evaluation process to the physician. -collaborate with physical on overall intervention plan

An OTR is providing supervision to a rehabilitation aide, who assists a patient with bathing. During the initial OT evaluation, the patient identified independence in bathing as a priority goal and was assessed to require 25% assistance. However, the aide tells the OTR that the patient wants total assistance during bathing sessions and frequently asks the aide to complete tasks that the patient can perform. What is the FIRST action the OTR should take to support progress toward the patient's bathing goal? A.Document the discrepancy in the patient's performance level between the OTR and the aide. B.Plan to re-evaluate the patient's level of assistance for bathing during the next scheduled session. C.Conduct a joint session with the patient and the aide to review goals for bathing.

C.Conduct a joint session with the patient and the aide to review goals for bathing.

A client is in the early stages of a slow, progressive upper motor neuron disease. Mild intention tremors and fatigue interfere with completion of typical daily tasks and ability to work a full day as an accountant. Currently, the client ambulates with a cane in the home and uses a wheelchair for community mobility. The client's goal is to remain working as long as possible. Which work accommodation meets the employer's obligation for this client as required by the Americans with Disabilities Act? A.Employee review to change the essential elements of the job description B.Modification of doorways throughout the workplace for maximal accessibility C.Consideration for modifying the client's current work schedule

C.Consideration for modifying the client's current work schedule

A client in an outpatient setting has hemiplegia secondary to a CVA. Over the past several weeks, there has been a decline in the client's energy level, ability to concentrate, and interest in intervention activities. When asked about the change, the client replies: "I just can't sleep at night thinking about the burden I am to my family." What INITIAL action should the OTR take based on this observed change? A.Advise the client to consult with a psychiatrist B.Adjust the timeframes for achieving short term goals C.Consult with the client's primary physician

C.Consult with the client's primary physician

What type of precautions MUST an OTR who works in a hospital setting take if an inpatient has been diagnosed with vancomycin-resistant enterococci (VRE)? A.Standard B.Airborne C.Contact

C.Contact

An inpatient had coronary artery bypass graft (CABG) surgery 3 days ago. While sitting on the edge of the bed during a grooming session, the patient's heart rate increases 10 beats per minute above the patient's resting heart rate. What INITIALaction should the OTR take based on this observation? A.Discontinue the activity and help the patient transfer to a side-lying position in bed. B.Document this response and other clinical observations in the client's medical record. C.Continue to monitor vital signs and proceed with the planned intervention session.

C.Continue to monitor vital signs and proceed with the planned intervention session.

An OTR is selecting a new seating system for a college student who has cerebral palsy and uses a wheelchair for all functional mobility. The client has flexible asymmetrical postures including scoliosis, kyphosis, and pelvic obliquity. The student wants to be able to sit upright for 12-14 hours per day. Which custom seating system would BEST support progress toward this goal? A.Modular B.Planar C.Contoured

C.Contoured -A contoured cushion is beneficial for this client who is at-risk for skin breakdown and has moderate positioning and seating needs.

An outpatient client has fixed range of motion at the elbow secondary to spasticity and prolonged immobility. What is the PRIMARY reason for this impairment? A.Rupture of forearm muscles B.Hypertrophy of triceps brachii C.Contracture of the bicep tendon

C.Contracture of the bicep tendon

A 5-year-old child sustained partial and full thickness burns on the volar surfaces of both wrists and forearms 3 months ago. Although the child wears pressure garments, scarring across the wrist is limiting wrist mobility. Which activity could be graded to MOST EFFECTIVELY help with increasing the child's wrist motion? A.Moving a parachute up and down during parachute games with peers B.Tossing a bean bag at a target placed at varying distances from the child C.Creeping on hands and knees through a play tunnel maze D.Bouncing a medium-size therapy ball from one hand to the other

C.Creeping on hands and knees through a play tunnel maze -weight bearing will assist in elongation

An OTR completed an initial assessment of a second-grade student with a learning disability. Results indicate the student has average fine motor skills, but test scores on a standardized test of visual perceptual skills are well below normal. Which of the following activities would be MOST challenging for the student based on the outcomes of the evaluation? A.Opening lunch containers B.Organizing school supplies C.Cutting on a bold, thick line

C.Cutting on a bold, thick line

A school-based OTR is selecting seating alternatives for a student who has moderate hypotonia and has just transitioned to a full-day kindergarten program. The student uses a wheelchair for mobility and does not tolerate an upright sitting position throughout the school day. What type of positioning system would be MOST BENEFICIAL for this student? A.Modular wheelchair with tilt-in-space feature in the mobility base B.Corner chair with high lateral supports that can be placed on the floor C.Dense foam lateral supports and gel cushion for the current wheelchair D.Lightweight chair with reclining back and reverse wheel configuration

C.Dense foam lateral supports and gel cushion for the current wheelchair -This allows positioning of the trunk posterior to the pelvis and accommodates for the forces of gravity against upright positioning.

An OTR, who works in a hospital setting, receives a phone call from an OT supervisor who has left for the day. The supervisor says to the OTR, "I forgot to enter a progress note for a patient who I provided treatment to earlier today. Could you enter the progress note in the chart for me?" How should the OTR respond to this request? A.Comply with the directive because it is requested by a supervisor. B.Enter the note into the patient's chart using the supervisor's name and credentials. C.Deny the request, and provide a reason why this practice is unacceptable.

C.Deny the request, and provide a reason why this practice is unacceptable.

A client sustained a severe hand injury 5 weeks ago. During an OT session, the client reports that family responsibilities make it impossible to complete the prescribed exercise and splinting program. What action should the OTR take in response to this comment in order to promote the client's successful participation in the home program? A.Analyze a 24-hour log to determine time management issues the client is experiencing. B.Suggest transition to a compensatory approach for dealing with residual deficits. C.Determine a home program that closely aligns with typical performance patterns. D.Advise the client to make the home program the highest priority for the short term.

C.Determine a home program that closely aligns with typical performance patterns.

A client who is recovering from a severe hand injury uses Medicaid as the primary source of reimbursement for OT services provided at a hand therapy clinic. The client plans to relocate to an adult child's home in another state even though the client requires continuation of OT services. Upon learning this information, what recommendation regarding transitional services should the OTR make to the client? A.Have a family member locate a comparable hand therapy clinic near the new address. B.Notify the current Medicaid provider to transfer benefits based on the new address. C.Determine the Medicaid benefits that will be authorized based on the new address.

C.Determine the Medicaid benefits that will be authorized based on the new address. -Medicaid benefits are a state-regulated program and eligibility requirements vary by state. The OTR should advise the client to determine the Medicaid benefits based on the new residential address.

An OTR is interpreting the results of a standardized test on visual perceptual skills administered to a student in second grade who has a developmental delay. The student scored within the average range for all subtests but scored below the norm on the visual closure subtest. Based on these results, which activity would be MOST DIFFICULT for this student? A.Recognizing the similarities between a picture of a mouse in a storybook and the class pet mouse in a cage B.Locating coat and hat hanging on a hook in the hallway before going outside to play on the playground C.Determining the difference between a crayon and a marker when both are partially covered by a book

C.Determining the difference between a crayon and a marker when both are partially covered by a book

Which medical conditions would TYPICALLY decrease the rate of wound healing? A.Guillain-Barré syndrome, chronic obstructive pulmonary disease (COPD), asthma B.Alzheimer's disease, rheumatoid arthritis (RA), depression C.Diabetes mellitus, acquired immune deficiency syndrome (AIDS), atherosclerosis

C.Diabetes mellitus, acquired immune deficiency syndrome (AIDS), atherosclerosis

An OTR working in a pediatric outpatient setting is teaching an 8-year-old child who has osteogenesis imperfecta to transfer from the floor to standing. At the start of the transfer, the child slips and falls, hitting the left shoulder on the floor. As the OTR attends to the child, the OTR observes that the child is wiping away tears, despite stating that the shoulder is fine. After comforting the child and completing an incident report, what action should the OTR take NEXT in this situation? A.Continue the session while monitoring the child's functional movement patterns. B.Complete a functional evaluation of the affected upper extremity. C.End the session until the incident is discussed with the referring physician.

C.End the session until the incident is discussed with the referring physician.

An OTR works for a large hospital system that provides a range of services including medical care and elective surgeries for pediatric and adult populations. For each scheduled shift, the OTR is assigned to a different patient care unit based on the daily census. What ethical obligation does the OTR have for upholding professional responsibilities in this situation? A.Complete at least one continuing education course prior to providing services on each unit. B.Seek clinical supervision from a more experienced OTR who works on the assigned unit. C.Engage in a variety of professional development activities to ensure continuing competence.

C.Engage in a variety of professional development activities to ensure continuing competence.

An OTR who works in a rehabilitation hospital is providing assistive technology recommendations to a patient who has relapsing-remitting multiple sclerosis. Which is the MOST EFFECTIVE method for the OTR use during the intervention session to evaluate the patient's understanding of the information presented? A.Explain to the patient evidence that supports the positive outcome associated with each device. B.Provide the patient with the written instruction manuals from the device manufacturer. C.Engage the patient in conversation to allow the opportunity for questions to be answered.

C.Engage the patient in conversation to allow the opportunity for questions to be answered.

An OTR who works in an outpatient setting is trying to obtain consent from an older adult client who reports occasional forgetfulness. What action should the OTR take before asking the client to sign the consent form? A.Print the written plan of care and allow time for the client to review. B.Explain details of the planned intervention to a family member. C.Ensure the client understands the recommended plan of care.

C.Ensure the client understands the recommended plan of care.

An OTR is critically reviewing evidence to provide recommendations for a client who has rheumatoid arthritis and who wants to know if attending a water aerobics class would be beneficial. The OTR searches a scholarly database and finds 14 relevant studies. What action should the OTR take to determine the MOST BENEFICIAL evidence to support water aerobics recommendations for this client? A.Ask experienced practitioners on a social media site to recommend the best study. B.Share the results of the search with the client and ask the client to select their preferred studies. C.Evaluate the studies using an organized methodology designed to appraise the evidence.

C.Evaluate the studies using an organized methodology designed to appraise the evidence.

An OTR is providing consultation to an early intervention program. The OTR has developed a weekly parenting skills group that is being facilitated by a COTA. After 3 weeks, the COTA reports that group attendance is poor. What action should the OTR take FIRST? A.Observe and provide feedback on the COTA's leadership skills. B.Suggest that the COTA advertise the group in the community C.Explore possible reasons for low attendance with the COTA

C.Explore possible reasons for low attendance with the COTA

An inpatient had a total hip replacement 3 days ago. During an intervention session, neither the patient nor the spouse appear interested in learning about the assistive devices for improving the patient's independence with BADL. What INITIAL action should the OTR take based on this observation? A.Document the reactions in the client's record and inform the care coordinator. B.Explain that assistive devices are essential to the patient's recovery. C.Explore the couple's feelings about using the equipment.

C.Explore the couple's feelings about using the equipment.

During the administration of a standardized assessment, a client demonstrates difficulty initiating a motor task. What action MUST the OTR take to obtain accurate results from this assessment tool? A.Modify the instructions to enhance clarity B.Provide prompts and tactile cues, as needed C.Follow the test protocol for giving additional directions

C.Follow the test protocol for giving additional directions

An OTR who works in an inpatient rehabilitation setting is completing an initial evaluation of a patient with relapsing-remitting multiple sclerosis. The OTR wants to measure the amount and type of assistance required for the patient during ADL. Which standardized assessment would be BEST for this purpose? A.Executive Function Performance Test (EFPT) B.Canadian Occupational Performance Measure (COPM) C.Functional Independence Measure (FIM™)

C.Functional Independence Measure (FIM™)

A patient in a skilled nursing facility had a CVA one week ago. An initial screening indicates the patient has hemiplegia, ambulates using a quad cane and has good memory. Nursing staff report the patient consistently has difficulty finding the way from the dayroom to the dining room. What type of assessment should be included as part of the initial evaluation to determine the underlying neurobehavioral problem associated with this difficulty? A.Cognitive-behavioral assessment of executive function during a familiar ADL task B.Attention and depth perception subtests from a standardized cognitive assessment C.Functional assessment of topographical orientation and visual perception

C.Functional assessment of topographical orientation and visual perception

An OTR is reviewing a research article investigating the role of peer support among members living in a retirement community. Data collection methods included observation, note-taking and semi-structured interviews conducted with members over a 6 month period. Constant comparison and open coding were undertaken to categorize core concepts. A conceptual model identifying 5 tiers of peer support was constructed based on members' experience. Which type of qualitative research design does this investigation represent? A.Critical theory study B.Phenomenological study C.Grounded theory study D.Participatory action research

C.Grounded theory study -Grounded theory is a research method in which there is continuous comparison between collected data and interpretation, which results in a set of categories and emerging theory.

An OTR is providing intervention for an infant who has lower extremity spasticity secondary to cerebral palsy. The OTR is teaching the caregiver, who is a kinesthetic learner, handling techniques to inhibit the lower extremity spasticity while changing the infant's diaper. Which is the MOST EFFECTIVE method for the OTR to promote the caregiver's understanding of the information presented? A.Offer to demonstrate the techniques at the contextually appropriate time when the diaper is soiled. B.Provide the instruction in the natural setting where diapers are typically changed. C.Have the caregiver demonstrate the inhibition method used during diaper change.

C.Have the caregiver demonstrate the inhibition method used during diaper change.

A high school student has spastic diplegia and cognitive delay. The student self-propels a wheelchair and requires moderate assistance for functional transfers. The student has been the target of peer-bullying in school and has been attending OT to help increase social participation. One of the student's goals is to attend a dance being planned by the dance committee of student volunteers. During a role-play session, the student tells the OTR, "I really want to go to the dance, but I am worried that I won't fit in." Which recommendation should the OTR suggest for addressing the student's concern? A.Ask the committee to recruit parent volunteers willing to attend the dance as facilitators of social inclusion. B.Require the committee leader to provide inclusion training for all students at a school assembly prior to the dance. C.Have the student partner with the committee to select a music playlist for the dance that includes modified line dances.

C.Have the student partner with the committee to select a music playlist for the dance that includes modified line dances. -This activity supports the student in social participation with peers both during the planning phase and at the school dance.

An outpatient client is participating in OT. During the intervention session, the client gestures toward another client participating in an intervention activity in the clinic and asks the OTR, "What happened to that client?" Which statement is an appropriate response for the OTR to provide? A.The client was involved in a motor vehicle accident. B.You may ask the client to explain personal information. C.I cannot share information about the client's situation.

C.I cannot share information about the client's situation.

An OTR working in an acute rehabilitation facility is planning intervention for an adolescent who sustained bilateral femur fractures one week ago. The adolescent is restricted to non-weight-bearing status for at least 6 weeks. Currently, the adolescent is independent with grooming while seated and requires moderate assistance with lower body dressing, bathing and transfers. One of the goals is for the adolescent to be independent with ADL prior to discharge. What action is MOST IMPORTANT for the OTR to take prior to selecting intervention activities for this adolescent? A.Complete a wheelchair seating and mobility assessment. B.Determine a projected timeline for full recovery. C.Identify barriers in the expected discharge context.

C.Identify barriers in the expected discharge context.

A student, who has autism spectrum disorder, becomes increasingly agitated during the school's holiday program. What is the FIRST action the OTR should take based on this observation? A.Instruct the student in visual imagery relaxation exercises. B.Provide a visual timer to cue the student when the program is over. C.Identify the behavioral trigger to develop a solution.

C.Identify the behavioral trigger to develop a solution.

An inpatient in a rehabilitation setting sustained a C7 spinal cord injury 2 months ago. One of the patient's goals is to be able to prepare family meals when discharged home. What INITIAL action should the OTR complete to support the patient's success with this goal? A.Provide the patient with assistive devices to use in the kitchen. B.Observe current physical skills and abilities during a typical kitchen task. C.Identify the patient's typical mealtime routines and habits.

C.Identify the patient's typical mealtime routines and habits.

An OTR is planning intervention for an outpatient client who had a CMC thumb arthroplasty 1 week ago. What should the OTR include as part of the INITIAL intervention plan at this stage of recovery? A.Encourage active range of motion of the affected hand within tolerance of pain. B.Begin gentle passive range of motion at the wrist and thumb CMC. C.Immobilize the wrist and thumb CMC and MCP joints while allowing AROM of the IP joint.

C.Immobilize the wrist and thumb CMC and MCP joints while allowing AROM of the IP joint.

A 2-year-old child has developmental delay due to mild spastic cerebral palsy. The child has mastered four-point positioning. Which movement component should the OTR plan to facilitate NEXT? A.Creeping on hands and knees B.Unsupported upright sitting C.Rocking on hands and knees D.Scooting on the belly

C.Rocking on hands and knees

A rheumatologist has prescribed bilateral nighttime resting hand orthotics for a child who has early stage juvenile rheumatoid arthritis. The parents ask the OTR who works at the child's school to fabricate the orthotics. Despite having this condition, the child is functioning at grade-level and is not on the OT caseload. What action should the OTR take in response to the parents' request? A.Schedule a time after school hours to fabricate the orthotics for the child. B.Initiate an IEP indicating the child's needs for school-based OT. C.Inform the parents to schedule an appointment at an outpatient OT clinic. D.Provide the parents with catalog information for ordering pre-fabricated orthotics.

C.Inform the parents to schedule an appointment at an outpatient OT clinic.

An OTR is working in the school system. A second-grade student is referred for an OT evaluation because of an awkward pencil grasp. Results of the evaluation indicate writing skills are at grade level and a 10% delay in fine motor skills. What action should the OTR take in this situation? A.Recommend the student use keyboarding as primary means of written assignments B.Develop IEP goals to improve the student's fine motor skill development C.Inform the teacher and parents that the student does not qualify for OT services

C.Inform the teacher and parents that the student does not qualify for OT services -Must not meet grade-level to qualify

A private practice OTR is developing goals for a 4-year-old child with autism spectrum disorder who has difficulty with emotional regulation, impulse control, and problem-solving skills. The child attends preschool, and academically is meeting expectations. Socially, the child has not developed friendships at school or within the neighborhood. Which task should be included as a short-term goal in the intervention plan for this child? A.Play at the sand table without acquiring an injury. B.Follow rules while playing an age-appropriate board game. C.Interact with peers during a structured play group.

C.Interact with peers during a structured play group.

An OTR is evaluating an inpatient who recently had a severe TBI and is emerging from coma. The patient is functioning at a Level III (Localized response) on the Rancho Los Amigos scale. Which is the FIRST cognitive function the OTR should assess? A.Ability to learn new information B.Orientation to place and time C.Level of arousal

C.Level of arousal

Which activity represents an effective sensory-based approach for improving tolerance to touch for a 5-year-old child who has mild tactile defensiveness? A.Swinging in a prone position in a net swing B.Playing with a feather boa during a dress-up activity C.Log-rolling to snugly wrap the body in a blanket D.Spinning in a seated position on a scooter board

C.Log-rolling to snugly wrap the body in a blanket -deep touch is better than light touch

Which of the following options represent a modifiable risk factor for readmission that an OTR working in an acute care setting should include in the discharge planning process for an inpatient who is returning home? A.High risk for co-morbid conditions B.Poor living situation C.Low health literacy

C.Low health literacy -MODIFIABLE

Which method would be MOST EFFECTIVE to use when grading an activity to improve muscular endurance? A.Decreasing repetitions and increasing resistance for short intervals of time B.Shortening the interval of time to complete a controlled set of isotonic exercises C.Maintaining the same resistance and increasing the number of repetitions D.Increasing resistance to 75% of maximal strength and maintaining repetitions

C.Maintaining the same resistance and increasing the number of repetitions

What MUST be included in the written Individualized Education Program (IEP) plan developed for a kindergarten student who needs special education? A.Report from primary care physician B.Applicable ICD-10 billing code C.Measurable goals and objectives

C.Measurable goals and objectives

An adolescent is in an acute setting after sustaining a recent C6 (ASIA B) spinal cord injury. The adolescent underwent surgery 4 days ago for cervical stabilization and is recovering from the surgery as expected. The adolescent has been on bedrest since the surgery, but now has medical clearance to transfer out of bed with assistance. One of the goals for the adolescent is to sit in a bedside chair for BADL. Prior to transferring the adolescent out of bed, what action MUST the OTR take? A.Evaluate sensation at key dermatomal segments. B.Complete a comprehensive manual muscle test. C.Measure the adolescent's resting vital signs.

C.Measure the adolescent's resting vital signs.

Which option represents the final phase of the occupational therapy process? A.Providing community-based resources to promote carry-over of abilities into everyday habits and routines B.Supporting the transfer of newly acquired skills to the environments and contexts that are important to the client C.Measuring outcome through assessment of participation, quality of life, performance, and / or satisfaction

C.Measuring outcome through assessment of participation, quality of life, performance, and / or satisfaction -OT process: outcomes

An OTR is developing a client-centered fall prevention program for community-dwelling older adults. The objective of the program is to reduce the participants' risk of falling. Topics the OTR plans to cover include the benefits of having a health care provider review a list of prescribed medications, the advantages of routinely completing an exercise program, and the purpose of having a regular vision examination. Which additional topic would be MOST BENEFICIAL to include in this type of fall prevention program? A.Reviews of healthy eating habits and nutritional information on prepared foods B.Resources to enhance social involvement and engagement in the community C.Methods and tips to modify the home environment to increase personal safety

C.Methods and tips to modify the home environment to increase personal safety

An OTR is collaborating with a home health client to select a new power wheelchair. The client has morbid obesity, chronic obstructive pulmonary disease, diabetes, and severe edema in both lower extremities. Caregivers assist the client to transfer into the chair with a mechanical lift, and the client uses the wheelchair full time for mobility. Based on this situation, which type of power wheelchair would be MOST BENEFICIAL for the OTR to recommend to this client? A.Rear-wheel drive with a seat elevator B.Front-wheel drive with power recline C.Mid-wheel drive with elevating leg rests

C.Mid-wheel drive with elevating leg rests -A mid-wheel drive power wheelchair provides greater stability as the center of gravity is positioned over the drivetrain

A student who is 8-years-old has severe diplegia and is non-ambulatory. The student is referred to OT after the family relocates to a new school district. Which area of occupation should be a priority for the school-based OTR to assess during the INITIAL evaluation? A.Prehensile skills for manipulating eating utensils during lunchtime B.Perceptual abilities for copying from the classroom lesson board C.Motor skills for managing clothing during toilet transfers

C.Motor skills for managing clothing during toilet transfers -related to kids diplegia

In addition to completing an occupational profile, which evaluation methods are TYPICALLY used in a comprehensive OT assessment of a student in preschool who has a developmental delay? A.Projective techniques and client self-report B.Review of medical records and diagnostic reports C.Naturalistic observation and standardized testing

C.Naturalistic observation and standardized testing

An OTR wants to measure the effectiveness of a pre-vocational group at a mental health day program. What data is MOST IMPORTANT to collect for this purpose? A.Client satisfaction ratings completed by participants after discharge B.Quality of the client's interview responses during role-play activities with peers C.Number of clients who successfully enter employment positions following the group D.Attendance levels and completion rate of homework assignments

C.Number of clients who successfully enter employment positions following the group

An OTR is providing recommendations to an outpatient client who has a chronic stage 3 pressure ulcer of the coccyx. One of the client's goals is to learn self-management strategies to promote wound healing. As part of the intervention plan, the OTR provides the client with a recommendation for a new seating system and instructions in pressure relief techniques. What additional topics should the OTR include in the client education to promote effective wound healing? A.Strengthening and therapeutic exercise B.Pain management and desensitization C.Nutritional intake and adequate hygiene

C.Nutritional intake and adequate hygiene

An OTR is administering the Functional Independence Measure (FIM™) as part of the initial evaluation of an inpatient who had a left CVA with aphasia 5 days ago. The patient has right hemiparesis and requires moderate assistance to maintain balance while standing. What action should the OTR take when completing this assessment? A.Provide the patient with visual and tactile cues to sequence each step of the task. B.Introduce a reacher and other adaptive equipment to maximize patient's level of performance. C.Observe as the patient performs ADL routines with the typical methods used prior to admission.

C.Observe as the patient performs ADL routines with the typical methods used prior to admission.

A 16-month-old child who has severe spasticity is receiving OT in the home. The parents, who both have intellectual disabilities, have been trained in a stretching program for the child. Which option represents the BEST method to assess the parent's understanding of the program? A.Request a home health aide perform routine monitoring. B.Have the parents complete an exercise log. C.Observe the parents demonstrate the stretching regimen.

C.Observe the parents demonstrate the stretching regimen.

An entry-level OTR accepts a new job that includes providing services at two skilled nursing facilities and supervising an experienced COTA. Prior to starting the new position, the OTR wants to know the supervision requirements for occupational therapy assistants. Where must the OTR locate regulations for COTA supervision? A.National Board for Certification in Occupational Therapy (NBCOT) standards of practice B.American Occupational Therapy Association (AOTA) code of ethics and ethical standards C.Occupational Therapy Practice Act of the state's licensure board

C.Occupational Therapy Practice Act of the state's licensure board

An OTR is developing an intervention plan for an inpatient who has severe post-traumatic stress disorder (PTSD). Symptoms of PTSD started several weeks after the patient was robbed in a convenience store where the patient was working. The patient's goal is to resume work at the store, but extreme fear and distrust interfere with the ability to interact with customers. Which environment is MOST conducive for promoting initial progress toward a return-to-work goal with this patient? A.During a role-play session in the therapy room B.Discussion group with several other patients C.One-on-one in the patient's hospital room

C.One-on-one in the patient's hospital room -INITAL then progress to role-play

A client who is legally blind reports having difficulty locating grooming items in the bathroom every morning, resulting in being late for work. Which recommendation should the OTR suggest for improving the client's occupational performance? A.Complete the majority of bathing and grooming tasks the night before. B.Wake-up at least one hour earlier so that grooming will not be rushed. C.Organize items in the bathroom so that there is a specific place for each item. D.Discuss with the employer options for implementing a later start time at work.

C.Organize items in the bathroom so that there is a specific place for each item.

An OTR is reviewing the clinical record of a client who has a wound on the dorsal surface of the hand that is slow to heal. Which of the following sets of medical conditions are TYPICALLY associated with delayed wound healing? A.Rheumatoid arthritis and gout B.Superficial partial thickness burn with exudate C.Peripheral vascular disease and diabetes

C.Peripheral vascular disease and diabetes

A student in sixth grade has autism spectrum disorder. During an OT intervention session, the student starts banging head on the floor. Which is the FIRST action the OTR should take in this situation? A.Provide calming verbal input. B.Call for the school counselor. C.Place a soft mat under student's head.

C.Place a soft mat under student's head.

A client in a home health setting had bilateral lung transplants one month ago and is participating in OT to improve activity tolerance for completing home management tasks. The client is currently able to perform laundry tasks but develops shortness of breath when standing for longer than 2 minutes. The client has a stacked washer and dryer located in the hallway between the kitchen and the main bedroom. Which energy conservation technique should the OTR teach this client to use to MOST EFFECTIVELY improve the client's activity tolerance during this home management task? A.Transfer clothing from the washer to the dryer all at one time to limit upward reach. B.Use a single basket for weekly laundry to minimize trips to the laundry room. C.Place clothing on a wheeled cart to bring the clothes from the bedroom to the laundry area.

C.Place clothing on a wheeled cart to bring the clothes from the bedroom to the laundry area.

An inpatient had a total hip arthroplasty 2 days ago with post- operative precautions that include touchdown weight-bearing (TDWB). Which option BEST represents instructions an OTR should provide to reinforce the touchdown weight-bearing (TDWB) precaution? A.Use pain as the limiting factor and place up to 100% of body weight through the operative extremity. B.Learn to apply up to 50% of body weight through the operative extremity. C.Place the toes of the operative extremity on the floor to maintain balance but do not bear weight.

C.Place the toes of the operative extremity on the floor to maintain balance but do not bear weight.

An OTR working in an adult day care center is preparing activities to use during a gardening group for clients who have moderately severe cognitive decline secondary to dementia. The goal of the group is to increase levels of arousal and engagement. Which activity would provide the BEST opportunity for achieving this goal? A.Assigning clients specific plants to water in the center B.Selecting seeds from an illustrated gardening catalog C.Planting flowers in an indoor raised garden bed

C.Planting flowers in an indoor raised garden bed -not complex and immediate gratification

A resident recently was admitted to a long term care facility after having an intracerebral hemorrhage secondary to a ruptured aneurysm. Currently, the resident is able to verbalize basic needs, can follow simple one-step directions, and has minimal functional movement of all extremities. The resident wants to be able to change channels on the television independently but is unable to hold the remote control device or apply enough pressure on the buttons to activate the device. The OTR is choosing a switch-activated selection device that will enable the resident to complete this task. Which type of switch is OPTIMAL to use with this device? A.Phonation switch B.Pneumatic switch C.Plate switch

C.Plate switch -A plate switch responds to minimal force, so it allows this resident to independently access the channels on the television.

During an initial evaluation, a resident who has poor balance and ataxia repeatedly attempts to stand up from the wheelchair. Which option BEST describes this behavior? A.Impaired attention B.Decreased concentration C.Poor judgment

C.Poor judgment

An inpatient in an intensive care unit is in the initial acute phase of Guillain-Barré syndrome. The OTR modified the nurse call bell to allow the patient to use available neck movement to push against a switch attached to the pillow. What additional interventions would be MOST BENEFICIAL for the patient during this phase of the disease process? A.Balance and vestibular training for self-care activities B.Therapeutic exercise program for strengthening C.Positioning and PROM to minimize contractures

C.Positioning and PROM to minimize contractures

An inpatient who had a left CVA one week ago is participating in a dressing session. After putting on a sock during lower body dressing, the patient repeatedly attempts to pull the sock up even though it is already in place. What neurobehavioral deficit is MOST CONSISTENT with these actions? A.Dressing apraxia B.Spatial inattention C.Premotor perseveration D.Somatagnosia

C.Premotor perseveration

Which of the following is the BEST example of a primary objective for a health promotion program for people with chronic conditions? A.Teach information about medical interventions and service providers B.Facilitate healing and recovery from existing disease processes C.Prevention of secondary conditions to enhance overall quality of life

C.Prevention of secondary conditions to enhance overall quality of life

What sensory system is TYPICALLY associated with a student who consistently stomps feet when walking in the hallway at school? A.Vestibular B.Tactile C.Proprioceptive

C.Proprioceptive -stomps to increase spatial awareness

Which option represents the code of professional behavior associated with the ethical principle of autonomy and confidentiality? A.Maintaining concern for the safety and well-being of the client throughout the occupational therapy process B.Attaining and maintaining competency in selecting, administering, and interpreting evaluation tools to avoid doing harm to the client C.Protecting the privacy of the client's information and being respectful of the client's independence

C.Protecting the privacy of the client's information and being respectful of the client's independence

An OTR is working with a 2-year-old toddler who has a gastrostomy feeding tube for nutritional support. The toddler has made improvement in oral motor function and has medical clearance to transition to oral feeding. During sessions, the toddler engages in oral play activities with a pacifier and blows bubbles. Which intervention technique would be MOST BENEFICIAL to include at this stage of the toddler's oral motor development for supporting successful transition to oral feeding? A.Have the toddler practice dry swallows by pretending to eat from a spoon. B.Place a compressed bolus of a sweet-tasting food between the cheek and gums. C.Provide flavors and tastes by dipping a spoon or chew toy in pureed foods.

C.Provide flavors and tastes by dipping a spoon or chew toy in pureed foods. -promote oral exploration -A is for motor skills -B is for sensory

A client has an upper extremity flexor synergy secondary to a CVA. The client has developed a severe soft-tissue contracture of the affected elbow. Which method would be MOST EFFECTIVE for increasing soft tissue length for improving elbow extension? A.Perform passive ROM to the affected elbow with high-load stretch for brief periods of time. B.Apply a long-arm cast while the affected elbow is being stretched to the terminal end range. C.Provide submaximal stretch to the contracted soft tissue for prolonged periods of time.

C.Provide submaximal stretch to the contracted soft tissue for prolonged periods of time. -Low load prolonged stretch is an effective technique for increasing the length of contracted soft tissue.

A client has moderately severe cognitive decline secondary to Alzheimer's disease. The caregiver reports that the client becomes agitated when hand-over-hand assistance and setup support are provided during routine ADL in the home bathroom. Which recommendation would be MOST BENEFICIAL to include as part of caregiver training for reducing the occurrence of these behavioral responses? A.Have the client use a checklist to mark off each step of a self-care activity as it is completed. B.Program an auditory timer and an electronic day planner to prompt the client to initiate self-care activities. C.Provide the client environmental cues by labeling drawers and posting pictures of tasks in the bathroom.

C.Provide the client environmental cues by labeling drawers and posting pictures of tasks in the bathroom. A client who has moderately severe cognitive decline secondary to Alzheimer's disease does not have the ability to use cognitive compensatory strategies but would benefit from environmental cues and modifications.

Which activity involves the use of a closed kinetic chain movement of the upper extremity? A.Lifting a suitcase off the floor to carry by the handle B.Hand-to-mouth patterns used for self-feeding C.Pulling up on an overhead trapeze during bed mobility

C.Pulling up on an overhead trapeze during bed mobility -distal portion is stable

A service competent OTR is completing a feeding reevaluation with an inpatient who has hemiplegia secondary to a CVA 5 days ago. A screening indicates the patient's cognitive and perceptual skills are intact. Observation during mealtime and results of a videofluoroscopy study show the patient has decreased tongue mobility, hypotonicity of the cheek and lips on the affected side, slowed swallow response, and a wet gurgly voice quality after swallowing several bites of food. Which types of food and liquid should the OTR recommend for the patient to reduce risk of aspiration? A.Normal foods cut in small pieces and nectar-thickened liquids B.Moistened soft foods and thin flavored liquids C.Pureed meat and vegetables and honey-thickened liquids

C.Pureed meat and vegetables and honey-thickened liquids

An OTR, who works in a large, urban hospital, is providing intervention to an inpatient who is preparing for discharge and has COPD, diabetes mellitus, and morbid obesity. The patient lives alone in a rural community and has been readmitted to the hospital for recurrent exacerbation of respiratory distress and poor glycemic control 3 times over the past year. What is the FIRST intervention priority the OTR should address to positively influence the risk of a future readmission? A.Ensure the patient can pay for prescription medications ordered by the physician. B.Discuss with the patient the option of moving to the home of a family member. C.Recommend durable medical equipment for the patient to use during ADL.

C.Recommend durable medical equipment for the patient to use during ADL. -within scope of practice

An outpatient OTR is working with an adult client who has spastic diplegia. One of the client's goals is to drive a car. The client has the necessary process skills to drive but requires adapted driving controls to access the gas and brake pedals. What is the FIRST action the OTR should take in this situation? A.Research funding options to pay for vehicle adaptations. B.Arrange for a trial of specialized equipment from a reputable vendor. C.Refer the client to a driving rehabilitation specialist.

C.Refer the client to a driving rehabilitation specialist.

A 5-year-old child has mild developmental delay. Motor and praxis skills are intact. Which action would be BEST to observe when screening emotional regulation skills? A.Sharing toys during unstructured play B.Taking turns while playing a board game C.Responding to the feelings of others

C.Responding to the feelings of others -Emotional self-regulation is BEST observed during a situation where the child is responding to the feelings of others

An OTR is reviewing the results of a single-subject case study. The study investigates the therapeutic effectiveness of sensory stories in children with autism spectrum disorder to influence target behaviors. The OTR wants to integrate sensory stories into daily clinical practice. Using the principles of evidence-based practice, what action should the OTR take NEXT to inform clinical decisions for practice? A.Locate the conclusions for several relevant pre- and post-tests and single-subject studies. B.Interview multiple experts in the field whose opinions on sensory approaches are valued. C.Search an evidence-based database for a meta-analysis or systematic review on the topic.

C.Search an evidence-based database for a meta-analysis or systematic review on the topic.

An inpatient sustained a complete T5 spinal cord injury one month ago and has been in the intensive care unit on extended bed-rest. The OTR recently initiated a program of graded bed mobility in preparation for beginning transfer training. The patient is now able to sit in bed with the head of the bed elevated to 45° for one-hour intervals without experiencing any dizziness. Which position is safest for the patient to be placed in NEXT? A.Upright on a tilt-table at 90° while wearing an abdominal binder and elastic stockings B.Seated on the edge of the bed with both legs unsupported and knees flexed to 90° C.Seated in a semi-reclining wheelchair with legs elevated D.Upright in a standard wheelchair with close monitoring

C.Seated in a semi-reclining wheelchair with legs elevated

An OTR completed an initial assessment of a student in the second grade. Results indicate the student has age-appropriate comprehension, visual object gnosia, and visual acuity, but standardized test scores on figure-ground subtests are well below the norm. Which of the following school art class activities would present the MOSTchallenge to this student based on the outcomes of this evaluation? A.Using plastic templates to trace basic geometric shapes on colored paper B.Placing tiles of the same color and shape in a straight line when making a trivet C.Selecting a round bead from a bag of multi-shaped beads to complete a necklace D.Painting a free form design on a clay pot using a variety of paint colors

C.Selecting a round bead from a bag of multi-shaped beads to complete a necklace

A client who has schizophrenia is participating in OT to improve functional living skills. The client's goal is to be able to independently prepare family meals. The client has successfully used a checklist strategy to gather and keep track of items while preparing a simple cold snack. Which of the following tasks would present a "just right" challenge when using the checklist strategy to progress towards the client's goal? A.Cooking a three-course meal for the family B.Purchasing groceries for a pre-set weekly menu C.Setting the dining table for a group meal

C.Setting the dining table for a group meal

An OTR working in an outpatient setting is completing an initial interview with an older adult client who has recently been diagnosed with a progressive neurological disease. What method should the OTR use during the course of the interview to communicate effective listening and client-centered understanding? A.Provide examples of how other clients have overcome similar adversity. B.Use head gestures to indicate empathy and understanding. C.Share examples of adaptations that help to overcome adversity. D.Offer suggestions as the client discloses concerns and problems.

C.Share examples of adaptations that help to overcome adversity. - not sure why this is right but NBCOT says so

An OTR is a contributing investigator for a unit-wide research project. The focus of the project is to determine if participation in rehabilitation is beneficial to a client's health, well-being, and general quality of life. Which standardized assessment should the OTR use for gathering the MOST RELIABLE evidence for this study? A.Kohlman Evaluation of Living Skills (KELS) B.Barthel Index of ADL (BI) C.Short Form-36 Health Survey (SF-36)

C.Short Form-36 Health Survey (SF-36)

A patient who has early Stage III Alzheimer's disease is being discharged home from an inpatient facility under the supervision of a caregiver. What recommendations should the OTR provide to the caregiver as part of the discharge instructions? A.Encourage the patient to explore new environments B.Assist the patient in developing leisure and social skills C.Simplify the home surroundings and reduce stimuli

C.Simplify the home surroundings and reduce stimuli

An OTR is preparing to measure the external entrance of a client's home to make recommendations for the design of a ramp. In addition to a measuring tape, which tools are MOST IMPORTANT for the OTR to gather in preparation for this evaluation? A.Light meter and safety glasses B.Force measure and sidewalk chalk C.Spirit level and string line

C.Spirit level and string line

An OTR is formulating a discharge plan for an inpatient who has dementia and is functioning at Allen Cognitive Level 4 (Goal-Directed Actions). The patient will be living at home with assistance from family. What type of caregiver education is MOST IMPORTANT for the OTR to provide prior to the patient's discharge from the inpatient facility? A.Techniques for establishing and posting a written emergency plan in the home environment B.Suggestions for modifying the environment to eliminate unnecessary household items C.Strategies for providing visual cues that will help the patient complete daily routines D.Methods for promoting the patient's problem solving for independence during ADL

C.Strategies for providing visual cues that will help the patient complete daily routines

An OTR working in a community hospital is contributing discharge recommendations for a patient who is generally deconditioned secondary to complications from a septic episode. The patient has been participating in OT for the past 3 weeks. Currently, the patient requires moderate assistance with transfers and tolerates up to 10 minutes of seated ADL activity prior to needing a rest break. The patient's goal is to return home to live independently. Which discharge setting would be MOST BENEFICIAL for the OTR to recommend for the patient's next level of care? A.Home health with skilled services B.Inpatient rehabilitation hospital C.Subacute rehabilitation facility

C.Subacute rehabilitation facility

Which of the following symptoms are associated with a client who is experiencing a simple focal (partial) seizure? A.Abruptly stopping an activity with a blank stare and fluttering eyelids B.Stiffening of extremities with rhythmic, bilateral jerking of limbs C.Sudden movement of a single body part with no change in vital signs

C.Sudden movement of a single body part with no change in vital signs

A third-party payer has denied reimbursement of pre-authorized occupational therapy services based on "insufficient information to substantiate payment". The OTR is writing a letter to appeal the denial. What type of client-related information is MOST IMPORTANT to include to increase the likelihood of reimbursement? A.Rate of progress compared to other clients who have the same diagnosis B.Outline of functional tasks used during each intervention session C.Summary of progress based on functional goals and medical necessity D.Annotated reference list indicating evidence-based best practice guidelines

C.Summary of progress based on functional goals and medical necessity

OTR Study Guide Domain 1: Practice Test An OTR working in an inpatient mental health setting is using a cognitive behavioral approach to select an intervention activity for a young adult undergoing treatment for symptoms associated with body dysmorphic disorder. The patient is employed in a successful career and enjoys outdoor activities such as jogging and hiking, but has difficulty forming and maintaining interpersonal relationships. One of the patient's goals is to improve self-image and the ability to express feelings. Which activity would be MOST BENEFICIAL to include as part of the intervention for supporting this goal when using this approach? A.Engaging the patient in an art project using a variety of media during which the patient is encouraged to release emotions B.Guiding the patient through avocational exploration of activities that provide opportunities for social interaction C.Teaching the patient to use a dysfunctional thought record for describing distressing situations and emotional responses

C.Teaching the patient to use a dysfunctional thought record for describing distressing situations and emotional responses

A school-based OTR is establishing a new goal for a student in middle school who has poor handwriting legibility secondary to delayed fine motor and visual motor skills. The student reports frustration with inability to complete written assignments on time, and the teacher reports that poor legibility impacts ability to grade the assignments. Which of the following is the BEST goal to meet the student's needs in the classroom by the end of the IEP cycle? A.The student will write a 5-sentence paragraph with 80% legibility when provided with a visual model and paper with a highlighted baseline. B.The student will rotate pencil from tip to eraser 10 times without dropping on 2 consecutive data days while completing dexterity activities. C.The student will type a 5-sentence paragraph in 20 minutes when provided with word prediction software and keyboarding device.

C.The student will type a 5-sentence paragraph in 20 minutes when provided with word prediction software and keyboarding device. - choice A is not achievable in timeframe

An OTR leads a leisure skills group with young adults who have intellectual and developmental disabilities. The OTR plans to use scaffolding techniques as part of the group process. What is the PRIMARY purpose for using these techniques? A.To use directed discussions to ensure each participant engages in the group B.To provide support to the participants if an error occurs during the group task C.To control task elements that exceed the learners' current capabilities

C.To control task elements that exceed the learners' current capabilities -scaffolding is breaking up the learning into chunks and providing a tool, or structure, with each chunk -OTR structures an activity to support for components of the task that are too difficult for the client,

What is the PRIMARY purpose for screening a new resident who was recently admitted to a skilled nursing facility? A.To identify client-centered intervention priority B.To contribute to the Minimum Data Set (MDS) C.To determine if further evaluation is warranted

C.To determine if further evaluation is warranted

An OTR is working with a client in a home health setting who has a wound on the lateral aspect of the proximal forearm. The wound is 4 cm (1.57 inches) in diameter and is being treated by the wound care nurse with a hydrogel sheet dressing. The wound margins are pink and flat, and the wound is draining a low volume of serosanguinous exudate. What is the PRIMARY reason for using a hydrogel sheet dressing type for this client? A.To provide hydration to the wound while offering pain relief with a hydrogel sheet B.To create a moist healing environment while absorbing discharge from the wound C.To keep bacteria out of the wound while promoting autolytic debridement

C.To keep bacteria out of the wound while promoting autolytic debridement

Which of the following is the BEST example of a primary objective for using the universal design for learning (UDL) principles with students who attend a middle school? A.To recommend desk heights, ergonomic computer workstations and adapted play environments that benefit most students in the school. B.To provide environmental design principles to adapt the classrooms and other learning spaces to include students who have a disability. C.To provide all students in the school the ability to learn and the opportunity to demonstrate academic performance abilities.

C.To provide all students in the school the ability to learn and the opportunity to demonstrate academic performance abilities.

An OTR wants to develop an intervention plan based on the evaluation results of a resident at a nursing home. The therapist and the resident collaborated to establish therapeutic goals that are meaningful to the resident, and now the OTR is trying to decide between individualized one-on-one intervention and group therapy. What responsibility does the OTR have when choosing the most effective method for providing intervention? A.To know the staffing levels during the expected duration of the resident's plan of care B.To follow the employer's guidelines on individual versus group intervention C.To select the intervention method that will best support progress toward the resident's goals

C.To select the intervention method that will best support progress toward the resident's goals

Which ADL would a client who has a complete C7 spinal cord injury be expected to complete independently? A.Dress lower extremity without adaptive equipment. B.Lean forward to wash legs while seated on a bath chair. C.Transfer out of bed to a wheelchair.

C.Transfer out of bed to a wheelchair.

In a healthy individual, which type of wound is expected to have the shortest healing time? A.Abdominal surgical wound left open to heal B.Vascular ulcer left uncovered during healing C.Traumatic laceration immediately closed with clips

C.Traumatic laceration immediately closed with clips

An OTR is evaluating a client who has an ulnar nerve injury at the wrist level of the right dominant extremity. During which task would this injury be MOST evident? A.Holding coins in the palm of the hand B.Carrying a briefcase C.Turning a key in the car ignition D.Operating a desktop calculator

C.Turning a key in the car ignition -difficulty twist/turning

What would be the MOST DIFFICULT activity for a typically developing 5-month-old infant who has congenital torticollis? A.Maintaining wakefulness during feeding time B.Babbling with predominantly vowel sounds C.Turning the head from side to side to see toys

C.Turning the head from side to side to see toys

A client has persistent pitting edema of the hand secondary to mild hemiplegia. Which method would be MOST RELIABLE for monitoring the client's edema over time? A.Measure the hand circumference at the MCP joint level. B.Trace an outline of the hand and fingers placed flat on a tabletop. C.Use a volumeter to measure water displacement.

C.Use a volumeter to measure water displacement.

An adult client with a moderate intellectual disability is beginning a supported employment program. The client typically has a hot lunch each day and wants to continue having hot lunches at work by bringing food from home that can be heated in the workplace microwave. An assessment indicates that the client requires some assistance to complete simple microwave meal preparation. Which strategy should the OTR implement next to MOST EFFECTIVELY achieve the client's goal? A.Talk with the client about purchasing a prepared hot meal in the workplace cafeteria. B.Provide the client with sequential picture cards indicating each step of lunch preparation. C.Use forward chaining techniques during meal preparation sessions with the client.

C.Use forward chaining techniques during meal preparation sessions with the client. -Forward chaining is an effective intervention technique that involves progressively teaching the client the steps of task starting with the first step.

An OTR, who works with children who have cerebral palsy, wants to stay abreast of research relevant to the practice area but is overwhelmed by the amount of evidence available. What is the MOST BENEFICIAL strategy for the OTR to use to integrate evidence into practice-based decisions? A.Test various search methods, and analyze the effectiveness of each approach. B.Locate a scholarly database that includes all levels of evidence (Level I - Level V). C.Utilize systematic reviews and resources that synthesize study results.

C.Utilize systematic reviews and resources that synthesize study results.

An OTR is providing consultative services to develop a wellness program at a community senior center. A needs assessment survey indicates center participants have a fear of falling, feel they are at risk for falls, and want to age in place for as long as possible. In addition to including fall prevention education, which topic should the OTR recommend as part of the educational programming offered at the center? A.Influence of aging on immediate and delayed memory B.Benefits of participating in socially stimulating activities C.Value of identifying effective home modifications

C.Value of identifying effective home modifications

An OTR is evaluating an inpatient who recently sustained a TBI. Diagnostic tests indicate that the injury primarily affects the occipital lobe. Which of the following neurological functions are typically associated with the occipital lobe? A.Language comprehension and motivation B.Executive function and execution of movement C.Visual reception and visual memory

C.Visual reception and visual memory

A client has a peripheral neuropathy of the dominant hand. A screening indicates thenar muscle atrophy with loss of thumb opposition and palmar abduction, inability to pick up a key or coin from a table top, and decreased grip and pinch strength compared to the non-affected hand. Based on these findings, where on the client's hand would an OTR expect to find sensory disturbances during a Semmes-Weinstein monofilament assessment? A.Volar and dorsal surfaces of the small finger and radial half of the ring finger B.Entire palm and tips of the index, long, ring, and small fingers C.Volar surface of the thumb, index, long, and radial half of the ring fingers

C.Volar surface of the thumb, index, long, and radial half of the ring fingers -median nerve injury and innervation

A patient who has a Borderline Personality Disorder has been hospitalized after exhibiting extreme mood swings, outbursts towards a roommate and self-harm behaviors. During her OT evaluation, the patient reports having trouble maintaining close ties with her family, being overwhelmed at work, and feeling a general lack of control in everyday situations. The results of a cognitive assessment indicate that she is functioning at Allen Cognitive Level V (Exploratory Actions). Which type of OT intervention would be MOST BENEFICIAL for this patient, at this stage? Structured crafts activity that requires 3 steps with a model A basic self-care activity that focuses on establishing a structured routine Coping skills groups that address a variety of adaptive strategies Since symptoms are a result of different personalities, care should be taken to assign a different social group to a different personality

Coping skills groups that address a variety of adaptive strategies -can problem-solve -learn new strategies first two are too easy

Individual recovering from TBI is assessed to be at level VI of Rancho Los Amigos Scale. What should the OT use to implement treatment? A. Sensory stimulation activities such as moving to music B. Repetitive self-care tasks such as brushing hair C. Community re-entry activities such as taking a bus D. Simple meal prep tasks such as making a sandwich

D. Simple meal prep tasks such as making a sandwich -C. is VII or VIII

A patient in the early stages of amyotrophic lateral sclerosis (ALS) presents with weakness in the thenar eminence. During which ADL task would the evaluating OTR® MOST LIKELY observe the effect on function? A. Transferring from the bed to the bedside commode B. Taking a sip from a glass of water C. Reaching overhead to don a T-shirt D. Squeezing toothpaste onto a toothbrush

D. Squeezing toothpaste onto a toothbrush

An OT is planning a group that will include some teens who pose a suicide risk. In selecting craft media, what activity would MOST likely be the safest? A. Leather checkbook cover with leather lacing. B. Macrame´ plant hanger. C. Ceramic vase. D. Stenciling greeting cards.

D. Stenciling greeting cards. -leather has cords so its dangerous -stenciling is self-harm not suicide

Which behavioral symptom might a client with early-stage dementia have? A. Pacing B. Difficulty choosing appropriate clothing C. Vulgar or rude language D. Suspicion

D. Suspicion -might think misplaced items were stolen -all others are for middle-stage dementia

The Affordable Care Act created mandated regulations that altered health insurance provisions in the United States. Which of the following is FALSE? A Preexisting conditions cannot limit ability to obtain coverage B Premium rates cannot be increased based on an individual's particular job C Insurance companies cannot determine a monetary limit to the quantity of health services covered D Mental health coverage is not included in federal health insurance changes

D Mental health coverage is not included in federal health insurance changes

The transition plan for a high school senior with developmental delay includes a referral to a vocational rehab workshop job setting. The student has set a goal to live indep. of family. Which is the best living environment for the OT to recommend for this student? A. An apartment in a subsidized housing project B. A group home w/ case managers available on-call C. A supported apartment w/ a roommate D. A group home w/ daily on-site sup

D. A group home w/ daily on-site sup -DD who need vocational rehab (aka. sheltered workshop) will need higher level of support and supervision for transition and IADLs. He hasn't lived on his own so start with most support first

An elementary school teacher was recently diagnosed with MS. Which adaptation is best for the OT to recommend to the teacher to accommodate for the effects of MS on classroom teaching? A. The use of anchoring techniques to compensate for scanning deficits B. A daily list of tasks to compensate for cog deficits C. A motorized scooter to compensate for decreased endurance D. A high stool to compensate for LE weakness

D. A high stool to compensate for LE weakness

An OTR® is working in a clubhouse with a group of clients who have mental health issues. Which leadership style would be MOST effective? A. Director B. Authority figure C. Facilitator D. Advisor

D. Advisor - clubhouse model means members have most control - clubhouse is work focused and staff and members work together

A client who uses an ultra lightweight wheelchair asks whether the chair can accommodate a wraparound lapboard. The OT is not familiar with this type of lapboard. Who on the rehabilitation team would be the BEST person for the OT to consult to obtain this information? A. Another OT B. A physical therapist C. A rehabilitation engineer D. A medical equipment supplier

D. A medical equipment supplier

A client presents at the occupational therapy clinic with a windswept deformity and needs to be assessed for a new wheelchair and a positioning device. In the documentation required for Medicare, the OT uses the term "windswept deformity" and also describes the impairment in body structure as follows: A. Pelvis rotates posteriorly, increasing trunk flexion B. Pelvis rotates anteriorly, increasing curvature of the lumbar spine C. One side of the pelvis is lower than the other D. Pelvis rotated to one side, resulting in the spine, trunk, and thighs moving to the opposite side

D. Pelvis rotated to one side, resulting in the spine, trunk, and thighs moving to the opposite side

When considering a wheeled mobility device with a client, the OT proposes a scooter. Which of the following client characteristics was the primary reason for proposing a scooter versus a manual wheelchair? A. The client is unable to safely ambulate in the home. B. The client's home can easily accommodate a wheeled mobility device for moving from room to room in the home. C. The client claims he will consistently use any wheeled mobility device he is prescribed. D. The client is able to walk around his home, but cannot walk any distance in the community.

D. The client is able to walk around his home, but cannot walk any distance in the community.

OT is working in a preschool with a child who was born with congenital cytomegalovirus (CMV) infection. As a result, the child has difficulty seeing. The child enjoys playing with classmates but has difficulty when the play activity is highly dep on vision. Which of the following is best for the therapist to rec for improving the child's play experiences with classmates? A. Train a personal assistant to provide verbal cues during play activities B. Have the child read books in braille aloud to classmates C. Train a classmate to guide the child during play activities D. Incorporate 3D objects into play activities

D. facilitate improvement in other senses to compensate so child will be better later on

Which child with neuromotor impairment would benefit MOST from using a prone scooter for exploratory play? A. A child with cerebral palsy with predominant extensor tone. B. A child with generalized low muscle tone who is easily fatigued. C. A child with cognitive limitations and poor sensory awareness. D. A child with spina bifida with lower extremity paralysis.

D. A child with spina bifida with lower extremity paralysis. -facilitate UE and neck strength without needing legs -A. is bad bc neck hyperextension B. is bad bc its a tiring position C. could injure themselves

A patient s/p spinal cord injury presents with more upper extremity deficits than lower extremity deficits. Which clinical syndrome is described? a. central cord syndrome b. cauda equina syndrome c. brown-sequard syndrome d. anterior cord syndrome

a. central cord syndrome

An OTR® is working with a client diagnosed with fibromyalgia who is experiencing disturbed sleep patterns. Which cognitive areas of functioning are disturbed sleep patterns MOST likely to interfere with? A. Arousal and orientation B. Sequencing and categorization C. Initiation of movement and language D. Attention span and short-term memory

D. Attention span and short-term memory -first three are related to TBI/CVA

An order came in for a hand splint for a new client. A newly graduated OTR® evaluated the client for the splint. The new OTR® had not made a splint on a client before and requested an experienced OTR® hand therapist's assistance. The newly graduated OTR® decided to attend a splinting continuing education class to improve splint-making skills and increase the OTR®'s comfort level. By which ethical principle is the OTR® abiding? A. Autonomy B. Social Justice C. Fidelity D. Beneficence

D. Beneficence

Whom would an OTR® working in an outpatient return-to-work program NOT consider a primary referral source for an FCE? A. Physician B. Case manager C. Attorney D. Career counselor

D. Career counselor

An older adult has lost significant functional vision over the last four years and complains of blurred vision and difficulty reading. The patient frequently mistakes images directly in front, especially in bright light. When walking across a room, the patient is able to locate items in the environment using peripheral vision when items are located on both sides. Based on these findings, which visual deficit should the occupational therapist report the client is exhibiting? A. Glaucoma. B. Presbyopia. C. Hemianopsia. D. Cataracts.

D. Cataracts.

The Beverly Foundation, a nonprofit group that promotes new ideas and options for older adult transportation, evaluates transportation programs using the Five As of Senior-Friendly Transportation. Which characteristic of a transportation system is categorized in the Acceptability category? A. Availability of transportation on evenings or weekends B. Affordability of the cost of travel C. Accessibility of the vehicles used for travel D. Cleanliness of the vehicles used for transportation

D. Cleanliness of the vehicles used for transportation

An adolescent-age student has athetoid cerebral palsy and has been participating in school-based OT since pre-school. After transitioning to the sixth grade in a new school, the teachers report the student is not using assistive devices, is sullen, and does not interact with peers. What INITIAL action should the OTR take in response to the teachers' reports about this student? A. Revise the IEP goals to focus on student preferences. B. Establish new goals to work toward self-sufficiency. C. Develop a transition plan for promoting autonomy. D. Complete an evaluation using a top-down approach.

D. Complete an evaluation using a top-down approach.

OT plans a task-oriented activity for a group of adolescents recently diagnosed with anorexia nervosa. Which is the best activity for the therapist to include in the initial session of this group? A. Making cards to send to veterans in a local hospital B. Making cookies for the residents in a homeless shelter C. Performing low impact aerobic exercises D. Composing lyrics and melody for a group song

D. Composing lyrics and melody for a group song -sharing thoughts, feelings, values, self-expression = task oriented

An OTR® is performing an initial evaluation of a client with an acute full-thickness burn to the dorsum hand involving all digits. The OTR® is applying Boutonniére precaution and avoiding having the patient form active or passive composite flexion of the fingers. What is the MOST APPROPRIATE clinical reasoning for this approach? A. The tensile strength of the burned skin will not allow for composite flexion of the fingers. B. A bulky dressing on the hand will limit the ability to form composite flexion of the fingers. C. Immobilization is critical for the initial wound healing to avoid wound widening from movement. D. Composite flexion of the fingers increases the risk of extensor dorsal hood disruption.

D. Composite flexion of the fingers increases the risk of extensor dorsal hood disruption.

School-based OT intervention for a child focuses on the treatment of visual perceptual deficits. The child show no progress over the past school year, does not complete homework assignments, and only wants to play with other children. Upon completion of the annual review, the OT determines that the child is ready for discharge for OT. Prior to discharge, what must the therapist do? A. Improve the child's decision-making skills B. Enrich play experiences at recess C. Challenge the child's creativity D. Contact the family to discuss the decision

D. Contact the family to discuss the decision

The OT is planning intervention to promote developmental skill acquisition for an infant in the neonatal intensive care unit. Which action will have the MOST therapeutic impact? A. Adjust infant's positioning to promote physio logical development. B. Recommend early intervention referral to as sess the infant when home following dis charge. C. Complete the neurobehavioral assessment and identify interventions. D. Create a comfortable collaborative relation ship with parents and promote parenting skills.

D. Create a comfortable collaborative relation ship with parents and promote parenting skills. -biggest long-term impact on infant -parents must see potential for care and nurture before beginning therapy

When designing and implementing an as sertiveness training program, what should be the focus of the FIRST session? A. Strategies for making a direct request for a de sired change. B. How to make I statements using the "when you , I feel " format. C. How to stand up for yourself in order to have more control in your life. D. Defining the three types of responses: passive, assertive, and aggressive.

D. Defining the three types of responses: passive, assertive, and aggressive.

OT working in school has been asked to rec technological devices for student with severe spastic quadriplegia and dysarthria. Which action should the OT take prior to recommending specific equipment? A. Determine access capabilities in collaboration with the SLP B. Identify funding source(s) in collaboration w/ the social worker C. Obtain family support in collaboration w/ the psychologist D. Determine intervention goals in collaboration w/ the student

D. Determine intervention goals in collaboration w/ the student -make sure its meaningful to client

Based on review of evaluation results, the OT believes that school-based services to support a student with autism spectrum disorder should include education for the teaching staff concerning ways to promote positive behavior throughout the school day. In preparation for the IEP meeting, what is the BEST plan to ad dress this concern? A. Collect handouts with strategies teachers can use to help alert the child in the classroom and distribute them at the meeting. B. Recommend the staff attend an in-service programs to learn ways to help all students promote self-regulation. C. Demonstrate for the teacher how the student can use fidget toys and other sensory strate gies to help him stay focused during seat work. D. Discuss the benefit of OT provision of staff ed ucation and determine how this can be imple mented and documented in the student's IEP

D. Discuss the benefit of OT provision of staff ed ucation and determine how this can be imple mented and documented in the student's IEP -none of the other choices involve collaboration with IEP

Which activity should the OT suggest to promote handwriting readiness skills for children in a preschool classroom? A. Hammering dowels into a Styrofoam board. B. Creating buildings with wooden blocks in the block center. C. Rolling clay into many different-sized balls. D. Drawing lines and shapes using shaving cream or sand.

D. Drawing lines and shapes using shaving cream or sand.

An OTR® is working with a community organization whose mission includes promoting universal access to transportation for people with disabilities. Which organization might the OTR® contact for training and technical assistance? A. Alzheimer's Association B. Association for Driver Rehabilitation Specialists C. Assistive Technology Industry Association D. Easter Seals Project ACTION

D. Easter Seals Project ACTION

While reviewing work hardening program documentation, an OTR realizes there is no entry in the client's medical record of a meeting conducted with the insurance case manager, client, and client's employer regarding light-duty options. According to best practice guidelines, how should the OTR amend the client's medical record to include a summary of the meeting? a. back-date the entry to correspond to the actual date of the meeting b. insert a summary of the meeting into the margins of the client's medical record. D. Enter the information as a late entry into the client's medical record.

D. Enter the information as a late entry into the client's medical record.

Parent of two elementary-aged children, receiving hospice services due to metastasized bone cancer. Client has pain and poor endurance and decreased muscle strength. Client required mod A for self-care and dressing. Which is the best intervention for the OT to incorp. into sessions w/ this parent? A. Training in energy conservation techniques for self-care and dressing B. Training in joint protection techniques for self-care and dressing C. Using biofeedback to reduce pain D. Exploring play activities for parent to do with their children

D. Exploring play activities for parent to do with their children -hospice focuses on QOL and control over life and values

A child with mild cerebral palsy (CP) receives OT intervention in a preschool setting. Which intervention approach should the therapist employ to facilitate development of typical grasp patterns? A. Place soft foam tubing around objects to be grasped. B. Analyze the present components of the child's grasp. C. Analyze the missing components of the child's grasp. D. Grade the sizes and shapes of objects to be grasped.

D. Grade the sizes and shapes of objects to be grasped.

A softball player sustained a deep partial-thickness burn to the anterior aspect of the right arm from the wrist, proximal to the ulnar styloid process, to the mid-upper arm. A split-thickness skin graft from thigh to mid-forearm was performed 3 days postinjury. To minimize the risk of graft rejection in the initial phase (7 days postinjury), in what should the client be instructed? A. Daily active pronation and supination exercises at least 5 times per day B. Desensitization using ice to gently rub the burned areas from distal to proximal C. Retrograde massage followed by elastic bandage wrapping from distal to proximal D. Immobilization using the elbow extension splint and avoiding forearm movement

D. Immobilization using the elbow extension splint and avoiding forearm movement

An OT is working with a child presenting with sensory-seeking behaviors and under-reactivity to touch and movement. The child has an unusually high activity level, inability to self-calm, motor impulsivity, and frequent touching/handling of items in the environment. Using Ayres classic sensory integrative approach which would be the most effective for the OT to use with this child to facilitate an adaptive response? A. A predetermined schedule of sensory activities designed by the OT B. The child's passive participation in a variety of vestibular/prop experiences C. Use of a sensory void environment to promote self-regulation D. Individualized therapeutic activities based on the interests of the child

D. Individualized therapeutic activities based on the interests of the child -Ayers focuses on inner drive and active involvement

An OT is developing a pilot program to promote positive behavior in students in the local elementary school. The program will address behavior needs of students with sensory processing challenges and will provide "prevention support" for all students in the classroom. Recognizing the setting and the students' capacities, which approach is MOST likely the OT's first consideration? A. An in-service training program for parents to help them learn about healthy limits in com puter and television time at home. B. Consultation with the physical education teacher to develop warm-up and cool-down exercises that promote self-regulation. C. Classroom screenings to identify students who need additional occupational therapy support. D. Instruct teacher and students to use the Alert program and include routine follow-up visits to classroom to colead sessions with teacher.

D. Instruct teacher and students to use the Alert program and include routine follow-up visits to classroom to colead sessions with teacher. -B. is good but only works for that specific class

OT works with a pt that has degenerative joint disease and incurred a R hand injury and also has CRPS Type I (presents with has severe pain, stiffness, extreme temp changes in hand, pitting edema, and blotchy, shiny skin). Which is the most appropriate activity for the OT to recommend for the person to complete at home? A. Doing light craft work B. Playing a tabletop game C. Performing relaxation exercises D. Manually washing a car

D. Manually washing a car -CRPS = edema, ROM, weight bearing

An OTR attended a continuing education class on cognitive strategies for children with attention deficit hyperactivity disorder. During the class, the instructor discussed anecdotal evidence for the use of oculovestibular treatment to improve learning ability. In researching the topic, the OTR reads a peer-reviewed article that indicates a correlation of 0.01 (r = 0.01) between the use of oculovestibular techniques and process skills. What do the research findings from the peer-reviewed article indicate for the level of correlation? A.High correlation B.Weak correlation C.Moderate correlation D. Negligible correlation

D. Negligible correlation -anything less than 0.70 is insignificant

During a group session at an adult day care program, an older adult consistently complains that everyone is mumbling. After the group, which action should the occupational therapist take in response to these statements? A. Notify the client's primary care physician that the person exhibited evidence of paranoia. B. Collaborate with the program director to remove groups from the client's program plan .C. Document objective data about the complaints in the person's chart. D. Notify the primary care physician that the person may need an audiological evaluation.

D. Notify the primary care physician that the person may need an audiological evaluation.

An occupational therapist provides intervention for a four-year-old with developmental delays characterized by the persistence of primitive postural reflexes. The child demonstrates age-appropriate cognitive skills. Which is the best play activity for the therapist to incorporate into the child's intervention? A. Spinning on a swing. B. Putting a puzzle together. C. Lying on the floor and playing a game of marbles. D. Pretending to be an explorer crawling through caves.

D. Pretending to be an explorer crawling through caves.

An individual is unable to read a standardized evaluation because he does not have his glasses with him. Which of the following options is MOST appropriate? A. Instruct the individual to bring his glasses to the next session and complete the evaluation then. B. Ask the OT aide to read the evaluation to the individual and check off his answers. C. Include the individual in the group session that is about to begin and remind the individual to bring his glasses to the next session. D. Read the questions to the individual and check off his responses.

D. Read the questions to the individual and check off his responses.

An OT hypothesizes that a child's hand writing difficulty is related to a writing posture and posture ergonomics that limit automaticity in handwriting skill. Which option does the OT MOST likely pursue? A. Promote work habits such as positioning pa per in midline area of table/desk, use of non dominant hand to stabilize paper, use of eraser when needed. B. Implement activities to refine grasp and ma nipulation of writing tool, gauge speed of work, avoid fatigue when writing. C. Help student learn strategies to ensure correct spacing, orientation, uniform size of letters drawn. D. Recommend student work station arrange ment that supports pelvis/shoulder/head alignment and optimizes arm/hand position for endurance in writing tasks.

D. Recommend student work station arrange ment that supports pelvis/shoulder/head alignment and optimizes arm/hand position for endurance in writing tasks.

A 3 y/o child with L spastic hemiplegia due to CP is evaluated for EI services. During the evaluation, the OT observes behaviors that seem to indicate the presence of visual deficits. Based on these observations, which action should the OT take? A. Completion of motor-free visual perceptual assessment B. Completion of developmental vision assessment C. Refer the child to an optician D. Refer the child to an optometrist

D. Refer the child to an optometrist

An OTR® is working on bed mobility with a client in the active phase of C8 spinal cord injury. The client becomes dizzy and nauseous when brought to the sitting position. What is the BEST action for the OTR® to take? A. Apply antiembolism stockings and continue activity. B. Return to the lying position and loosen clothing. C. Transfer to wheelchair and elevate legs. D. Return to the lying position and elevate legs.

D. Return to the lying position and elevate legs.

A patient has endured a burn to the volar aspect of her hand. What contracture is she MOST at risk for developing? a. Wrist flexion b. Wrist extension c. Palmar contracture d. Claw hand deformity

c. Palmar contracture

Individual is evaluated for a repetitive stress disorder. Individual complains of numbness and tingling in thumb, index, middle, and radial half of ring finger and aching pain in proximal forearm. The client states that these symptoms are not evident at night. The OT notes a positive Tinel's sign. Which site would the therapist document as the location of this sign for this client? A. The wrist B. The Guyon's canal C. The elbow D. The forearm

D. The forearm -PTS usually dosnt have night pain and is in forearm -Tinels can be done at forearm (PTS), wrist (CTS), or medial elbow (golfer's elbowl)

An OTR® assesses a client using the Allen Disability Framework. The client scores an Allen Cognitive Level (ACL) of 4.0. Given this score, what discharge arrangement would the OTR likely recommend for this client? A. This client could be discharged home alone with weekly checks for novel safety issues and health maintenance needs and reminders. B. This client could be discharged home with family for support in all areas of IADLs and personal care including bathing, dressing, and hygiene. C. This client should remain in the psychiatric facility under close 24-hour supervision for safety and personal care needs. D. This client could be discharged to a supported living arrangement with the expectation that the client can independently complete the morning self-care routine.

D. This client could be discharged to a supported living arrangement with the expectation that the client can independently complete the morning self-care routine.

Which clinical situation would be LEAST amenable to positioning and splinting solutions for decreasing soft-tissue contracture to improve functional use for occupational performance? A. Use of a volar antispasticity hand splint while weight bearing to decrease flexor tone in the wrist and fingers B. Use of a C-bar splint to stretch the first web space of the hand secondary to a medial nerve injury C. Use of a knee extension splint for a client with a below-the-knee amputation while sitting in a wheelchair D. Use of an elbow extension splint post fracture to release a boney block

D. Use of an elbow extension splint post fracture to release a boney block -boney block is surgical only

During a home visit, client who has incurred a CVA reports difficulty finding objects during BADL and IADL. Client reports that directions (e.g., look on refrigerator's door, look in medicine cabinet) are not helpful. Which cognitive perceptual ability should the OT further evaluate? A. Stereognosis B. Organization C. Spatial relations D. Visual Closure

D. Visual Closure -person might not be able to find items if they are covered by other things in fridge/cabinet - spatial relations is more body awareness and directions should help this

An individual reports that back pain dur ing sexual activity is so severe that it prevents any enjoyment. The BEST strategy to recom mend is to: A. use a side-lying position. B. time sexual activity for periods of high energy. C. avoid discussing pain with the sexual partner because it may be a "turn off." D. encourage client to identify alternative meth ods with the partner for meeting sexual needs that do not cause pain.

D. encourage client to identify alternative meth ods with the partner for meeting sexual needs that do not cause pain.

An ot completes a home assessment has recommended a hospital bed, lightweight wc, bedside commode, reachers, long-handed sponge, shower chair, and hand held shower. The family states they can only afford the items that can be billed as DME. The ot should: A. explain to the family that they will need to pay for all the items B. order only the reachers and long handed sponge. C. order on the shower chair and hand held shower D. order on the light weight wc and hospital bed.

D. order on the light weight wc and hospital bed. -medicare might not cover adaptive equipement

A client expresses an interest in playing a computer game with another group member during a leisure skills group. The OT reviews the client's cognitive evaluation and agrees that the game is a good choice for the client's cognitive level. After 15 mins of engaging in the computer game, the client rubs both eyes, looks around, and reports trouble focusing. Which should the therapist do in response to these observations? A. provide verbal encouragement for client to complete game before taking a break B. suggest the client and the other member play a different video game that is easier for the client C. discontinue session and advise client to select different leisure activity to do with other member D. suggest the client and other member pause the game and talk about the game's progress after significant plays

D. suggest the client and other member pause the game and talk about the game's progress after significant plays -best for engagement and promoting occupational outcomes

An individual diagnosed with Guillain Barre´ acute syndrome exhibits good upper ex tremity strength. The activity MOST appropri ate for further strengthening and endurance building would be: A. peeling potatoes. B. bed making. C. polishing furniture. D. washing windows.

D. washing windows. -most resisted and elevated to promote function -all others aren't elevated or resistive enough

A client who sustained a Colles' fracture of the dominant extremity has been participating in OT. Reevaluation results indicate that the client's active wrist extension and supination increased by 5°, grip strength increased by 5 lbs (2.27kg), and pinch strength improved by 1 lb (0.45kg). The client is able to touch fingertips to palm, but stiffness and swelling limit tight gripping. Pain interferes with the client's ability to complete typical work tasks using the affected hand, but the client compensates for most tasks, except writing, by using the non-dominant hand. Which statement is BEST to include in the assessment section of the client's weekly progress note? A."Decreased ROM and strength interfere with client's ability to write at work. Client would benefit from training activities to improve fine motor skills." B."ROM and strength of the affected hand and wrist are improved slightly compared to previous evaluation. Pain and swelling interferes with ability to make a full fist." C."Client is able to complete work tasks by using the non-dominant extremity. Recommend continuation of OT for pain management and to improve ROM and strength of the dominant hand." D."Improvements are noted in ROM and strength, but persistent stiffness, swelling, and pain interfere with full functional use of the affected hand for completion of daily tasks."

D."Improvements are noted in ROM and strength, but persistent stiffness, swelling, and pain interfere with full functional use of the affected hand for completion of daily tasks."

What critical functional advantage is TYPICALLY observed in a client who has a complete C6 spinal cord injury compared to a client who has a complete C5 spinal cord injury? A.Ability to use triceps strength during transfers B.Improved trunk control to bend side to side without falling C.Improved gross grasp from innervation of the extrinsic flexors D.Ability to use the radial wrist extensors to supplement grasp

D.Ability to use the radial wrist extensors to supplement grasp

An OTR has completed a developmental assessment of a 6-year-old child who has Down syndrome. Results indicate the child is dependent in all self-care tasks. The parents do not place a high priority on dressing independence, but the OTR does. What action should the OTR take as part of the intervention planning process? A.Ask for the parents' consent to begin working on specific self-care skills with the child. B.Inform the parents that school-age children are expected to be independent with self-care. C.Talk with the parents about establishing independence in self-care skills as a primary goal. D.Collaborate with the parents to identify mutually acceptable treatment goals for the child.

D.Collaborate with the parents to identify mutually acceptable treatment goals for the child.

An OTR wants to measure the effectiveness of a leisure education program for clients who attend an outpatient chemical dependency unit. What method should the OTR use to obtain a valid measure of the program's effectiveness? A.Administer a questionnaire about the perceived importance of leisure activities to clients who complete the program. B.Track the number of clients referred to the program with the number of clients who complete the program. C.Monitor the frequency of client attendance and level of participation during each session of the program. D.Compare a measurement of client leisure involvement at the start of the program with leisure involvement upon completion of the program.

D.Compare a measurement of client leisure involvement at the start of the program with leisure involvement upon completion of the program.

An inpatient in a rehabilitation facility has a C6 tetraplegia with a rating of "A" on the ASIA impairment scale. The patient has achieved BADL goals and now wants to be as independent as possible with homemaking tasks. Which intervention approach would be effective to use as the PRIMARY strategy for promoting progress toward the patient's goal? A.Biomechanical B.Behavioral C.Remedial D.Compensatory

D.Compensatory

After becoming initially certified as an OTR, an individual worked in an outpatient OT clinic for 4 years. After that time, the individual did not work as an OT and has not participated in occupational therapy professional development activities for the past 6 years. The individual wants to resume OT clinical practice. What ethical responsibility does the individual have prior to obtaining a job in an outpatient setting? A.Identify professional development units that can be carried over from the previous job. B.Volunteer as an OT in a clinical setting to establish service competence. C.Submit an application to renew national certification as an OTR. D.Create an effective learning plan related to skills needed for the desired practice setting.

D.Create an effective learning plan related to skills needed for the desired practice setting. -OT practitioners have the ethical responsibility of perceiving their current level of professional competence.

An OTR is using a top-down approach to select interventions for a client who has unilateral neglect secondary to a CVA. Which intervention would be MOST BENEFICIAL to include as part of the client's intervention when using this approach? A.Place commonly used toiletry items to the client's affected side during self-care tasks. B.Use tactile-kinesthetic guiding to the client's involved extremity during a dressing task. C.Teach drills for practicing head turning to find an object placed near the affected side. D.Determine compensatory options the client can use in the home environment.

D.Determine compensatory options the client can use in the home environment.

An OTR administered a criterion-referenced standardized developmental checklist to a 3 year-old-child who has mild developmental delay. The child did not meet the standard for snipping with scissors. For what purpose would these results be MOST USEFUL? A.Linking outcome measures to other typically developing children B.Identifying functional tasks that would be most difficult for the student C.Comparing the child's performance to that of an age-equivalent population D.Determining developmentally appropriate activities to use in therapy

D.Determining developmentally appropriate activities to use in therapy -purpose of criterion-referenced tests is to identify specific tasks to use as the focus of intervention.

An OTR is interpreting scores of a developmental test that was administered to a 3-year-old child. The child scored at the 89th percentile for the child's age and gender group. What can the OTR conclude based on this score? A.This child displays above-average developmental skills compared to similar children. B.These scores are sensitive for measuring small changes in the child's overall development. C.The child has minor developmental deficits compared to the normative sample group. D.Eleven per cent of the children in the sample group scored higher than this child.

D.Eleven per cent of the children in the sample group scored higher than this child.

A client in an outpatient setting sustained a frontal lobe TBI 2 months ago. The client has good motor control but has residual problems with executive functioning. One of the client's goals is to be independent with homemaking tasks. During a meal preparation session, the client cooks a meal, but makes no attempt to clean the cooking utensils and dishes or put the food items away after completing the cooking task. Which area of executive function appears to be MOST affected by the TBI as evidenced by this behavior? A.Episodic memory B.Selective attention C.Environmental gnosia D.Emergent awareness

D.Emergent awareness

An inpatient had a total hip replacement 2 weeks ago. The OTR is discussing home set-up and seating options with the patient as part of the discharge instructions. What type of seat should the OTR recommend the patient sit on when watching television at home? A.Upholstered reclining chair with extra pillows to elevate the affected leg B.Sofa with enough length to allow the patient to elevate both legs on the seat cushions C.Cushioned, armless dining chair elevated on one inch (2.54 cm) high blocks D.Firm raised armchair with a wedge pillow roll between the cushion and back of the chair

D.Firm raised armchair with a wedge pillow roll between the cushion and back of the chair

An inpatient has a TBI and is functioning at Level V (Confused-inappropriate) on the Rancho Los Amigos scale. Which approach would be MOST EFFECTIVE for facilitating the patient's success with grooming tasks based on the patient's current cognitive level? A.Use forward chaining techniques if the patient is distracted from the task. B.Demonstrate a portion of the activity then ask the patient to return demonstration. C.Provide repeated verbal instructions until the patient completes the task. D.Give one-step instructions and hand-over-hand cueing throughout the task.

D.Give one-step instructions and hand-over-hand cueing throughout the task.

A client sustained a hand injury 8 weeks ago and has been participating in an outpatient OT program several times a week for the past 3 weeks. The OTR has just fabricated a dynamic orthotic to correct a PIP joint contracture. What information is MOST IMPORTANT to include in the contact note for this visit? A.Results of a sensory evaluation of the affected hand B.Orthotic construction methods and care instructions C.Thickness and type of material used for the orthotic D.Goniometric measurements of the affected hand

D.Goniometric measurements of the affected hand

A patient in an inpatient rehabilitation setting is in the recovery phase of intervention after an acute onset of Guillain-Barré syndrome one month ago. The OTR advises the patient that using assistive devices will improve independence, but the patient refuses to use the devices stating: "My wife is happy to help me whenever I need it." How should the OTR respond to the patient's comment? A.Focus intervention sessions on strengthening and ROM activities. B.Discuss the patient's comment with family members. C.Convince the patient to try the devices at least once. D.Identify other strategies for improving occupational performance.

D.Identify other strategies for improving occupational performance. -base interventions on pts. needs and priorities

A student in the first grade has moderate hypotonicity resulting in poor oral motor control. One of the student's goals is to be able to "eat the same foods as the other kids" at lunchtime in the cafeteria at school. Currently, the student requires moderate assistance when eating and eats a soft diet. What should be the INITIAL focus of intervention for progressing toward this goal? A.Providing the student with assistive devices to use when eating in the cafeteria B.Asking the student to identify specific food preferences from the cafeteria menu C.Determining which cafeteria foods have the textures the student can eat D.Identifying seating and positioning options for the student in the cafeteria

D.Identifying seating and positioning options for the student in the cafeteria

An outpatient client has an acute flare-up of stage I rheumatoid arthritis. Initial evaluation results indicate the client's MCP joints bilaterally are red and swollen. The client lacks 10° active extension of the MCP joints on the second through fifth digits bilaterally. The client works as a florist and reports pain as 9 out of 10 on a visual analog scale when completing activities requiring grasp and prehensile patterns. The client will be participating in OT twice weekly. Which therapeutic exercise should be included as part of the intervention plan for the client to complete by the end of the first week of therapy? A.Passive motion and stretch of the MCP joints through the full arc of motion B.Tendon gliding exercises of the fingers against light resistance therapy putty C.Pinching and gripping a soft sponge in warm water within pain tolerance D.Isotonic and isometric exercises of both hands within pain-free ranges of motion

D.Isotonic and isometric exercises of both hands within pain-free ranges of motion -keep within pain free

An OTR is developing a program for clients who are considering bariatric surgery as a weight-loss option. Which type of program represents a comprehensive client-centered approach that would be effective for improving surgical outcomes or potentially eliminating the need for surgery? A.Self-actualization education B.Weekly support groups C.Stress management classes D.Lifestyle modification program

D.Lifestyle modification program

A young adult client was diagnosed with axonotmesis of the ulnar nerve secondary to a crush injury of the forearm 2 weeks ago. After obtaining baseline assessment information, which technique would be MOST IMPORTANT for the OTR to teach to the client as part of the intervention during the initial phase of the client's rehabilitation? A.Sensory re-education B.Hand-dominance retraining C.Isometric strengthening D.Visual compensation

D.Visual compensation - use visual skills as a compensatory means for protecting the hand from further injury. - sensation will recover within 6 months - axonotmesis is loss of protective sensation

An OTR is providing home-based services to an adult client who had an open reduction and internal fixation of a tibia fracture. The client lives at home and can ambulate for short distances in the house using crutches. The client's short-term goal is to be able to take a bath independently. What should the OTR teach to the client during the INITIAL bathing session? A.Sterile techniques for using hydrogen peroxide to clean the skin surrounding the pin sites B.Proper method for setting up an inflatable bed bath with the assistance of a caregiver C.Positioning of the affected leg over the side of the tub after transferring to a tub bench D.Methods for wrapping the affected leg in a moisture barrier prior to transferring to a tub bench

D.Methods for wrapping the affected leg in a moisture barrier prior to transferring to a tub bench

An inpatient in a rehabilitation facility is preparing for discharge to home. The patient has hemiplegia, uses a wheelchair for mobility and completes self-care independently with assistive devices. The patient's home bathroom has a standard bathtub and a separate walk-in shower with a safety glass door and a 6-inch (15.24 cm) high doorsill. Both the shower and the tub have safety grab bars. Which piece of durable medical equipment would be MOST BENEFICIAL for this patient to use at home? A.Transfer board and plastic shower stool with a contoured seat B.Plastic bath chair with armrests and accessory caddy C.Shower chair that can be used in the shower or bathtub D.Padded transfer bench with swivel seat for the bathtub

D.Padded transfer bench with swivel seat for the bathtub -safest transfer method

An OTR plans to use a sensorimotor approach to improve the handwriting skills of a 6-year-old student who has a mild learning disability. The student maintains a very tight grip on a pencil when writing, consistently uses a palmar grasp when holding the pencil, and has directional confusion when forming letters. Which activity would be effective to include as part of the INITIAL intervention when using this approach? A.Using spring-opening blunt-edge scissors to cut out geometric paper shapes B.Rolling out colored modeling dough and making cookie cutter shapes on a tabletop C.Providing hand-over-hand assistance during writing assignments D.Painting letters using a wide-barrel brush on paper attached to an upright easel

D.Painting letters using a wide-barrel brush on paper attached to an upright easel

Which of the following environmental adaptations will improve safety during meal preparation for a client who has low vision? A.Using large-sized bowls and pots for mixing and stove-top cooking B.Arranging items on the pantry and cabinet shelves in alphabetical order C.Installing a microwave that has preprogrammed cooking options D.Placing tactile markings on the operating features of appliances

D.Placing tactile markings on the operating features of appliances

A 6-year-old child who has mild ataxic cerebral palsy has been referred to OT for handwriting skill development. When implementing treatment, what action should the OTR take FIRST? A.Attach a wrist cuff weight to promote forearm stability. B.Fabricate a thermoplastic splint for the dominant hand. C.Facilitate letter formation by having the child trace simple lines. D.Position the child`s wrist in slight extension and the forearm in 45° of pronation.

D.Position the child`s wrist in slight extension and the forearm in 45° of pronation. -Prior to implementing compensatory and rehabilitative approaches, the OTR should FIRST ensure the child is optimally positioned to support progress toward the handwriting goal.

An inpatient in a rehabilitation facility has hemiplegia secondary to a CVA. The patient is independent with BADL. The OTR, who uses the ecology of human performance model, is preparing the patient's discharge summary. What information reflects this approach and should be included as part of this report? A.Support groups for promoting the patient's acceptance of the physical impairments B.Exercise protocols for maintaining physical strength and cardiovascular endurance C.Current functional status and anticipated occupational performance upon return home D.Recommendations for home modifications to maximize accessibility and task performance

D.Recommendations for home modifications to maximize accessibility and task performance

A client with stage 2 Parkinson's disease is preparing to move from sitting on a tub transfer bench in the bathtub to standing up at a walker placed outside the bathtub. Which method would be effective for the client to use at the start of the transfer? A.Pulling forward on the grab bar mounted to the bathroom wall B.Bearing weight through both arms by pressing down on the bench C.Using a towel to quickly rub the larger muscles of the thighs D.Rocking rhythmically back and forth on the bench a few times

D.Rocking rhythmically back and forth on the bench a few times -This is a technique used with clients who have difficulty initiating motion due to rigidity associated with this stage of Parkinson's disease.

An OTR has completed an evaluation of a patient who is experiencing complications from pneumonia and was recently admitted to a Medicare funded skilled nursing facility. The patient was living independently prior to hospitalization and wants to return home. Evaluation results indicate the patient is generally deconditioned and fatigues quickly during activity. The patient ambulates slowly using a walker, and requires frequent verbal and physical cueing for safety when using the walker during ADL. What criteria should the OTR use to prioritize the goals for this patient's intervention plan? A.Patient's desire to improve strength, ROM, and endurance prior to discharge from the facility. B.Amount of time the patient will need to maximize strength and endurance prior to returning to independent living. C.Amount of assistance that will be available to the patient to maintain progress after discharge. D.Skilled services the patient currently requires for completion of basic functional tasks.

D.Skilled services the patient currently requires for completion of basic functional tasks. - in medicare facility, you must must prioritize goals based on medical necessity and skilled services needed for function.

A client has relapsing-remitting multiple sclerosis and recently transitioned from assisted ambulation to using a standard wheelchair for mobility. A recent onset of fatigue, upper extremity weakness, and back and neck discomfort is beginning to interfere with job performance. The client is employed as a magazine editor, and spends much of the day sitting in the wheelchair while working at the computer monitor positioned at eye level. The client wants to continue sitting in a wheelchair to avoid having to complete transfers when moving from the desk to other parts of the work area. Which modification would be MOST BENEFICIAL for this client? A.Voice-controlled computer system and telephone headset B.Deltoid aid and a split design computer keyboard C.Power scooter with padded seat and electric tilt-in-space control D.Solid seat insert, lumbar support and bilateral forearm supports

D.Solid seat insert, lumbar support and bilateral forearm supports

An OTR has completed the initial evaluation of an inpatient who has leukemia. The OTR plans to work with the patient on a daily basis prior to the patient's planned discharge to home in one week. What information is MOST IMPORTANT to include in the initial evaluation report? A.Details about interventions for promoting goal attainment B.Descriptions of community support services available C.Recommendations for post-discharge OT services D.Summary data as it relates to the occupational profile

D.Summary data as it relates to the occupational profile

An OTR is providing a home program to a client who has stage 1 complex regional pain syndrome. What information should be included in the home program to MOST EFFECTIVELY help the client with symptom management? A.Instructions for incorporating energy conservation into daily tasks B.Recommendations for one-handed activities to protect the affected hand during ADL C.Pictures illustrating passive ROM exercises for each joint of the affected hand D.Techniques for elevating the hand and completing active ROM exercises

D.Techniques for elevating the hand and completing active ROM exercises -In the presence of complex regional pain syndrome (CRPS), vasospasm and vasodilation results in an abnormal persistence of edema. Edema control is important to include in a home program. If edema is not controlled, the protein-rich exudates that cause swelling will result in collagen formation, which will cause joint stiffness and decreased functional mobility.

An inpatient has been participating in rehabilitation since having bilateral transfemoral amputations 2 months ago. The patient has good balance and Fair plus (3+/5) upper extremity strength, is independent with bed mobility and self-care using adaptive equipment, and requires stand-by assistance during wheelchair transfers and with wheelchair management. The patient is preparing for discharge to live at home with the spouse and an adult son. Modifications have been made to the main entrance of the home and the bathroom. The OTR plans to provide family education for promoting the patient's safe transition to the home environment. What information would be MOST BENEFICIAL to include as part of this process? A.Wrapping techniques for shaping and protecting the residual limb B.Methods for improving the patient's independence with transfers C.Energy conservation techniques for the patient to use during ADL D.Techniques the patient uses to transfer to a variety of surfaces

D.Techniques the patient uses to transfer to a variety of surfaces -most important for transition to home environment

An OT has determined that a patient is functioning at Level 8 on the Rancho Los Amigos scale. For today's session, the patient is taken on an outing to a grocery store and handed a grocery list with 5 items written on it. The patient demonstrates that she is able to read the list, but she is unable to initiate looking for any of the items on the list. The OT simplifies the task by covering the list with a piece of paper, leaving only the top item visible. The patient is then able to successfully retrieve that item from the shelf. Based on this behavior, what should the OT document about this patient's performance in the grocery store? Ideational apraxia interfering with task initiation Visual perception is improved by anchoring techniques Deficits in executive functioning interfering with performance Attention deficits are compensated for by adaptive strategies

Deficits in executive functioning interfering with performance -document about PERFORMANCE; not specific skills

Marva is a 76-year-old woman who lives in a long-term care facility. Her score on the Allen Cognitive Levels screening test is a 4.0. What abilities would you expect to observe while watching Marva's daily routine? Becomes distracted by other people in the hallway, learns a new task through trial and error Dresses self with minimal assistance, locates the dining room, therapy clinic, and own room within the facility with a few cues from staff, and becomes distracted by other people in the hallway Becomes distracted by other people in the hallway, locates the dining room Assists caregiver with dressing, tries to walk towards dining room but doesn't seem to know where she has to go

Dresses self with minimal assistance, locates the dining room, therapy clinic, and own room within the facility with a few cues from staff, and becomes distracted by other people in the hallway

An OT is treating a patient who recently sustained a C6 spinal cord injury. The patient has active wrist extension but no hand function. What can the OT recommend to assist this patient in picking up and holding objects independently? Tenodesis splints A universal cuff A reacher Gripping aids

Tenodesis splints

What is the main purpose of using the ACLS for a developmentally disabled adult? Identifying the person's difficult behaviors Improve the person's prevocational skills Estimate the person's cognitive level Improve the person's social skills

Estimate the person's cognitive level

If an individual with cognitive dysfunction can demonstrate a whipstitch, but cannot imitate a cordovan stitch, which of the following is most likely indicated? The patient can think of hypothetical situations and do mental trial and error problem solving Further evaluation should focus on determining whether the patient can perform simple tasks independently using visual cues There is some awareness of large objects in the environment and the patient may assist the caregiver in simple tasks The patient has a minimal response to the external environment

Further evaluation should focus on determining whether the patient can perform simple tasks independently using visual cues

An OTR working in a community mental health setting is using dialectical behavior therapy to guide intervention planning for a client who has a borderline personality disorder. The client works in a university setting and is at risk of losing the job due to verbal outbursts and erratic mood swings. The client is aware that behavioral changes are needed and reports a willingness to work toward these changes. Which type of intervention would be MOST BENEFICIAL for supporting progress toward this objective when using this approach? A.Group skills training modules in mindfulness, interpersonal effectiveness, emotion modulation, and distress tolerance B.Role-play sessions using prepared scripts for practicing problem-solving and decision-making related to workplace situations C.Individual sessions focusing on Socratic questioning and guided discovery for addressing problematic situations and emotions

Group skills training modules in mindfulness, interpersonal effectiveness, emotion modulation, and distress tolerance -DBT uses mindfulness skills to manage the period of intense emotion and also encourages meditative activities

An OTR is fabricating an orthotic for a client who has a claw-hand deformity secondary to an ulnar nerve injury several months ago. Which type of orthotic is indicated for this client? A.Dorsal-based MCP joint blocking orthotic with a dynamic component to pull the fourth and fifth digit IP joints into extension B.Volar-based forearm and hand static orthotic that blocks MCP joint hyperextension while allowing IP joint motion C.Hand-based orthotic that positions the fourth and fifth digits in 30° - 40° of MCP joint flexion while allowing IP joint motion D.Low-profile dynamic orthotic that blocks hyperextension of the second through fifth digits and has an IP joint extension outrigger

Hand-based orthotic that positions the fourth and fifth digits in 30° - 40° of MCP joint flexion while allowing IP joint motion -A claw-hand deformity is characterized by hyperextension of the fourth and fifth digits and is typically secondary to an ulnar nerve injury. Blocking the fourth and fifth MCP joints in slight flexion allows the extensor digitorum communis tendon to extend the IP joints in the absence of the ulnar innervated intrinsic muscles. The orthotic will enable the client to have a more functional grasp.

Which computer keyboard adaptation would help a child who cannot use his hands? Magnification Head pointer or switch Screen reading software program Synthesized speech device

Head pointer or switch

What is the BEST way to react if a patient begins to slip from the OTA's grasp during a commode-to-bed transfer? It would be best to have the patient return to the commode It would be best to ease the patient to the floor, then get assistance It would be best to continue with the transfer, and try to get the patient to the bed as quickly as possible It would be best to call for help, and get the attention of the patient's nurse

It would be best to ease the patient to the floor, then get assistance

A 42-year-old woman who works at a law firm as a paralegal is having visual difficulties working at the computer. What would be the MOST useful AT device she could use? Larger font Voice activated Wireless mouse Word prediction software

Larger font

What type of activity can an individual at Allen Cognitive Level 5 perform? Get items from the grocery store Wash their face Learn a new recipe for dinner Put cereal into a bowl

Learn a new recipe for dinner

An OT conducts a screening on a patient who recently sustained a traumatic brain injury. The patient is observed to be awake on and off during the day, turns towards the TV when it is on, and is able to squeeze the OT's hand when instructed to. What Rancho Los Amigos level is this patient functioning at? Level 3: Localized Response Level 2: Generalized Response Level 1: No Response Level 5: Confused and Inappropriate

Level 3: Localized Response

A patient is able to copy demonstrated directions presented to him one step at a time and is interested in doing simple two dimensional projects. At what ACL level is this patient functioning? Level 3 Level 4 Level 5 Level 6

Level 4

During an OT session, a patient is able to perform a goal-directed activity of getting items at a grocery store. What ACL are they at? Level 2 Level 3 Level 4 Level 5

Level 4

When working with patients who sustained a TBI, OT practitioners commonly use both the Rancho Los Amigos Scale and Allen Cognitive Levels (ACL) to guide their intervention. If a patient is functioning at RLA 7, which ACL level does this correspond to? Level 5 Level 4 Level 3 Level 6

Level 5

At what ACL is a patient functioning if they are able to anticipate and avoid hazardous situations? Level 7 Level 6 Level 5 Level 4

Level 6

Max is a 52-year-old patient who has been admitted to an inpatient facility for rehabilitation after sustaining a severe TBI. Max is aware that he has been injured and that is why he is in the rehab facility, but he has little insight into the extent of his impairments. He is starting to recognize some of the nursing staff, but he is unable to recall their names. In terms of his ability to learn, he is able to follow simple instructions and he has re-learnt to brush his teeth independently but he is not yet able to learn new tasks. Max enjoys watching the morning news on a daily basis, but in order to focus on the news, he has to watch the TV in his room where there is the least amount of distractions. After about 30 minutes however, Max becomes disinterested in watching TV and he starts walking around the facility. At what Rancho Los Amigos level is Max functioning? Level 7 Level 6 Level 5 Level 4

Level 6

If a patient is able to follow a set schedule and follow a self-care routine independently, what Rancho level is he functioning at? Level 6: Confused and Appropriate Level 7: Automatic and Appropriate Level 5: Confused and Inappropriate Level 1: No response

Level 7: Automatic and Appropriate

At what Rancho Los Amigos level does a patient who sustained a TBI develop insight into their impairments? Level 6 Level 8 Level 5 Level 7

Level 8

Which Rancho Los Amigos Level would you assign to a patient who is able remember the main theme of stories but confuses the details and who can pay attention for about 30 minutes unless the environment or complexity of the task distract them? Level VI Level III Level IV Level V

Level VI

A patient with Muscular Dystrophy is having difficulty typing on the computer due to decreased strength. What type of adaptive keyboard would be MOST useful to recommend to this patient? Oversized keyboard Light touch keyboard One-handed keyboard Large print keyboard

Light touch keyboard

An OT is working with a 75-year-old female patient who presents with weakness in her upper extremities and torso. As a result, she has great difficulty bending down and picking up objects inside her home, and playing corn hole (a game of tossing bean bags into a hole) with her grandchildren has become challenging. What type of adaptive equipment would be best to recommend to this patient? Stander lever extender Long-handled reacher Environmental control unit Magnifier

Long-handled reacher

An individual with bilateral proximal weakness identifies a goal of independence in self-feeding. Which equipment is most beneficial for the occupational therapist to recommend for goal attainment? a. Extended long-handled utensils. b. Built-up handled utensils. c. An electric feeder. d. Mobile arm supports

Mobile arm supports -no shoulder needed

Seating biomechanics take into account the client's stability, postural support, and mobility needs. When determining seating requirements, what would the OT FIRST assess? A. Head and neck B. Upper extremities C. Trunk D. Pelvis

Pelvis The pelvis and lower extremities need to be stabilized first for overall postural support, followed by the trunk, and then the head, neck, and upper extremities can be addressed for both stability and mobility needs.

When administering an evaluation of upper extremity function to a newly admitted patient with Guillain-Barre syndrome, it is MOST important to: Test proximal muscle strength first. Perform the evaluation over several sessions. Include sensory testing. Evaluate range of motion.

Perform the evaluation over several sessions.

A patient who was recently involved in a MVA in which he sustained a TBI, is functioning at a Level II according to the Rancho Los Amigos scale. What is the MOST USEFUL pre-feeding technique that could be used for graded stimulation of the patient's cranial nerves? Place small pieces of the patient's favorite foods on the front of the patient's tongue Place items with strong smells (i.e. cinnamon, lavender, orange) under the patient's nose for 2-3 seconds Gently rub different flavorings of salt and sweet foods on the back of the patient's tongue Provide tactile massage on the patient's jaw, nose, and ears to prepare for mastication

Place items with strong smells (i.e. cinnamon, lavender, orange) under the patient's nose for 2-3 seconds The patient is probably not able to respond appropriately to stimuli placed in the mouth, as this requires a higher level of response, but will be able to respond to a strong smell.

What adaptive eating dishes and utensils would be appropriate for a patient with a diagnosis of Parkinson's Disease? Select the best 3 choices. Knife/fork combo Plate guard Weighted utensils Coated spoon Non-slip placemat Redware

Plate guard Weighted utensils Non-slip placemat

An OTA is working with a patient who is recovering from a recent TBI. The focus of the intervention is currently on meal preparation. The patient has successfully prepared cold cereal. In order to appropriately grade this activity, what is the next type of meal the OTA should select, to help this patient progress to the next level? Prepare a hot one-dish meal, such as oatmeal. Prepare a hot beverage, soup, or prepared dish, such as hot chocolate. Prepare a hot multi-dish meal, such as a family dinner. Access a prepared meal, such as opening an already heated microwave dinner.

Prepare a hot beverage, soup, or prepared dish, such as hot chocolate.

Billy, a 26-year-old auto mechanic who recently sustained a TBI, is currently functioning at Rancho Los Amigos IX (Purposeful-Appropriate). Billy is participating in a 90-minute treatment session which is focusing on developing strategies to improve his problem-solving skills. The session is taking place at a supermarket and so far, Billy has been successful in making shopping lists and finding the items on his list. When he arrives at the soda aisle however, he becomes increasingly frustrated as there is a wide variety of root beer to choose from and he is unsure which brand of root beer to select. What should the OTR® do NEXT to help Billy continue on with his shopping? Provide a reward to reinforce improved behavior regulation Give him a time constraint by turning on an audible countdown timer Probe his thinking by asking, "What steps should you follow to narrow down your choice?" Patient is exhibiting decreased carryover and requires 1-to-2 step directions

Probe his thinking by asking, "What steps should you follow to narrow down your choice?"

An OT screens a patient using the ACL and determines that the patient is functioning at a level 3 (manual actions). What is the best advice the therapist can provide to the patient's family upon discharge? Develop a grooming routine for the patient in the morning. Provide the patient with a structured task at home. Make sure the patient has leisure activities. Provide the patient with 24-hour supervision.

Provide the patient with 24-hour supervision.

At which Rancho level would you place a recent TBI patient who blinks at light, turns towards sound, responds to family members, and whose eyes follow an object? Rancho level 4 Rancho level 5 Rancho level 6 Rancho level 3

Rancho level 3

At which Rancho level is a patient functioning if they are able to turn towards sound, respond to family members, and follow an object visually? Rancho level IV Rancho level V Rancho level VI Rancho level III

Rancho level III

For which patient would you recommend the use of a rocker knife, to use as an adaptive eating utensil? Cerebral palsy Right below elbow amputation Macular degeneration Type II diabetes

Right below elbow amputation

An OT has evaluated an elderly patient who presents with generalized muscle weakness. During the initial consultation, the patient reported that she is having great difficulty cutting her food. What adapted utensil should the OT recommend to this patient, to compensate for her weak grasp and to enable her to maintain her independence when preparing meals and eating? Swivel utensil Rocker knife Weighted utensil Universal cuff

Rocker knife

An OT is working with a patient who has a strong tonic bite reflex. In order to feed this patient applesauce without eliciting the reflex, what type of utensil should the OT use? Rubber-coated spoon Weighted utensil Long handled utensil Spork

Rubber-coated spoon

An OT is performing a physical examination as part of a positioning and seating assessment. For which trio of conditions would the OT apply manual pressure to the pelvis to determine whether a deformity is flexible or inflexible? A. Scoliosis, lordosis, sarcoidosis B. Lordosis, exocytosis, windswept deformity C. Kurtosis, kyphosis, lordosis D. Scoliosis, windswept deformity, kyphosis

Scoliosis, windswept deformity, kyphosis -With scoliosis, the pelvis rotates anteriorly; with windswept deformity, the pelvis rotates laterally; and with kyphosis, the pelvis rotates posteriorly.

An OTA has been assigned to work with a patient who is recovering from a recent TBI. The patient is alert and the plan is for them to be transferred to an outpatient rehab facility. When treating this patient, which deficit would have the MOST impact on the patient's ability to contribute to and participate in their OT intervention plan? Motor function Sensory function Memory Self-awareness

Self-awareness

According to Abraham Maslow's hierarchy of needs, which level involves the need for and understanding of one's self-worth? a. physiological b. love and belonging c. self-esteem d. self-actualization

c. self-esteem

In order for an indv sitting in a wc to achieve maximal postural positioning, the ot should position the indv pelvis in a a. moderate posterior tilt b. neutral position c. slight anterior tilt. d. slight posterior tilt.

c. slight anterior tilt.

Which professional would facilitate discharge planning from a hospital to a sub-acute rehab facility, based on the recommendations of the medical team? a. psychiatrist b. substance abuse counselor c. social worker d. psychologist

c. social worker

A 35-year-old airport shuttle driver sustained an injury to his left upper limb resulting in an above- elbow amputation of that limb. In order for this patient to return to work, what type of adaptation needs to be added to the vehicle? Hand controls Pedal extender Automatic car door opener Spinner knob

Spinner knob

Michael is a 53-year-old inpatient who was recently involved in a MVA which resulted in him sustaining a TBI and a minor injury to his left non-dominant upper limb. Michael has stated that his main long-term goal is to return to his job, working as a sales assistant in a men's clothing store. On a cognitive level, Michael is functioning at Rancho Los Amigos IX - Purposeful Appropriate. His static hand splint which he has been wearing for the past 6 weeks, has only just been removed and the focus of OT intervention is on facilitating bilateral hand function, and improving his sensory perceptual and attention skills. Which of the following activities would MOST EFFECTIVELY utilize an integrated functional approach involving the use of these skills in order to help Michael achieve his long-term goal? Drawing a map on how to get from his home to his work Sorting through utensils in a drawer and making scrambled eggs on the stove Folding heavy loads of laundry and placing them on shelves in cabinets Playing a familiar game of chess while seated for 20 minutes

Sorting through utensils in a drawer and making scrambled eggs on the stove

An OT practitioner is working with a patient who has generalized low muscle tone which is affecting their ability to use utensils in an efficient manner. What utensil would be the MOST appropriate to recommend to this patient to promote their independence during meal times? Cylindrical foam Universal cuff Weighted utensil Swivel spoon

Swivel spoon -helps when clients can't use muscles to balance food

A patient who sustained a head injury as a result of a sports injury, is currently functioning at Level VI (Confused-Appropriate) on the Rancho Los Amigos Scale. The patient is due to be discharged and the plan is for him to go live with his 44-year-old sister. As she works from home, she has agreed to be available 24/7, to help with her brother's care. Before the patient is discharged, what is the most important information the patient's sister should be given to BEST equip her to take care of her brother? Effective methods of cueing to help the patient initiate and end BADLs List of mental health resources to ensure the patient maintains regular visits to prevent depression Strategies to use to reduce or prevent caregiver burnout The importance of using visual cues as reminders

Strategies to use to reduce or prevent caregiver burnout

What adapted utensil is designed to keep the head of the utensil level in order to prevent spills. It is best suited for patients who have a tremor, upper limb weakness or poor co-ordination? Swivel utensil Rocker knife Weighted utensil Universal cuff

Swivel utensil

A patient with a TBI takes 23 minutes to brush his hair, despite being able to pick up his hair brush, comb his hair and apply hairspray independently. In which of the following areas does this behavior MOST likely indicate difficulty? Task completion Difficulty following verbal cues Manual dexterity Short-term memory

Task completion

A patient presents with numbness and tingling along the median nerve pathway in the hand, as well as pain in the forearm. What is the likely condition? a. pronator teres syndrome b. carpal tunnel syndrome c. guyon's canal d. cubital tunnel syndrome

a. pronator teres syndrome -compression in forearm vs wrist (carpal tunnel)

A middle-aged office assistant with muscular dystrophy is finding using her standard keyboard challenging as she is quickly becoming fatigued. The patient has stated that she wants to continue to use her keyboard as she is accustomed to it and she does not like change. With this in mind, what recommendation can be made to enable this patient to conserve her energy whilst still using her favored keyboard? The best option would be a word prediction program to save keystrokes The best option would be a screen reading program The best option would be a switch access with a scanning program The best option would be a screen enlargement program

The best option would be a word prediction program to save keystrokes

a college is converting a historical building into wheelchair accessible dormitory space. To allow for a 360 degree turning radius, the ot recommends that the space between the student's desk and bed be a minimum of a. 4×4 b. 5×5 c. 6×6

b. 5×5

An OT practitioner has received an order to evaluate an individual who is 2 days post total hip arthroplasty, and expected to be discharged to home in 2 days. The most important areas to focus on are: a. Cognitive and perceptual functioning. b. Work, self-care, and leisure performance. c. Upper and lower extremity strength, ROM, and coordination. d. The need for adaptive equipment and assistance from others.

The need for adaptive equipment and assistance from others.

Jeremy a 32-year-old musician was admitted to an inpatient facility, a week ago, following a MVA. Jeremy sustained TBI in the accident and he is currently functioning at Rancho Los Amigos V - Confused, Inappropriate, Non-agitated. The OTR® is working on BADLs with him in a structured environment. Previously, Jeremy required maximal assist to total assist with grooming and hygiene, requiring 100% cueing. However, during his most recent session, after the OTR® rolled Jeremy's wheelchair to the sink, he spontaneously reached for his toothbrush and started to brush his hair. What is the MOST IMPORTANT information the OTR® should include in the documentation based on Jeremy's behavior during this session? The patient is demonstrating ideational apraxia The patient is showing initiative and beginning task-directed behavior The patient is improving cognitive abilities indicative of trial and error performance The patient regressing to behaviors and abilities associated with Rancho Los Amigos IV

The patient is showing initiative and beginning task-directed behavior -although he does have ideational apraxia, it is more important to say he imitated for goal attainment -document progress towards goals

What activity would a patient who is functioning at Rancho Los Amigos 8 (Purposeful and Appropriate) be able to accomplish? The patient will be able to shake your hand and pay attention for a few minutes The patient will be able to play a game of checkers and pay attention for 30 minutes The patient will be able to follow their morning grooming activity with some assistance The patient will be able to come up with several solutions to a problem such as being out of milk when trying to cook macaroni and cheese

The patient will be able to come up with several solutions to a problem such as being out of milk when trying to cook macaroni and cheese

Jorge is a 26-year-old man who sustained a C6 SCI and one of his goals is to feed himself which can be achieved with the use of a universal cuff. In order to teach Jorge how to use a universal cuff to scoop food with a spoon, where on the hand should the universal cuff be placed and where should the spoon be positioned? The universal cuff should be placed on the volar surface of the hand and the spoon placed in the ulnar side of the pocket The universal cuff should be placed on the dorsal surface of the hand and the spoon placed in the ulnar side of the pocket The universal cuff should be placed on the volar surface of the hand and the spoon placed in the radial side of the pocket The universal cuff should be placed on the dorsal surface of the hand and the spoon placed in the radial side of the pocket

The universal cuff should be placed on the volar surface of the hand and the spoon placed in the radial side of the pocket

At what age would one expect an infant to display raking when grasping a pellet, according to Erhardt Prehension Developmental Levels? a. 9 months b. 6 months c. 8 months d. 12 months

b. 6 months

What is the earliest age a child could cut out a complicated picture? a. 8-9 years b. 6-7 years c. 4-6 years d. 2-3 years

b. 6-7 years

For a patient who recently suffered a CVA, which type of adaptive medical equipment would be the most helpful in supporting them to transfer into the tub at home, to bathe themselves? Hydraulic bath lift Three-in-one shower commode Shower chair with armrests Tub transfer bench

Tub transfer bench

In regard to object use, at what age is a child's participation with a toy characterized by the movement patterns as the child completes simple tasks with the toy? a. 3-6 months b. 9-12 months c. 6-9 months d. 12-15 months

b. 9-12 months

Sharon, a 35-year-old patient who recently sustained a coup-contrecoup brain injury, has been assessed to be functioning at Rancho Los Amigos V (confused, inappropriate). As a result of her injury, Sharon's level of alertness is erratic and fluctuates between periods when she has decreased alertness and attention, to periods when she is more alert and responsive. At this stage, Sharon requires maximal assistance for her BADLs. She is generally cooperative when completing these tasks but becomes agitated when confronted with the more challenging ADL tasks, such as brushing her teeth. Taking advantage of those periods when Sharon is more alert and receptive, which strategy would be the MOST effective to incorporate into her treatment plan to help her progress in performing her ADL tasks? Post-It® Notes placed on the mirror indicating steps in the procedure for hair care Use of the strategy of guiding the toothbrush followed by a one-step command such as "to mouth" with OT periodically stating, "Let's focus" Use of visual imagery and rhythmic cues to reduce hesitation to ensure the patient follows the sequence of brushing his teeth Use a self-instructional approach where the patient records a step, checks it, and refers to it when needed during application of makeup

Use of the strategy of guiding the toothbrush followed by a one-step command such as "to mouth" with OT periodically stating, "Let's focus"

An adult has been referred to occupational therapy. The individual demonstrates decreased ROM in the dominant hand secondary to a nerve injury. Active thumb ROM for the IP and MP is within normal limits. Active ROM of the IPs of all four fingers is 0°-60°. The individual wants to be able to hold a knife, spoon, and fork. Which utensils are best for the therapist to recommend to this person? a. Utensils with cylindrical foam handles, 1½" in diameter. b. Standard utensils with no adaptations. c. Utensils with custom-built handles made of low temperature thermoplastic splinting material. d. Utensils held in a universal cuff.

Utensils with cylindrical foam handles, 1½" in diameter.

An OT is working with a patient who presents with visual perceptual deficits secondary to a traumatic brain injury (TBI). During an intervention, the patient is handed a variety of small plastic shapes and boards on which patterns and pictures have been drawn. The patient is then asked to arrange the shapes directly onto the board to replicate these pictures and patterns. By using this activity, what 3 visual perceptual skills are being addressed? Select the 3 best answers Visual Sequential Memory Form constancy Visual Discrimination Figure ground Visual motor integration Visual memory

Visual motor integration Visual Discrimination Figure ground: see an object on top of background or scene

An OTR® is working with Mary, a college student who is recovering from a recent TBI. Mary has a grade 4+ strength (MMT) in both her upper extremities but due to marked intention tremors and postural instability her ability to grasp and manipulate a pen is being affected. As Mary is majoring in creative writing, continuing to write in her journal is an important goal for her. Which writing adaptation would be MOST appropriate to recommend to Mary to help her compensate for her deficits? Universal cuff with a pen loop attachment Computer keyboard with tilt feature Weighted pen and weighted wrist cuffs Forearm orthosis balanced on a sling with a built-up marker

Weighted pen and weighted wrist cuffs

A patient with a C5 SCI who has met all their occupational therapy goals, is preparing to be discharged to live at home with caregiver assistance. What type of device should be recommended for this patient to maximize their independence during self-care activities at home? Power wheelchair with head control Wheelchair-mounted mobile arm supports Electronic aid to daily living Custom-fitted tenodesis splint

Wheelchair-mounted mobile arm supports

Regarding quality improvement, which term includes strategies to enhance the effectiveness of a program? a .Performance assessment and improvement (PAI) b. Total quality management (TQM) c. Goal attainment scaling (GAS) d. Professional review organization (PRO)

a .Performance assessment and improvement (PAI)

An individual who is acutely psychotic has been brought to the hospital by a legal guardian. The individual neither responds to questions nor attends to visual stimuli in the room. Who should the therapist collaborate with to determine the individual's short-term goals? a .The guardian. b. The individual. c .The psychiatrist. d. The case manager.

a .The guardian.

An occupational therapist observes that an 18-month-old child is not able to creep more than a few steps. When the child looks up, both hips and knees flex and the child ends up W sitting with both arms extended and propped forward. When documenting this observation, which is most accurate for the occupational therapist to report the child is demonstrating? a .The influence of the symmetrical tonic neck reflex (STNR) resulting in delayed gross motor skills. b. An obligatory asymmetrical tonic neck reflex (ATNR) resulting in delayed gross motor skills. c. An intact tonic labyrinthine reflex which facilitates balance response

a .The influence of the symmetrical tonic neck reflex (STNR) resulting in delayed gross motor skills.

An OT practitioner is assessing an individual who demonstrates normal range of motion when flexing the elbow, but hyperextends by 15 degrees when the elbow is extended. The practitioner will MOST likely record the measurement as: a. -15 to 0 to 140 degrees. b. 0 to 140 degrees. c. 15 to 140 degrees. d. -15 to 120 degrees.

a. -15 to 0 to 140 degrees.

At what age does a child begin to try multiple actions to achieve his goal, continuing until one is successful? a. 12-15 months b. 18-21 months c. 9-12 months d. 21-24 months

a. 12-15 months

A child has recently begun to build with blocks both horizontally and vertically but cannot yet build a tower of nine cubes, what age is this child? Select one: a. 24-30 months b. 6-9 months c. 21-24 months d. 12-15 months

a. 24-30 months

In regard to typical development of toileting skills, at what age is a child first able to go to the bathroom himself, but continues to require assistance with hygiene and clothing management skills? a. 3 years b. 4 years c. 1 year d. 2.5 years

a. 3 years

During an occupational therapy screening session, the therapist observes that a child bangs objects on a tabletop but is unable to give up a toy upon request. The occupational therapist documents these behaviors. Which developmental level is indicated by the child's observed behaviors and would be the most accurate for the therapist to report? a. 3-4 months. b. 7-8 months. c. 9-10 months.

a. 3-4 months.

An occupational therapist is providing intervention to properly adapt workstations to maintain ergonomic standards. What is the recommended table height for doing seated precision work? a. 31-37 inches b.28-31 inches c. The height is not important d. 21-28 inches

a. 31-37 inches

In regard to self-feeding, at what age does an infant first display the ability to hold a bottle? a. 6-8 months b. 12-14 months c. 9-13 months d. 5-7 months

a. 6-8 months

An occupational therapist is educating an individual on wheelchair mobility, focusing on making turns in tight areas. What MINIMUM area dimensions are required for a wheelchair to adequately make a 360-degree turn? a. 60 inches x 60 inches b. 36 inches x 36 inches c. 48 inches x 48 inches d. 42 inches x 42 inches

a. 60 inches x 60 inches

Which of the following is NOT considered a normal vital sign value? a. 70 yr old male with a resting respiratory rate of 24 breaths per minute b. 30 yr old female with BP of 118/76 c. 58 yr old male with resting HR of 78 bpm d. newborn infant with HR of 120 bpm

a. 70 yr old male with a resting respiratory rate of 24 breaths per minute

At what age would one expect an infant to display an inferior pincer grasp when grasping a pellet, according to Erhardt Prehension Developmental Levels? a. 9 months b. 6 months c. 8 months d. 12 months

a. 9 months

Which intervention strategy is MOST appropriate to assist with work functions for a 30-year-old teacher who has recently been diagnosed with multiple sclerosis? a. A stool in the classroom for reducing fatigue b. A rolling walker to compensate for walking instability c. Large-print written handouts to compensate for visual impairment d. A motorized wheelchair to compensate for balance impairment

a. A stool in the classroom for reducing fatigue

An occupational therapist establishes a program for a new acute psychiatric unit at a community hospital. The therapist designs the physical layout of the occupational therapy department to include storage for arts and crafts materials. Which of the following should the therapist recommend to store these materials? a. A ventilated locked metal cabinet accessible only to staff. b. Open shelving accessible to patients. c. Shelving next to a sink for easy clean up. d. A locked closet outside of the intervention area to ensure safety.

a. A ventilated locked metal cabinet accessible only to staff. -most secure

The parents of an 18-month-old bring their child to a free community developmental screening. The child can attend to shapes and use them appropriately. However, the parents are worried because the child cannot match shapes or manipulate different shaped objects into a shape sorter. Which is the best response for the occupational therapist to make in response to the parents' expressed concern? a. Advise the parents that the child is showing a typical, age-appropriate skill. b. Complete an occupational therapy evaluation of the child's cognitive skills. c. Refer the child to the early intervention program for developmental delay. d. Provide the parents with activity recommendations to develop shape recognition.

a. Advise the parents that the child is showing a typical, age-appropriate skill. -shape sort is 2.5-3 years

An OT practitioner is evaluating two-point discrimination in an individual with median nerve injury. The MOST appropriate procedure is to: a. Apply the stimuli beginning at the little finger and progress toward the thumb. b. Test the thumb area first, then progress toward the little finger. c. Present test stimuli in an organized pattern to improve reliability during retesting. d. Allow the individual unlimited time to respond.

a. Apply the stimuli beginning at the little finger and progress toward the thumb. -median nerve effects thumb side so go from good side to bad side

For the following goal, which piece of the SMART goal acronym is missing? The patient will increase lower body dressing from Max Assist to Independent within one week. a. Attainable b. Measurable c. Relevant d. Time-limited

a. Attainable

An occupational therapist receives a referral to provide home-based services to an older adult who lives alone in a fourth floor walk-up apartment. Upon entering the apartment, the therapist notes the sweltering heat. The apartment has no fans or air conditioners. The client's skin is hot, dry, and red, and breathing is labored. The therapist offers the client a glass of water and places ice compresses on the arterial pressure points to help with cooling. Which is the most important action for the therapist to take next? a. Cancel the intervention session and call for an ambulance to provide emergency medical services. b. Proceed with the planned intervention session and include documentation about client's environmental conditions in the intervention report. c. Contact the home health agency's case manager to report the client's environmental conditions and then proceed with the planned intervention session. d. Cancel the intervention session and advise the client to speak to a doctor on how to best address the impact of hot weather on personal health.

a. Cancel the intervention session and call for an ambulance to provide emergency medical services.

As defined by Yalom, which "curative factor of groups" involves the expression of emotion? a. Catharsis b. Guidance c. Universality d. Altruism

a. Catharsis

An OT practitioner measures an individual's elbow PROM three times, and gets three different measurements, varying by up to 10 degrees. The BEST action for the therapist to take is to: a. Check the alignment of the goniometer. b. Use a larger goniometer. c. Use a smaller goniometer. d. Attempt to force the individual's arm further into flexion.

a. Check the alignment of the goniometer.

Which of the following assessment tools uses observation of behavior to rate the presence and severity of autism in children two years old and older? a. Childhood Autism Rating Scale (CARS) b. Revised Knox Preschool Play Scale (RKPPS) c .Early Coping Inventory d. Toddler and Infant Motor Evaluation (TIME)

a. Childhood Autism Rating Scale (CARS)

A patient presents with shoulder impingement and has pain and limited shoulder range of motion. What is an appropriate initial intervention to address these limitations? a. Codman's exercise b. Strengthening of the shoulder with a resistance band c. Shoulder splinting d. Isometric strengthening for the shoulder

a. Codman's exercise

An OT is evaluating a client who is complaining of severe pain, edema and temperature changes in their L UE after a fall. Upon evaluating the clients' arm, you also notice discoloration and tropic changes. What UE injury do you suspect the client to have? Select one: a. Complex Regional Pain Syndrome (CRPS) b. Carpal Tunnel Syndrome (CTS) c. Skier's Thumb d. Dupurytren's Disease

a. Complex Regional Pain Syndrome (CRPS)

Proprioceptive neuromuscular facilitation (PNF) is used therapeutically for many individuals with neurological conditions; it focuses on using movement patterns to improve motor skills for functional tasks. Which upper extremity pattern is used when the therapist is trying to promote improved function in reaching above the head and behind the body, like when reaching for a seat belt using the arm closest to the belt's origin? a. D2 flexion b. D1 flexion c. D1 extension d. D2 extension

a. D2 flexion

A middle school-aged child with right upper extremity amelia attends occupational therapy to learn how to dress independently. Which of the following is most beneficial for the occupational therapist to focus on during intervention? a. Donning and doffing a variety of shirt types of personal preference. b. Donning and doffing only shirts that can be donned overhead. c. Donning and doffing shirts that button in the front. d. Donning and doffing shirts with Velcro tabs sewn on to replace buttons.

a. Donning and doffing a variety of shirt types of personal preference. -Amelia is congenital absence of part of arm

A seven-year-old child with spina bifida at the C7 level receives home-based occupational therapy services. Which ability is most relevant for the occupational therapist to focus on during intervention? a. Dressing the lower body. b. Dressing the upper body. c. Playing tabletop games.

a. Dressing the lower body.

Which of the following hand and wrist disorders has poor success with conservative treatment? a. Dupuytren's disease b. De Quervain's tenosynovitis c. Carpal tunnel syndrome d. Trigger finger

a. Dupuytren's disease

Which model of practice views disability as an impaired skill base and focuses on helping the person learn new skills? a. Education model b. Medical model c. Community model d. Telehealth model

a. Education model

An occupational therapist is intervening to improve self-care for a patient with a spinal cord injury at C7-C8. What is an appropriate self-care intervention for this patient? a. Education on the use of a button hook for independent dressing b. Education on the use of angled utensils for feeding c. Education in self-advocacy for instructing others on preference of care d. Practice using a straw for drinking liquids

a. Education on the use of a button hook for independent dressing - probably already independent in eating

An occupational therapist leads a social skills group for children aged 10-12 with conduct disorders. One of the children complains that the group activity is stupid and boring. Which is the most effective response for the occupational therapist to provide in response to this complaint? a. Encourage the child to complete the activity with the group. b. Allow the child to leave the group since they are uninterested. c. Allow the child to suggest a different group activity. d. Tell the child the complaint will be discussed at the next family meeting.

a. Encourage the child to complete the activity with the group.

According to Abraham Maslow's hierarchy of needs, what is the lowest level of need that one must attain before being able to pursue any higher levels? a. Physiological b. Love and belonging c. Safety d. Self-esteem

a. Physiological

A client in the mid-stage of Alzheimer's disease lives at home with a spouse as the primary caregiver. The spouse wants the client to remain at home as long as possible; despite a progressive decline in the client's cognition. What strategy should be included in the recommendations for supporting the client's aging-in-place? A. Establish a routine schedule of familiar daily activities to encourage engagement. B. Have the client use a variety of assistive devices to minimize task demands. C. Plan most activities for the morning to avoid the effects of sundowning.

a. Establish a routine schedule of familiar daily activities to encourage engagement.

A young adult with a ten-year history of serious mental illness is being discharged home in two days. The client collaborates with the care coordination team to plan discharge with the client's primary family members. The team consists of a psychiatrist, a registered nurse, a social worker, and an occupational therapist. The team conducts a predischarge family meeting to provide family members with information to assist them in supporting the client's recovery. Which is the most relevant information for the occupational therapist to provide to the client's primary family members at this meeting? a. Family role activity suggestions and potential adaptations. b. The therapeutic effects and potential side effects of medications. c. Advocacy strategies and consumer/family resources. d. Family dynamics information and family support groups.

a. Family role activity suggestions and potential adaptations. -OT is only one in care team who can do this -others can take care of other things

Which principle of the Occupational Therapy Code of Ethics refers to the manner in which you approach clients, professionals, and colleagues when acting as a healthcare provider? a. Fidelity b. Nonmaleficence c. Veracity d. Justice

a. Fidelity

An occupational therapist provides intervention to develop independent feeding skills in an 18-month-old child with significant developmental delays. The child has mastered the ability to hold a spoon and bang it on the tray of the high chair. The child can also hold and suck on a cracker. Which activity is best for the therapist to provide next during intervention? a. Finger-feeding soft foods. b .Scooping food and bringing it to the mouth. c. Taking cereal from a spoon held by the therapist. d. Bringing a filled spoon to the mouth.

a. Finger-feeding soft foods. -The typical developmental sequence of feeding is taking cereal from a spoon (5-7 months), self feeding by sucking on a cracker (6-9 months), holding and banging a spoon (6-9 months), finger feeding soft foods (9-13 months), bringing a filled spoon to mouth (12-14 months), and scooping food and bringing to mouth (15-18 months).

An occupational therapist conducts an in-service at an outpatient wheelchair clinic for individuals with central nervous system dysfunction. According to the principles of wheelchair prescription, which of the following statements is accurate for the therapist to make during the presentation? a. Firm seats are needed to provide stability. b. Soft seats are needed to prevent decubiti. c. Back heights should be extended to facilitate weight shifting. d. Seat angles should be 45° to prevent falling forward.

a. Firm seats are needed to provide stability. -the rest are false

Which type of upper extremity amputation is described when the amputation includes the arm as well as the scapula and clavicle? a. Forequarter b. Elbow disarticulation c. Shoulder disarticulation d. Above-elbow

a. Forequarter

An occupational therapist is seeing a new patient for evaluation. Upon reviewing medical records, the OT reads that the patient fell in her home and now has a Smith's fracture. What should the OT expect to see?Select one: a. Fracture of the distal radius with volar displacement b. Fracture of the distal ulna c. Fracture of the distal radius with dorsal displacement d. Fracture of the 5th metacarpal

a. Fracture of the distal radius with volar displacement

Which area of the brain is responsible for voluntary motor movement? a. Frontal lobe b. Parietal lobe c. Temporal lobe d. Occipital lobe

a. Frontal lobe -premotor cortex

Which simple edema reduction strategy is typically NOT included in a home exercise program for a patient with moderate edema present in the hand? a. Heat modality b. AROM c. Isotoner glove d. Elevation of the hand

a. Heat modality

A patient is unable to identify common objects by the sense of touch without utilizing vision. This is an example of: a. astereognsis b. topographical disorientation c. ideational apraxia d. asomatognosia

a. astereognsis

Which of the following is NOT a factor contributing to decreased nutrition and the desire to eat in the elderly? a. Increased saliva production b. Decreased ability to taste food c. Decreased ability to smell food d. Dentures that do not remain fixed in place

a. Increased saliva production

Which is a function of the sympathetic division of the autonomic nervous system? a. Increases heart rate b. Increases peristalsis c. Reduces blood pressure d. Stimulates flow of saliva

a. Increases heart rate

After six months of rehabilitation for a T2 spinal cord injury, a patient is being discharged. The occupational therapist conducts a home visit to evaluate accessibility. The individual lives with two roommates in an apartment in a private home. The doorway measurements currently range from 30 to 32 inches throughout the apartment. The patient's landlord is amenable to making changes in the apartment but has no financial resources. Which recommendation is best for the therapist to make for independent accessibility in the apartment? a. Install offset hinges on all doors. b. Remove doorframes of doorways less than 32 inches and install wider frames. c. Remove all doors except for the apartment entrance door.Remove all doorframes and install 36-inch-wide doorframes.

a. Install offset hinges on all doors.

An individual with a spinal cord injury at C7 reports noticeable redness on the ischial tuberosity during self-exam with a mirror. Which action is most effective for the occupational therapist to recommend in response to client's observations? a. Integrate weight shifting into daily activities. b. Use a tilt-in-space wheelchair. c. Use an angled foam cushion. d. Self-direct caregivers to assist with weight shifting at least once every 30 minutes.

a. Integrate weight shifting into daily activities.

An individual recovering from hepatitis, type C has decreased upper and lower extremity muscle strength and hypertension. Six months ago the client had an angioplasty and is very fearful of having a heart attack. Which should the occupational therapist instruct the client to perform to increase muscle strength? a. Isotonic exercises. b. Isometric exercises. c. Contract-relax exercise. d. Muscle contractions and holds.

a. Isotonic exercises. -weights

In a gross motor screening of a two-year-old, which tasks would he likely be able to demonstrate? a. Jumping downward off a step and walking up stairs one foot at a time unsupported b. Skipping and walking down stairs unsupported c. Creeping up stairs on hands and knees and attempting to jump d. Jumping off the floor with both feet and walking up stairs with alternating feet

a. Jumping downward off a step and walking up stairs one foot at a time unsupported

A therapist is looking to promote extensor control for an infant and wants to use reflexive properties to do so. The infant is seven months old. The therapist suspends the infant in a horizontal prone position. Which reflex is being incorporated into treatment? a. Landau b. Symmetric tonic neck c. asymmetric tonic neck d. tonic labyrinthine-prone

a. Landau

Which of the following upper extremity disorders is caused by repetitive use of the wrist extensors? a. Lateral epicondylitis b. Carpal tunnel syndrome c. Medial epicondylitis d. De Quervain's tenosynovitis

a. Lateral epicondylitis

A patient was recently admitted to a hospital following a traumatic brain injury. After approaching his hospital room for an initial evaluation, the OT identifies that the patient blinks his eyes when light crosses his visual field, responds inconsistently when asked to wiggle his fingers, and turns his head towards music being played on a visitor's phone in the room. According to the Rancho Level of Cognitive Functioning, at what stage may this patient be described within?Select one: a. Level III: Localized Response b. Level IV: Confused/Agitated c. Level II: Generalized Response d. Level VI: Confused, Inappropriate Non-Agitated

a. Level III: Localized Response

An OT practitioner is preparing to complete an initial evaluation on an individual diagnosed with obsessive-compulsive disorder. Which strategy for modifying the environment is likely to be MOST effective while interviewing this individual? a. Limit the time available to answer each question. b. Instruct the individual to take her time and not rush. c. Utilize open-ended questions. d. Minimize environmental distractions.

a. Limit the time available to answer each question. -OCD will spend forever coming up with anwser

An occupational therapist working in a skilled nursing facility conducts an inservice on validation therapy for the recently hired staff of a new psychogeriatric unit. Which fundamental principle of validation therapy is important for the therapist to include in this presentation? a. Listen to the words an individual uses to ascertain the person's underlying message. b. Provide highly structured activities to refocus the individual on reality. c. Provide unstructured activities to facilitate the expression of feelings. d. Listen to the words an individual uses and provide reality orientation for invalid statements.

a. Listen to the words an individual uses to ascertain the person's underlying message. -validation therapy was founded on the principles that the unspoken messages an individual conveys in their speech are more important than the actual content of the speech

And OT completes testing for touch sensation using two point discrimination for client following a nerve repair. Which rationale best describes why this assessment was chosen for the client? a. Maps nerve repair and receptor density b. Quantifies and locates pain sensory receptors c. Identifies tolerance to varying temperature d. determines propiroception and kinesthesia

a. Maps nerve repair and receptor density

An occupational therapist designs a dining rehabilitation program in a long-term care facility. The occupational therapist instructs paraprofessional staff in proper feeding techniques. Which point is most important for the therapist to include in this staff training? a. Meals should occur in a homelike environment with staff conversing with the residents being fed. b. Residents with swallowing difficulties should be fed in a group so that staff can remind them to swallow at the beginning of each meal. c. Placing three fingertips on the throat and pressing firmly will stimulate a swallow response. d. The head should be tilted slightly backward during feeding to facilitate an assisted swallow.

a. Meals should occur in a homelike environment with staff conversing with the residents being fed. -all others are false

In regard to statistical analysis for evidenced-based practice, which of the following is NOT a measure of variability? a. Mean b. Range c. Standard deviation d. Normal distribution

a. Mean - does not show spread of scores

An OTR is planning an initial intervention session with a client who is in the acute manic phase of bipolar disorder. Which intervention strategy is most important to achieve maximum participation? a. Minimize environmental distractions during task performance b. Allow the client to choose an activity of interest during therapy c. Encourage self-expression and creativity in social groups d. Talk to the client about the influence of mania on participation

a. Minimize environmental distractions during task performance

Which type of healthcare team involves physical, occupational, and speech therapists who provide interventions to a patient but do not work collaboratively with each other? a. Multidisciplinary b. Intradisciplinary c. Transdisciplinary d. Interdisciplinary

a. Multidisciplinary

An occupational therapist is treating a patient who has suffered an anterior neck burn. She is trying to prevent a contracture from developing. What is the appropriate position to encourage? a. Neck extension b. Neck side-bending c. Neck rotation d. Neck flexion

a. Neck extension

An OT practitioner needs to identify why an individual who attends a day program for adults with developmental disabilities has difficulty participating in group activities. The BEST way to determine whether the individual is experiencing anxiety during group sessions would be to: a. Observe the individual's body language during group sessions. b. Ask the individual to complete a questionnaire rating her anxiety level after each session. c. Have group members provide feedback to the individual and OT about her anxiety. d. Allow the individual to select two other clients she feels she'd be comfortable with in a small group.

a. Observe the individual's body language during group sessions.

An OTR is completing an evaluation of a client with MS. The client's medical information was already reviewed prior to the session. What information is MOST important for the OTR to collect prior to intervention plan? a. Performance patterns b. Sensorimotor skills c. Cognitive-perceptual skils d. Psychological issues

a. Performance patterns

A homemaker and parent is hospitalized for a major depressive disorder and prescribed Parnate to treat depressive symptoms. The patient's hobbies are gardening and jogging. Upon discussing the functional effects of medications with the patient, which is the most important precaution for the occupational therapist to review? a. Photosensitivity. b. Dietary restrictions. c. Orthostatic hypotension.

a. Photosensitivity. -idk I didn't get a pharmD why do I need to memorize this

An individual has had a brain tumor removed from the cerebellum. The occupational therapist conducts a screening to determine the need for further evaluation. During the screening the therapist observes deficits that indicate the need for further evaluation. Which would be most relevant for the therapist to evaluate based on this screening? a. Proprioception and coordination. b. Tactile and sensory integration. c. Vision and visual-perception. d. Audition and communication.

a. Proprioception and coordination.

An adult incurred an injury to the anterior spinal artery at the T12 level. The occupational therapist completes a sensory evaluation with this client. Which sensation is most likely for the therapist to document that the individual has retained? a. Proprioception. b. Pain. c. Crude touch. d. Temperature.

a. Proprioception.

A high school student with a diagnosis of borderline personality disorder and a history of self-abusive behaviors attends a transitional school-to-work program co-led by an occupational therapist and a COTA®. During the vocational skills group, the student expresses feelings of hopelessness about the future and questions the point of participating in the program. The student asks to leave the group due to being too tired to concentrate as a result of sleepless nights. The occupational therapist asks the COTA® to assume leadership of the group while the therapist addresses the student's concerns. Which action is best for the therapist to take in response to the student's statements? a. Pull the student aside from the group and ask if the student is feeling self-destructive. b. Allow the student to leave the group after reminding the student to relay concerns to the guidance counselor. c. Support the validity of the student's feelings and encourage the student to remain in the group. d. Remind the student that in a work setting the norm is to work even if fatigued.

a. Pull the student aside from the group and ask if the student is feeling self-destructive. -ask directly to deal with potential crisis -C. does not deal with safety of client

A patient in a rehabilitation facility has a complete C8-T1 spinal cord injury. The patient is learning techniques to maximize independence with dressing. Which technique should the OTR teach the patient to use to support progress toward this goal? a. Put on undergarments and trousers in bed, then transfer to a wheelchair to complete upper body dressing b. Use a universal cuff with a button-hook and zipper-pull to fasten front-opening clothing c. Wear a wrist-driven flexor hinge splint for pulling on and fastening clothing d. While supine in bed, put on socks and shoes; then put on underpants and trousers by rolling side to side in bed

a. Put on undergarments and trousers in bed, then transfer to a wheelchair to complete upper body dressing

An acute inpatient has been referred to OT after surgery for trauma to the right upper extremity. After surgery, the patient is reporting an inability to extend her fingers to release objects like her phone and toothbrush, as well as, difficulty holding and manipulating those objects. Upon quick examination, the OT notices that the client's wrist drops every time she lifts her right upper extremity off the bed; she is unable to perform wrist extension. What injury do you suspect the patient has suffered?Select one: a. Radial nerve laceration b. Radial nerve palsy c. Median nerve laceration d. Ulnar nerve laceration

a. Radial nerve laceration

An adult who incurred a severe traumatic brain injury (TBI) is entering the second week of care at a longterm TBI rehabilitation center. The patient's family visit regularly and frequently asks multiple questions of the treatment team. A team and family conference is planned to address family concerns. Which is the most important information for the team to share with the family? a. Realistic and clear information about the individual's current status and care plan. b. Each team member's expert opinion about the expected prognosis and discharge recommendations. c. Reimbursement information about each professional service to assist in determining treatment choices. d. Community resources that are available to support the family and provide respite care.

a. Realistic and clear information about the individual's current status and care plan.

A preschool-aged child with recurring headaches and decreased gross and fine motor skills is hospitalized on an acute care unit for a diagnostic workup. Just prior to the occupational therapy evaluation, the parents are told that their child has cancer. The parents are upset when they bring their child to the evaluation session. Which are the best actions for the therapist to take in response to this situation? a. Recommend the parents speak to their spiritual advisor or the social worker and proceed with the OT session. b. Cancel the OT session and recommend the parents speak to their spiritual advisor or the social worker. c. Spend the OT session providing support to the parents and addressing the parents' acceptance of the diagnosis. d. Recommend the parents speak to their spiritual advisor or the social worker and reschedule the OT session for later in the week.

a. Recommend the parents speak to their spiritual advisor or the social worker and proceed with the OT session.

Two COTAs working for a school district are assigned to two different schools. One school has a supervising occupational therapist, the other school does not. The COTA without a supervising therapist expresses concern to the COTA who is supervised about the lack of supervision. Which is the best response for the supervised COTA to take in response to this situation? a. Report the situation to their OT supervisor. b. Share information acquired during supervisory sessions with the unsupervised COTA. c. Advise the unsupervised COTA to contact the state regulatory board. d. Report the situation to the school district's administration.

a. Report the situation to their OT supervisor.

An individual is transferred from an acute care hospital to a subacute rehabilitation unit in a long-term care facility. The patient incurred a left cerebral vascular accident (CVA) in the middle cerebral artery (MCA) one week ago. The individual is referred to occupational therapy. The referral states that the patient has right hemiplegia and a subluxed right shoulder.B93. The occupational therapist also advises the direct care staff on proper positioning of the patient's right arm while the patient is seated in a wheelchair. Which is the most appropriate recommendation for the therapist to make for positioning of the patient's right arm? a. Rest the person's arm on a wheelchair lap board. b. Rest the arm in the person's lap with the hands folded. c. Have the person wear a shoulder sling throughout the day.

a. Rest the person's arm on a wheelchair lap board. -avoids scapular downward rotation and shoulder traction (pulling downwards)

An OT working in a school setting is observing a new client in the classroom. The OT notices that the child is reluctant to fully participate in the school environment. In order to assess functional performance to promote increased participation in the school environment, which assessment should the OT use?Select one: a. School Function Assessment b. Participation Scale c. Test of Playfulness Revised Version 3.5 d. Coping Inventory

a. School Function Assessment

A student has spasticity and extensor imbalance secondary to his cerebral palsy. As a result, the student is unable to stand independently. He uses a stander but experiences difficulty with hip and pelvic stability while standing. The student's teacher would like him to be able to stand and write on the Smartboard in her classroom. What adaptation could be made to the student's stander to allow him to complete this activity? a. Secure a pelvic band around the student's pelvis. b. Provide lateral trunk support to the stander c. Offset the axle on the student's wheelchair d. Tighten the student's chest strap

a. Secure a pelvic band around the student's pelvis.

A teacher requests you to screen a student in their classroom who they believe would benefit from OT services. You do an informal observation while the student is in class and notice the student is applying a lot of pressure into the pencil while writing his name and he gets up tries to bounce on a ball whenever the teacher is looking the other way. Which neurological threshold and behavioral response is this student exhibiting?Select one: a. Sensory seeking: high neurological thresholds and active behavioral responses b. Poor registration: high neurological thresholds and passive behavioral responses c. Sensory sensitivity: low neurological thresholds and passive behavioral responses d. Sensory avoiding: low neurological thresholds and active behavioral responses

a. Sensory seeking: high neurological thresholds and active behavioral responses

A 60-year-old auto-mechanic with diabetes and impaired sensation in the residual lower limb has been referred to OT following an above-knee amputation. The FIRST item the OT practitioner should address is: a. Skin inspection. b. Grooming techniques (shaving, trimming toenails, etc.). c. Retirement planning. d. Returning to work.

a. Skin inspection.

Which type of wrist fracture is characterized by the fractured segment being pushed forward toward the volar side of the wrist? a. Smith's fracture b. Colles' fracture c. Boxer's fracture d. Scaphoid fracture

a. Smith's fracture

Which form of cerebral palsy is characterized by hypertonia and spasticity? a. Spastic cerebral palsy b. All forms of cerebral palsy are characterized by hypertonia c. Ataxic cerebral palsy d. Dyskinetic cerebral palsy

a. Spastic cerebral palsy

Which of the following is NOT a component of an effective occupational therapy interview during an initial assessment? a. Speaking before listening in order to establish the tone of the interview b. Observing the patient's actions and tendencies throughout the interview c. Questioning as needed to obtain pertinent information d. Asking specific questions regarding functional performance

a. Speaking before listening in order to establish the tone of the interview

An occupational therapist is working with a child diagnosed with Spina Bifida. As the OT is doing her evaluation, she notices that the patient has some motor and sensory deficits in him. While interviewing the patient's mother, she stated that the child has bladder incontinence often. What classification of Spina Bifida does the patient exhibit?Select one: a. Spina bifida with a myelomeningocele b. Spina bifida meningocele c. Spina bifida occulta d. Spina bifida cystica

a. Spina bifida with a myelomeningocele

Which classification of burn involves only the epidermis? a. Superficial b. Deep partial-thickness c. Full-thickness d. Superficial partial-thickness

a. Superficial

An OT is seeing an 8-year-old girl diagnosed with Osteogenesis Imperfecta after recent surgery to fix a bone malformation. The OT is educating the family about safe exercises for the patient to engage in. Which of the following exercises or activities should the OT suggest to the patient's family considering the symptoms of Osteogenesis Imperfecta?Select one: a. Swimming/Water Therapy b. Gymnastics c. Cycling d. Ice Skating/Ice Hockey

a. Swimming/Water Therapy

In regard to research design, which type of sampling is being performed when a researcher selects individuals from a population list by selecting every eighth name from the list? a. Systematic sampling b. Random sampling c. Purposive sampling d. Stratified sampling

a. Systematic sampling - when the researcher obtains participants through a particular systemic method -stratified Is when they are obtain based on a certain trait

An occupational therapist provides bedside BADL training to a patient recovering from multiple injuries incurred during a motor vehicle accident. The patient's children arrive for a visit and ask the therapist to let them look at their parent's chart while they wait outside the room for the session to conclude. Which response is best for the therapist to make in response to this request? a. Tell the family members that they must have the permission of their parent before they can look at the chart. b .Commend the family members for their interest in their parent's status and give them the chart to read. c. Tell the family members to ask the unit's charge nurse for permission to look at the chart.

a. Tell the family members that they must have the permission of their parent before they can look at the chart.

Which muscle of the rotator cuff assists with external rotation and is innervated by the axillary nerve? a. Teres minor b. Supraspinatus c. Subscapularis d. Infraspinatus

a. Teres minor

A client participates in occupational therapy for intervention following a rotator cuff injury. The therapist provides progressive resistive exercises. When grading these exercises, which of the following is best for the therapist to increase? a. The amount of resistance provided with a stronger level of therapy band. b. The proximal load on the muscles the client uses during the exercises. c. The repetitions of external rotation exercises with less distal weight.

a. The amount of resistance provided with a stronger level of therapy band. -increase level of resistance. B. is load and C. is reps

An adult with amyotrophic lateral sclerosis frequently coughs and chokes when eating finely chopped foods and drinking thin liquids. The speech pathologist and occupational therapist collaborate and recommend a videofluoroscopy procedure. Which behavioral information would be most relevant for the occupational therapist to include on the referral? a. The client demonstrates minimal limitations in cognitive level. b. The client is able to consume chopped foods and apple juice with no difficulty. c. The client demonstrates only oral stage problems during eating. d. The client cannot tolerate therapy focused on improving feeding and swallowing skills.

a. The client demonstrates minimal limitations in cognitive level. -must be min cog limits to have procedure

An occupational therapist plans intervention for an individual with cognitive perceptual deficits. In deciding whether to use a dynamic interactional approach or a deficit-specific approach, which is most important for the occupational therapist to consider? a. The client's auditory processing skills. b. The availability of familial support. c. The client's social interaction skills. d. The client's problem-solving skills.

a. The client's auditory processing skills. The dynamic interactional approach utilizes awareness questioning to help the individual detect errors, estimate task difficulty and predict outcomes. Therefore, the therapist must consider the client's level of auditory processing skills to determine if adaptations or modifications are needed when implementing this approach. If an individual has severe auditory processing deficits, it may indicate a need to use a deficit specific approach.

An OT is working with an 88-year-old female client who was admitted to a SNF s/p R THA. During the evaluation, you ask the client what her goals are for therapy and she replies with: "I want to get stronger so I can go back home and be able to take care of myself." Which of the following statements is TRUE that the OT should keep in mind when designing her client-centered goals and exercise program?Select one: a. The exercise program should consist of strength training at 70-80% of the client's one-rep max in order to see improved strength with predictable results. b. The exercise program should focus more on cardio rather than strength training c. Elders ages 80 and above should not participate in any kind of strength training because they are frail and it is unsafe for their health. d. Improvements in strength does not correlate with improving ability to complete functional tasks.

a. The exercise program should consist of strength training at 70-80% of the client's one-rep max in order to see improved strength with predictable results.

An adult is hospitalized and diagnosed with mild chronic obstructive pulmonary disease (COPD). During the discharge planning session, the person identifies a desire to exercise regularly. Which of the following should the occupational therapist recommend the client pursue? a. The hospital wellness program's yoga group. b. Low-impact aerobics at a local gym. c. Weight-lifting under the direction of a personal trainer. d. Jogging in a local park with friends.

a. The hospital wellness program's yoga group. -mointored

An individual with COPD has identified a long-term goal of being able to shop independently for groceries. Which statement is the BEST short-term goal for this individual? a. The individual will purchase 10 items at the supermarket with supervision. b. The individual will cook a one-dish meal with items purchased at the supermarket. c. The individual will identify food items needed for developing a shopping list. d. Instruct the individual in energy conservation techniques that apply to grocery shopping.

a. The individual will purchase 10 items at the supermarket with supervision. - small list, in actual environment

An occupational therapist designs a qualitative research study to examine the efficacy of an after-school playbased program for the development of social interaction skills. Which method of data collection is best for the therapist to use? a. The therapist's observations of the participating children in the classroom and during recess. b. The completion of a social skills Likert-scale questionnaire by the participants' teachers. c. The completion of a social skills Likert-scale questionnaire by the participants' parents. d. The administration of a social skills evaluation pre-and postintervention.

a. The therapist's observations of the participating children in the classroom and during recess. -only qualitative answer

The occupational therapist works with a wide range of professional healthcare workers. Which professional would work to design a new e-stim device to be used therapeutically? a. biomedical engineer b. prosthetist c. chiropractor d. creative arts therapist

a. biomedical engineer

Several residents of a skilled nursing facility report that they are bored with their individual daily range of motion exercise programs. The occupational therapist collaborates with the physical therapist to design a group format to facilitate participation in range of motion exercises. Which would be most beneficial for the therapist to recommend incorporating into the proposed group? a. The use of several exercise videos with diverse exercise styles and music. b. The pairing of residents to perform gentle range of motion on each other. c. The use of exercises performed in rhythm to a marching band video d. The provision of coffee and cake after the group.

a. The use of several exercise videos with diverse exercise styles and music

Which motor evaluation tool assesses for potential motor delays and includes subtests identifying difficulties in mobility, stability, motor organization, social and emotional skills, and functional performance? a. Toddler and Infant Motor Evaluation (TIME) b. Hawaii Early Learning Profile, Revised (HELP) c. Bruininks-Oseretsky Test of Motor Proficiency, 2nd Edition (BOT-2) d. Denver Developmental Screening Test II

a. Toddler and Infant Motor Evaluation (TIME)

An OTR is working with a 12-year-old child diagnosed with emotional disturbance. The child has a history of being off-task, poor social skills, and becoming overly upset about things when something does not go as anticipated. The child attends middle school and the teacher reports that the child has difficulty following directions, completing schoolwork, and has frequent outburst resulting in visits to the principals office. What intervention or strategy is most effective for improving school behavior and performance? a. Token economy system to reinforce cooperation and task completion b. A therapy ball to sit on in the classroom so he could move while working c. Movement breaks throughout the day for self regulation d. Referral to social work or school counselor for further intervention

a. Token economy system to reinforce cooperation and task completion

What is an appropriate self-care intervention for a patient with tremors in order to promote improved independence with self-feeding? a. Use weighted utensils b. Educate the caregiver on how to feed the patient c. Puree the food d. Use a thickening agent for liquids

a. Use weighted utensils

A client attends a group intervention program to address substance-abuse. The client is aware that the number therapy sessions are limited. What is the most beneficial assessment for the OTR to conduct with this client based on the limited number of sessions? a. Utilize a tool that allows a client to self identify occupational challenges b. Complete a predictive assessment tool to determine the likelihood of relapsing c. Assessments to determine the level of independence and safety d. Compile an evaluative assessment to evaluate changes and symptoms over time

a. Utilize a tool that allows a client to self identify occupational challenges - focus on client

An OT observes that an individual who had been doing well on a pureed diet has demon strated a gurgle, or wet voice, after swallowing a second time. What is the MOST appropriate recommendation for the OT to make? a. Videofluoroscopy. b. Diet change to include thin liquids. c. Tracheostomy tube. d. Advancement to a regular diet.

a. Videofluoroscopy. -wet voice is sign of aspiration -should be reevaluated using imaging

An occupational therapist working in an outpatient cardiac rehabilitation center develops an intervention plan for an individual who has entered Phase 2 of cardiac recovery. Which activities should the therapist recommend be included in this intervention plan? a. Weeding a garden and doing low-impact aerobics. b. Putting away groceries and keyboarding. c. Washing dishes and playing tabletop board games. d. Carrying groceries upstairs and playing basketball.

a. Weeding a garden and doing low-impact aerobics. -B. and C. are phase 1, D. is 6.0-10 MET

An OT practitioner is assessing the range of motion of an individual who actively demonstrates internal rotation of the shoulder to 70 degrees. The practitioner would MOST likely document this measurement as: a. Within normal limits.- b/c degrees b. Within functional limits. - fxl ROM, no degrees c, Hypermobility that requires further treatment. d. Limited mobility that requires further treatment.

a. Within normal limits.- b/c degrees

Which of the following statements is FALSE regarding the preprosthetic treatment of a patient who has undergone an amputation? a. Wrapping should be completed proximal to distal b. Complete AROM as available in residual limb c. Reduce tension in wrappings toward the proximal limb d. Desensitize the skin on the residual limb

a. Wrapping should be completed proximal to distal

An occupational therapist is providing intervention to improve ergonomics for an individual working seated at a computer desk. Which of the following positions, regarding proper posture, is NOT conducive to an ergonomic workspace? a. Wrists are in 20-degree extension b. Feet are flat on the floor c. Legs below the knee are perpendicular to the floor d. Legs above the knee are parallel to the floor

a. Wrists are in 20-degree extension

A patient presents with an inability to read after sustaining a traumatic brain injury, although he used to read for pleasure regularly, before the accident. What term defines this condition? a. alexia b. agraphia c. acalculia d. preservation

a. alexia

An OT practitioner is evaluating an individual who has undergone a total hip replacement to determine compliance with hip precautions prior to discharge. When the individual is observed leaning forward and stopping at 90 degrees of hip flexion to use the long-handled shoe-horn, the therapist determines that the individual: a. demonstrates compliance with hip precautions. b. requires verbal cueing to observe hip precautions. c. requires a longer shoe horn. d. demonstrates cognitive deficits.

a. demonstrates compliance with hip precautions. -technically, they stopped at 90 - I hate this question its mean

Which is an appropriate task to assign to an occupational therapy assistant? a. direct a patient in therapeutic strengthening exercises for the hand in order to improve grip strength b. determine long-term goals for a set plan of care c. design a functional strengthening protocol for a patient recovering from a CVA d. administrate and interpret the pediatric evaluation of disability inventory

a. direct a patient in therapeutic strengthening exercises for the hand in order to improve grip strength

Validity is an important element of standardized assessments to ensure you are obtaining an accurate measurement of whichever skill or ability you wish to measure. Which type of validity refers to if the assessment has the appearance of obtaining the information it was designed to obtain? a. face validity b. criterion validity c. content validity d. predictive validity

a. face validity

An OTR is seeking a pressure-relieving seat cushion for a client who uses a wheelchair throughout the day. The primary objective is to select a lightweight, inexpensive cushion that provides even distribution of pressure relief to the buttocks when the client is seated in the wheelchair. What is the MOST EFFECTIVE type of cushion for this purpose? A.Foam B.Air-filled C.Gel

a. foam Foam material is lightweight, inexpensive and provides a stable base of support.

A OTR is planning treatment sessions for adolescence diagnosed with eating disorders in an outpatient, voluntarily community mental health complex. Which goal is most important to address initially? a. increasing self-awareness with expressive activities b. developing awareness of personal nutritional issues c. recommending a referral for family therapy

a. increasing self-awareness with expressive activities

An OT receives a referral to evaluate an individual's executive functioning following a mild CVA. The therapist will most likely assess the person's: a. initiation and planning b. orientation, attention, and memory c. job interests and efficacy d. spatial relations and praxis

a. initiation and planning -mild CVA

A patient presents with rhythmic, oscillating movements in her hands when attempting to brush her teeth. What does this describe? a. intention tremor b. dyskinesia c. Tic d. myoclonus

a. intention tremor

An occupational therapist supervisor implements a system in which fellow occupational therapists complete chart audits on coworkers to maintain documentation standards. What management system is being used? a. peer review b. risk management c. utilization review d. prospective review

a. peer review

A client with left homonymous hemianopsia following a cerebrovascular accident is experiencing difficulty with reading. Previously, the client enjoyed reading novels, newspapers, and magazines. Which is the most effective intervention to promote the client returning to reading? a. provide a vertical anchor on the left side of the page only b. provide a vertical anchor along the right side of the page only c. provide number lines along the right side of the page only d. provide vertical anchors on the right and left side of the page

a. provide a vertical anchor on the left side of the page only -i.e. bright sticky note to encourage reading on left side

A patient presents with the benediction sign. What is the likely injury or disorder? a. proximal median nerve injury b. ulnar nerve lesion c. radial nerve lesion d. distal median nerve injury

a. proximal median nerve injury

At four months of age, which form of reaching is observed? a. reaching with bilateral arms b. unilateral reaching with the shoulder flexed and externally rotated and the forearm supinated c. batting at toys with fingers flexed d. unilateral reaching with fingers extended and minimal shoulder abduction and internal rotation

a. reaching with bilateral arms

An individual is transferred from an acute care hospital to a sub-acute rehabilitation unit in a long-term care facility. The patient incurred a left cerebral vascular accident (CVA) in the middle cerebral artery (MCA) one week ago. The individual is referred to occupational therapy. The referral states that the patient has right hemiplegia and a subluxed right shoulder. an ot advises the direct care staff on proper positioning of the patient's right arm while the patient is seated in a wc. What is the most appropriate position for the ot to recommend for placement of the patients right arm? a. rest the arm in an inclined, padded arm trough attached to the right wc armrest throughout the day b. rest the arm on the wc lapboard throughout the day c. cross the patient's arms across the chest and have the patient cradle the affected extremity with the unaffected extremity wear a shoulder sling 24 hours a day to avoid weight on the shoulder

a. rest the arm in an inclined, padded arm trough attached to the right wc armrest throughout the day -no weight on shoulder and no traction

An occupational therapist uses early intervention programs to address infant present developmental concerns. Which type of intervention does this describe? a. tertiary prevention b. management intervention c. secondary prevention d. primary prevention

a. tertiary prevention -addressing a concern

Which disorder of the peripheral nervous system affects the corticospinal tracts and anterior horn cells and can cause difficulty swallowing and difficulty with speech production? a.Amyotrophic lateral sclerosis (ALS) b. Guillain-Barré syndrome c. Myasthenia gravis d. Post-polio syndrome

a.Amyotrophic lateral sclerosis (ALS)

A therapist is testing for the forward parachute response by suddenly tipping the infant forward from an initial vertical position. What is the intact infant response? a.Extension of the fingers, upper extremities, and neck b. Flexion of the fingers, upper extremities, and neck c. Extension of the upper extremities, finger flexion, and neck flexion d. Flexion of the upper extremities, finger flexion, and neck extension

a.Extension of the fingers, upper extremities, and neck

Intervention should follow what type of progression? a. A "top down" approach focused on skills and abilities b. A "top down" approach focused on occupations c. A "bottom up" approach focused on occupations d. A "bottom up" approach focused on skills and abilities

b. A "top down" approach focused on occupations

An OT administrator is designing a patient satisfaction questionnaire to be administered upon discharge from the OT program. The administrator designs the questionnaire so that individuals will indicate their level of agreement with a series of statements by circling a number with 1 = very dissatisfied, 2 = dissatisfied, 3 = neutral, 4 = satisfied, and 5 = very satisfied. Which method of data collection is the administrator using? a. Gutman scale rank ordering. b. A Likert scale. c. A semantic differential.

b. A Likert scale.

When assessing burn wound size, what would be the expected percentage for an individual who has suffered a burn to both of his arms? a. 72% b. 18% c. 9% d. 36%

b. 18%

a religious congregation obtained private funding to build a ramp so that members with disabilities can attend services. The entrance to the congregation building has 6 steps with a rise of 8 inches each. The most appropriate recommendation for the ot consultant to make is to for construction of a ramp that is: a. 48 feet long b. 48 feet long with a 4 x4 landing at the ramp's mid point c. 60 feet long d. 60 feet long with a 4×4 landing at the ramps mid point.

b. 48 feet long with a 4 x4 landing at the ramp's mid point

A child with a brachial plexus injury at your outpatient facility has begun to develop contractures in the left elbow. Using your clinical judgment based on current research trends, you decide to treat the patient with the following: a. serial casting to regain full ROM, then maintenance ROM stretching b. A combination of serial casting and nighttime splinting to initially regain full ROM and then maintain it in the future c. PROM once a day at therapy and school d. Nighttime splints to maintain the current ROM the patient has

b. A combination of serial casting and nighttime splinting to initially regain full ROM and then maintain it in the future

Occupational therapy services are provided to the residents of a psychogeriatric unit in a skilled nursing facility. An occupational therapist presents an inservice on restraint reduction to the unit's direct care staff. Which of the following would the therapist identify as a permissible use of a restraint? a. A bed guardrail to prevent a confused resident from wandering in the evening. b. A lap board to enhance a resident's engagement in self-determined activities. c. A wheelchair with a lap belt to prevent a resident with ataxic gait from falling.

b. A lap board to enhance a resident's engagement in self-determined activities. -increases engagement

An individual prepares for discharge home following rehabilitation for a left cerebrovascular accident (CVA). Residual difficulties include fair dynamic balance and decreased proximal upper extremity (UE) strength. The individual's stated priority is to be able to ambulate safely to the senior center located in the client's apartment building. Which ambulatory aid would be most effective for the occupational therapist to recommend to this client? a. A hemi-walker. b. A rolling walker. c. A side-stepper walker. d. A standard walker

b. A rolling walker. -good if you can't lift a standard walker

An occupational therapist conducts a sensory evaluation of an individual recovering from a left cerebral vascular accident. The individual has right hemiplegia and expressive aphasia. During the evaluation of stereognosis, which should the therapist have the client use to identify responses to the testing stimuli? a. Pictures of the objects. b. A set of identical objects. c. Cards with "one" and "two" printed on them. d. Cards with "yes" and "no" printed on them.

b. A set of identical objects. -pictures are higher level of cog functioning -3D easier

All of but which of the following can result in temporary or permanent demenia: a. meningitis b. ALS c. HIV d. Parkinson's disease e. CVA

b. ALS

Which major construct of Jean Piaget's process of cognitive development is described as the ability to make a change in response to new schemes or experiences? a. Operations b. Accommodation c. Equilibrium d. Assimilation

b. Accommodation

An individual with rheumatoid arthritis (RA) is currently in a stage of remission. During this inactive chronic phase of this disease, the occupational therapist works with the client to maintain range of motion (ROM) and muscle strength. Which of the following is most effective for the therapist to recommend the client include in a daily home exercise program? a. Passive ROM. b. Active ROM. c. Isotonic exercises. d. Progressive resistance.

b. Active ROM. -isotonic exercises are controversial and need to be monitored (not HEP)

Which of the following activities would MOST effectively evaluate group interaction skills during an OT session? a. The clients make individual collages, sharing a set of magazines to complete the activity. b. All group members construct one tower that incorporate all of the pieces provided in a set of constructional materials (e.g., Legos or Erector set). c. All group members work together to make pizza and salad for their lunch that day. d. Each client selects a short-term craft activity from four available samples.

b. All group members construct one tower that incorporate all of the pieces provided in a set of constructional materials (e.g., Legos or Erector set).

In which situation is it okay to use either soap and water or an alcohol-based sanitizer to clean the hands? a. If hands become noticeably dirty after cleaning a patient b. Before assisting a patient with cutting food c. After exiting the room of a patient diagnosed with C-diff d. After a patient accidentally sneezes on your hands

b. Before assisting a patient with cutting food

Which hand deformity would be expected in a patient with a high median nerve laceration? a. Claw hand b. Benediction sign c. Ape hand d. Wrist drop

b. Benediction sign

The occupational therapist plans intervention for a client with a recent diagnosis of complex regional pain syndrome (CRPS) Type I. Which intervention approach is most effective to use to reduce pain and increase function? a. Hot packs. b. Biofeedback. c. Paraffin. d. Passive range of motion.

b. Biofeedback.

An adult who incurred a traumatic brain injury three months ago is referred to a home care agency to receive OT services. The client's referral states that cognition is at Level VII of the Rancho Los Amigos scale and that grasp and shoulder mobility are limited. During the initial interview, the client reports being frustrated by the inability to independently engage in the previously enjoyed and personally meaningful activity of cooking. The client reports frequently losing place when reading recipes and an inability to "find" things in the kitchen.B46. During an intervention session focused on the development of grasp and shoulder mobility, the occupational therapist asks the client to move numerous identical one pound cans of vegetables from the counter top to the cabinet shelf above the counter. According to contemporary motor learning approaches, which type of practice has the therapist designed this activity to provide to the client? a. Random practice. b. Blocked practice. c. Contextual practice.

b. Blocked practice. -blocked=repetition of same motor skill -contextual isnt a thing

An individual's PIP joint appears flexed, and the DIP joint appears hyperextended. The OT can BEST document this condition as a: a. Mallet deformity. b. Boutonniere deformity. c. Subluxation deformity. d. Swan neck deformity.

b. Boutonniere deformity.

Which of the following movements allows the OTR to evaluate the strength and control of the client's trunk flexors? a. Client is instructed to set upright, maintain erect spine, and lean forward. b. Client leans backwards and holds the position, then forward to sit upright c. The client is instructed to sit and lean forward then lean back to sit upright

b. Client leans backwards and holds the position, then forward to sit upright

An OT practitioner is conducting a predischarge interview with a patient who has been treated on the psychiatric unit for schizophrenia. The question which the therapist asks, "Are you ready to go home today?" is an example of a(n): a. Open question. b. Closed question. c. Directed or leading question. d. Double question.

b. Closed question.

During an interview following a total hip replacement, it is important that the OT practitioner determine FIRST the individual's: a. Marital status. b. Cognitive status. c. Leisure interests. d. Work responsibilities.

b. Cognitive status.

Which type of wrist fracture is characterized by the fractured segment being pushed forward toward the dorsal side of the wrist? a. Smith's fracture b. Colles' fracture c. Boxer's fracture d. Scaphoid fracture

b. Colles' fracture

A person fell and sustained bilateral Colles' fractures. The client wore bilateral short-arm casts for six weeks. After cast removal, the client began OT sessions to increase endurance and strength prior to returning to work. The client tends to work hard when performing resistive exercises with both wrists. The therapist monitors the client for overexertion. Which behavior would indicate overexertion? a. Increased ability to achieve full ROM of the wrist. b. Complaints of pain in the wrist extensors. c. Consistent strength in wrist extension activities.

b. Complaints of pain in the wrist extensors.

An individual recovering from myasthenia gravis has fair minus (F-) muscle strength in both upper extremities. The occupational therapist develops an intervention plan to include the goal of increasing muscle strength. According to the biomechanical approach, which should the therapist work on with the patient during intervention? a. Complete active ROM with gravity decreased. b. Complete active ROM against gravity. c. Incomplete active ROM against gravity. d. Complete active ROM against gravity and slight resistance.

b. Complete active ROM against gravity. -next step is 3/5 (ROM against gravity)

An occupational therapist provides home care services to a neonate with significant developmental delays. Two hours before the next scheduled home visit, the child's parent informs the therapist that one of three older children has developed chicken pox. While the other children do not show signs of chicken pox, the parent expresses concern that they are contagious. Which is the therapist's best response to this situation? a. Cancel the scheduled session and reschedule after two weeks have passed. b. Complete the scheduled session using airborne precautions. c. Complete the scheduled session using standard precautions. d. Complete the scheduled session using droplet precautions

b. Complete the scheduled session using airborne precautions.

An individual is transferred from an acute care hospital to a subacute rehabilitation unit in a long-term care facility. The patient incurred a left cerebral vascular accident (CVA) in the middle cerebral artery (MCA) one week ago. The individual is referred to occupational therapy. The referral states that the patient has right hemiplegia and a subluxed right shoulder.B94. While completing the screening, the occupational therapist observes that the patient uses only their left side to participate in activities. The therapist suspects that the patient has unilateral neglect and difficulties with body scheme. Which should the therapist have the patient do during the OT evaluation to determine if these deficits are present? a. Point to various body parts named by the therapist. b. Complete upper and lower extremity dressing. c. Complete the draw-a-person test.

b. Complete upper and lower extremity dressing.

Which type of validity refers to how well scores and data match on two separate but similar assessments administered in close proximity? a. Face validity b. Concurrent validity c. Predictive validity d. Content validity

b. Concurrent validity

Regarding qualitative research, which criterion is defined as the study's accuracy of the findings in relation to the information received the study? a. Dependability b. Confirmability c. Credibility d. Transferability

b. Confirmability

Regarding qualitative research, which criterion is defined as the study's accuracy of the findings in relation to the information received through the study? a. Dependability b. Confirmability c. Credibility d. Transferability

b. Confirmability

Which type of sampling is being used when the researcher obtains participants who are readily accessible? a. Stratified sampling b. Convenience sampling c. Random sampling d. Systematic sampling

b. Convenience sampling

Which of the following groups of cranial nerves are sensory nerves associated with the sense of smell, vision, hearing, and the vestibular system? a. Cranial nerves III, IV, VI b. Cranial nerves I, II, VIII c. Cranial nerves V, VII, IX, X d. Cranial nerves XI, XII

b. Cranial nerves I, II, VIII

A child with developmental delay has mastered the ability to cut simple figure shapes with scissors. Which scissor activity is best for the occupational therapist to introduce to the child during their next intervention session? a. Cutting simple geometric figures. b. Cutting complex figure shapes. c. Cutting multiple circles.

b. Cutting complex figure shapes.

Proprioceptive neuromuscular facilitation (PNF) is used therapeutically for many individuals with neurological conditions; it focuses on using movement patterns to improve motor skills for functional tasks. Which upper extremity pattern is used when the therapist is trying to promote improved function in reaching across the body, such as when pulling on pants or reaching for a pocket? a. D1 extension b. D2 extension c. D2 flexion d. D1 flexion

b. D2 extension

Best practice in documentation includes wording to justify skilled intervention and the need for services. Which of the following terms is the BEST choice to justify continued skilled occupational therapy? a. Requires repetition b. Demonstrated recall c. Chronic impairment d. Minimal change

b. Demonstrated recall -potential for improvement

The director of OT services for a large metropolitan home health agency is conducting a supervisory visit and notices the OTR is writing progress notes on a persona electronic tablet. OT is excited about using tablet and shares with the director that it saves a great deal of time. Most appropriate response from director? a. Applaud the OT for using technology efficiently b. Develop a policy prohibiting the use of personal electronic tablets for note writing c. Ask the OT to describe the use of electronic recording to the other OT at the agency d. Ask home health clients for their opinions regarding electronic record keeping

b. Develop a policy prohibiting the use of personal electronic tablets for note writing

Which vision condition is associated with diabetes mellitus and can potentially cause further damage to the retina, such as scarring or detachment? a. Macular degeneration b. Diabetic retinopathy c. Glaucoma d. Cataracts

b. Diabetic retinopathy

A school-based occupational therapist receives an evaluation referral for a third-grader. The teacher reports that the student has illegible handwriting, poor attending behaviors, questionable visual skills, and problems with pencil management. After speaking with the teacher, reviewing classroom work samples, and reading the student's history, which action should the therapist take next? a. Provide pencil grips and specialized paper as a trial to determine interventions. b. Directly observe the student during a naturally occurring writing time. c. Administer a standardized visual perceptual and visual motor assessment. d. Administer a standardized handwriting assessment.

b. Directly observe the student during a naturally occurring writing time.

An occupational therapist is scheduled to give a one-hour presentation to a support group of parents of infants with a diversity of developmental disabilities. Which of the following is the most important focus of the therapist's presentation? a. Demonstration of infant positioning techniques. b. Discussion of typical areas of concern addressed by OT practitioners. c. Demonstration of different types of developmental assessments. d. Discussion of the Individual Family Service Plan (IFSP)

b. Discussion of typical areas of concern addressed by OT practitioners. -promotion of OT services

A method that an OT practitioner can use to document total finger flexion without recording the measurement in degrees would be to measure the: a. Passive flexion at each joint and total the numbers. b. Distance from the fingertip to the distal palmar crease with the hand in a fist. c. Active flexion at each joint and total the measurements. d. Distance between the tip of the thumb and the tip of the fourth finger. (opposition)

b. Distance from the fingertip to the distal palmar crease with the hand in a fist.

An adult recently diagnosed with scleroderma receives occupational therapy services to deal with the functional changes caused by this disease. Which recommendation is best for the occupational therapist to make to this individual? a. Dress in lightweight clothing for thermal comfort. b. Dress in layers for neutral warmth. c. Use pull-on clothing to ease donning and doffing. d. Use Velcro or a button hook to ease fastening.

b. Dress in layers for neutral warmth. -poor circulation due to hardening of the skin

Which hand disorder can result in flexion contractures in the digits as the fascia within the hand thickens and becomes less elastic? a. Colles' fracture b. Dupuytren's disease c. Skier's Thumb/Gamekeeper's Thumb d. De Quervain's tenosynovitis

b. Dupuytren's disease

A patient presents with normal sensory and motor function. What type/grade of spinal cord injury is described, based on the ASIA Impairment Scale? a. C b. E c. D d. A

b. E

An occupational therapist is working with a preschool child and notices his aggression toward peers and the adult language he uses in anger when another child wants to play with the same toy. He suspects this child may be enduring which form of child abuse? a. Neglect b. Emotional abuse c. Sexual abuse d. Physical abuse

b. Emotional abuse

An older adult with a diagnosis of osteoarthritis in both knees is referred to inpatient occupational therapy. During screening, the patient expresses a desire to return home to live alone independently. Which should the occupational therapist do first in response to the patient's stated goal? a. Recommend adaptations to the patient's home environment to increase safety. b. Evaluate the patient's BADL and IADL using a standardized measure. c. Teach the patient energy conservation techniques to use during IADL tasks. d. Train the patient in a home resistive exercise program to build strength and ROM.

b. Evaluate the patient's BADL and IADL using a standardized measure.

An occupational therapy administrator implements a quality improvement program at a large private hand therapy clinic. The administrator determines that the clinic's certified occupational therapy assistants (COTA®s) are not completing their assigned initial screenings in a timely manner. This has resulted in scheduling delays for complete functional evaluations. Which initial action is most effective for the administrator to take in response to this situation? a. Counsel the COTA®s on the need to adhere to screening schedules. b. Examine the organizational structure of the screening process. c. Assign the occupational therapists to complete all screenings. d. Redesign the screening to simplify the process

b. Examine the organizational structure of the screening process.

The residents of a halfway house plan a community leisure activity for a Saturday. Two residents state that they cannot participate in Saturday activities due to religious observances. The other residents express strong interest in the activity. Which is the occupational therapist's best response to this situation? a. Schedule an in-house Saturday leisure activity for the two observant residents. b. Explore with the group an alternative schedule for a community leisure activity. c. Schedule an in-house Saturday leisure activity for all residents. d. Recommend the two observant members seek approval from their religious leadership to attend the Saturday activity.

b. Explore with the group an alternative schedule for a community leisure activity.

Regarding shoulder adhesive capsulitis, which shoulder motion is limited the MOST? a. Flexion b. External rotation c. Internal rotation d. Abduction

b. External rotation

A college is converting an historical building into wheelchair-accessible dormitory space. To allow for a 360° turning radius, which dimensions are best for the occupational therapist to recommend as the minimum space between the students' desk and bed? a. Four feet by four feet. b. Five feet by five feet. d. Six feet by six feet.

b. Five feet by five feet.

While observing an individual who has just been admitted to the rehabilitation unit after a right CVA with left hemiplegia, the OT practitioner notices that the individual's left arm lies limply by his side. This MOST likely indicates: a. Normal upper extremity function. b. Flaccidity. c. Subluxation. d. Spasticity.

b. Flaccidity.

Several adolescents with behavior problems attend a school-based after-school program. They work at an egocentric-cooperative level in a group dealing with issues related to school performance and peer pressure. Which of the following would be most likely for the occupational therapist to observe the participants doing in the group? a. Actively taking on roles such as energizer, coordinator, or opinion giver. b. Focusing on the group tasks rather than the feelings of the participants. c. Making decisions with minimal to no supervision from the group leader.

b. Focusing on the group tasks rather than the feelings of the participants. -A. is mature group

An OT is treating an 86-year-old male who has difficulty visually scanning objects outside of his central vision. After providing a visual screen, the OT notices that the client has tunnel vision and has difficulty seeing objects in the outer borders of his visual field in both eyes. This leads the OT to believe that the client has loss of peripheral vision. What is the most likely cause of this vision loss?Select one: a. Macular degeneration b. Glaucoma c. Cataracts d. Diabetic retinopathy

b. Glaucoma

An adult diagnosed with bipolar disorder has been taking lithium for five years. Prior to a weekly occupational therapy vocational planning group, the client reports noticeable functional changes since the last group session. In describing these changes, the client reports symptoms that may be indicative of a possible lithium overdose. With the client's permission, the therapist contacts the psychiatrist to describe the client's concerns. Which symptom would the therapist most likely report as indicative of this problem? a. Reduction in mood swing. b. Gross hand tremors. c. Decreased velocity of speech. d. Fine hand tremors.

b. Gross hand tremors. -sign of lithium overdose

An OT practitioner is selecting treatment activities to use with a young adult diagnosed with schizophrenia that would help to increase the ability to receive, process, and respond to sensory information. What are the MOST suitable types of activities to address this area? a. Social skills training. b. Gross motor exercises. c. Life skills. d. Expressive projects.

b. Gross motor exercises.

A cooking group meets for 1 1/2 hours each week at a partial hospitalization program. During the group, members do not smoke, they wait for everyone to be served before eating, and they clean up after the meal. When reporting these observations, which of the following is the most accurate statement for the therapist to make? a. The group protocol is clear. b. Group norms are being followed. c. Group sanctions are effective.

b. Group norms are being followed.

When working on cooking skills, an individ ual with a history of traumatic brain injury ex hibits moderate upper extremity incoordina tion. Which of the following recommendations would be MOST beneficial for this individual? a. Built-up utensil handles. b. Heavy utensils, pots, and pans. c. Use of a high stool to work at counter height. d. Placement of commonly used items on shelves just above and below the counter.

b. Heavy utensils, pots, and pans -more proprioception

A treatment plan for a child with a visual discrimination problem would MOST likely include which adaptation of visual materials? a. Low contrast and defined borders. b. High contrast and defined borders. c. High contrast and shaded borders. d. Low contrast and shaded borders.

b. High contrast and defined borders.

An OT practitioner is performing an environmental assessment to determine accessibility for an individual who will be returning home. The FIRST step in this process is to: a. Identify the barriers to movement and function in the home environment. b. Identify and analyze the tasks and occupations that the individual will be performing in the home. c. Identify the aspects of the environment that support movement and function in the home. d. Determine the social environment of the individual.

b. Identify and analyze the tasks and occupations that the individual will be performing in the home.

An older client with a diagnosis of major neurocognitive disorder lives with family members. On the last three occasions, the client attended OT sessions with bruises and cuts on both legs. When asked about this, the client replies that a family member caused these injuries. Which is the first action the occupational therapist should take? a. Confirm the client's statements with the family. b. Immediately report potential abuse according to the facility policy. c. Call the police to report elder abuse. d. Administer a cognitive evaluation before taking further action.

b. Immediately report potential abuse according to the facility policy.

A toddler with severe congenital anomalies and an irreparable cleft palate has a do not resuscitate (DNR) order. While being fitted for a molded seat for a wheelchair, the child stops breathing and turns blue. The entry-level occupational therapist determines that the child has a brachial pulse. Which of the following is the first action the therapist should take in response to this situation? a. Inform the physician about the situation and the child's DNR order. b. Implement the facility's emergency procedure. c. Perform obstructed airway maneuver and monitor heart rate for five minutes.

b. Implement the facility's emergency procedure. -DNR means there is no pulse

Which gross motor skill would NOT be expected in an eight-month-old typically developing infant? a. Cruising sideways with handheld support b. Independent stepping c. Unsupported sitting d. Rolling from prone to supine

b. Independent stepping

Which of the following is NOT true of the Individuals with Disabilities Education Act? a. It developed IEP guidelines b. It required occupational therapists to serve as child abuse reporters c. It required schools to provide testing accommodations as necessary for children with disabilities d. It included services for children under two years old

b. It required occupational therapists to serve as child abuse reporters - this was in the child abuse prevention and treatment act

An OT is working with a client with complex regional pain syndrome. Which of the following is NOT an OT intervention for complex regional pain syndrome?Select one: a. ADL to encourage pain-free activities b. Joint mobilization c. AROM to involved joints d. Splinting to prevent contractures and enable ability to engage in occupation-based activities

b. Joint mobilization

An individual who had a stroke is copying a picture of a clock. The drawing appears as a lopsided circle with a flat side on the left. The numbers one through eight are written in numerical order around the right side of the clock. The hands are correctly drawn on the clock to represent three o'clock. The individual's performance seems to demonstrate: a. Right hemianopsia.: visual field cut (right side of both eyes) b. Left unilateral neglect. (visual attention) c. Cataracts in the left eye. d. Bitemporal hemianopia. (inside of both eyes)

b. Left unilateral neglect. (visual attention)

An individual recovering from flexor tendon repair surgery is two days post-operation. The surgeon refers the client to occupational therapy with a prescription to use the Kleinert protocol to guide intervention.B152. The client is now seven weeks postoperation. Which are the most appropriate intervention activities for the occupational therapist to use with this client? a. Home management activities such as doing laundry. b. Light ADL such as grooming. c. Strengthening exercises using high-resistance TheraBand. d. Passive exercises using a dynamic splint.

b. Light ADL such as grooming.

The BEST method for the OT practitioner to evaluate the presence of unilateral neglect is by using which of the following evaluations? a. Six-block assembly. b. Line bisection. c. Proverb interpretation. d. Identification of the square in four overlapping figures.

b. Line bisection.

One interventional role of an occupational therapist is to provide consultation for universal design setup. What principle of universal design involves the end product's ability to decrease the potential for muscular or postural strain with use? a. Perceptible information b. Low physical effort c. Size and space for approach and use d. Tolerance for error

b. Low physical effort

Which major construct of Jean Piaget's process of cognitive development is described when a child develops beliefs based on home or life experiences? a. Accommodation b. Mental schemes c. Equilibrium d. Adaptation

b. Mental schemes

Which Occupational Therapy Code of Ethics principle refers to a clinician not providing services that they know are a detriment to a patient's physical outcomes? a. Justice b. Nonmaleficence c. Autonomy d. Fidelity

b. Nonmaleficence

A non-English-speaking family attends a discharge planning session. The assigned occupational therapist does not share the language of the family. Which action should the therapist take first? a. Make a referral for a home care therapist to visit the family to provide in-home education. b. Obtain a translator to communicate with the family during the session. c. Attempt to communicate with the family through nonverbal communication. d. Consult with the case manager to develop a discharge plan.

b. Obtain a translator to communicate with the family during the session.

Regarding occupational therapy documentation, which statement is TRUE? a. Occupational therapy assistants (OTAs) are not qualified to write notes in medical charts b. Occupational therapy notes and records may be subpoenaed if a patient takes legal action c. As long as the facility allows abbreviations, any form can be used to reduce time constraints for documentation d. Informed consent can be given by minors as long as they are of sound mind

b. Occupational therapy notes and records may be subpoenaed if a patient takes legal action

Which legislation concerns residents of skilled nursing facilities? a. Age Discrimination in Employment Act b. Omnibus Budget Reconciliation Act of 1990 c. No Child Left Behind Act d. Freedom to Work Act

b. Omnibus Budget Reconciliation Act of 1990

The key symptoms to assess when evaluating an individual with suspected complex regional pain syndrome (reflex sympathetic dystrophy) are: a. Night pain and tingling of thumb, index, and middle fingers. b. Pain, edema, skin temperature, and skin color. c. Ability to pinch a piece of paper between thumb and index finger. d. Decreased range of motion and strength.

b. Pain, edema, skin temperature, and skin color.

An occupational therapist employed in a pediatric clinic participates in an initial performance appraisal. The supervisor identifies an area needing improvement as handling skills in working with children with various types of cerebral palsy. Which is the most effective way for the occupational therapist to improve handling skills? a. Observe an experienced occupational or physical therapist use handling techniques. b. Participate in an advanced-level experiential course on handling techniques. c. Participate in a teleconference on handling techniques for children with cerebral palsy.

b. Participate in an advanced-level experiential course on handling techniques. -visual and hands on learning

An OT who is working in a pediatric clinic is completing an evaluation on a new 5-year-old client. The OT needs an assessment that will detect the client's functional deficits, determine developmental level and assess capabilities. Which overall development assessment is most appropriate for the OT to use? Select one: a. Hayley Scales of Infant Development (BSID-III) b. Pediatric Evaluation of Disabilities Inventory (PEDI) c. Miller Assessment for Preschoolers (MAP) d. Denver Developmental Screening Test II

b. Pediatric Evaluation of Disabilities Inventory (PEDI)

When providing intervention for sexual expression and sexual activity, occupational therapists use the PLISSIT model as a guide for appropriate interventions. What does the P stand for? a. Protection b. Permission c. Partner d. Privacy

b. Permission Permission, limited information, specific suggestions, intensive therapy

Which type of qualitative methodology in research studies an individual's experiences and responses to situations without further analysis by the researcher? a. True experimental research b. Phenomenological research c. Ethnographic research d. Heuristic research

b. Phenomenological research

For best practice in documentation, it is recommended to include specific words to justify skilled intervention and the need for services. Which of the following words/terms is the MOST appropriate to include in documentation in order to reflect the potential for improvement? a. Decreased participation b. Progressing c. Required repetitive instruction d. Maintenance

b. Progressing

The residents of an urban homeless shelter include individuals with histories of chronic alcohol abuse who are at risk for developing peripheral neuropathy. The occupational therapist consulting at this shelter monitors the residents' status to ensure early detection of this problem. Which is the most important observed status change for the therapist to report? a. Progressive deterioration in visual acuity. b. Progressive deterioration of sensorimotor functions of the lower extremities. c. Rapid onset of intention tremors. d. Rapid loss of sensorimotor functions of the facial and neck muscles.

b. Progressive deterioration of sensorimotor functions of the lower extremities.

An OT working in acute care is assigned a new evaluation for a patient with a SCI. While reviewing the medical records prior to seeing the client for the first time, the OT reads that the patient has suffered a posterior cord injury. According to that information, the OT can suspect that the patient will present with the following symptoms:Select one: a. Motor function, pain, and temperature sensation loss bilaterally below the lesion b. Proprioceptive loss; pain, temperature, and touch are preserved c. Lower extremity motor and sensory loss and areflexic bowel and bladder d. More upper extremity deficits versus lower extremity deficits

b. Proprioceptive loss; pain, temperature, and touch are preserved

An individual is transferred from an acute care hospital to a subacute rehabilitation unit in a long-term care facility. The patient incurred a left cerebral vascular accident (CVA) in the middle cerebral artery (MCA) one week ago. The individual is referred to occupational therapy. The referral states that the patient has right hemiplegia and a subluxed right shoulder.B92. The occupational therapist meets with the nursing staff that will be providing primary care to the patient at the patient's bedside. The occupational therapist recommends that the direct care staff position the patient in left sidelying. Which is the best bed position for the therapist to recommend for placement of the patient's right arm? a. In 90° of humeral abduction and internally rotated. b. Protracted with arm forward on a pillow and the elbow extended or slightly flexed. c. On the person's side, adducted and internally rotated. d. In 90° of abduction of the humerus with neutral rotation.

b. Protracted with arm forward on a pillow and the elbow extended or slightly flexed.

An occupational therapist works with a child with pervasive developmental disabilities in order to develop self-care skills. In teaching the child to brush teeth, the therapist places the toothbrush in the child's hand and guides it to the mouth. To help the child learn to complete the activity the therapist uses the somatosensory system. Which of the following is most effective for the therapist to use next during intervention with this child? a. Tell the child to brush up and down. b. Provide hand-over-hand assistance to brush the child's teeth. c. Touch the child's hand to prompt hand-to-mouth movements. d. Instruct the child to follow a pictorial sequence card depicting toothbrushing.

b. Provide hand-over-hand assistance to brush the child's teeth.

A patient presents with an inability to extend the wrist and fingers. What is the likely condition? a. Carpal tunnel syndrome b. Radial nerve palsy c. Pronator teres syndrome d. Cubital tunnel syndrome

b. Radial nerve palsy

The family of a two-year-old in a spica cast asks the occupational therapist to modify the child's car seat. The child cannot fit in the car seat due to the cast. Which action is best for the therapist to take in response to this request? a. Pad the area between the car seat and the child's back with a pillow to accommodate for the lack of hip flexion. b. Recommend the family purchase a car seat designed for a child with a spica cast. c. Cut down the sides of the car seat to allow the cast to hang out of the sides of the car seat.

b. Recommend the family purchase a car seat designed for a child with a spica cast. -safety

The parents of a five-year-old with attention deficit with hyperactivity disorder (ADHD) express difficulty managing the child's aggressive behavior toward older siblings. Which is the most effective strategy for the occupational therapist to recommend to the parents? a. Allow the child to vent aggressive feelings on a stuffed animal or doll. b. Redirect the child's energy into acceptable and safe play activities. c. Provide consistent punishment for aggressive behavior. d. Send the child to stay with a family member or close friend for an extended "time-out."

b. Redirect the child's energy into acceptable and safe play activities

In an acute inpatient psychiatric facility, an occupational therapist designs a therapeutic activity group for individuals with poor orientation to reality. Which is the best activity choice for the therapist to provide in this group? a. A discussion of the effects of hospitalization on occupational roles. b. The assembly of wooden toys for a children's unit. c. Guided imagery for stress management. d. Structured verbalizations of personal assets and limitations.

b. The assembly of wooden toys for a children's unit.

An adult who incurred a traumatic brain injury three months ago is referred to a home care agency to receive OT services. The client's referral states that cognition is at Level VII of the Rancho Los Amigos scale and that grasp and shoulder mobility are limited. During the initial interview, the client reports being frustrated by the inability to independently engage in the previously enjoyed and personally meaningful activity of cooking. The client reports frequently losing place when reading recipes and an inability to "find" things in the kitchen.B43. Upon evaluation, the client exhibits difficulty with the letter cancellation task. Which visual deficit should the occupational therapist document as present? a. Imagery b. Scanning. c. Cognition. d. Memory.

b. Scanning.

Which type of intervention is described when a facility establishes a regular rotating position schedule for an individual with preliminary skin redness on his lower buttock? a. Primary prevention b. Secondary prevention c. Maintenance d. Tertiary prevention

b. Secondary prevention

When observing attention, multiple components must be evaluated. An individual is unable to study for an exam in the presence of background noises at a busy library. Which skill is impaired? a. Divided attention b. Selective attention c. Attentional switching d. Sustained attention

b. Selective attention

A single parent of two school-aged children is employed as a truck driver working the 11 p.m.-7 a.m. shift. The client was hospitalized for a major depressive episode following the sudden death of the client's spouse one year ago. The deceased spouse's primary role had been home maintainer. The client was rehospitalized this past weekend for another major depressive episode. Which skills are most relevant for the occupational therapist to evaluate? a. Interpersonal. b. Self-management. c. Cognitive. d. Leisure.

b. Self-management.

A person with a traumatic brain injury (TBI) is assessed to score a 6 on the Glasgow Coma Scale. Which should the occupational therapist use to initiate intervention with this person? a. Demonstrated directions. b. Sensory stimulation. c. Verbal cues. d. Hand-over-hand assistance.

b. Sensory stimulation.

Upon initial evaluation of an individual who sustained lacerations of the flexor tendons of the right index, middle, and ring fingers, the OT practitioner should FIRST be concerned with the individual's: a. Attitude toward the use of adaptive equipment. b. Skin integrity, pain, ROM, and sensory loss. c. Ability to engage in fine motor coordination tasks with his right hand. d. Ability to perform resistive tasks with his right hand.

b. Skin integrity, pain, ROM, and sensory loss.

An individual diagnosed with substance abuse exhibits difficulty gluing two pieces of a birdhouse together, becomes increasingly agitated, and finally storms out of the room to get a cigarette. This behavior would provide most information about the person's: a. Eye-hand coordination. b. Stress management. c. Visual perception. d. Fine motor skills.

b. Stress management.

A four-month-old with arthrogryposis remains in position when placed and shows little spontaneous movement. The occupational therapist implements intervention to work on rolling. Which positional changes should the therapist include in the intervention session? a. Prone to supine. b. Supine to side-lying. c. Prone to side-lying. d. Supine to prone.

b. Supine to side-lying.

Which muscle of the rotator cuff functions for shoulder abduction and flexion and is innervated by the suprascapular nerve? a. Teres minor b. Supraspinatus c. Subscapularis d. Infraspinatus

b. Supraspinatus

An occupational therapist plans individual and group activities for a child with oppositional defiant disorder. Which is most important for the therapist to address during group activities? a. The child's willingness to take on a variety of group roles. b. The child's ability to attend to and complete a task. c. The child's self-regulation of energy and activity levels.

b. The child's ability to attend to and complete a task.

An individual recently discharged from an acute psychiatric unit interviews for a position in a transitional employment program (TEP). The person answers the interviewing therapist's questions in a direct yet subdued manner and rarely looks at the therapist. Which is most accurate for the therapist to document in the summary of the interview? a. The individual should have medications evaluated before starting the TEP. b. The individual demonstrated limited eye contact. c. The individual exhibited poor social interaction skills. d. The individual appeared depressed.

b. The individual demonstrated limited eye contact.

An occupational therapist goes into a treatment room in a hospital setting to evaluate a patient who has recently suffered a heart attack. She finds that the patient does not speak English, so she is unable to effectively communicate verbally with the patient. What is the appropriate action? a. The occupational therapist should use nonverbal communication for the evaluation b. The occupational therapist should seek out a translator to assist with the evaluation c. The occupational therapist should perform a gross screening instead, based on functional movement d. The occupational therapist should ask the family to translate

b. The occupational therapist should seek out a translator to assist with the evaluation

An occupational therapist is working with veterans who have posttraumatic stress syndrome. The therapist may work with these individuals on an individual basis or in a group setting format. Which of the following would indicate that the intervention should be provided on an individual basis rather than in a group setting? a. The veteran wants to meet others with the same experiences b. The veteran has many specific goals to address during therapy c. The veteran wants to discuss symptom management with peers d. The veteran wants to meet other veterans who have worked through their symptoms

b. The veteran has many specific goals to address during therapy

An occupational therapy professional education program provides an after-school play program for typically developing children to help students understand typical development. The students observe a child who is beginning to use blunt scissors to snip paper. The child opens and closes the scissors and moves them in a controlled forward motion, but the child cannot cut circles or figure shapes. At which age are these behaviors typical? a. Two years old. b. Three years old. c. Four years old. d. Five years old.

b. Three years old. -2 is not controlled

A patient suffered a brain injury and is no longer able to find his way walking home from the library, even though he used to do this every day for five years. Which of the following is this an example of? a. Asomatognosia b. Topographical disorientation c. Astereognosis d. Ideational apraxia

b. Topographical disorientation

An occupational therapist creates handouts to promote his practice. On the handouts, he shows statistics claiming 50% improvement in hand function and 75% improvement in functional independence in his patients, even though there is no evidence to support these claims. What type of marketing is he using? a. Differentiated marketing b. Unethical marketing c. Undifferentiated marketing d. Concentrated marketing

b. Unethical marketing

Which reaching skill is displayed in a typically developing six-month-old infant? a. Reaching with bilateral arms b. Unilateral reaching with an open hand and minimal abduction and internal rotation c. Batting at a toy with fingers flexed d. Unilateral reaching with shoulder flexion, external rotation, supination, and wrist extension

b. Unilateral reaching with an open hand and minimal abduction and internal rotation

Which professional would be LEAST likely to be part of an occupational therapist's interdisciplinary team in a school-based setting? a. Social worker b. Special education teacher c. Audiologist d. Speech-language pathologist

c. Audiologist

Many individuals who participate in outpatient cardiac rehabilitation programs take beta blockers. The best method for evaluating tolerance for exercise with these individuals is to: a. Perform isometric testing. b. Using a perceived exertion scale. c. Monitor heart rate and blood pressure. d. Calculate maximum age-adjusted heart rate.

b. Using a perceived exertion scale. - HR/BP could be inaccurate; person can be overexercised but still low HR -client centered

Following an acute hospitalization for the medical management of a CVA, an individual receives homebased occupational therapy services. The occupational therapist is working on dressing skills with the patient. During one session, the therapist has the individual dress in the bedroom and during the next session the therapist has the client dress in the bathroom. During the following session, the therapist has the client don and doff a sweater and coat in the living room. Which motor learning technique is the therapist using? a. Variable activities. b. Variable conditions. c. Repetition. d. Generalization.

b. Variable conditions. -generalization is a desired outcome; not a technique

Ten members of a community reintegration group are not working well together and show decreased levels of trust. The occupational therapist's goal is to enhance the level of cohesiveness in the group. To begin the next group session, which is the best action for the therapist to take? a. Read inspirational phrases to increase motivation. b. Verbally review the goals and purposes of the group. c. Have each person contribute a line about childhood memories to group poem. d. Ask each person to talk about silly mistakes to provide some levity.

b. Verbally review the goals and purposes of the group.

An OT working for an assisted living corpo ration needs to design programs to engage resi dents with dementia who wander and pace the halls throughout the day. What is the BEST type of movement-oriented programming to employ to engage these residents? a. Reminiscing about previous jobs. b. Walking as part of a walking club. c. Singing oldies in a group. d. Participating in a craft activity requiring con centration.

b. Walking as part of a walking club.

A patient has endured a burn to the dorsal aspect of his wrist. What contracture is he MOST at risk for developing? a. Wrist flexion b. Wrist extension c. Claw hand deformity d. Palmar contracture

b. Wrist extension

a student with charcot-marie tooth disease participates in a re-evaluation session. He reports that he has difficulty keeping his feet on the wc footrests. The most appropriate recommendation for the ot to make is a. an exercise routine to strengthen lower extremities b. ankle straps on foot rests. c. heel loops on the foot rests d. elevating foot rests.

b. ankle straps on foot rests.

A patient suffered a spinal cord injury due to hyperflexion during an accident. She has no movement below the injury site and cannot determine the difference between hot and cold. She can, however, feel light touch. What is the likely spinal cord syndrome? a. central cord syndrome b. anterior cord syndrome c. cauda equina syndrome d. brown-sequard syndrome

b. anterior cord syndrome

A patient s/p spinal cord injury presents with loss of movement and decreased bowel and bladder control. Which clinical syndrome is described? a. central cord syndrome b. cauda equina syndrome c. brown-sequard syndrome d. anterior cord syndrome

b. cauda equina syndrome

Which type of validity refers to how well scores and data match on two separate but similar assessments administered in close proximity? a. predictive validity b. concurrent validity c. face validity d. content validity

b. concurrent validity

When treating individuals in the acute phase of cardiac rehabilitation post-myocardial infarction (MI), it is important for the OT practitioner FIRST to select activities that: a. prompt dyspnea. b. decrease the effects of prolonged inactivity. c. promote strength and ROM. d. assist with independence in relearning daily activities

b. decrease the effects of prolonged inactivity.

When evaluating a patient who has suffered a burn to his arms, the occupational therapist notes that the area looks white and waxy. The patient displays no sensation on the affected area. What is the likely classification of burn? a. superficial partial-thickness b. full-thickness c. superficial d. deep partial-thickness

b. full-thickness

Client presents for occupational therapy intervention with age related macular degeneration. Which strategy is most beneficial to promote successful engagement and the intervention during therapy sessions? a. decrease lighting b. increase contrast c. provide magnifiers d. recommend glasses

b. increase contrast

An occupational therapist is working on scissor skills in an early preschool classroom setting. The 2.5-year-old child should demonstrate which skill with scissors? a. moves the scissors forward when cutting paper b. makes snips on the edge of the paper c. cuts a circle d. cuts along the line

b. makes snips on the edge of the paper

an individual is being evaluated for a wheelchair to use for functional mobility. She expresses concern that she will not be able to continue her volunteer work at her local church. The church's doorways are 31 inches wide, and the client knows, from remodeling her home, that 32 inches is the minimum width recommended for wheelchair access. The most appropriate recommendation for the therapist to make is to: a. have the church widen its doorways to comply with ada requirements b. order a wheelchair with wrap around armrests c. order a customized narrow adult wheelchair d. have the client explore alternative volunteer activities in accessible locations.

b. order a wheelchair with wrap around armrests

A patient presents with skin that is red and blistered following a burn. He also complains of increased pain. What classification of burn is described? a. Full Thickness b. superficial partial thickness burn c. superficial d. deep partial thickness burn

b. superficial partial thickness burn

An OTR, who works in an acute care setting, measures the resting heart rate of an inpatient and notes that it is 130 bpm. What is the BEST option to describe this heart rate measurement? A.Atrial fibrillation B.Tachycardia C.Sinus rhythm

b. tachycardia

An OT practitioner documents that an individual exhibits elbow flexion strength of grade 1. according to the manual muscle test system of letters and numbers, the word that would be the equivalent of grade 1 would be: a. absent b. trace c. good d. normal

b. trace

A patient presents with claw hand deformity. What is the likely injury or disorder? a. proximal median nerve injury b. ulnar nerve lesion c. radial nerve lesion d. distal median nerve injury

b. ulnar nerve lesion

At nine months of age, which form of reaching is observed? a. reaching with bilateral arms b. unilateral reaching with the shoulder flexed and externally rotated and the forearm supinated c. batting at toys with fingers flexed d. unilateral reaching with fingers extended and minimal shoulder abduction and internal rotation

b. unilateral reaching with the shoulder flexed and externally rotated and the forearm supinated

A client's chart indicates that they have homonymous hemianopsia. Which visual skill is impaired? a. scanning b. visual fields c. visual acuity d. oculomotor function

b. visual fields

Following medical treatment for a brain tumor, a client is referred to OT home care services for a functional evaluation. During the initial interview, the person reports that he cannot locate items that her wants and needs. For example, at lunchtime when she went to the pantry to find a can of soup he could not locate it. Based on this self-report, the OT determines the need for further evaluation to assess: a. visual acuity b. visual scanning c. spatial relations d. topographical orientation

b. visual scanning -spatial relations is relationship of body to objects

An occupational therapist is providing intervention to properly adapt workstations to maintain ergonomic standards. What is the recommended table height for doing extensive writing when seated? a. 31-37 inches b.28-31 inches c. The height is not important d. 21-28 inches

b.28-31 inches

In regard to payment for occupational therapy, which term describes the patient receiving therapy? a. Capitation b. Diagnostic related groups c. Beneficiary d. Provider

c. Beneficiary

A Level II fieldwork student's first assigned case is an individual with right hemiplegia. The supervising therapist reminds the student that primitive reflexes can emerge when someone incurs a CVA. The therapist demonstrates this point by rotating the client's head to the right and stating that the observed response demonstrates a subtle asymmetrical tonic neck reflex (ATNR). The therapist asks the student to describe the client's reaction that resulted in the therapist's interpretation. Which is most accurate for the student to state the client is exhibiting based on this observation? a. Increased flexor tone of the right upper extremity. b .Increased extensor tone of the left upper extremity. c .Increased extensor tone of the right upper extremity. d. Increased extensor tone in both upper extremities.

c .Increased extensor tone of the right upper extremity.

Which payment system is used when each type of healthcare provider is paid the same amount for specific services? a. Capitation b. Private payment c. Fee for service d. Deductible

c. Fee for service

In a subjective section of a daily progress note in a mental health facility, an OT practitioner documents an individual's initial refusal to participate in OT activities. Which of the following subjective statements would MOST likely cause the therapist to consider depression as a factor in the evaluation process? a. "I had an argument with another group member and I'm too angry." b. "I do not want to participate because I do not know how to do the activity." c. "I'm just too tired." d. "I'm waiting for my visitor to come."

c. "I'm just too tired."

An OTR is supervising an OT student for Level II FW. The OTR notices that the student did not gather complete info. Regarding a client's Occ. History. Which comment best represents effective feedback? a. "you missed some info when you were recording the client's occ. History b. "in the future, you need to ask better questions about the client's occ. History c. "in the future, you may want to use a checklist to ensure that your occ. History interviews are comprehensive. d. you should practice doing occ. History interviews

c. "in the future, you may want to use a checklist to ensure that your occ. History interviews are comprehensive.

An occupational therapist is working with a cardiac patient in an acute setting. The therapist is monitoring heart rate with standing self-care tasks. What is the maximum heart rate for light activity for a patient who had an MI 10 days ago? a. 100 bpm b. 130 bpm c. 120 bpm d. 110 bpm

c. 120 bpm - keep it under 120 if it is less than six weeks ago. High risk cardiac patients are recommended to maintain a heart rate under 100 bpm. Keep it under 130 if it is CABG or CHF

A local pharmacy hires an ot to consult on a redesign of the pharmacy's customer service area. The ot recommends that the pharmacy counter be no higher than: a. 29 inches b. 33 inches c. 31 inches d. 35 inches.

c. 31 inches -30-32 for wheelchair users

an ot is conducting a home eval for an sci patient. Upon arriving at the house, it is observed that the only entrance has 5 steps, each with a 7 inch rise. A ramp to allow accessibility is recommended to the family. The ramp length according to standards is a. 72 b. 48 c. 35 d. 60

c. 35

A 45-year-old client sustained burns to the front of her left arm, posterior trunk, front of her left leg, and back of her left leg. Using the Rule of Nines, calculate the total body surface area percentage that is burned.Select one: a. 66% b. 45% c. 40.5% d. 62.5%

c. 40.5%

In regard to object use, at what age is a child's participation with a toy characterized by the child's focus on the toy's properties? a. 3-6 months b. 9-12 months c. 6-9 months d. 12-15 months

c. 6-9 months

At what age would one expect an infant to display a scissors grasp when grasping a pellet, according to Erhardt Prehension Developmental Levels? a. 9 months b. 6 months c. 8 months d. 12 months

c. 8 months

In regard to self-feeding, at what age does an infant first display the ability to feed himself finger foods? a. 6-8 months b. 12-14 months c. 9-13 months d. 5-7 months

c. 9-13 months

A child with congenital anomalies has severe developmental delay. The child demonstrates motor and cognitive skills at the nine-month level. Which is the best adaptation for the occupational therapist to use during intervention to develop the child's visual and auditory awareness? a. A hand-held rattle of the child's favorite cartoon character. b. A wrist bracelet with blinking lights that makes noise when moved. c. A button switch that activates a CD player when the switch is pressed.

c. A button switch that activates a CD player when the switch is pressed. -most developmentally appropriate

An occupational therapist provides intervention for an individual with a swallowing disorder. To elicit a swallowreflex, the occupational therapist provides sensory input to the inferior faucial arches. Which should the therapist use to provide this intervention? a. A tongue depressor. b. A moistened cotton swab. c. A chilled dental examination mirror. d. A warmed metal teaspoon.

c. A chilled dental examination mirror. -cold is good

To develop social interaction skills, an occupational therapist implements a group program for students with autism spectrum disorder (ASD). Which group is best for the therapist to include in this program? a. A directive group. b. A topical group. c. A developmental group. d. A task-oriented group.

c. A developmental group. -achieve social skills development

An OT practitioner is assessing an individual who has schizophrenia and appears to be experiencing positive symptoms of the disease. During the initial evaluation, the individual might exhibit: a. A toneless voice. b. Very little facial expression. c. A false perception of reality. d. Difficulty concentrating.

c. A false perception of reality.

An OT practitioner is treating an individual who has suddenly been diagnosed with a disabling condition. Which would be the adaptive response that would MOST likely pass in time without intervention? a. A dependency reaction. b. A stress reaction. c. A mourning response. d. A desire to set unrealistic goals.

c. A mourning response.

An occupational therapist working for a home care agency provides an inservice to new employees on Medicare reimbursement guidelines for durable medical equipment (DME). Which item would the therapist describe as reimbursable by Medicare? a. A raised toilet seat for a patient after a hip replacement. b. A reacher for a person with arthritis in both hips. c. A walker for a person who cannot ambulate in the home without one. d. Grab bars in the bathroom for a person who cannot bathe or toilet without them.

c. A walker for a person who cannot ambulate in the home without one. -dont think grab bars count as DME

A OTR is working with an adult with substance abuse disorder in a community-based setting. The client has maintained sobriety for 1 year, however, the evaluation results indicate that the client has inadequate adaptive skills, poor hygiene, and low self-esteem. What INITIAL goal is MOST BENEFICIAL to include in the intervention plan to improve occupational performance and participation? a. Provide education about coping skills to prevent any relapses b.Increase engagement in social and leisure activities with friends c. Acquisition of basic life skills for greater independence

c. Acquisition of basic life skills for greater independence

An individual recovering from flexor tendon repair surgery is two days post-operation. The surgeon refers the client to occupational therapy with a prescription to use the Kleinert protocol to guide intervention.B150. The occupational therapist plans the client's early mobilization program. Which is the most appropriate exercise routine for the occupational therapist to use within the limits of a dorsal block splint? a. Active flexion/passive extension. b. Active flexion/active extension. c. Active extension/passive flexion. d. Passive flexion/passive extension.

c. Active extension/passive flexion.

The OT is observing a 3-year-old child dur ing toothbrushing. The child demonstrates good bilateral upper extremity/hand strength, but decreased dexterity. Which piece of equip ment would the OT MOST likely encourage the child to use during toothbrushing? a. A small soft bristle toothbrush. b. A toothbrush with universal cuff. c. An electric toothbrush. d. A soft sponge-tipped toothette.

c. An electric toothbrush. -can clean without having to rotate brush

An entry-level occupational therapy assistant (COTA®) recently hired for an outpatient rehabilitation clinic requires supervision during the direct supervisor's scheduled vacations. To maximize departmental efficacy, who should provide supervision to the COTA®? a. The rehabilitation clinic's administrator. b. A COTA® with one year of experience. c. An occupational therapist. d. A COTA® with advanced credentialing.

c. An occupational therapist.

A young adult with a T9-T10 spinal cord injury wishes to engage in sports activities. Which wheelchair features are best for the occupational therapist to recommend to this client? a. A heavy-duty foldable frame with a high back. b. An ultra-light foldable frame with a high back. c. An ultra-light rigid frame with a low back. d. A heavy-duty rigid frame with a low back.

c. An ultra-light rigid frame with a low back.

When conducting a structured interview, it is MOST important for the OT practitioner to: a. Rephrase the interview questions in his or her own words. b. Ask questions the therapist thinks are pertinent to this patient. c. Ask the questions as they are stated on the interview sheet. d. Ask additional questions to gain further insight into the patient.

c. Ask the questions as they are stated on the interview sheet.

In regard to developmental assessments of neonates, which outcome measurement tool is an extension and refinement of the Neonatal Behavioral Assessment Scale (NBAS) and assesses an infant's pattern of developing behavioral organization in response to increasing sensory and environmental stimuli? a. Denver Developmental Screening Test II b. Bayley Scales of Infant Development, 3rd Edition (BSID-III) c. Assessment of Preterm Infants' Behavior (APIB) d. Neurological Assessment of Pre-term and Full-term Newborn Infant (NAPFI)

c. Assessment of Preterm Infants' Behavior (APIB)

When performing an infant occupational therapy evaluation, the occupational therapist turns the infant's head and holds it for 5 seconds while the infant is positioned on her back. Which reflex is he assessing? a. Tonic labyrinthine-supine b. Tonic labyrinthine-prone c. Asymmetric tonic neck d. Landau

c. Asymmetric tonic neck

Which form of cerebral palsy is characterized by hypotonia? a. Spastic cerebral palsy b. All forms of cerebral palsy are characterized by hypotonia c. Ataxic cerebral palsy d. Dyskinetic cerebral palsy

c. Ataxic cerebral palsy

An individual with hemiplegia and her spouse need to learn how to perform transfers. The MOST important transfer(s) to learn would be to: a. The unaffected side of the individual's body. b. The affected side of the individual's body. c. Both sides of the individual's body. d. The side of the body from which the individual will be approaching the transfer.

c. Both sides of the individual's body. -dependent on environment

A patient presents with an incomplete spinal cord injury, with movement below the injury level, but his manual muscle strength is grossly 2/5. What type/grade of spinal cord injury is described, based on the ASIA Impairment Scale? a. A b. B c. C d. D

c. C

A patient presents with numbness and tingling in the first four fingers but does not complain of pain in the forearm. Which is the likely condition? a. Pronator teres syndrome b. Guyon's canal c. Carpal tunnel syndrome d. Cubital tunnel syndrome

c. Carpal tunnel syndrome

Which of the following spinal cord injury syndromes originates in the lumbar vertebrae, resulting in a loss of movement below the injury and decreased control of the bowel and bladder? a. Brown-Sequard syndrome b. Anterior cord syndrome c. Cauda equina syndrome d. Central cord syndrome

c. Cauda equina syndrome

Which type of hand injury would be expected in a patient with an ulnar nerve laceration? a. Wrist drop b. Ape hand c. Claw hand deformity d. Benediction sign

c. Claw hand deformity

Which term related to payment for occupational therapy services refers to the amount a patient must pay for services received from a healthcare provider? a. Usual and customary rate b. Denial c. Co-insurance d. Beneficiary

c. Co-insurance

Which type of play describes when a child begins to play in groups and to expand social, motor, sensory, and cognitive processing skills? a. Exploratory play b. Game-based play c. Creative play d. Symbolic play

c. Creative play

Regarding qualitative research, which criterion is defined as the study's ability to provide an accurate overview of information received, regardless of whether it supports the theory or purpose of the study? a. Dependability b. Confirmability c. Credibility d. Transferability

c. Credibility

Which is the MOST important policy to review with an entry-level occupational therapist who is newly hired at an acute care facility? a. Reimbursement for services b. Productivity standards c. Crisis intervention d. Employee benefits

c. Crisis intervention

A patient presents with an incomplete SCI with motor function preserved below neurological level and the majority of key muscle groups below the neurological level have a muscle grade of greater than or equal to 3/5. What type/grade of spinal cord injury is described, based on the ASIA Impairment Scale? a. C b. B c. D d. A

c. D

Which type of data would be MOST useful for an OT practitioner consultant in an adult day care facility for the purpose of identifying the overall needs of the adult day-care population at the site? a. ADL and IADL performance-based tests results. b. Leisure and recreational checklist filled out by clients. c. Data about overall occupational performance, activity needs, and health issues of clients. d. Results of standardized cognitive level assessments for each client.

c. Data about overall occupational performance, activity needs, and health issues of clients.

A client with left upper extremity hemiplegia is referred to occupational therapy to regain use of the left arm following a stroke. Which of the following is the least appropriate intervention to facilitate functional use of the left arm? a. Instructing the client to use the left arm to push up from sideline to sitting during bed mobility b. reminding the client to use the left upper extremity for postural support when wiping the table c. Demonstrating a home exercise program that incorporates self range of motion and overhead pulleys d. Encouraging the client to use the left hand to wash the right arm and apply lotion during bathing and grooming

c. Demonstrating a home exercise program that incorporates self range of motion and overhead pulleys -overhead use may lead to impingement

Which developmental assessment tool is used to identify the potential for developmental delay and consists of 125 test items to assess the infant's ability to complete age-appropriate tasks in four developmental subsets? a. Assessment of Pre-term Infants' Behavior (APIB) b. Neurological Assessment of Pre-term and Full-term Newborn Infant (NAPFI) c. Denver Developmental Screening Test II d. Bayley Scales of Infant Development, 3rd Edition (BSID-III)

c. Denver Developmental Screening Test II

Which of the following is FALSE regarding driver rehabilitation intervention? a. Necessary driving skills can be influenced by physical and cognitive factors b. An occupational therapist wishing to complete on-the-road driving evaluations must become licensed by the state c. Driving is a basic activity of daily living d. Driver rehabilitation involves consistent hands-on instruction

c. Driving is a basic activity of daily living

An occupational therapist works in a school system with a child with developmental delays. One of the goals of treatment is to develop prewriting skills. The child exhibits the ability to grasp a pencil proximally with crude approximation of the thumb, index, and middle fingers and the ring and little fingers slightly flexed. The therapist develops an intervention plan. Which grasp should be the focus for the implementation of intervention? a. Digital pronate grasp. b. Static tripod posture grasp. c. Dynamic tripod grasp. d. Palmar supinate grasp.

c. Dynamic tripod grasp. -has ability to make position so do the best option

The administrator of a large rehabilitation hospital reviews the occupational therapy staffing schedules. The administrator determines that of the 12 full-time therapists, six work full-time in the inpatient department, two work full-time in the outpatient department, two divide their hours equally between the inpatient and outpatient department, and two divide their hours equally between administrative/managerial work and direct care provision on the inpatient unit. Which is most accurate staffing for the administrator to document for the occupational therapy inpatient department in the annual report? a. Twelve full-time equivalent employees. b. Ten full-time equivalent employees. c. Eight full-time equivalent employees. d. Six full-time equivalent employees.

c. Eight full-time equivalent employees.

A school-based occupational therapist consults with a teacher regarding a nonspeaking student who uses a wheelchair and an augmentative communication device. The teacher reports that the student has been making many errors on the communication device but that no difficulties had been observed when the student used the device in the past. Which is the most effective initial action for the therapist to take in response to the teacher's report? a. Advise the teacher to contact the student's parents and recommend that they bring the child to a physician for an exam. b. Reassess the student's motor and communication abilities to determine needed modifications. c. Evaluate the position of the student in the wheelchair and the device on the wheelchair. d. Reposition the communication device on the wheelchair to facilitate access and increase accuracy.

c. Evaluate the position of the student in the wheelchair and the device on the wheelchair.

An individual is referred by a physician to an outpatient occupational therapy program for an evaluation. The person's insurance company does not cover outpatient OT services. Which action is best for the therapist to take in response to this situation? a. Follow the physician's order and let the person pay the bill when it is received. b. Refuse to accept the referral and advise the person to seek additional insurance coverage. c. Explain the limitation of the insurance coverage and let the person decide whether to complete the evaluation. d. Request that the program's billing department submit a bill using a code that is covered by the insurance company.

c. Explain the limitation of the insurance coverage and let the person decide whether to complete the evaluation.

An OT begins seeing a 2-year-old client for feeding. Upon initial observation, the OT notices that the client pockets a large amount of food in his mouth. The client's parents report that he typically holds food in his mouth for hours at a time and refuses to swallow in between bites. What is the most appropriate intervention strategy to apply during treatment sessions?Select one: a. Prevent tongue retraction to avoid choking b. Refer the parents for a VFSS c. Facilitate swallow by placement and slight downward pressure of the spoon on the middle aspect of the tongue d. Facilitate chewing by placement of long, crunchy-textured food on lateral molars

c. Facilitate swallow by placement and slight downward pressure of the spoon on the middle aspect of the tongue

During a functional assessment of strength, the OT observes that the individual can move the arm through the full range of motion to reach a high bathroom shelf, but can lift and place nothing heavier than a can of spray deodorant on the shelf. The strength according to the MMT would be documented as: a. Fair minus (3-). b. Fair (3). c. Fair plus (3+). d. Good minus (4-).

c. Fair plus (3+).

An individual with rheumatoid arthritis has developed several boutonniere deformities. Which of the following is the most accurate description for the occupational therapist to include in documentation of the individual's presenting signs? a. Hyperextension of the PIP joint and flexion of the DIP joint. b. Ulnar deviation and subluxation of the MCP joints. c. Flexion of the PIP joint and hyperextension of the DIP joint. d. Heberden's nodes at the DIP joints and Bouchard's nodes at the PIP joints.

c. Flexion of the PIP joint and hyperextension of the DIP joint.

A patient has endured a burn around the foot and ankle. What contracture is she MOST at risk for developing? a. Dorsiflexion contracture b. Inversion contracture c. Foot drop d. Knee flexion contracture

c. Foot drop

An OTR is teaching a client an exercise program as part of a lifestyle modification plan to control diabetes. During the exercise program, the OTR noticed that the client suddenly looked pale, sweaty, and agitated. The client spoke rapidly and reported feelings of hunger. What should the OTR do FIRST? a .Encourage the client to finish the exercise b .Tell him to stop and take his insulin c. Give the client a cup of orange juice d. Place the client in supine position with the legs up

c. Give the client a cup of orange juice

A client is admitted to the hospital exhibiting the following symptoms: symmetric muscular weakness, distal sensory loss, stocking-glove distribution (tingling from the feet and legs to the upper body), and deep tendon reflex loss. What neurological system disorder might this client have?Select one: a. Post-polio Syndrome b. Amyotrophic Lateral Sclerosis (ALS) c. Guillan-Barre Syndrome d. Muscular Dystrophy

c. Guillan-Barre Syndrome

An occupational therapy evaluation reveals findings consistent with ulnar nerve compression at the wrist. Which peripheral nerve disorder is consistent with this finding? a. Carpal tunnel syndrome b. Pronator teres syndrome c. Guyon's canal d. Cubital tunnel syndrome

c. Guyon's canal

A young adult recently diagnosed with schizophrenia is referred to an occupational therapy day treatment program. Which should the occupational therapist do first with the client? a. Determine short-term and long-term goals for program participation. b. Model desired behaviors during occupational therapy groups. c. Have the client complete an occupational interest checklist. d. Encourage the client to maintain a daily log of medication intake.

c. Have the client complete an occupational interest checklist.

An individual is status post carpal tunnel release. When the occupational therapist conducts a sensory test for sharp/dull (pain), the person reports dull as sharp on the palmar surface of the thumb and index finger. All other responses were correct. Which is accurate for the therapist to document about the individual's sensation? a. Impaired for pain along C5 and C6 dermatomes. b. Hypersensitive along the ulnar nerve distribution of the palmar surface of the hand. c. Hypersensitive along the median nerve distribution of the thumb and index fingers. d. Absent for pain along the median nerve distribution

c. Hypersensitive along the median nerve distribution of the thumb and index fingers.

When working with a client with chronic pain, the OTR needs to set a tone that will facilitate a therapeutic relationship with the client. Which statement sets an open and appropriate tone when working with a client who has chronic pain? a. your pain is all in your head. All you have to do is ignore it b. I have pain when I wake up in the morning. I have arthritis. It usually takes a while before it gets any better. Maybe you should try to exercise in the morning to get things moving like I do c. I realize that you are in pain. Lets try this activity and see what happens. If you need to stop, let me know, but I would like to see you push yourself. d. I think we should slowly increase the time you are spending at work-related tasks

c. I realize that you are in pain. Lets try this activity and see what happens. If you need to stop, let me know, but I would like to see you push yourself.

Several newly homeless veterans with a variety of mental health diagnoses attend an occupational therapy community reentry group conducted in a community-based shelter. Which should be the primary focus of the initial group session? a. Development of home management skills such as meal preparation. b. Determination of financial assets and money management skills. c. Identification of local resources such as soup kitchens and thrift stores. d. Exploration of vocational interests and employment possibilities.

c. Identification of local resources such as soup kitchens and thrift stores. -I guess D. doesn't focus on basic needs first (i.e. food/shelter)

An OT practitioner is conducting a perceptual function screening with an individual who has had a CVA. Which of the following informal screening activities would the therapist ask the individual to perform in order to identify the presence of agnosia? a. Demonstrate common gestures such as waving. b. Identifying objects through touch only. c. Identify or demonstrate the use of common household objects. d. Read a paragraph and explain its meaning.

c. Identify or demonstrate the use of common household objects.

Determining a developmentally disabled adult's level of cognitive function according to Allen's Cognitive Disability theory provides information about the person which is MOST helpful for: a. Identifying the client's difficult behaviors that might interfere with intervention. b. Planning a training program to improve prevocational skills. c. Identifying the type of environmental support that can maximize the client's level of function. d. Developing an educational plan to improve social skills.

c. Identifying the type of environmental support that can maximize the client's level of function. -number 1 priority of Allen is client safety (what support is needed)

During an evaluation of shower safety following a knee replacement, the individual begins to fall toward the operated leg. The OT practitioner determines that the individual is demonstrating: a. Unilateral neglect. b. Lower extremity weakness. c. Impaired balance. d. Impaired gross motor coordination.

c. Impaired balance.

A single parent with rheumatoid arthritis and two school-aged children reports difficulty completing a home exercise program. The parent states that multiple familial, work, and home management responsibilities fill the day and additional activities cannot fit into the day. Which is the best action for the occupational therapist to take in response to these realities? a. Explain and reinforce the importance of active range of motion exercises for remediation of dysfunction. b. Provide intervention to develop time management skills and enable temporal adaptation. c. Incorporate the parent's engagement in a diversity of role activities into the home program. d. Increase the frequency of OT sessions to compensate for lack of follow-through with the home program.

c. Incorporate the parent's engagement in a diversity of role activities into the home program.

A nine-year-old child is referred to an outpatient pediatric clinic first screening for occupational therapy services the mother reports to the OTR that the child constantly breaks pencils while doing math homework, frequently falls out of the chair at home, and hugs family and friends and appropriate times. Based on the mother's observations, which intervention goal area is most appropriate to focus on for the child? a. Improving motor planning or praxis b. Decreasing sensory awareness c. Increasing proprioceptive awareness

c. Increasing proprioceptive awareness

An individual with paraplegia is learning to perform wheelchair-to-car transfer. What is the highest level of independence this individual is likely to achieve? a. Minimal to moderate assistance. b. Independent transfer unrealistic; recommend public transportation. c. Independent with use of a sliding board. d. Independent using tenodesis functions

c. Independent with use of a sliding board.

An OT practitioner interviewing an individual diagnosed with Alzheimer's disease about his ADL performance realizes that the client is confabulating. Which of the following options in MOST appropriate? a. Complete the interview using closed-ended questions. b. Stop the interview and complete it the next day. c. Interview a reliable informant instead of the individual. d. Administer a written questionnaire using a checklist format.

c. Interview a reliable informant instead of the individual.

An occupational therapist is treating an individual with Parkinson's disease in an outpatient setting. The therapist observes that the person attends regularly but has little energy and is difficult to engage during intervention. The person reports limited performance of the activities prescribed for their home program. Which is the best action for the therapist to take in response to these observations and the person's self-report? a. Ask the person's physician to complete a referral for a psychiatric evaluation. b. Tell the person that the completion of the home program is vital to recovery. c. Interview the person and complete a standardized depression scale. d. Defer intervention until the person's depression is treated.

c. Interview the person and complete a standardized depression scale. -find reasons for little energy/occupational engagement -defer to MD if it indicates depression

During a topical work preparation group for individuals recovering from mental illness, a member expresses concern about answering questions related to personal psychiatric history during a job interview. Which action is best for the occupational therapist to take in response to these expressed concerns? a. Refer the client to a vocational rehabilitation counselor. b. Encourage the other members of the group to share their interview experiences. c. Lead a group discussion on the legal rights afforded in the interview process. d. Support the client in not disclosing past psychiatric history.

c. Lead a group discussion on the legal rights afforded in the interview process. - topical groups develop knowledge about specific performance area (i.e. interview)

For the following goal, which piece of the SMART goal acronym is missing? The patient will increase the 9 hole peg test score in two weeks for improved coordination. a. Attainable b. Time-limited c. Measurable d. Short

c. Measurable

Which of the following upper extremity disorders is caused by repetitive use of the wrist flexors? a. Lateral epicondylitis b. Carpal tunnel syndrome c. Medial epicondylitis d. De Quervain's tenosynovitis

c. Medial epicondylitis

An individual is transferred from an acute care hospital to a subacute rehabilitation unit in a long-term care facility. The patient incurred a left cerebral vascular accident (CVA) in the middle cerebral artery (MCA) one week ago. The individual is referred to occupational therapy. The referral states that the patient has right hemiplegia and a subluxed right shoulder.B95. During the initial ADL evaluation, an occupational therapist notes that the patient cuts food awkwardly and consistently spills food when moving the food from the plate to the mouth. When recording this observation,which deficit should the therapist identify as a primary focus for intervention? a. Ideational apraxia. b. Asomatognosia. c. Motor apraxia. d. Tactile agnosia

c. Motor apraxia.

A patient presents with a variety of neurologic symptoms, including parasthesia, with periods of remission and exacerbations. Which disease of the central nervous system are they likely experiencing? a. Amyotrophic lateral sclerosis b. Guillain-Barré syndrome c. Multiple sclerosis d. Myasthenia gravis

c. Multiple sclerosis

Which disorder of the peripheral nervous system is an autoimmune disease when the body attacks acetylcholine receptors, resulting in weakness, especially in the facial and proximal muscles? a.Amyotrophic lateral sclerosis (ALS) b. Guillain-Barré syndrome c. Myasthenia gravis d. Post-polio syndrome

c. Myasthenia gravis

An OT is working with a client in a sub-acute unit of the hospital who is experiencing pain. The pain they are describing is persistent, aches deep in the muscle, and is highly sensitive in tender spots. What type of pain is this client experiencing?Select one: a. Fibromyalgia pain syndrome b. Acute pain c. Myofascial pain syndrome d. Chronic pain

c. Myofascial pain syndrome

A young child is reported to be left alone at home after school for long hours and regularly comes to school hungry, hoarding leftover lunch food in his pockets. Which type of child abuse might the occupational therapist suspect? a. Physical abuse b. Sexual abuse c. Neglect d. Emotional abuse

c. Neglect

Which infant neurological evaluation tool involves assessing an infant's response to items presented in three stages: during the infant's "quiet or sleep state," "items not influenced by state," and during the infant's "awake state"? a. Denver Developmental Screening Test II b. Bayley Scales of Infant Development, 3rd Edition (BSID-III) c. Neurological Assessment of Pre-term and Full-term Newborn Infant (NAPFI) d. Assessment of Pre-term Infants' Behavior (APIB)

c. Neurological Assessment of Pre-term and Full-term Newborn Infant (NAPFI)

When performing a house assessment for an occupational therapy evaluation, the therapist notices the house has a ramp for wheelchair access. The ramp is 10 feet long from the front door, over the stairs. The stairs have four steps, each with a 7-inch rise. Is this an appropriate ramp? a. Yes. The length does not matter, as long as it covers the stairs. b. No. The ramp should be 7 feet long. c. No. The ramp should be 28 feet long. d. Yes. This is an appropriate length for this situation.

c. No. The ramp should be 28 feet long.

Visual foundation skills must be evaluated to differentiate perceptual dysfunction and visual system deficits. Which component identifies control of eye movements? a. Scanning b. Visual acuity c. Oculomotor function d. Visual fields

c. Oculomotor function

In regard to fetal sensorimotor development, what sense has NOT been at least partially formed by the second trimester? a. Vision b. Taste c. Olfactory d. Auditory

c. Olfactory

An eight-year-old was admitted to a skilled nursing facility after surgery to repair a fractured hip. The client was transferred to the facility from the hospital to Reagan independence in activities of daily living in mobility. The client was working in the garden and lost balance; however, did not sustained a fall to the ground. Although the client did not fall, the sudden movement led to a broken hip. During the evaluation, the client was unsure why the heck broke since all falls down occur. Which of the following conditions most likely is the largest contributor to the injury? a. osteopenia b. Osteosarcoma c. Osteoporosis d. Osteoarthritis

c. Osteoporosis

According to Exner's Classification System, which manipulation skill, present by 2.5 years of age, is used when putting a quarter into a piggy bank? a. Simple rotation b. Shift c. Palm-to-finger translation d. Finger-to-palm translation

c. Palm-to-finger translation

Which pediatric developmental assessment, intended for children ages 6 months to 7.5 years, assesses a child's developmental ability via a behavior checklist administered through observation and interview? a. Denver Developmental Screening Test II b. Hawaii Early Learning Profile, Revised (HELP) c. Pediatric Evaluation of Disability Inventory (PEDI) d. Bayley Scales of Infant Development, 3rd Edition (BSID-III)

c. Pediatric Evaluation of Disability Inventory (PEDI)

An occupational therapist completes a cognitive screening for a person with chronic schizophrenia, undifferentiated type. The therapist uses Allen's cognitive disabilities model to guide the evaluation process. During the screening, the person is able to imitate the whipstitch but cannot imitate the single cordovan stitch. Based on these results, the occupational therapist determines that further evaluation is indicated. Which ability is most relevant for the therapist to assess? a. Performance of multistep tasks using overt trial and error problem-solving. b. Performance of multistep tasks using analytical reasoning. c. Performance of simple tasks independently using visual cues. d. Performance of simple tasks with long-term repetitive training.

c. Performance of simple tasks independently using visual cues. -level IV

Which complication of amputation is described as the sensation that an amputated limb is still attached? a. Neuroma b. Phantom limb pain c. Phantom limb syndrome d. Contracture

c. Phantom limb syndrome

An occupational therapist working in an outpatient clinic observes the clinic's administrative assistant leaving patient records open on the clinic's reception counter. The assistant has left the clinic to go for lunch. Which action is best for the therapist to take in response to this observation? a. Remind the administrative assistant of the need to keep patient records private when they return from lunch. b. Immediately contact the administrative assistant's direct supervisor to report this observation. c. Pick up the records and place them in a location out of public view. d. Discuss the issue with the clinic's director during their next scheduled supervision session.

c. Pick up the records and place them in a location out of public view.

According to SOAP note documentation, in which section of a note should the goals be documented? a. Assessment b. Subjective c. Plan d. Objective

c. Plan

A 75-year-old male client is complaining of ringing in both of his ears during treatment and cannot comprehend the OT's directions when engaging in therapy. Which type of hearing loss is the client exhibiting?Select one: a. Tinnitus b. Conductive c. Presbycusis d. Sensorineural

c. Presbycusis

A toddler with developmental delays attends an early intervention program. Over the past two weeks, the toddler has successfully completed the activities the occupational therapist has provided in order to develop a palmar grasp. Which action should the therapist take next in response to the child's progress? a. Continue providing the child with the activities to refine palmar grasp. b. Review the initial evaluation to determine new goals. c. Provide activities to develop a radial palmar grasp. d. Provide activities to develop an ulnar palmar grasp.

c. Provide activities to develop a radial palmar grasp.

An OT practitioner is evaluating a young cabinetmaker who complains of sensory changes over the dorsal thumb and proximal phalanx of the index, long, and half of the ring finger. The practitioner will MOST likely suspect involvement of the: a. Ulnar nerve. b. Median nerve. c. Radial nerve. d. Brachial plexus.

c. Radial nerve.

An older teenager with a congenital right below-elbow amputation had never wanted a prosthesis before. Now the teen wants a prosthesis "to look good at the prom and more 'normal' when doing things with my friends." Which action would be most beneficial for the occupational therapist to take to meet the teen's expressed needs? a. Recommend a prosthesis with a cosmetic passive hand. b. Recommend a prosthesis with a voluntary opening hook. c. Recommend a prosthesis with a myoelectrically controlled hand. d. Recommend counseling to explore the client's sudden preoccupation with body image.

c. Recommend a prosthesis with a myoelectrically controlled hand. -will look the most normal out of every option

An individual with borderline personality disorder has been referred to occupational therapy. Which of the following would be MOST important to evaluate? a. Activities of daily living b. Instrumental ADLs c. Relationships with others d. Sensorimotor skills

c. Relationships with others -BPD effects this the most - self-image issues, difficulty managing emotions and behavior, and a pattern of unstable relationships.

A home-care occupational therapist plans intervention for an individual with agoraphobia with panic attacks. The occupational therapist plans to use a cognitive-behavioral approach. Which approach is best for the therapist to use with the client during the initial intervention session? a. A token reward system. b. Behavioral extinction. c. Relaxation techniques. d. Systematic desensitization.

c. Relaxation techniques.

.An occupational therapist provides consultation services to a psychogeriatric unit for individuals with moderately severe cognitive decline. In designing the activity program, which groups are best for the occupational therapist to include? a. Reality orientation. b. Sensory stimulation. c. Reminiscence. d. Coping skills.

c. Reminiscence. -level III can remember pictures/long-term

An individual with a spinal cord injury (SCI) at the level of T1 is practicing a stand pivot transfer in the OT department of a rehabilitation center. The patient complains of dizziness and nausea. Which action is most important for the occupational therapist to take first? a. Call for help according to facility procedures. b. Return the patient to the wheelchair for a five-minute rest break. c. Return the person to the wheelchair and immediately recline it. d. Return the patient to the wheelchair and transport the patient back to rest in bed.

c. Return the person to the wheelchair and immediately recline it.

A patient s/p CVA presents with hemiplegia, impaired sensation, neglect and impaired spatial awareness. What is the likely site of infarct? a. Posterior cerebral artery b. Anterior cerebral artery c. Right middle cerebral artery d. Left middle cerebral artery

c. Right middle cerebral artery

An occupational therapist is providing intervention for positioning s/p a posterior total hip replacement. What is NOTan appropriate position? a. Sitting in a tall chair b. Lying supine c. Rolling toward the non-operated side d. Using a hip abductor pillow between legs

c. Rolling toward the non-operated side

A 10-year-old with congenital anomalies wears bilateral ankle-foot orthoses. The parents want the child to be able to don and doff shoes independently, but the child cannot tie shoes. Which is the best footwear recommendation for the therapist to make for the child to wear? a. Leather slip-on loafers. b. Slip-on tennis shoes with no laces. c. Running shoes with Velcro shoe closures. c. Hi-rise sneakers with sliding adapters on the laces.

c. Running shoes with Velcro shoe closures.

Visual foundation skills must be evaluated to differentiate between visual information processing impairments and visual system deficits. Which describes the ability of an individual to search for and locate hazards when driving? a. Oculomotor function b. Visual fields c. Scanning d. Visual acuity

c. Scanning

According to Abraham Maslow's hierarchy of basic human needs, what is the highest level of pursuit, in which an individual can reach their highest level of capability? a. Physiological b.Love and belonging c. Self-actualization d. Self-esteem

c. Self-actualization

An individual with developmental disabilities scores a Level 3 on the Allen Cognitive Level Test. Which activities should the occupational therapist include in the intervention plan to help meet the client's functional needs? a. Community mobility activities such as taking a bus. b. Home management activities such as preparing a food shopping list. c. Self-care activities such as brushing teeth. d. Leisure activities such as completing a 50-piece puzzle.

c. Self-care activities such as brushing teeth.

Which term describes the steady physical decline of the body as it ages? a. Dementia b. Gerontology c. Senescence d. Life expectancy

c. Senescence

An individual with Parkinson's disease exhibits difficulty moving from sitting in a chair to standing. Which technique is best for the therapist to recommend the person use to help successfully complete this functional mobility activity? a. Rise from the chair while sitting with buttocks against the back of the chair. b. Extend both legs so that both feet are away from the chair while rising. c. Sit at the edge of the chair and rock back and forth before rising. d. Rise while weight-bearing on one foot and pushing up with both arms.

c. Sit at the edge of the chair and rock back and forth before rising.

Which is an appropriate driver adaptation for a person who has limited use of his left arm? a. Hand control steering ring b. Hand controls c. Standard round spinning knob d. Pedal extensions

c. Standard round spinning knob

The occupational therapist completes an intake interview for a work hardening program. As the individual is leaving, the person gives the therapist a hug and expresses much gratitude. The individual then tries to kiss the therapist on the lips. Which action is best for the therapist to take in response to this situation? a. Forcibly push the individual away while telling the person that the behavior is inappropriate and unacceptable. b. Say nothing but decline the person's admission to the work hardening program based upon the person's inappropriate behavior. c. State that the individual's behavior oversteps professional boundaries and makes the therapist uncomfortable. d. Tell the person the behavior is inappropriate and unacceptable and decline the person's admission to the program.

c. State that the individual's behavior oversteps professional boundaries and makes the therapist uncomfortable.

As part of an intervention, an occupational therapist is teaching driving adaptations to a person who has a prosthetic upper extremity and fully functional legs. She would like to use both arms for driving. What is the MOST appropriate adaptation? a. Pedal extensions b. Hand controls c. Steering ring d. Standard round spinning knob

c. Steering ring

In regard to visual impairments that may occur in children with cerebral palsy, which condition is characterized by misalignment of the eyes? a. Presbyopia b. Myopia c. Strabismus d. Nystagmus

c. Strabismus

Which muscle of the rotator cuff functions for internal rotation and is innervated by the suprascapular nerve? a. Teres minor b. Supraspinatus c. Subscapularis d. Infraspinatus

c. Subscapularis

Which type of play occurs when the child begins to play side by side with other children and forms a greater understanding of thoughts, emotions, and actions? a Exploratory play b. Game-based play c. Symbolic play d. Creative play

c. Symbolic play - 2-4 years old

An individual who recently experienced an MI has been referred to OT. To determine the individual's current endurance level, the OT practitioner plans to monitor performance of self-care activities. The first step is to: a. Take the individual's vital signs after performance of self-care activities. b. Observe the individual for signs and symptoms of exhaustion during self-care activities. c. Take the individual's vital signs at rest. d. Take vital signs 5 minutes after the individual has completed self-care activities.

c. Take the individual's vital signs at rest.

Which area of the brain controls understanding of auditory information? a. Parietal lobe b. Frontal lobe c. Temporal lobe d. Occipital lobe

c. Temporal lobe

An individual recovering from flexor tendon repair surgery is two days post-operation. The surgeon refers the client to occupational therapy with a prescription to use the Kleinert protocol to guide intervention.B151. The occupational therapist meets with the client to ensure compliance with the prescribed splinting protocol. Which is the most important outcome of this session? a. The client's adherence to a written splint wearing schedule. b. The client's ability to independently don and doff the splint. c. The client's understanding of the purpose(s) and procedure(s) of the splint protocol. d. The completion of functional training in the use of the splint.

c. The client's understanding of the purpose(s) and procedure(s) of the splint protocol. -ensure client buy-in and collaboration -others are good but not most important

An individual with borderline personality disorder incurred a back injury while working as a stockperson for a large warehouse. The individual attends a work hardening program. The occupational therapist evaluates the individual and determines that the client's level of productivity is just below the warehouse minimum standards. The individual complains of pain when lifting the heaviest of boxes. The client frequently becomes angry and verbally abusive in response to directions or feedback. Which is the most important initial focus of the work hardening program for this individual? a. An increase in productivity to meet minimum standards. b. The development of strength and ergonomic lifting abilities. c. The development of affective work behavior skills.

c. The development of affective work behavior skills.

An occupational therapist is working with a five-year-old child in a school setting. She notices the child talks about touching his genitals and exposes himself to his fellow classmates. The occupational therapist may suspect which form of child abuse? a. neglect b. physical abuse c. sexual abuse d. emotional abuse

c. sexual abuse

When evaluating motor control in a beginning-level patient with TBI (Rancho Los Amigos Scale score of 1-3), the OT practitioner will MOST likely observe for: a. Response to simple verbal commands. b. Response to pain or temperature. c. Tone, rigidity, and reflexes. d. Eye contact.

c. Tone, rigidity, and reflexes.

A school-based occupational therapist is teaching orientation and mobility skills to an adolescent with a degenerative visual disorder. Which is the most effective motivational technique for the therapist to use with this student? a. Provide concrete structure and frequent feedback to ensure accurate orientation and safe functional mobility. b. Keep sessions short to allow time for emotional adjustment to orientation and mobility challenges. c. Treat the student as an adult and incorporate the student's orientation and mobility goals into intervention sessions. d. Limit anxiety by practicing the techniques in a quiet and self-contained environment (e.g., an empty classroom).

c. Treat the student as an adult and incorporate the student's orientation and mobility goals into intervention sessions. -most client centered; A. is too much structure

You are an occupational therapist working with a client who is 5 years old and has a history of simple febrile seizures. During an obstacle course activity in the OT gym, the client starts to have involuntary generalized jerking. Which of the following should be avoided?Select one: a. Allow the seizure to happen; protect the head and/or extremities if injury could occur from violent shaking b. Remove dangerous objects from the area c. Turn the client on his/her back d. Protect the individual from harm, without interfering with the individual's movements

c. Turn the client on his/her back

An occupational therapist is promoting the field of occupational therapy with informational handouts at a village community event. She is not looking to promote her own practice but to raise awareness of the profession. What type of marketing is she using? a. Unethical marketing b. Differentiated marketing c. Undifferentiated marketing d. Concentrated marketing

c. Undifferentiated marketing

As defined by Yalom, which "curative factor of groups" involves the realization that others may be experiencing similar emotions or events? a. Catharsis b. Guidance c. Universality d. Altruism

c. Universality

Which principle of the Occupational Therapy Code of Ethics refers to honesty in depicting the services and purpose of occupational therapy? a. Fidelity b Justice c. Veracity d. Beneficence

c. Veracity

A patient who is status-post left frontal lobe ischemia has difficulty bearing weight through the right lower extremity during reaching activities (e.g., standing at a sink during morning self-care routine). The occupational therapist implements a Motor Re-Learning Program (MRP). Which is the best intervention for the therapist to provide according to this approach? a. Therapeutic handling to affect the central nervous system. b. A stool to sit on during reaching activities. c. Joint compression to the right lower extremity during reaching activities. c. Verbal and visual feedback while practicing reaching.

c. Verbal and visual feedback while practicing reaching. -A. is NDT approach -B. is compensatory -C. is sensory

An OT practitioner is evaluating a group of individuals with Parkinson's disease in an aquatic therapy program. Which of the following would be MOST important for successfully walking across the pool a. Strength, fine motor coordination, and kinesthesia. b. Auditory processing, postural control, and gross motor coordination. c. Vestibular processing, postural control, and muscle tone. d.Normal range of motion, praxis, and crossing the midline.

c. Vestibular processing, postural control, and muscle tone.

An OTR is working with an adult client with sensory processing difficulties who finds it uncomfortable to commute to a job via train because of the noise of the train and of other passengers talking. Which environmental modification would be appropriate? a. encourage the client to obtain a driver's license and avoid train b. petition the train to set off a section of silence c. recommend the use of noise-cancelling headphones with a device that plays music or books the client enjoys d. get a closer job

c. recommend the use of noise-cancelling headphones with a device that plays music or books the client enjoys

An occupational therapist is working with a patient who is cognitively intact but lacks full range of motion in her shoulders and has impaired upper extremity muscle strength and endurance. What accommodation would be MOST appropriate for driving? a. pedal extensions b. steering ring c. reduced effort steering d. hand controls

c. reduced effort steering -alters wheels position

Which type of work program is provided by one therapy discipline, focuses on return-to-work activities, and does not require CARF accreditation? a. Rehabilitation workshop b. Ergonomic program c. Work conditioning program d. Work hardening program

c. Work conditioning program -work hardening requires CARF

With a newborn, which form of reaching is observed? a. reaching with bilateral arms b. unilateral reaching with the shoulder flexed and externally rotated and the forearm supinated c. batting at toys with fingers flexed d. unilateral reaching with fingers extended and minimal shoulder abduction and internal rotation

c. batting at toys with fingers flexed

A patient s/p CVA presents with difficulty performing dressing tasks, using only the unaffected upper extremity. When cued, she does not attempt to use her affected upper extremity. The evaluating therapist suspects unilateral neglect. Which standardized assessment tool would be BEST indicated to evaluate for neglect in this patient? a. Rivermead behavioral memory test b. mini-mental state examination c. behavioral inattention test d. Allen cognitive level test

c. behavioral inattention test

A patient s/p spinal cord injury presents with loss of movement, light touch sensation and proprioception on the same side of injury. Which clinical syndrome is described? a. central cord syndrome b. cauda equina syndrome c. brown-sequard syndrome d. anterior cord syndrome

c. brown-sequard syndrome

a school age child with duchennes muscular dystrophy . although he is able to use a manual chair for distances between classes, he is tired on arrival. What would be the best recommendation the ot could make for the wc use at school. a. retain the manual chair to build up strength b. change to an ultralight sports model because it requires less strength c. change to a power wc to reduce effort. d. encourage walking with a walker to alternate mobility methods.

c. change to a power wc to reduce effort. -progressive disease

An adult is experiencing difficulties with grocery shopping. He has trouble remembering his list, matching his coupons to the products, and finding item categories in the store. Which ability is impaired? a. execution b. opinion c. cognition d. perception

c. cognition

Which of the following statements is FALSE regarding the obtaining and use of assistive technology devices and electronic aids to daily living? a. stability of positioning and seating must be assessed as it will affect the ability to use the device b. devices may have specific instructions involving charging time and modalities c. devices are typically paid for out-of-pocket d. multiple devices should be tried to determine what best serves the individual's needs

c. devices are typically paid for out-of-pocket

When transferring an individual from one seat to another, OT practitioners can BEST pro tect themselves from injury by: a. stepping back from the individual. b. keeping the back in a flexed position. c. keeping the knees bent. d. maintaining a narrow base of support.

c. keeping the knees bent.

An occupational therapist is recommending DME for a patient with a decubitus ulcer. Which pressure-reducing device is intended for people with full-thickness ulcers or those at moderate to high risk for ulcers? a. air-fluidized bed b. standard mattress c. low-air-loss mattress d. foam mattress

c. low-air-loss mattress

In order to ensure best practice, the advanced-level occupational therapist should be provided with what level of supervision? a. general b. routine c. minimal d. close

c. minimal

Regarding self-dressing skills, which of the following tasks is it NOT appropriate to expect of a two-year-old child? a. taking off loosened shoes b. taking off a coat after an adult unzips it c. pulling on socks d. pushing arms through correct holes in a Tshirt

c. pulling on socks - 3 yrs of age

An occupational therapist completes regular weight check on a premature infant to monitor and address diet changes as needed. Which type of intervention does this describe? a. tertiary prevention b. management intervention c. secondary prevention d. primary prevention

c. secondary prevention -addressing concerns early on to reduce effects

An OTR who has passed the initial NBCOT exam has decided not to be recertified through NBCOT. Which consequences will result from the OTR decision? a. the OTR will not be allowed to practice OT in the USA b. The OTR practice opportunities and credentials will remain unchanged c. the OTR may practice OT in the state that does not require certification, but the OTR's credential will change d. the OTR may practice OT in a state that does not require certification and the OTR credentials will not change

c. the OTR may practice OT in the state that does not require certification, but the OTR's credential will change

A middle school student with learning disabilities exhibits no behavioral problems in the classroom. However, whenever the class is in a line waiting to switch classrooms, the student becomes agitated and often pushes classmates. The occupational therapy consultant advises the teacher that this behavior may be indicative of an underlying disorder. Which of the following is most accurate for the therapist to identify as a potential disorder warranting further evaluation? a. Gravitational insecurity. b. A conduct disorder. c. Antisocial tendencies. d .Tactile defensiveness.

d .Tactile defensiveness.

An individual with a TBI is able to pick up a toothbrush and apply toothpaste independently, but takes 15 minutes to brush his teeth. This behavior MOST likely indicates difficulty in which of the following areas? a. Sequencing. b. Following directions. c. Problem solving. d .Termination of activity.

d .Termination of activity.

An individual who has been receiving treatment for an overuse syndrome is about to be discharged. Which of the following is the BEST example of the assessment section of a discharge summary? a. "Pt. can work for up to 3 hours at the computer using periodic stretch breaks." b. "Pt. will take stretch breaks every 30 minutes when working at the computer." c. "Pt. reports being able to work at the computer much longer and more comfortably than initially." d. "Pt. has improved significantly in his ability to work at the computer by using periodic stretch breaks."

d. "Pt. has improved significantly in his ability to work at the computer by using periodic stretch breaks."

An occupational therapist is working as a licensed driving instructor for driver rehabilitation. He is instructing his patient in car safety in relation to seat position and posture. What is the recommended distance between a driver and the steering wheel for safety in the event of a motor vehicle accident? a. 8 inches b. 6 inches c. 10 inches d. 12 inches

d. 12 inches

An occupational therapist is treating a patient who has suffered a deep partial-thickness burn to her axilla and is in the healing phase, so positioning is an important component of intervention. Which is the MOST appropriate position to promote healing and prevent contractures? a. 120-degree abduction with slight internal rotation b. Arm positioned at patient's side c. 60-degree abduction with slight internal rotation d. 120-degree abduction with slight external rotation

d. 120-degree abduction with slight external rotation

Upon evaluating a client for a wheelchair, the OT determines that a standard narrow adult chair would be suitable for the individual. The dimension of this chair will be: a. 18" wide x 18" deep x 20" high b. 16" wide x 16" deep x 18.5" high c. 14" wide x 16" deep x 18.5" high d. 16" wide x 16" deep x 20" high

d. 16" wide x 16" deep x 20" high -standard 18x16x20 -narrow 16x16x20

An occupational therapist measures a person for a wheelchair. The widest point across the person's hips and thighs is 16 inches and the greatest length from the person's posterior portion of the buttocks to the popliteal fossa is 18 inches. Which wheelchair seat dimensions should the therapist recommend? a. 18 inches wide by 20 inches deep. b .18 inches wide by 18 inches deep. c. 16 inches wide by 18 inches deep. d. 18 inches wide by 16 inches deep

d. 18 inches wide by 16 inches deep

An occupational therapist is creating a resting hand splint. What is the appropriate position for the wrist? a. 30-45 degrees extension b. 30-45 degrees flexion c. 20-30 degrees flexion d. 20-30 degrees extension

d. 20-30 degrees extension

An occupational therapist is providing intervention to properly adapt workstations to maintain ergonomic standards. What is the recommended table height for seated and typing at a computer? a. 31-37 inches b.28-31 inches c. The height is not important d. 21-28 inches

d. 21-28 inches

A patient presents with burns the whole length of both legs. What would be the determined burn wound size? a. 18% b. 27% c. 9% d. 36%

d. 36% -the rule of nines

Regarding typical toileting skill development, at what age can a child complete all toileting aspects—hygiene, clothing management, and hand washing—independently? a. 3 years b. 2 years c. 2.5 years d. 4-5 years

d. 4-5 years

An occupational therapist wants to evaluate the ability of an infant to release a toy from his hand. The infant uses his left hand to reach for and grasp a toy that's in his right hand. His right hand does not release the toy until his left hand has a firm grasp on it. At what age does an infant FIRST display this skill? a. 1-4 months b. 0-1 month c. 1 year d. 5-6 months

d. 5-6 months

In regard to self-feeding, at what age does an infant first display the ability to eat from a spoon with assistance? a. 6-8 months b. 12-14 months c. 9-13 months d. 5-7 months

d. 5-7 months

egarding self-feeding skills of growing infants, at what age does an infant begin to feed himself finger foods? a. 12-14 months b. 5-7 months c. 6-8 months d. 9-13 months

d. 9-13 months

A patient presents with a complete spinal cord injury, with no sensation or sacral function. What type/grade of spinal cord injury is described, based on the ASIA Impairment Scale? a. C b. B c. D d. A

d. A

Which of the following patients is NOT eligible for Medicare coverage? a. A 35-year-old man with end-stage renal disease on a waiting list for kidney transplant b. A 66-year-old man with no known health issues c. A 68-year-old woman with Parkinson's disease d. A 40-year-old woman with coronary artery disease

d. A 40-year-old woman with coronary artery disease

A person with scleroderma has limited upper extremity ROM. Coordination is within functional limits. The occupational therapist assesses the individual's computer inputting capabilities and provides a recommendation to improve efficacy. Which adaptation would be most effective for this person? a. An expanded keyboard. b. A concept keyboard. c. A key guard. d. A contracted keyboard.

d. A contracted keyboard.

A patient is recovering from a right CVA resulting in severe left hemiplegia and visuospatial deficits. Their left lower extremity has pitting edema. Which wheelchair would be best for the occupational therapist to recommend for this patient? a. A powered wheelchair with a joystick control and dual elevating leg rests. b. A lightweight active duty wheelchair with dual elevating leg rests. c. A one-arm drive chair with an elevating leg rest on the left. d. A hemiplegic chair with an elevating leg rest on the left.

d. A hemiplegic chair with an elevating leg rest on the left. -not sure what the difference is between this and one-arm but whatever

An adolescent with spina bifida at the C8 level wants to access the new computerized play system that was received as a birthday gift. Which is the best adaptation for the occupational therapist to recommend the adolescent use to access this system? a. A chin switch. b. A tenodesis splint. c. A universal cuff. d. A joystick control.

d. A joystick control.

An individual cannot independently get from a supine position to a sitting position. The person has good scapular, shoulder, and elbow muscle strength. Which of the following should the occupational therapist recommend as most effective for the client to use to improve bed mobility? a. A leg lifter. b. A bed rail assist. c. A log roll technique. d. A rope ladder.

d. A rope ladder. -bed rail is for sitting to standing -log roll requires assistance of someone else (grab someone else to roll over due to lack of core strength)

A Sensory Profile completed by a caregiver indicates that an elementary school-aged child has modulation impairments and sensation seeking patterns. The occupational therapist observes the child frequently wandering, bumping into objects in the room, and fidgeting. Which is the best intervention approach for the therapist to use to address this child's deficits? a. Strategies to increase random sensory input and encourage high physical activity at home. b. An obstacle course that requires diverse movements for varied proprioceptive and tactile input. c. Sensory experiences that focus on body awareness and grading control during play activities. d. A sensory diet that includes controlled sensory input integrated into the child's daily routine.

d. A sensory diet that includes controlled sensory input integrated into the child's daily routine. -overall sensory improvement (not just one area)

An OT begins to see a client for inpatient rehabilitation after the client has undergone a CABG. Which of the following interventions are appropriate for treatment while adhering to precautions?Select one: a. Isometric muscle work, breath holding b. Overhead exercises and holding UEs over head for extensive time periods c. Lateral arm movements and exercises that stretch chest d. AROM, bed mobility, and static standing exercises

d. AROM, bed mobility, and static standing exercises

What exercise could be utilized to help decrease the amount of energy a patient uses during activity participation?Select one: a. Self-propel in wheelchair b. AROM/warm-up exercises c. Static Standing d. Abdominaldiaphragmatic breathing

d. Abdominaldiaphragmatic breathing

A newly referred patient complains of frequently dropping lightweight items and reports a numb feeling in both hands. Which of the following instruments is MOST important for evaluating this individual? a. Goniometer. b. Dynamometer. c. Pinch meter. d. Aesthesiometer.

d. Aesthesiometer.

As defined by Yalom, which "curative factor of groups" involves concern for others? a. Catharsis b. Guidance c. Universality d. Altruism

d. Altruism - also known as selflessness

An OT practitioner is working with a motivated and alert adult who recently had a CVA. Despite many ADL training sessions, the individual still seems unaware of his limitations, is unsafe and unable to learn compensation techniques for neglect, leading the therapist to assess that the individual is MOST likely exhibiting: a. A visual field cut b. Apraxia. c. Aphasia. d. Anosognosia

d. Anosognosia -lack of insight about injury

An OT is conducting an evaluation on a client post-CVA. The client present with contralateral hemiplegia, grasp reflex, incontinence, confusion, and apathy. What is the involved artery and the site of the infarct? Select one: a. Posterior cerebral artery (PCA) b. Middle cerebral artery (MCA) c. Internal carotid artery (ICA) d. Anterior cerebral artery (ACA)

d. Anterior cerebral artery (ACA)

An occupational therapist working in a skilled nursing facility observes a resident with cognitive disabilities don slippers by putting them on the wrong feet. The resident plans to go visit a friend on another floor and does not seem aware that the slippers are on the wrong feet. Which is the best action for the therapist to take in response to this situation? a. Say nothing for this error may embarrass the resident. b. Say nothing but follow the resident to the friend's room to ensure a safe arrival. c. Ask the resident to look at the slippers to see if the error is noticed. d. Approach the resident and advise the resident to reverse the slippers.

d. Approach the resident and advise the resident to reverse the slippers. -safety first

An individual who has recently had a stroke seems to have difficulty with selecting utensils while he eats and the OT practitioner would like to perform a screening for agnosia. Which of the following screening techniques would be MOST appropriate for this purpose? a. Ask the person to demonstrate to you how he would use a knife and fork. b. Ask the person to pick out a spoon and fork from a drawer containing many utensils. c. Ask the person to tell you the steps involved in using the utensils. d. Ask the person to identify several common objects by sight only.

d. Ask the person to identify several common objects by sight only.

An individual with mental illness wants to travel to the library independently, but keeps getting lost. Which of the following actions should the OT practitioner take FIRST in the evaluation process: a. Take the individual to the library and obtain a library card. b. Assess the individual's ability to read. c. Identify the bus that goes to the library and obtain a bus schedule. d. Assess the individual's topographical orientation skills.

d. Assess the individual's topographical orientation skills.

My client was an extensive history of alcohol abuse is referred for an occupational therapy evaluation what type of assessment is most appropriate for the OTR to chose? a. Quantitative measure to determine the alcohol severity b. Sesame to determine the level of dysfunction and relation to expectations of roles c. Trait based or diagnostic basis assessment to determine "normal" d. Assessment measuring occupational performance, self efficacy, and role competence

d. Assessment measuring occupational performance, self efficacy, and role competence

An individual recovering from an exacerbation of multiple sclerosis is referred by a primary care physician to an outpatient OT clinic. The referral requests that the occupational therapist complete a functional capacity evaluation. Which is the most likely reason for this referral? a. Determination of cognitive level. b. Determination of disability status. c. Assessment of instrumental activities of daily living. d. Assessment of return-to-work capabilities.

d. Assessment of return-to-work capabilities.

When assessing the sense of proprioception at an individual's joint, movement within the range would BEST be performed: a. Until pain is elicited. b. Until the stretch reflex is elicited. c. At the end ranges of the joint. d. At the midrange of the joint

d. At the midrange of the joint i.e. shoulder flexion at 90 for FNF test

An older adult recovering from a myocardial infarction is referred to occupational therapy for a home care evaluation. The referral states that the client has high blood pressure and medication-related orthostatic hypotension. Which precaution is most important for the occupational therapist to observe with this client? a. Adherence to dietary restrictions during meal preparation activities. b. Avoidance of activities that require movement against gravity. c. Delay of the OT evaluation until the client's medications are stabilized. d. Avoidance of activities that require sudden postural changes.

d. Avoidance of activities that require sudden postural changes.

A patient presents with difficulty getting his shoes on during an evaluation. Which area of occupation is involved? a. wok b. IADLs c. education d. BADLs

d. BADLs

Which type of fracture is an injury to the fifth metacarpal in the hand? a. Smith's fracture b. Scaphoid fracture c. Colles' fracture d. Boxer's fracture

d. Boxer's fracture

An individual is transferred from an acute care hospital to a subacute rehabilitation unit in a long-term care facility. The patient incurred a left cerebral vascular accident (CVA) in the middle cerebral artery (MCA) one week ago. The individual is referred to occupational therapy. The referral states that the patient has right hemiplegia and a subluxed right shoulder.B96. Upon evaluation, the therapist determines that the patient has right homonymous hemianopsia. The therapist provides recommendations to modify the patient's room to enhance independence. Which are the most appropriate recommendations for the therapist to make for the placement of the patient's call button and cell phone? a. Call button on the left side and the cell phone on the left side. b. Call button on the right side and the cell phone on the right side. c. Call button on the right side and the cell phone on the left side. d. Call button on the left side and the cell phone on the right side.

d. Call button on the left side and the cell phone on the right side. -call button for safety; cell phone to promote scanning

An OTR is reviewing hard-copy of client charts before a group session. While reviewing the charts, the OTR receives an urgent phone call and must leave the chart room. How can the OTR ensure that the charts remain protected during the OTR's absence? a. Close the charts and stack them on the table b. Lock the door to the chart room c. Cover the charts with other documents to obscure them from view d. Close the charts and return them to their original secure location

d. Close the charts and return them to their original secure location

Which of the following scenarios suggests that a patient would benefit more from an individual occupational therapy setting than from a group-based intervention session? a. Generalized exercise program determined by diagnosis b. Desire to interact with others experiencing aphasia c. Basic financial training in a mental health facility d. Completion of a difficult problem-solving task

d. Completion of a difficult problem-solving task

In an outpatient rehabilitation clinic, an occupational therapist is treating a high school student with spina bifida resulting in full motor paralysis and sensory deficits below L1 spinal cord level. The client is a competitive swimmer and is able to transfer independently from the wheelchair to the pool without an assistive device. The client's goal is to learn how to mount and ride a horse. Which is best for the therapist to do initially to help the client attain this goal? a. Implement a home-based transfer training program for the client's family to learn how to assist with mounting a horse. b. Develop an exercise program for the client to do at home on a daily basis to increase upper extremity strength. c. Encourage the client to extend scheduled swimming sessions and practice mounting large inflatable tubes in the pool. d. Consult with a stable owner to discuss alternative methods of mounting a horse.

d. Consult with a stable owner to discuss alternative methods of mounting a horse.

According to Jean Piaget, the sensorimotor period of cognitive development is from birth to two years of age. In which developmental stage are schemes initiated following reflexive movements? a. Primary circular reactions b. Secondary circular reactions c. Coordination of secondary schemata d. Reflexive stage

d. Reflexive stage

An occupational therapist provides bed mobility training for an individual recovering from a left CVA. The therapist notes that the person's right calf is swollen and warm. The person complains that it is painful. Which action should the therapist take initially? a. Elevate the leg and provide retrograde massage. b. Advise the person to tell the physician about the symptoms during the physician's next bedside visit. c. Continue with the training and document the symptoms in the medical record. d. Contact the charge nurse immediately to report symptoms.

d. Contact the charge nurse immediately to report symptoms.

A person with arthrogryposis undergoes serial casting with weekly cast changes of the right wrist. Upon cast removal during the fourth week, the therapist notes a small open area 0.25 cm by 0.25 cm and a red rash over the ulnar styloid. Which is the therapist's best response to these observations? a. Pad the area and apply another cast. b. Refer the individual to the wound care team. c. Fabricate a static splint that does not impede on the ulnar styloid. d. Contact the physician and describe the observations.

d. Contact the physician and describe the observations. -MD will make decision about how to reduce wound (static cast vs new cast)

An older adult diagnosed three years ago with a neurocognitive disorder has been admitted to the hospital for regulation of medication. The occupational therapist determines that the person demonstrates diminished memory skills since the previous evaluation but is still able to live independently at home with support and supervision. During the discharge planning meeting, which activity should the therapist recommend family members perform for the patient? a. Weeding the garden. b. Sorting and folding laundry. c. Preparing cold sandwiches. d. Cooking hot meals.

d. Cooking hot meals.

An occupational therapy evaluation reveals findings consistent with ulnar nerve compression at the elbow. Which peripheral nerve disorder is consistent with this finding? a. Carpal tunnel syndrome b. Pronator teres syndrome c. Guyon's canal d. Cubital tunnel syndrome

d. Cubital tunnel syndrome

Proprioceptive neuromuscular facilitation (PNF) is used therapeutically for many individuals with neurological conditions; it focuses on using movement patterns to improve motor skills for functional tasks. Which upper extremity pattern is used when the therapist is trying to promote improved function in reaching low and behind the body, like when placing an arm in a jacket sleeve? a. D1 flexion b. D2 flexion c. D2 extension d. D1 extension

d. D1 extension

Proprioceptive neuromuscular facilitation (PNF) is used therapeutically for many individuals with neurological conditions; it focuses on using movement patterns to improve motor skills for functional tasks. Which upper extremity pattern is used when the therapist is trying to promote improved function of bringing food to mouth with self-feeding? a. D2 extension b. D2 flexion c. D1 extension d. D1 flexion

d. D1 flexion

Which of the following hand injuries is characterized by pain and swelling along the radial styloid of the wrist and a positive Finkelstein's Test? a. Dupuytren's disease b. Colles' fracture c. Skier's Thumb/Gamekeeper's Thumb d. De Quervain's tenosynovitis

d. De Quervain's tenosynovitis

A high school teacher diagnosed with a right-hemisphere CVA is given a paper with letters of the alphabet displayed in random order across the page and is instructed to cross out every "M." The individual misses half of the "M"s in a random pattern, which MOST likely indicates: a. A left visual field cut b. A right visual field cut c. Functional illiteracy d. Decreased attention

d. Decreased attention -no pattern

An OT practitioner in an out-patient clinic is evaluating problem-solving and sequencing skills of a woman with a longstanding history of mental illness as she makes brownies from a mix. The therapist observes the patient put the whole egg, shell and all, into the bowl. Which of the following actions should the therapist take FIRST? a. Evaluate the individual's cognitive function. b. Determine that the individual is interested in meal preparation as a goal, then develop short- and long-term goals and a treatment plan. c. Schedule the individual for meal preparation group sessions to improve skill level. d. Determine the individual's home environment and her need for meal preparation skills.

d. Determine the individual's home environment and her need for meal preparation skills. -client centered

An occupational therapist provides home-based services to a home maintainer who incurred a right CVA eight months ago. The individual and the therapist have chosen to focus on kitchen activities during the intervention session. The therapist has the client stand in front of the counter with an open dishwasher to the left. The therapist asks the client to put the clean dishes into an overhead cabinet to the right of the client using the affected UE. By setting up the activity in this manner, which proprioceptive neuromuscular facilitation (PNF) technique is the therapist using? a. Heavy work/mobility superimposed on stability. b. Reciprocal inhibition/innervation. c. Diagonal patterns of D2 flexion/extension. d. Diagonal patterns of D1 flexion/extension.

d. Diagonal patterns of D1 flexion/extension.

An OTR is planning group programming in an acute care psychiatric setting for individuals with sever mental illnesses who display disorganized thinking and difficulty functioning in many occupation areas. What type of group is MOST APPROPRIATE for this population? a. Activity group b. Psychoeducational group c. Neurodevelopment group d. Directive group

d. Directive group

Evaluation results for a person with arthritis will MOST accurately reflect true functional abilities if scheduled: a. Early morning (8 to 10 a.m.). b. Afternoon. c. Late morning (10 to 11 a.m.). d. Early morning and again in the afternoon.

d. Early morning and again in the afternoon.

An OT practitioner is observing dressing skills in an individual with COPD. While putting on his shirt, the individual becomes short of breath and stops to rest before finishing with the shirt and going on to his trousers. This behavior MOST likely indicates a deficit in: a. Postural control. b.Muscle tone. c. Strength. d. Endurance.

d. Endurance.

A graduate student with an anxiety disorder reports feeling confused about the future. During the OT evaluation, the client relates decreased feelings of competence for their chosen field of study and overall poor personal causation. Which is the best initial action for the therapist to take in response to the client's stated concerns? a. Administer a vocational interest inventory. b. Provide activities related to the client's chosen field of study. c. Refer the client to the state office of vocational and educational services. d. Establish short-term goals with high potential for attainment.

d. Establish short-term goals with high potential for attainment.

A patient who is nonverbal has been successfully using an augmentative communication device, however the device stopped working. What should be done to help this patient? a. Immediately physically examine the patient b. Reassess visual and motor abilities c. Check the patient's ability to use the augmentative device d. Evaluate the patient's augementative device setting

d. Evaluate the patient's augementative device setting

An OTR is working with a 4-year old child with developmental delays. The goal of OT for this child is promote hand function, allowing the client to hold a crayon more efficiently. Which activity is the MOST EFFECTIVE in developing this hand skill? a. Rolling clay into a ball b. Finger painting c. Crumpling paper d. Eye dropper painting

d. Eye dropper painting

Which Occupational Therapy Code of Ethics principle refers to the matter in which you approach any relevant persons when acting as a healthcare provider including the attributes of compassion respect good judgment and honesty? a. Justice b. Nonmaleficence c. Autonomy d. Fidelity

d. Fidelity

A patient has agraphia. Which activity should the therapist modify to facilitate success? a. Reading a home exercise program b. Expressing their needs to the health team c. Performing calculations d. Filling out paperwork

d. Filling out paperwork

A child with developmental delay has poor oral motor control. Which should the occupational therapist do to facilitate lip closure? a. Give pressure with the index finger under the jaw. b. Firmly place a spoon with food on the back part of the tongue. c. Place the thumbs on the lateral ends of the mandibles. d. Give a slight upward sweep of the index finger from the lower jaw to the lower lip.

d. Give a slight upward sweep of the index finger from the lower jaw to the lower lip.

Which condition related to vision loss is associated with increased pressure in the eye and an initial decrease in peripheral field vision? a. Diabetic retinopathy b. Macular degeneration c. Cataracts d. Glaucoma

d. Glaucoma

Which type of qualitative methodology in research involves great immersion of the researcher into a particular situation or experience in order to obtain a firsthand account? a. True experimental research b. Phenomenological research c. Ethnographic research d. Heuristic research

d. Heuristic research

An OTD conducts a comprehensive home assessment with a physical therapist and social worker for an elderly client with long-standing herniated nucleus pulpous of C5-7 with partial paralysis of the upper extremities. The home assessment was conducted prior to the client's planned discharge home from a SNF. The assessment reveals that the client is unable to drive and lives alone in a multi-level home without strong family support in the area. What would be the MOST APPROPRIATE recommendation for the OTR to make to facilitate the client's safe return home? a. Inpatient rehab prior to discharge home b. Outpatient therapy for a PT to address mobility and an OTR to address activities of family living c. Client remain in SNF with ongoing PT, OT, and nursing d. Home health services with a live-in caregiver until the client is deemed safe to be alone

d. Home health services with a live-in caregiver until the client is deemed safe to be alone

Which principle of the Occupational Therapy Code of Ethics requires the clinician to be the advocate for a patient to receive services? a. Beneficence b. Veracity c. Autonomy d. Justice

d. Justice

A rehabilitation hospital is interested in starting a driver rehabilitation program. Which must the occupational therapist hired to develop this program do first? a. Determine the cost of commercially available driving rehabilitation programs. b. Develop admission criteria for program participants. c. Develop a marketing plan to obtain referrals. d. Learn the state's driving laws and requirements.

d. Learn the state's driving laws and requirements.

A patient s/p CVA presents with hemiplegia, impaired sensation, and aphasia. What is the likely site of infarct? a. Posterior cerebral artery b. Anterior cerebral artery c. Right middle cerebral artery d. Left middle cerebral artery

d. Left middle cerebral artery

While evaluating an individual's ADL status on the first day of treatment following open heart surgery, it is MOST important for the OT practitioner to: a. Keep the MET level below 3.0. b. Observe for shortness of breath. .c Listen for complaints of chest pain. d. Monitor heart rate, blood pressure, and symptoms.

d. Monitor heart rate, blood pressure, and symptoms.

A 10-year old child with CP is working in the classroom on signing work. The OTR has been facilitating written name production during direct intervention times; however, the task is tedious and frustrating to the child. The OTR has consulted with the teacher and decided to incorporate some adaptive equipment to provide positive feedback to the child as well as identify the child's work. Which is the MOST appropriate adaptive equipment to use with this child? a. Pencil grip b. Raised notebook c. Weighted pencil d. Name stamp

d. Name stamp

An OT receives a home health referral for a 78-year-old male client with general weakness. During the evaluation, the OT notices that the client is dehydrated, malnourished, has poor personal hygiene, and is living in unsafe conditions. The OT suspects that this is a case of elderly abuse. Given these signs and symptoms, what form of elderly abuse is suspected?Select one: a. Physical abuse b. Sexual abuse c. Psychological abuse d. Neglect

d. Neglect

In regard to releasing skills, what is the correct sequence of progression? a. No release, to involuntary release, to one-stage transfer, to two-stage transfer, to voluntary release b. Involuntary release, to no release, to two-stage transfer, to one-stage transfer, to voluntary release c. No release, to two-stage transfer, to one-stage transfer, to involuntary release, to voluntary release d. No release, to involuntary release, to two-stage transfer, to one-stage transfer, to voluntary release

d. No release, to involuntary release, to two-stage transfer, to one-stage transfer, to voluntary release

During a group session at an adult day care program, an older adult consistently complains that everyone is mumbling. After the group, which action should the occupational therapist take in response to these statements? a. Notify the client's primary care physician that the person exhibited evidence of paranoia. b. Collaborate with the program director to remove groups from the client's program plan. c. Document objective data about the complaints in the person's chart. d. Notify the primary care physician that the person may need an audiological evaluation.

d. Notify the primary care physician that the person may need an audiological evaluation. -documenting does not deal directly with issue

An occupational therapist works in a program for survivors of domestic violence. The therapist applies a client-centered approach to guide intervention. When using this approach, which is best for the therapist to do? a. Offer specific concrete behavioral suggestions for dealing with confrontations. b. Respond to the participants' self-deprecating comments with positive feedback on personal characteristics. c. Reinforce only the participants' neutral comments about themselves and personal skills. d. Reflectively paraphrase the participants' statements and ask for their verification.

d. Reflectively paraphrase the participants' statements and ask for their verification.

OTR and OTA work together for a home health company. The state in which they work requires OTR's and OTA's to obtain a physical agent modality certification to use and apply PAMs to clients. The OTR has received PAMs certification but the OTA has not. The OTR to properly manage her caseload and supervise the OTA, which of the following to be the best supervisory strategy? a. OTR should provide close supervision when the OTA is using PAMS with clients b. OTR should provide supervision and instruction for the OTA for the proper use of PAMS c. OTR should review precautions and contraindications for the use of PAMS with the OTA d. OTR should be responsible for selection and direct application of PAMs

d. OTR should be responsible for selection and direct application of PAMs

The FIRST step an OT practitioner should take in screening visual problems with a developmentally disabled adult would be to: a. Assess and adapt lighting in the client's living environment. b. Administer formal visual assessment of acuity and visual fields. c. Refer the client to an optometrist or eye care professional. d. Observe for signs of visual deficits during daily living activities.

d. Observe for signs of visual deficits during daily living activities.

Which of the following patients should be placed on droplet precautions? a. Patient with chickenpox virus b. Patient with tuberculosis c. Patient with measles virus d. Patient with influenza

d. Patient with influenza

An occupational therapist is working with a three-year-old child and notices burn marks on her arm and a large area of bruising. He should suspect what type of child abuse? a. Neglect b. Emotional abuse c. Sexual abuse d. Physical abuse

d. Physical abuse

Which Occupational Therapy Code of Ethics principle requires clinicians to provide, maintain, and ensure best-practice standards for all patients receiving OT services? a. Principle 3: Autonomy b. Principle 2: Nonmaleficence c. Principle 4: Justice d. Principle 1: Beneficence

d. Principle 1: Beneficence

The administrator of a home care agency tells the occupational therapist to submit all intervention plans to the client's third party payers, prior to the implementation of treatment. Which is the most accurate term for the therapist to use when documenting these actions? a. Concurrent review. b. Peer review. c. Utilization review. d. Prospective review.

d. Prospective review. -PRIOR to treatment -concurrent is ongoing

A person recovering from a cerebral vascular accident has left-sided weakness and dysphagia. Which of the following is the most effective direct intervention approach to help the person successfully swallow ingested food? a. Provide pureed, thick liquids. b. Provide thermal stimulation to the inferior faucial arches. c. Tilt the person's head back and toward the left side. d. Provide small, warm boluses.

d. Provide small, warm boluses. -oral motor control direct intervention that you can modify act, consistency, and temperature - A. and B. are indirect

When analyzing and interpreting data for descriptive statistics, which term provides information regarding the scope of reported data? a. Mean b. Mode c. Median d. Range

d. Range

What should an acute care OT practitioner do FIRST to obtain accurate information about a patient's family situation and about his occupational, cultural, and educational backgrounds? a. Read the medical history record. b. Interview the patient's family. c. Interview the patient. d. Read the social worker's report.

d. Read the social worker's report. -most accurate

An elementary school-aged child with Duchenne's muscular dystrophy receives occupational therapy services. The family establishes a goal of maintaining the child's leisure and social participation. Which is the best activity for the occupational therapist to recommend the family pursue with this child? a. Electronic sports (e.g., Wii bowling). b. Adapted little league baseball. c. Wheelchair basketball. d. Recreational swimming

d. Recreational swimming

When selecting activities for an 8-year-old child with Duchenne muscular dystrophy, which developmental issue is MOST important to consider? a. Establishment of basic trust. b. Freedom to use his initiative. c. Development of self-identity. d. Reinforcement of competence

d. Reinforcement of competence -trust is infancy, initiative is toddlers, self-identity is adolescence

According to Jean Piaget, there is a hierarchical development of cognition. During the concrete operations period (age 7-11 years), which term describes greater understanding of the conservation of objects? a. Empirical-inductive thinking b. Classification c. Seriation d. Reversibility

d. Reversibility an Objects property stays the same even when it changes

An individual with obsessive-compulsive personality disorder participates in a vocational program. The client asks the occupational therapist to speak to the supervisor of the transitional employment program (TEP). The client is concerned that compulsive behaviors are interfering with job performance and may result in the loss of a new TEP placement. Which is the therapist's best response to these expressed concerns? a. Instruct the client to speak directly to the TEP supervisor about the right to receive reasonable accommodations. b. Schedule a reevaluation of the client's work behaviors and skills. c. Assure the client that it is natural to have initial difficulties at a new job. d. Schedule an appointment with the client and the TEP supervisor.

d. Schedule an appointment with the client and the TEP supervisor.

Which is an appropriate intervention for energy conservation and work simplification for a patient who has congestive heart failure? a. Perform household IADLs while standing b. Lift items when relocating them c. Rest after onset of exhaustion d. Schedule rest breaks throughout the day

d. Schedule rest breaks throughout the day

Upon completion of the initial interview and chart review, the NEXT step to be taken in the OT process is to: a. Analyze the data. b. Develop the treatment plan. c. Perform selected assessments. d. Select appropriate evaluation procedures.

d. Select appropriate evaluation procedures.

An occupational therapist is working with a client who had a LE amputation. Which of the following treatment is NOT considered an appropriate treatment? Select one: a. Transfer training using standing pivot b. Desensitization c. Wrapping the residual limb to decrease swelling d. Strengthening exercises on the residual limb

d. Strengthening exercises on the residual limb

An OT practitioner has been asked to perform an ergonomic evaluation and provide ergonomic interventions to a job site where the rate of cumulative trauma disorders is unusually high. Which of the following actions BEST address this request? a. Introduce relaxation seminars for employees to decrease stress while on the job. b. Treat corporate clients for cumulative trauma disorders. c. Provide work-simulation activities. d. Suggest furniture and accessories that promote better positioning at work.

d. Suggest furniture and accessories that promote better positioning at work. -cumulative trauma = repetitive injury

The supervisor of an acute inpatient unit requests that a recently hired entry-level therapist write summaries of several evaluation sessions that were completed by another therapist. The evaluating therapist had to leave work unexpectedly due to a medical emergency and is not expected to return to work. Which is the best response for the therapist to make in response to this request? a. Comply with the supervisor's request but ask for the supervisor to co-sign the notes b. .Request time to complete an independent evaluation of each individual previously evaluated. c. Report the supervisor's request to the facility's administration. d. Suggest that the therapist's evaluation results be documented by the supervisor.

d. Suggest that the therapist's evaluation results be documented by the supervisor.

The OT is observing a typical 3-year-old child playing house with her doll. The child is using a teapot filled with cotton balls that symbolizes the "tea" she is serving. She uses a variety of words that she did not use before, as well as phrases. While there are other children in the clinic, the child is not playing with them. What stage of play is the child experiencing?Select one: a. Creative Play b. Exploratory Play c. Games d. Symbolic Play

d. Symbolic Play

Which is NOT a recognized neurobehavioral subsystem? a. Emotional state b. Motor system c. Autonomic system d. Sympathetic system

d. Sympathetic system - neurobehavioral is autonomic, motor, emotional state, attention-interaction, & self-regulation

A patient s/p spinal cord injury presents with loss of movement, as well as loss of pain and temperature sensation below the injury (bur maintains light touch and propiroception). Which clinical syndrome is described? a. central cord syndrome b. cauda equina syndrome c. brown-sequard syndrome d. anterior cord syndrome

d. anterior cord syndrome

Which is NOT a useful intervention for redirecting an agitated individual? a. provide interesting activities as a means of distraction b. speak in a calm voice with short sentences c. provide opportunities to engage in interesting discussion d. approach the patient from behind to catch them off guard

d. approach the patient from behind to catch them off guard

A child with a diagnosis of traumatic brain injury (TBI) is evaluated by an occupational therapist. The child presents with extension of both upper extremities and flexion of both lower extremities following a stimulus of neck extension. When interpreting this observation, which statement is most accurate for the therapist to document? a. The presence of + ATNR, which is "abnormal" and has reappeared after the TBI. b. The presence of a + STNR, which is "normal" and not affected by the TBI. c. The presence of a + ATNR, which is "normal" and not affected by the TBI. d. The presence of a + STNR, which is "abnormal" and has reappeared after the TBI.

d. The presence of a + STNR, which is "abnormal" and has reappeared after the TBI.

As part of occupational therapy intervention, therapists often assist in consulting for proper environmental safety, including ramp design. Which of the following statements about ramp design is INCORRECT? a. Four inches is adequate for curb height b. Railings should be between 34"-38" c. The ramp should have 12 inches in length for every 1 inch in height d. The ramp should have a 2' x 2' platform before a doorway

d. The ramp should have a 2' x 2' platform before a doorway - needs to be 5x5

An occupational therapist is assisting with the selection of an appropriate wheelchair for her patient. The patient is 10 years old and has spastic cerebral palsy. She is nonambulatory and requires assistance with self-care and transfers. She is unable to self-propel a wheelchair. What is the BEST option for a wheelchair? a. One-arm-drive b. Stair-climbing c. Sports wheelchair d. Tilt-in-space

d. Tilt-in-space

Which of the following is NOT a reason to conduct family interviews in the home setting for a pediatric occupational therapy patient? a. To determine the level of support for the child within the household b. To better understand the child's home environment c. To determine relevant cultural preferences d. To discuss the occupational therapy center's goals and priorities

d. To discuss the occupational therapy center's goals and priorities

Which form of amputation is considered the MOST common? a. Hip disarticulation b. Transfemoral c. Hemipelvectomy d. Transtibial

d. Transtibial

COTA working on an inpatient rehab has had 2 patients fall in last month. OTR has concerns about the COTA's safety awareness during certain interventions. Which approach is the best for the OTR? a. speak with the rehab manager about possible disciplinary action b. review the documentation of the 2 patients who fell and discuss their cases thoroughly with the COTA to determine cause of the falls c. ask the COTA whether more thorough supervision is needed and then proceed to provide close supervision d. collaborate with the COTA to determine potential causes of the falls and provide direct supervision with more deliberate feedback

d. collaborate with the COTA to determine potential causes of the falls and provide direct supervision with more deliberate feedback

An OTR is working in a school with high incidence of violence in the district with increasing numbers of students diagnosed with ODD. The superintendent announces the formation of a task force to study the problem. Which response to the announcement would be best? a. document the progress of the OTR has made with students with ODD to justify continuing work with these students b. increase the frequency of visits to the students on the OTR caseload with his diagnosis because the task force will scrutinize intervention with these students. c. research intervention strategies to make sure that the intervention the OTR has been providing to these students is considered d. contact the superintendent and request to be appointed to a seat on the task force to study violence and incidence of ODD.

d. contact the superintendent and request to be appointed to a seat on the task force to study violence and incidence of ODD.

A patient presents with ape hand deformity. What is the likely injury or disorder? a. proximal median nerve injury b. ulnar nerve lesion c. radial nerve lesion d. distal median nerve injury

d. distal median nerve injury

An individual who works as a nurse reports difficulty squeezing the bulb of the sphygmomanometer when taking blood pressures and difficulty opening pill bottles. Which of the following instruments would be MOST appropriate for assessing this individual? a. goniometer. b. aesthesiometer. c. volumeter. d. dynamometer.

d. dynamometer.

Which type of work program focuses on prevention of overuse injuries in the workplace? a. rehabilitation workshop b. work conditioning program c. work hardening program d. ergonomic program

d. ergonomic program

You read in a TBI patient's chart that they are at Level III of the Rancho Los Amigos Levels of Cognitive Function Scale. Which of the following is NOT an appropriate intervention for a patient at this level of cognition? a. Simple ADL to promote automatic responses b. ROM to prevent contracture/tone and spasticity management c. Sensory stimulation such as tactile, visual, auditory, temperature, and proprioception d. multi-step command following

d. multi-step command following

Which statement is FALSE regarding the use of personal protective equipment (PPE)? a. do not wear gloves that are too large and unable to provide tight coverage b. wear personal protective equipment when assisting with toileting and self care tasks c. when wearing a gown and gloves, remove the gown first before removing gloves following patient care d. personal protective equipment must be carefully removed outside the patient room

d. personal protective equipment must be carefully removed outside the patient room -inside to prevent contamination

Validity is an important element of standardized assessments to ensure you are obtaining an accurate measurement of whichever skill or ability you wish to measure. Which type of validity refers to the ability of an assessment to predict scores for a similar evaluation? a. face validity b. criterion validity c. content validity d. predictive validity

d. predictive validity

What form of grasp is displayed when a four-month-old infant holds a small toy block but does not use the thumb? a. radial-palmar grasp b. radial-distal grasp c. palmar grasp d. primitive squeeze grasp

d. primitive squeeze grasp

Child is referred to occupational therapy to address poor hand writing skills. The OT observed the child holding the pencil with a lateral grass, the paper moves on the desk when writing, and the child posture is slumped over the desk. What is the most appropriate initial activity to address this child handwriting? a. Playdouh activities to strengthen the hands and prehension b. teaching the child to hold the paper more firmly with the non-dominant hand c. using a stant board during the writing process in the classroom d. prone activities on the scooter board to build core strength

d. prone activities on the scooter board to build core strength -work distal to proximal need to strengthen core before UE can be stable

An occupational therapist finds that his patient has poor control of upward rotation of the scapula. Which of the following would NOT be a contributing muscle? a. upper trapezius b. middle trapezius c. lower trapezius d. rhomboids

d. rhomboids -they assist with downward rotation

What is the site of referred pain from a gallbladder attack? a. low back pain b. right side lower abdominal pain c. left arm pain d. right subscapular pain

d. right subscapular pain - referred pain means in a different location than where issue takes place

An occupational therapist is providing intervention for positioning s/p a posterior total hip replacement. What is NOT an appropriate position? a. using a hip abductor pillow between legs b. sitting in a tall chair c. lying supine d. rolling toward the non-operated side

d. rolling toward the non-operated side

According to Abraham Maslow's hierarchy of needs, which level involves the culmination of personal development, only obtained after all pervious needs are met? a. physiological b. love and belonging c. self-esteem d. self-actualization

d. self-actualization

At six months of age, which form of reaching is observed? a. reaching with bilateral arms b. unilateral reaching with the shoulder flexed and externally rotated and the forearm supinated c. batting at toys with fingers flexed d. unilateral reaching with fingers extended and minimal shoulder abduction and internal rotation

d. unilateral reaching with fingers extended and minimal shoulder abduction and internal rotation

A client exhibits visual deficits following a cerebrovascular accident. Which of the following are most important for the OT to include in the initial occupational therapy evaluation? a. visual memory and visual field loss b. visuocognition and visual field loss c. visuocognition and oculomotor control d. visual field loss and oculomotor control

d. visual field loss and oculomotor control -lowest level and most functional

an ot provides caregiver training to the spouse of an indv. With cerebellar cortical degeneration. The focus of the session is on community mobility using a wheelchair. The individual is dependent upon his spouses assistance for mobility. The ot advises the spouse that when descending a steep grade the best method is to: a. go down backwards with all wc wheels maintaining contact with ground surface b. tilt the wc backwards to its gravitational balance point and then go down forward. c. tilt the wc backwards to its gravitational balance point and then go down backwards. d. push forward as on flat surfaces but lean body back for extra lag.

go down backwards with all wc wheels maintaining contact with ground surface -enables the spouse to use her body weight to slow the chair's momentum.


संबंधित स्टडी सेट्स

Ch 42: Management of Patients with Musculoskeletal Trauma (2)

View Set